You are on page 1of 495

Chapter 1

ASSURANCE SERVICES

1. Which of the following statements best describes assurance


services?
A. Independent professional services that are intended to
enhance the credibility of information to meet the needs of an
intended user.
B. Services designed to express an opinion on the fairness of
historical financial statements based on the results of an
audit
C. The preparation of the financial statements or the
collection, classification and summarization of other
financial information.
D. Services designed for the improvement of operations
resulting in better outcomes.

Assurance engagements performed by professional accountants


are intended to enhance the credibility of information about
subject matter by evaluating whether the subject matter
conforms in all material respects with suitable criteria,
thereby improving the likelihood that the information will
meet the needs of the intended user.
Services performed by professional accountants that are not
assurance engagements include the following:
1. Agreed-upon procedures
2. Compilation of financial or other information
3. Preparation of tax returns where no conclusion is
expressed, tax consulting
4. Management consulting
5. Other advisory services

2. Which of the following is not an assurance service?


A. examination of prospective financial information
B. Audit of historical financial statements
C. Review of financial statements
D. Compilation of financial information.
3. Which of the following professional services would be
considered an assurance engagement?
A. A management consulting engagement to provide IT advice to
a client
B. An engagement to report on compliance with statutory
requirements.
C. An income tax engagement to prepare tax returns.
D. A compilation of financial statements from a clients
accounting record

4. Which of the following best describes the objective of an


assurance engagement?
A. Improve the company outcomes
B. Compare the companys information and policies with those
of other entities
C. Enhance the credibility of information in order to improve
the likelihood that the information will meet the needs of
intended user.
D. Assist in preparing the companys financial statements.

The objective of an assurance engagement is for a professional


accountant to evaluate or measure a subject matter that is the
responsibility of another party against identified suitable
criteria, and to express a conclusion that provides the
internal users with a level of assurance with the subject
matter
An assurance engagement is intended to enhance the credibility
of information about a subject matter by evaluating whether
the subject matter conforms in all material respects with
suitable criteria, thereby improving the likelihood that the
information will meet the needs of the an intended user

5. Assurance services differ from consulting services in that


they:
I. Focus on providing advice
II. Involve monitoring of one party by another
A. I only C. Both I and II
B. II only D. Either 1 and II
Assurance services differ from consulting services in that
they:
1. Focus on enhancing the credibility of information
rather that providing advice
2. Typically involve situations in which one party wants
to monitor another
Consulting services are usually two-party arrangements that
focus on providing advice on how to use the information for
better outcomes.

6. How many separate parties are involved in an assurance


engagement?
A. 2 C. 4
B. 3 D. 5

7. An assurance engagement should have which of the following


elements?
Subject Matter Criteria
A. Yes No
B. No Yes
C. Yes Yes
D. No No

An assurance engagement should have the following elements:


1. A three-party relationship involving:
A. a professional accountant
B. a responsible party
C. intended users
2. A subject matter
3. Suitable criteria
4. Sufficient appropriate evidence
5. An assurance report
8. The Philippines Framework for Assurance Engagements
A. Contains basic principles, essential procedures and related
guidance for the performance of assurance engagements.
B. Define and describes the elements and objectives of an
assurance engagements and identifies engagements to which
PSAs, PSREs and PSAEs apply.
C. Provides a frame of reference of CPAs in public practice
when performing audits, review and compilations of historical
financial information.
D. Establishes standards and provides procedural requirements
for the performance of assurance engagements.

The Framework defines and describes the elements and objective


of an assurance engagement and identifies engagements to which
Philippine Standards of Auditing (PSAs), Philippines Standards
on Review Engagements (PSREs) and Philippine Standards on
Assurance Engagements (PSAEs) apply.
The Framework does not itself establish standards or provide
procedural requirements for the performance of assurance
engagement. PSAs, PSREs and PSAEs contain basic principles,
essential procedures, and related guidance, consistent with
the concepts in the Framework, for the p [performance of
assurance engagements.

9. CPAs in public practice who perform assurance engagements


are governed by the following, except:
A. Philippine Framework for Assurance Engagements
B. Code of Ethics for Professional Accountants in the
Philippines
C. Philippine Standards in Related Services
D. Philippine Standards in Quality Control

10. in an assurance engagements, the responsible party and the


internal users:
A. Should be from different entities
B. Should be from the same entity
C. May be from the same entity or different entities
D. Are both responsible for determining the nature, timing and
extent of the procedures to be performed?

According to the Philippine Framework for Assurance


Engagements, the responsible party and the intended users may
be from different entities or the same entity.
Answer D is incorrect because the practitioner is responsible
for determining the nature, timing and extent of procedures to
be performed.

11. The subject matter of an assurance engagement may include:


Financial Information Internal Control Compliance with
Regulation
A. Yes Yes Yes
B. No No No
C. Yes No Yes
D. No Yes No

According to the Philippine Framework for Assurance


Engagement, an assurance engagements subject matter may
include the following:

Financial performance or conditions such as historical or


perspective financial position, financial performance and
cash flows
Non-financial performance or conditions, for example,
performance of an entity
Physical characteristics, for example, an entities
internal control or IT system
Behavior, such as corporate governance, compliance with
regulation, human resource practices.

12. For assurance engagement regarding historical financial


information, reasonable assurance engagements are called:
A. Audit C. Compilation
B. Review D. Examination
13. When performing as assurance service, professional
accountants use standards or benchmarks to evaluate or measure
the subject matter of an assurance engagement. These are
referred to in the Framework as
A. Criteria C. Conditions
B. Norms D. Gauges

Criteria are the standards or benchmarks used to evaluate or


measure the subject matter of an assurance engagement. These
are important because they establish and inform the intended
users of the basis against which the subject matter has been
evaluated or measured in forming the conclusion.

14. The criteria against which the subject matter of the


assurance engagements is to be evaluated or measured should
process which of the following characteristics?
Relevant Concise Neutral
A. Yes No Yes
B. No Yes No
C. Yes No No
D. No Yes Yes

15. Relevant criteria contribute to conclusions that are


A. Free from bias
B. Clear and comprehensive
C. Subject to different interpretations
D. Useful for decision making

According to the Philippine Framework for Assurance


Engagements, suitable criteria should have the following
characteristics:
1. Relevance

Relevant criteria contribute to conclusions that assist


decision-making by the intended user
2. Completeness

Criteria are sufficiently complete when relevant factors


that could affect the conclusions in the context of the
engagement are not omitted.
Complete criteria include, where relevant, bench-marks
for presentation and disclosure
3. Reliability

Reliable criteria allow reasonable consistent evaluation


or measurement of the subject matter including where
relevant, presentation disclosure, when used in similar
circumstance by similar qualified practitioners.
4. Neutrality

Neutral criteria contribute to conclusions that are free


from bias.
5. Understandability

Understandable criteria contribute to conclusions that


are clear, comprehensive, and not subject to
significantly different interpretations.

16. Criteria that are embodied in laws or regulations or


issued by authorized or recognized body of experts that follow
a transparent due process are called
A. Suitable Criteria
B. Established Criteria
C. Specifically developed criteria
D. General criteria

The Framework states that criteria can either be established


or specifically developed. Established criteria are those
that are embodied in laws or regulations, or issued by
authorized or recognized bodies. Specifically developed
criteria are those designed for the purpose of the engagement.

17. in an assurance engagements, the person or persons, either


as individuals or representative of an entity, responsible for
the subject matter is the
A. Intended user
B. Responsible party
C. Professional Accountant
D. Client

The responsible party is the one responsible for the subject


matter of an assurance engagement. For example, an entitys
management is responsible for the preparation and presentation
of financial statements or the establishment and
implementation of internal control.
The responsible party may or may not be the party who engages
the professional accountant.

18. In an assurance engagement, the person or class of persons


for whom the professional account prepares the report for a
specific user or purpose is the
A. Intended user
B. Responsible party
C. Management
D. Client

The intended user is the person or a class of persons for whom


the professional accountant prepares the report for a specific
use or purpose.

19. In an assurance engagement, the outcome of the evaluation


or measurement of a subject matter against criteria is called
A. Subject matter information
B. Subject matter
C. Assurance
D. Conclusion

The term subject matter information is used in the Framework


for Assurance Engagements to mean the outcome of the
evaluation or measurement of a subject matter.
According to the Framework, it is the subject matter
information about which the practitioner gathers sufficient
appropriate evidences to provide a reasonable basis for
expressing a conclusion in an assurance report.
20. In some assurance engagements, the evaluation or
measurement of the subject matter is performed by the
responsible party, and the subject matter information is in
the form of an assertion by the responsible party that is made
available to the untended users. It is called:
A. Direct reporting engagements
B. Assertion-based engagements
C. Non-assurance engagements
D. Recurring engagements

21. The following are characteristics of direct reporting


assurance engagements, except
A. The subject matter information is in the form of an
assertion by the responsible party that is made available to
the intended users.
B. The subject matter information is provided to the intended
users in the assurance report
C. The practitioner either directly performs the evaluation or
measurement of the subject matter our obtains a representation
from the responsible party that has performed the evaluation
or measurement
D. The representation of the responsible party that has
performed the evaluation or measurement of the subject matter
is not available to the intended users.

22. What type of assurance engagement is involved when the


practitioner expresses a positive form of conclusion?
A. Limited assurance engagement
B. Positive assurance engagements
C. Reasonable assurance engagements
D. Absolute assurance engagements

According to the Framework, the objective of the reasonable


assurance engagement is the reduction in the assurance
engagement risk to an acceptable low level in the
circumstances of the engagements the basis for a positive
form of expression of the practitioners conclusion.
23. What type of assurance engagements is involved when the
practitioner expresses a negative form of conclusion?
A. Reasonable Assurance engagements
B. Negative assurance engagements
C. Assertion-based assurance engagements
D. Limited assurance engagements

According to the Framework, the objective of the limited


assurance engagement is the reduction in the assurance
engagement risk to the level that is acceptable in the
circumstances of the engagement, but where the risk is
greater than for reasonable assurance engagement, as the
basis for a negative form of expression of the
practitioners conclusion.

24. A practitioners assurance report contains the following


conclusions:
Based on our work described in this report, noting has come
to our attention that causes us to believe that internal
control is not effective, in all material aspects based on ABC
criteria
A. Limited assurance engagements
B. Reasonable assurance engagements
C. Negative assurance engagements
D. Positive assurance engagements

25. In assertion-based engagements, the evaluation or


measurement of the subject matter against criteria is
performed by the
A. Intended users C. Practitioner
B. Responsible party D. AASC

In assertion-based assurance engagements, the evaluation or


measurement of the subject matter against criteria is
performed by the responsible party and the subject matter in
formation (outcome) is in the form of an assertion by the
responsible party that is made available to the intended
users.
26. The following statements relate to the three parties
involved in an assurance engagement. Which is correct?
A. The responsible party and the intended users should be from
different entities.
B. A practitioner should decline a proposed assurance
engagement when the subject matter requires specialized skills
and knowledge beyond those ordinarily possessed by the
practitioner.
C. A responsible party is the person who is responsible for
the subject matter or the subject matter information
D. The responsible party, not intended users, determines the
nature of the procedures to be performed

According to the Framework, the responsible party is the


person (or persons) who:

In a direct reporting engagement, is the repos bile for


the subject matter
In an assertion-based engagement, is responsible for the
subject matter information (the assertion) and may be
responsible for the subject matter.

27. A proposed assurance engagements can be accepted when the


practitioners preliminary knowledge about the engagement
circumstances indicates that relevant ethical requirements
will be satisfied and:
I. The subject matter of the engagement is appropriate
II. The criteria to be used are suitable and are available to
the intended users
III. The practitioner has access to sufficient appropriate
evidence to support the conclusion.
IV. The conclusion is to be contained in a written report
V. There is a rational purpose for the engagement

A. I, II, III
B. I, II, IV, V
C. I, II, III, IV
D. I, II, III, IV, V
A proposed assurance engagement can be accepted when the
practitioners preliminary knowledge of the engagement
circumstances indicates that relevant ethical requirements
such as independence and professional competence will be met
and the engagement exhibits all of the characteristics
described in statements I to V.

28. A practitioner should accept an assurance engagements only


if
A. The subject matter is in the form of financial information
B. The criteria to be used are not available to the intended
users.
C. The practitioners conclusion is to be contained in a
written report
D. The subject matter is the responsibility of either the
intended users or the practitioner.

29. Which of the following statements is true concerning


evidence in assurance engagement?
A. Sufficiency is the measure of the quantity of evidence
B. Appropriateness is the measure of the quantity of evidence,
that is, its reliability and persuasiveness
C. The reliability of evidence is influenced not by its nature
but by its source
D. Obtaining more evidence may compensate for its poor quality

Sufficiency is the measure of the quantity if evidence. The


quantity of evidence needed is affected by the quality of such
evidence (the higher the quality, the less may be required.)
However, merely obtain more evidence may not compensate for
its poor quality.
Appropriateness is the measure of the quality of evidence,
that is, its relevance and the reliability. The reliability of
evidences influent by its source and by its nature.

30. Assurance engagement risk is the risk


A. That the practitioner expresses an inappropriate conclusion
when the subject matter information is materially misstated
B. Of expressing an inappropriate conclusion when the subject
matter information is not materially misstated
C. Through the loss from litigation, adverse publicity or
other events rising in connection with a subject matter
reported on
D. Of expressing an inappropriate conclusion when the subject
matter of information is either materially misstated or not
materially misstated

31. The following are components of assurance engagement risk,


except
A. Inherent risk C. Detection risk
B. Control risk D. Business risk

Assurance engagement risk has the following components:


1. The risk that the subject matter information is materially
misstated. This consist of:
A. Inherent Risk- the susceptibility of the subject
matter information to a material misstatement, assuming that
there are no related controls.
B. Control Risk- the risk that a misstatement will occur
will not be prevented, or detected and corrected, on a timely
basis by related internal controls.
2. Detection Risk- the risk that the practitioner will not
detect a material misstatement that exists.

32. An unqualified conclusion is not appropriate for either


reasonable or limited assurance engagement when
A. Circumstances prevent the practitioner from obtaining
evidence required to reduce assurance engagement risk to the
appropriate level
B. The responsible party or the engaging party imposes the
restriction that prevents the practitioner from obtaining
evidence required to reduce assurance engagement risk to the
appropriate level
C. Both A and B
D. Neither A nor B
According to the Framework, an unqualified conclusion is not
appropriate for either type of assurance engagement in the
case of a material limitation on the scope of the
practitioners work, whether imposed by the engagement
circumstances or the engaging party or the responsible party.

33. The following statements relate to the performance of an


assurance engagement other than an audit or review of
historical financial information covered by PSAs and PSREs.
Which is incorrect?
A. Those persons who are to perform the engagement should
collectively possess the necessary professional competence
B. The practitioner is precluded from using the work of
persons from other professional disciplines
C. The practitioner should consider materiality and assurance
engagement risk when planning and performing an assurance
engagement
D. The assurance report should be in writing and should
contain a clear expressions of the practitioners conclusion
about the subject matter information

The subject matter and related criteria of some assurance


engagements may include aspects inquiring specialized
knowledge and skills in the accumulation and evaluation of
evidence. The standards allow a practitioner to engage persons
from other professional disciplines, referred to as experts.

34. Reducing assurance engagement risk to zero is very rarely


attainable or cost beneficial as a result of the following
factors except
A. The use of selective testing
B. The fact that much of the evidence available to the
practitioner is persuasive rather that conclusive
C. The practitioner may not have required assurance knowledge
and skills to gather and evaluate evidence
D. The use of judgment in gathering and evaluating evidence
and forming conclusions based on that evidence

35. After an accepting an assurance engagement, a practitioner


is not allowed to change the engagement to a non-assurance
engagement, or from a reasonable assurance engagement to a
limited assurance engagement except when there is reasonable
justification for the change. Which of the following
ordinarily will justify a request for the change in the
engagement?
I. A change in circumstances that affects the intended users
requirements
II. A misunderstanding concerning the nature of the engagement

A. I only C. Both I and II


B. II only D. Neither I nor II

B. AUDITING AND RELATED SERVICES


36. Which of the following standards are to be applied, as
appropriate, in the audit or historical financial information?
A. PSREs c. PSRSs
B. PSAEs D. PSAs

The Philippine Standards of Auditing (PSAs) are to be applied,


as appropriate, in the audit of historical financial
information.

37. Which of the following standards are to be applied to


compilation engagement, engagements to apply agreed-upon
procedures of information, and other related services
engagements as specified by the AASC?
A. PSRSs C. PSAEs
B. PSAs D. PSREs

The Philippine Standards on Related Services (PSRSs) are to be


applied to compilation engagement, engagements to apply
agreed-upon procedures of information, and other related
services engagements as specified by the AASC

38. The Philippine Standards on Review Engagements (PSREs) are


to be applied in
A. The audit of historical financial information
B. Assurance engagements dealing with the subject matters
other that the historical financial information
C. The review of historical financial information
D. The review of both historical and prospective financial
information

39. PSRE 2400 (Engagement to Review Financial Statements), as


amended by the AASC in February 2008, applies to
A. Reviews of any historical financial information of an audit
client
B. Reviews of any historical financial information by a
practitioner other than the entitys auditor.
C. Reviews of historical financial or other information by a
practitioner other than the entitys auditor
D. Reviews of historical financial or other information of an
audit client

PSRE 2400 (Engagement to Review Financial Statements), and


PSRE 2410 (Review on Interim Financial Information Performed
by the Independent Auditor of the Entity) were amended by the
AASC in February 2008. The objective of the amendment made is
to clarify to which engagements each of the standards is to be
applied.
The effect of the amendments is summarized as follows:

PSRE 2400 applies to reviews of historical financial


information by a practitioner other than the entitys
auditor.
PSRE 2410 applies to reviews of historical financial
information by the entitys auditor.
Reviews of other historical information fall under PSAs
3000 (Revised), Assurance Engagement other than Audits
and Reviews of Historical Financial Information.
40. The Philippine Standards on Assurance Engagements (PSAEs)
are to be applied in
A. Assurance engagement dealing with the subject matters other
than historical financial information
B. Compilation engagement and agreements to apply agreed-upon
procedures to information
C. The audit or review of historical financial information
D. Assurance engagement dealing with historical financial
information

41. The Philippine Standards on Quality Control (PSQSs) are to


be applied to
A. Assurance engagements only
B. Review engagements only
C. Compilation and review engagements only
D. All services that fall under the AASCs engagement
standards

PSAs, PSREs, PSAEs, and PSRSs are collectively referred to as


the AASCs Engagement Standards. PSQSs are to be applied for
all services under these Engagement Standards.

42. These statements are issued by the AASC to provide


interpretive guidance and practical assistance to auditors in
the implementations of PSAs and to promote good practice

A. PREPSs C. PAEPs
B. PAPSs D.PRPSPs

The AASC issues Practical Statements to provide interpretative


guidance and practical assistance to practitioners in
implementing the Engagement Standards and to promote good
practice. The following are the AASC engagement standards and
the related Practice Statements.

Engagement Standards Practice Statements


1. Philippine Standards Philippine Auditing Practice

on Auditing (PSAs) Statements (PAPSs)

2. Philippine Standards on Philippine Review Engagement

Review Engagements (PSREs) Practice Statements (PREPSs)


3. Philippine Standards on Philippine Assurance Engagements

Assurance Engagements (PAEPSs) Practice Statements (PAEPSs)

4. Philippine Standards on Philippine Related Services


Related Services (PSRSs) Practice Statements (PRSPSs)

43. The auditors satisfaction as to reliability of an


assertion being made by one party for use by another party is
called
A. Opinion C. Examination
B. Assurance D. Verification

The term assurance means means the practitioners


satisfaction as to the reliability of an assertion being made
by one party for use by another party. The procedures
performed and the evidence collected by the practitioner.

44. What level of assurance is provided by the auditor in an


audit engagement?
A. Absolute C. Moderate
B. High, but not absolute D. No assurance

In an audit engagement, the auditor provides a high, but not


absolute level of assurance that the financial statements are
free of material misstatement. That expressed positively in
the audit report as the reasonable assurance.

45. What level of assurance is provided by the practitioner in


a review engagement?
A. No assurance C. Reasonable
B. High, but not absolute D. Moderate

In a review engagement, the practitioner provides a moderate


level of assurance that the information subject to review is
free of material misstatements. This is expressed in the form
of a negative (also limited) assurance.
46. For the purpose of expressing negative assurance in the
review report, the practitioner should obtain sufficient
appropriate evidence primarily through
A. Inquiry and confirmation
B. Analytical procedures and substantive tests of details of
transactions and account balances
C. Confirmation and test of controls
D. Inquiry and analytical procedures

47. In reviewing a companys financial statements, a


practitioner is required to
A. Send bank confirmations
B. Obtain knowledge of the clients industry
C. Obtain a signed engagement letter from the client
D. Observe clients physical inventory

48. In reviewing engagement, the practitioner performs which


of the following?

Obtain an understanding Test of internal Test of


Of Internal Control controls Transactions

A. Yes Yes No
B. Yes No Yes
C. No Yes Yes
D. No No No

49. A practitioners review of an entitys financial


statements does not provide assurance that he/she will become
aware of all significant matters that would be disclosed in an
audit. However, if the practitioner has become aware that
information coming to his/her attention may be materially
misstated, the practitioner should
A. Carry out additional or more extensive procedures as are
necessary to achieve limited assurance
B. Withdraw immediately from the engagement
C. Perform a complete audit and issue a modified auditors
report
D. Downgrade the engagement to a compilation and issue the
appropriate report

According to PSRE 2400, if the practitioner has reason to


believe that the information subject to review may be
materially misstated, he/she should carry out additional or
more extensive procedures as are necessary to be able to
express negative assurance or to confirm that a modified
report is required.

50. The following statements relate to a review of financial


statements. Which is incorrect?
A. The objective of a review of financial statements is to
enable a practitioner to state whether anything has come to
the practitioners attention that causes the practitioner to
believe that the financial statements are not prepared in
accordance with an identified financial reporting framework
B. A review comprises inquiry and analytical procedures which
are designed to review the reliability of an assertion that is
the responsibility of one party for use by another party
C. A review ordinarily involves an assessment of accounting
and internal control systems
D. The level of assurance provided in review report is less
than that given in an audit report

While a review involves the application of audit skills and


techniques and the gathering of evidence, it does not
ordinarily involve an assessment of accounting and internal
control systems, test of recorded and of responses to
inquiries by obtaining corroborating evidence through
inspection, observation, confirmation and computations which
are procedures ordinarily performed during an audit.

51. The following statement relate to a review of an interim


financial information performed by the entitys independent
auditor. Which is incorrect?
A. Similar to a financial statement audit, a review of interim
financial information is designed to obtain reasonable
assurance that the interim financial information is free from
material misstatement
B. A review of interim financial information does not provide
a basis for expressing an opinion whether the financial
information is presented fairly, in all material respects, in
accordance with an applicable financial reporting framework.
C. In a review of interim financial information. The auditor
should have an understanding of the entity and its
environment, including its internal control
D. A review of interim financial information may bring
significant matters affecting the interim financial
information to the auditors attention, but it does not
provide all of the evidence that would be required in an audit

In contrast to an audit, a review of interim financial


information is to enable the auditor to express a conclusion
whether, on the basis of the review, anything has come to the
auditors attention that causes the auditor to believe that
the interim financial information is not prepared, in all
material aspects, in accordance with an applicable financial
reporting framework.

52. In a compilation engagement, the accountant is engaged to


use accounting expertise to collect, classify, and summarize
financial information. What type of assurance is provided by
the accountant when he/she performs this engagement?
A. Positive assurance
B. Negative assurance
C. No assurance
D. Limited assurance

A compilation engagement ordinarily entails reducing detailed


data to a manageable and understandable form without a
requirement to test the assertions underlying that
information. The procedures employed are not designed and do
not enable the accountant to express any assurance on the
financial information.
However, the accountants involvement provides some benefit to
users of complied financial information because the work has
been performed with due professional skill and care.

53. Which of the following statements concerning compilation


engagement is incorrect?
A. In a compilation engagement, the accountant us engaged to
use accounting expertise as opposed to auditing expertise to
collect, classify and summarize financial information
B. The procedures employed in the compilation engagement
enable the accountant to express a moderate level of assurance
in a compiled financial information
C. Users of the compiled financial information derived some
benefit as a result of the accountants involvement because
the service has been performed with due professional skills
and care.
D. A compilation engagement ordinarily engaged reducing
detailed data to a manageable and understandable form without
a requirement to test the assertions underlying that
information.

The procedures employed in a compilation engagement are not


designed and do not enable the accountant to express any
assurance in the financial information.

54. When performing a compilation engagement, the accountant


is required to
A. Assess internal controls
B. Make inquiries of management to assess the reliability and
completeness of the information provided
C. Verify matters and explanations
D. Obtain a general knowledge of the business and operations
of the entity

According to PSRS 4410 (Engagements to obtain a Compile


Financial Information), The accountant should obtain a
general knowledge of the business and operation of the entity
and should be familiar with the accounting principles and
practices of the industry in which the entity operates and
with the form and content of the financial information that is
appropriate in the circumstances.
The standards further provides that, The accountant
ordinarily obtains knowledge of these smatters through
experience with the entity or the inquiry of the entitys
personnel.
PSRS 4410, par. 13, provides that the accountant is not
ordinarily required to:
a) Make inquiries to the management to assess the
reliability and completeness of the information provided;
b) Assess internal controls;
c) Verify any matters; or
d) Verify any explanations.

55. Each page of the financial information complied by the


accountant should include the following reference, except
A. Unaudited
B. Compiled without Audit or Review
C. Refer to Compilation Report
D. Compiled, Negative Assurance Expressed

According to PSRS 4410 (Engagements to compile Financial


Information), the financial information compiled by the
accountant should contain a reference such as:

Unaudited
Compiled without Audit or Review
Refer to Compilation Report on each page of the financial
information or on the front of the complete set of
financial statements.

56. An accountant who performs a compilation engagement


A. Should read the compiled information and consider whether
it appears to be appropriate in form and free from obvious
material misstatements.
B. Should use his/her auditing expertise in testing the
assertions underlying the compiled financial information
C. Include his/her report a listing of the specific procedures
performed
D. Need not obtain an acknowledgement form the management of
its responsibility of its appropriate presentation of the
financial information

57. What assurance is provided by the author in the agreed-


upon procedures engagement?
A. Reasonable C. Moderate
B. Absolute D. No assurance

In an agreed-upon procedures engagement, the auditor simply


provides a report of the factual findings and expressed no
assurance in his/her report. Users of the report make an
assessment of the procedures and findings reported by the
auditor and draw their own conclusions from the auditors
work.

58. In an engagement to perform agreed- upon procedures, an


auditor is engage to
A. Carry out those procedures of an audit nature to which the
auditor, the entity and any appropriate third party have
agreed and to report factual findings.
B. Use accounting expertise as opposed to auditing expertise
to collect, summarize and classify financial information
C. Provide a moderate level of assurance that the information
is free from any material misstatement
D. Provide a high, but not absolute, level of assurance that
the information is free of material misstatement

In an engagement to perform agreed- upon procedures, an


auditor is engage to carry out those procedures, an auditor is
engaged to carry out those procedures of an audit nature which
the auditor and the entity and any appropriate parties have
agreed and to report factual findings.
The report contains no assurance and is restricted to those
parties that have agreed to the procedures to be performed,
since others, unaware of the reasons for other procedures, may
misinterpret the results. Users of the report must form their
own conclusions from the auditors work.
59. A report may be based upon applying agreed-upon procedures
to specified elements, accounts or items of a financial
statement. The users of the report should participate in
establishing the procedure to be performed. If the auditor
cannot discuss the procedures with all the parties who will
receive the report, he/she may
I. Discuss the procedures to be applied with appropriate
representatives of the parties involved.
II. Review relevant correspondence from the parties involved.
III. Distribute a draft of the type of report that will be
issued to the parties involved.

A. I and II only C. II and III


only
B. I and III only D. I, II and III

PSRS 4400 (Engagements on Agreed- upon Procedures) states, in


certain circumstances for example, when the procedures have
been agreed to between the regulator, industry representatives
and representatives of the accounting profession, the auditor
may not be able to discuss the procedures with all the parties
who will receive the report . In such cases, the auditor may
consider, for example, discussing the procedures to be applied
with appropriate representatives of the parties involved,
reviewing relevant correspondence from such parties or sending
them a draft of the type of report that will be issued.

60. An auditor may accept an engagement to perform specifies


procedures on the specific subject matter of specified
elements, accounts or items of a financial statement if
A. The report does not list the procedures performed
B. The financial statements are prepared in accordance with a
special purpose framework
C. Use of the report is restricted
D. The auditor is also the entitys continuing auditor
PSRS 4400 states that the report is restricted to those
parties that have agreed to the procedures to be performed,
since others, unaware of the reasons for other procedures, may
misinterpret the results.
Answer A is incorrect because the report should include a
listing of the specific procedures performed.
Answer B is incorrect because the financial statements need
not be prepared in accordance with a special purpose
framework.
Answer D is incorrect because the auditor need not be the
entitys continuing auditor.

61. Reports on agreed- upon procedure are intended to be


distributed
A. To only involved parties, who are aware of the reasons for
the procedures
B. Only to the stockholders of the entity
C. To any part to whom the client wishes
D. Only to the entitys management

62. An engagement to perform an agreed-upon procedures may


involve the auditor in performing certain procedures
concerning
I. Individual items of the financial data
II. A single financial statement
III. A complete set of financial statement

A. I and II only C. I and III only


B. II and III only D. I, II and III

63. The report on an agreed0upon procedures engagement should


contain
A. Identification of the purpose for which the agreed-upon
procedures were performed
B. An expression of positive assurance based on the specific
procedures performed
C. A statement that the auditor is independent of the entity
D. A general description of the procedures performed
According to PSRS 4400, the report on an agreed- upon
procedures engagement needs to be described the purpose and
the agreed-upon procedures of the engagement in sufficient
detail to enable the users of the report to understand the
nature and extent of the work performed.
Answer B is incorrect because the report should include a
statement that the procedures performed do not constitute
either an audit or a review and, as such, no assurance is
expressed.
Answer C is incorrect because the report should contain the
statement that the auditor is not independent of the entity if
such is the case.
Answer D is incorrect because the report should include a
listing of the specific procedures performed.

64. Which of the following engagement does not require


compliance with independence requirements?
A. Compilation of financial information
B. Review of financial statements
C. Examination of prospective financial information
D. Audit of financial statement

Independent is not a requirement for compilation and agree-


upon procedures engagement. However where the accountant or
auditor is not independent, a statement to that effect would
be made in the report.

65. Which of the following services, if any, may a


practitioner who is not independent provide?
A. Compilations but not reviews
B. Reviews but not compilations
C. Reviews but not financial statement audits
D. Agreed-upon procedures but not compilations

66. A practitioner is associated with financial information


when
I. The practitioner attaches a report to that financial
information
II. The practitioner consents to the use of his/her name in a
professional connection

A. I only C. Either I or II
B. II only D. Neither I or II
CHAPTER 2
THE ACCOUNTANCY PROFESSION

A. Philippine Accountancy Act of 2004

1. Republic Act 9298 is known as the


A. Revised Accountancy Law
B. Revised Accountancy Act
C. Philippine Accountancy Act of 2004
D. Philippine Accountancy Law of 2004

2. Which of the following is not an objective of the Philippine


Accountancy Act of 2004?
A. The standardization and regulation of accounting
education.
B. The examination for registration of certified public
accountants.
C. The supervision, control, and regulation of the practice
of accountancy in the Philippines.
D. The development and improvement of accounting standards
that will be generally accepted in the Philippines

3. The practice of Accountancy includes

I. Practice of Public Accountancy


II. Practice in Commerce and Industry
III. Practice in Education/Academe

A. I and II only
B. II and III only
C. I and III only
D. I,II, and III

Section 4 of RA 9298 provides that the practice of accountancy


shall include, but not limited to, the following:
a) Practice of Public Accountancy
b) Practice in Commerce and Industry
c) Practice in Education/Academe
d) Practice in the Government

4. A CPA is in public accounting practice when he/she


A. Represents his/her employer before government agencies on
tax and other matters related to accounting.
B. Represents his/her clients before government agencies on
tax and other matters related to accounting.
C. Teaches accounting, auditing, management advisory
services, accounting aspect of finance, business law,
taxation, and other technically related subjects.
D. Holds, or is appointed to, a position in an accounting
professional group in government or in a government-owned
and/or controlled corporation where decision making
requires professional knowledge in the science of
accounting.
5. Section 4 of the Rules and Regulations Implementing RA 9298
(IRR) provides that any position in any business or company
in the private sector which requires supervising the
recording of financial transactions, preparation of financial
statements, coordinating with the external auditors of the
audit of such financial statements, and other related
functions should be occupied by a duly registered CPA. It
provides further that the business or company where such
position exists has a
A. Paid-up capital of at least P5,000,000 and/or an annual
revenue of at least P10,000,000.
B. Paid-up capital of at least P10,000,000 and/or an annual
revenue of at least P5,000,000.
C. Paid-up capital and/or an annual revenue of at least
P10,000,000.
D. Paid-up capital and/or an annual revenue of at least
P5,000,000.

Section 4 of the IRR states that:


a) The business or company where such position exists has a
paid-up capital of at least P5 million and/or annual
revenue of at least P10 million.
b) The provision shall apply only to persons to be employed
after the effectivity of the IRR.
c) The provision shall not result to deprivation of the
employment of incumbents to the position.

6. Which of the following statements concerning the practice of


accountancy in the academe/education is incorrect?
A. Members of the Integrated Bar of the Philippines are
prohibited from teaching business law and taxation
subjects.
B. Members of the Integrated Bar of the Philippines may be
allowed to teach business law and taxation subjects.
C. The position of either the Dean or the Department Chairman
or its equivalent that supervises the Bachelor of Science
in Accountancy program of an educational institutional is
deemed to be in practice of accountancy in the
academe/education.
D. The position of either the Dean or the Department Chairman
or its equivalent that supervises the Bachelor of Science
in Accountancy program of an educational institution must
be occupied only by a duly registered CPA.
7. A CPA is in the practice of accountancy in commerce and
industry when he/she

A B C D
1. Is involved in decision making
requiring professional knowledge
in the science of accounting Yes No No Yes
2. Represents his/her employer
before government agencies on
tax and other matters related to
accounting Yes No Yes No
3. Renders professional services as
a CPA to more than one client on
a fee basis. No Yes No Yes

Section 4 (b) of the IRR states that Practice in Commerce and


Industry- shall constitute in a person,
i. Involved in decision making requiring professional
knowledge in the science of accounting, as well as the
accounting aspects of finance and taxation, or
ii. When he/she represents his/her employer before
government agencies on tax and other matters related to
accounting, or
iii. When such employment or position requires that the
holder thereof must be a certified public accountant.
A CPA is in public accountancy when he/she renders his/her
professional services as a CPA to more than one client on a
fee basis.

8. Section 5 of the Accountancy Act of 2004 states that the Board


of Accountancy shall be composed of a chairman and
A. 2 members
B. 4 members
C. 6 members
D. 8 members

9. The members of the Board of Accountancy shall be appointed by


the
A. Philippine Institute of CPAs (PICPA)
B. Professional Regulation Commission (PRC0
C. President of the Philippines
D. Association of CPAs in Public Practice (ACAPP)

10. If the PICPA fails to submit to the PRC its own nominees
within 60 days prior to the expiry of the term of an incumbent
chairman or member of the Board of Accountancy (BOA), the PRC
in consultation with the BOA shall submit to the president a
list of how may nominees for each vacant position?
A. 2
B. 3
C. 4
D. 5
11. A member of the BOA shall, at the time of his/her
appointment, possess which of the following qualifications?
A. Must be a natural-born citizen of the Philippines.
B. Must be a Filipino citizen.
C. Must be a Filipino citizen and a resident of the
Philippines.
D. Must be a natural-born citizen and a resident of the
Philippines.

According to Section 6 of the Accountancy Act of 2004, am


member of the Board shall, at the time of his/her appointment,
possess the following qualifications:
a) Must be a natural-born citizen and a resident of the
Philippines;
b) Must be a duly registered Certified Public Accountant
with at least ten (10) years of work experience in any
scope of practice of accountancy.
c) Must be of good moral character and must not have been
convicted of crimes involving moral turpitude; and
d) Must not have any pecuniary interest, directly or
indirectly, in any school, college, university, or
institution conferring an academic degree necessary for
admission to the practice of accountancy or where review
classes in preparation for the licensure examination
are being offered or conducted, nor shall he/she be a
member of the faculty or administration thereof at the
time of his/her appointment to the Board.

12. The Chairman and members of the Board of Accountancy shall


hold office for a term of
A. 2 years
B. 3 years
C. 4 years
D. 5 years

13. According to Section 7 of the IRR, no person who has served


two successive complete terms as chairman or member of the
Board of Accountancy shall be eligible for reappointment as
chairman or member until the lapse of
A. 1 year
B. 2 years
C. 3 years
D. 4 years

14. According to Section 7 of the IRR, no person shall serve


in the Board of Accountancy for more than
A. 3 years
B. 5 years
C. 10 years
D. 12 years

15. Which of the following is a function of the Board of


Accountancy?
I. To prescribed and adopt the rules and regulations
necessary for carrying out the provisions of the
Philippine Accountancy Act of 2004.
II. To supervise the regulation, licensure, and practice of
accountancy in the Philippines.
III. To issue, suspend, revoke, or reinstate the certificate
of registration for the practice of the accountancy
profession.

A. I and II only
B. II and III only
C. I and III only
D. I, II, and III

Section 9 of the Accountancy Act of 2004 states that the Board


of Accountancy shall exercise the following specific powers,
functions and responsibilities:

a) To prescribe and adopt rules and regulations necessary


for carrying out the provisions of the Act;
b) To supervise the registration, licensure and
practice of accountancy in the Philippines;
c) To administer oaths in connection with the
administration of the Act;
d) To issue, suspend, revoke, or reinstate the Certificate
of Registration for the practice of the accountancy
profession;
e) To adopt an official seal of the Board;
f) To prescribe and/or adopt a Code of Ethics for the
practice of accountancy;
g) To monitor the conditions affecting the practice of
accountancy and adopt such measures, rules and
regulations and best practices as may be deemed proper
for the enhancement and maintenance of high
professional, ethical, accounting and auditing
standards.
h) To conduct an oversight into the quality of audits of
financial statements through a review of the quality
control measures instituted by auditors in order to
ensure compliance with the accounting and auditing
standards and practices;
i) To investigate violations of the Act and the IRR;
j) To make such investigations as it deems necessary to
determine whether any person has violated any provision
of the Act, any accounting or auditing standards or
rules duly promulgated by the Board as part of the rules
governing the practice of accountancy;
k) To issue a cease or desist order to any person,
association, partnership or corporation engaged in
violation of any provision of the Act, any accounting
or auditing standards or rules duly promulgated by the
Board as part of the rules governing the practice of
accountancy in the Philippines;
l) To punish for contempt of the Board, both direct and
indirect, in accordance with the pertinent provisions
of and penalties prescribed by the Rules of the Court;
m) To prepare, adopt, issue or amend the syllabi of the
subjects for examinations in consultation with the
academe, determine and prepare questions for the
licensure examination which shall directly be within
the scope of the syllabi of the subjects for
examinations as well as administer, correct and release
the results of the licensure examinations;
n) To ensure, in coordination with the Commission on Higher
Education (CHED) or other authorized government offices
that all higher educational instruction and offering of
accountancy comply with the policies, standards and
requirements of the course prescribed by CHED or other
authorized government offices in the areas of
curriculum, faculty, library, and facilities; and
o) To exercise such other powers as may be provided by law
as well as those which may be implied from, which are
necessary or incidental tot the carrying out of, the
express powers granted to the Board to achieve the
objectives and purpose of the Act.

16. The Board of Accountancy shall elect a vice-chairman from


among its members for a term of
A. Two (2) years
B. One (1) year
C. Three (3) years
D. Five (5) years

17. According to Section 9(A) of the IRR, the Commission, upon


the recommendation of the Board, shall create an accounting
standard setting body to be known as the
A. Accounting Standards Council
B. Financial Reporting Standards Council
C. Accounting Standards Board
D. Financial Reporting Standards Board

18. According to Section 9(A) of the IRR, the Commission, upon


the recommendation of the Board, shall create an auditing
standard setting body to be known as the
A. Auditing and Assurance Standards Council (AASC)
B. Auditing Standards and Practices Council (ASPC)
C. Auditing Standards Board
D. Auditing Standards Council
19. This standard setting body shall have a chairman who had
been or presently a senior accounting practitioner in any of
the scope of accounting practice.
A. FRSC
B. AASC
C. PICPA
D. ACPAPP

20. This standard setting body shall have a chairman who had
been or presently a senior accounting practitioner in public
accountancy.
A. AASC
B. FRSC
C. ACPAE
D. BOA

21. The Chairman and the members of the FRSC and AASC shall
have a term of
A. 3 years
B. 5 years
C. 6 years
D. 7 years

22. Which of the following is not represented in the AASC?


A. Bangko Sentral ng Pilipinas
B. Board of Accountancy
C. Bureau of Internal Revenue
D. Securities and Exchange Commission

23. Who has the power to suspend or remove any member of the
Board of Accountancy?
A. The Chairman of the FRSC
B. The Chairman of the PRC
C. The Chairman of the AASC
D. The President of the Philippines

24. An applicant for the CPA licensure examination should be

I. A Filipino citizen
II. Of good moral character
III. A holder of the degree of Bachelor of Science in
Accountancy

A. I and II only
B. I and III only
C. II and III only
D. I, II, and III
Section 14 of the IRR states that any person applying for
examination shall be establish the following requisites to
the satisfaction of the Board that he/she:
a) Is a Filipino citizen;
b) Is of good moral character;
c) Is a holder of the degree of Bachelor of Science in
Accountancy conferred by a school, college, academy or
institute duly recognized and/or accredited by the
CHED or other authorized government offices;
d) Has not been convicted of any criminal offense
involving moral turpitude.

25. The following documents shall be submitted by applicants


for the CPA licensure examination, except
A. Certificate of Live Birth in National Statistics Office
(NSO) security paper.
B. Marriage contract in NSO security paper for married male
applicants.
C. NBI clearance.
D. Transcript of records with indication therein of date of
graduation and Special Order number unless it is not
required.

Section 14 of the IRR requires applicants to submit the


following:
a) Birth certificate in NSO security paper;
b) Marriage contract in NSO security paper for married
female applicants;
c) College diploma;
d) Transcript of records;
e) NBI clearance;
f) Other documents that the Board may require.

26. Section 16 of the IRR states that to be qualified as having


passed the licensure examination for accountants, a candidate
must obtain a
A. General average of seventy-five percent (75%), with no
grades lower than sixty percent (60%) in any given subject.
B. General average of seventy-five percent (75%), with no
grades lower than sixty-five percent (65%) in any given
subject.
C. General average of seventy-five percent (70%), with no
grades lower than sixty percent (60%) in any given subject.
D. General average of seventy-five percent (75%), with no
grades lower than sixty percent (60%) in any given subject.
27. A candidate who obtains the rating of seventy-five percent
(75%) and above in at least a majority of the subjects shall
receive a conditional credit for the subjects passed. He/she
shall take an examination in the remaining subjects within
how many years from the preceding examination?
A. 1
B. 2
C. 3
D. 5

28. Any candidate who fails in two (2) complete CPA board
examinations shall be disqualified from taking another set of
examinations unless he/she has completed at least how many
units of subjects given in the licensure examination?
A. 4
B. 8
C. 16
D. 24

29. The Board of Accountancy shall submit to the PRC the


ratings obtained by each candidate within how many calendar
days after the examination?
A. 1
B. 2
C. 5
D. 10

30. The Certificate of Registration issued to successful


examinees
A. Is renewable every three years.
B. Is renewable every five years.
C. Shall remain in full force and effect until withdrawn,
suspended or revoked in accordance with RA9298.
D. Shall bear the signature of the PRC Chairperson and the
two PRC Commissioners.

According to Section 20 of the IRR, the Certificate of


Registration shall:
a) Bear the signature of the Chairperson of the PRC and
the Chairman and Members of the Board of Accountancy,
stamped with the official seal of the PRC and of the
Board of Accountancy, indicating that the person named
therein is entitled to the practice of the profession
with all the privileges appurtenant thereto;
b) Remain in full force and effect until withdrawn,
suspended or revoked in accordance with RA 9298.
31. The Professional Identification Card issued to successful
examinees
A. Is renewable every three years.
B. Is renewable every five years.
C. Shall remain in full force and effect until withdrawn,
suspended or revoked in accordance with RA 9298.
D. Shall bear the signature of the PRC Chairperson and the
Chairman and Members of the Board of Accountancy

Section 20 of the IRR provides that a Professional


Identification Card bearing the registration number, date of
issuance, expiry date, duly signed by the Chairperson of the
PRC, shall be issued to every registrant. It provides further
that the reissuance or renewal of the said card shall be
subject to payment of the annual registration fees for another
and every after three (3) years thereafter.

32. The Board of Accountancy may reinstate the validity of a


revoked Certificate of Registration after the expiration of
how many years from the date of revocation?
A. 1
B. 2
C. 3
D. 5

Section 25 of the IRR provides that the Board of Accountancy


may, after the expiration of two (2) years from the date of
revocation of a Certificate of Registration and upon
application and for reasons deemed proper and sufficient, and
after being convinced of applicants remorse and
rehabilitation, reinstate the validity of a revoked
Certificate of Registration and in so doing, may, in its
discretion, exempt the applicant from taking another
examination.

33. The following statements relate to the practice of public


accountancy. Which is incorrect?
A. Single practitioners and partners of partnerships
organized for the practice of public accountancy shall be
registered CPAs in the Philippines.
B. A partnership engaged in the practice of public accountancy
may be carried on in the form of a general partnership
(GP) but not a limited liability partnership (LLP).
C. A CPA is in public accounting practice when he/she
represents his/her clients before government agencies on
tax and other matters related to accounting.
D. The Securities and Exchange Commission shall not register
any corporation organized for the practice of public
accountancy.
Section 28 of the IRR provides that partnership engaged in in
the practice of public accountancy may be carried on in the
form of a general partnership (GP) or a limited liability
partnership (LLP).

34. Which of the following statements concerning ownership of


working papers is incorrect?
A. All working papers made by a CPA and his/her staff in the
course of an examination remain the property of such CPA
in the absence of any agreement (written or oral) between
the CPA and the client to the contrary.
B. Working papers include schedules and memoranda prepared
and submitted by the client of the CPA.
C. All working papers, except reports submitted by a CPA to
his/her client shall be treated confidential and
privileged.
D. Working papers shall be treated confidential and
privileged and remain the property of the CPA unless such
documents are required to be produced through subpoena
issued by any court, tribunal, or government regulatory or
administrative body.

35. All registered CPAs shall obtain and use a seal which shall
be circular in form with a smaller circle within bearing the
registrants name, registration number and title. Which of
the following is correct?
A. Engraved in the lower portion of the space between the
circles is the CPAs name.
B. Engraved in the upper portion of the space between the
circles is the CPA registration number.
C. Engraved in the middle of the smaller circle are the
letters CPA.
D. Engraved in the middle of the smaller circle are the CPAs
name and registration number.
E. Engraved in the middle of the smaller circle is the CPAs
name

Section 33 of the IRR states that the seal of a CPA shall be


circular in form with a smaller circle within. In the upper
portion of the space between the circles shall be engraved
the name of the individual CPA, firm or partnership as the
case maybe, the lower portion thereof shall be engraved the
CPA registration number of the individual CPA, proprietor of
the firm and the signing partner of the partnership, and in
the middle of the smaller circle shall be engraved the letters
CPA.

The auditors reports shall be stamped with said seal,


indicating therein his/her current Professional Tax Receipt
(PTR) number, date/place of payment when filed with
government authorities or when used professionally.

36. Section 36 (Penal Provision) of RA 9298 states that any


person who shall violate any of the provisions of the Act or
any of its implementing rules and regulations shall, upon
conviction, be punished by
A. A fine of not less than P50,000
B. Imprisonment for a period not exceeding two (2) years.
C. A fine of not less than P50,000 or by imprisonment for a
period not exceeding two (2) years.
D. A fine of not less than P50,000 or by imprisonment for a
period not exceeding two (2) years, or both.

37. Which of the following is the accredited national


professional organization of CPAs (APO)?
A. PICPA
B. ASC
C. AASB
D. FRSC

38. Tanya, Sam, and Jervi, CPAs, a newly formed partnership,


applied for registration with the Commission and the Board
which was approved on June 30, 2012. The partnership should
apply for renewal of its registration on or before
A. September 30,2014
B. September 30,2015
C. December 30,2016
D. June 30,2015

39. Which of the following statements concerning the use of


firm or partnership name is incorrect?
A. In the case of an individual CPA, he/she shall do business
under his/her registered name with the BOA and the PRC and
as printed in his/her CPA certificate (for example, Juan
Puruntong, CPA).
B. In the case of a firm, it shall do business under its duly
registered and authorized firm name appearing in the
registration documents issued by the Department of Trade
and Industry (DTI) and other government offices and such
firm name shall include the real name of the sole
proprietor as printed in his/her CPA certificate (for
example, Amulfo Gumamela and Associates).
C. In the case of registered partnership, it shall do business
under its name as indicated in its current Articles of
Partnership and Certificate of Registration issued by the
Securities and Exchange Commission (SEC) (for example,
Tanya, Sam, and Jervi, CPAs).
D. A CPA shall practice only under an individual, firm, or
partnership name in accordance with Philippine law and
shall not include any fictitious name but may indicate
specialization.
40. A partner surviving the death or withdrawal of all the
other partners in a partnership may continue to practice under
the partnership name for a period of not more than ___ years
after becoming a sole proprietor.
A. 1
B. 2
C. 3
D. 4

41. The death or disability of an individual CPA and/or the


dissolution and liquidation of a firm or partnership of CPAs
shall be reported to the BOA not late than ____ days from the
date of such death, dissolution or liquidation.
A. 15
B. 30
C. 60
D. 90

42. As defined in Annex C of the IRR, this refers to the


inculcation, assimilation and acquisition of knowledge,
skills, proficiency and ethical and moral values, after the
initial registration of a professional that raise and enhance
the professionals technical skills and competence.
A. Professional Development
B. Continuing Professional Education
C. Continuing Professional Development
D. Professional Growth and Development

43. The PRC CPE Council shall assist the BOA in implementing
its CPE program. Which of the following statements is
incorrect concerning the Councils compostion?
A. The PRC CPE Council shall be composed of a chairperson and
two (2) members.
B. The Chairperson shall be chosen from among the members of
the BOA by the PRC.
C. The first member shall be the President or, in his/her
absence or incapacity, any officer chosen by the Board of
Directors of PICPA.
D. The second member shall be the President or, in his/her
absence or incapacity, any officer of the organization of
Deans or Department Heads of schools, colleges, or
universities offering the degree requiring licensure
examination.

44. The following statements relate to CPE credit units. Which


is incorrect?
A. The total CPE credit units for registered accounting
professionals shall be sixty (60) credit units for three
(3) years, provided that a minimum of fifteen (15) credit
units shall be earned in each year.
B. Any excess credit units in one year may be carried over to
the succeeding years within the three-year period.
C. Excess credit units earned may be carried over to the next
three-year period except credit units earned for doctoral
and masters degrees.
D. One credit hour of CPE program, activity or source shall
be equivalent to one (1) credit unit.

45. The following statements relate to the exemption from CPE


requirements. Which is false?
A. A registered professional shall be permanently exempted
from CPE requirements upon reaching the age of 60 years
old.
B. A registered professional applying for permanent CPE
exemption is required to submit an authentic or
authenticated copy of his/her birth certificate or, if not
available, his/her voters ID or drivers license.
C. A registered professional who is working or practicing
his/her profession abroad shall be temporarily exempted
from compliance with CPE requirements during the period of
his/her stay abroad provided that he/she has been out of
the country for at least two years immediately prior to
the date of renewal.
D. A registered professional who is furthering his/her
studies abroad shall be temporarily exempted from
compliance with CPE requirements during the period of
his/her stay abroad provided that he/she has been out of
the country for at least two years immediately prior to
the date of renewal.

A registered professional shall be permanently exempted from


CPE requirements upon the reaching the age of 65 years old.

46. Which of the following statements concerning a CPAs


disclosure of confidential client information is ordinarily
correct?
A. Disclosure may be made to any party on consent of the
client.
B. Disclosure should not be made even if such disclosure will
protect the CPAs professional interests in legal
proceedings.
C. Disclosure should be made only if there is a legal or
professional duty to make disclosure.
D. Disclosure may be made to any government agency without
subpoena.

47. Listed below are names of four CPA firms and pertinent
facts relative to each firm. Unless otherwise indicated, the
individuals named are CPAs and partners, and there are no
other partners. Which is a violation of the Implementing Rules
and Regulations of RA 9298?
A. Tin, Ton and Tan, CPAs (Tin died about five years ago; Ton
and Tan are continuing the firm.)
B. Poe and Que, CPAs (The name of Cua, a third partner, is
omitted from the partnership name.)
C. Joni and Jona, CPAs (Joni died about three years ago; Jona
is continuing the firm as a sole proprietor.)
D. Elias and Co., CPAs (The firm has ten other partners who
are all CPAs).

Quality Assurance Review (QAR)


(Revised Rules and Regulations for the Conduct by the
Professional Regulatory Board of Accountancy of Oversight into
the Quality of Audits of Financial Statements)

BOA Resolution No. 23, Series 2010

48. Which of the following has the power to conduct an


oversight into the quality of audits of financial statements
through the review of the quality control measures instituted
by auditors?
A. Bureau of Internal Revenue
B. Securities and Exchange Commission
C. Board of Accountancy
D. Insurance Commission

Under RA 9298, the Board of Accountancy (BOA) has the power


to conduct an oversight into the quality of audits of
financial statements through a review of the quality control
measures instituted by auditors in order to ensure compliance
with the accounting and auditing standards and practices.

49. In the exercise of its power to conduct an oversight into


the quality of audits, the BOA organized the _______
Department of the Philippine Institute of Certified Public
Accountants (PICPA) to conduct an independent study,
appraisal, or review of the quality of audit of financial
statements.
A. Quality Assurance Review
B. Quality Control Review
C. Independent Quality Assurance Review
D. Independent Quality Control Review

Section 2 of the Revised Rules and Regulations for the Conduct


by the Professional Regulatory Board of Accountancy of
Oversight into the Quality of Audits of Financial Statements
defines Quality Assurance Review as a study, appraisal, or
review by an independent Quality Assurance Review Department
organized by the Board, of the quality of audit of financial
statements through a review of the quality control measures
instituted by member-CPA Practitioners engaged in the
practice of public accountancy to ascertain compliance with
prescribed professional, ethical and technical standards of
public practice.
50. The operation of the Quality Assurance Review Department
(QARD) of the PICPA shall be supervised by a/an
A. Executive Committee
B. Supervisory Board
C. Oversight Committee
D. Accountability Board

Section 5 provides, The Executive Committee shall have the


full power and authority to set policies and to supervise the
operation of QARD.

51. Which of the following is not a responsibility of the


Executive Committee?
A. To set policies that will ensure effective implementation
of the quality assurance review program.
B. To monitor the quality of audits through its QARD.
C. To recommend to the BOA a rolling Quality Assurance Review
(QAR) plan for 3 years.
D. To select the CPA practitioners to be reviews for the year,
in accordance with the three-year plan.

Listed under Section 5 are the following duties and


responsibilities of the Executive Committee:
a) To set policies that will ensure effective
implementation of the quality assurance review program;
b) To maintain independence of the QARD;
c) Through its QARD, to monitor the quality of audits;
d) To hire the Chief Inspector and Chief of Administration
of QARD including other personnel thereof;
e) To recommend to the Board a rolling Quality Assurance
Review (QAR) plan for three (3) years, which may be
approved by the Board without referral to the
Commission.
f) To implement the approved QAR plan through the QARD;
g) To obtain independent technical advice on the subject
of quality assurance when needed and appropriate;
h) To receive and evaluate the reports and recommendations
of the Chief Inspector;
i) Submit regularly to the Board a list of erring member-
CPA practitioners including the findings and actions
taken by the QARD. The Committee shall likewise
regularly submit to the Board policies that it adopted
and implementing issues that it settled for the
oversight review of the Board. It shall also report to
the Board any form of interference by the APO on its
policy making or on QARDs operation;
j) To issue through the Board, annual reports for the
benefit of the general public.

The selection of the CPA-Practitioners to be reviewed for


the year is a responsibility of the Chief Inspector of the
QARD.
52. Which of the following is an incorrect statement concerning
the term of the office of the Executive Committee members?
A. The term of office of the Chairman and members shall be
co-terminus with their respective term in the BOA or PICPA
unless replaced earlier by the Board or PICPA.
B. The term of office of the Chairman and members in no case
shall be more than 3 years.
C. The Chairman and members may continue to be in active
practice of public accountancy during their term, subject
to certain restriction imposed by the Board.
D. The Chairman of the Executive Committee shall be from the
Boards appointees.

Section 6 provides that the Chairman and members of the


Executive Committee should not be in active practice of public
accountancy during his term in the Executive Committee.

53. The Head of the QARD who will be appointed by the Executive
Committee is the
A. Chief Inspector
B. Chief of Administration
C. Chief Auditor
D. Chief Quality Assurance Reviewer

Under Section 7, the QARDs personnel shall be composed of:


1) Head (the Chief Inspector)
2) Assistants to the Chief Inspector
3) Chief of Administration
4) Assistants to the Chief of Administration
5) Staff auditors
6) Such other employees that may be necessary to carry out
effectively the functions of the QARD.

54. Which of the following is an incorrect qualification of


the Chief Inspector of the QARD?
A. Must have at least 5 years of experience in the practice
of public accountancy.
B. Must be of good character.
C. Must not have been found guilty of violating any
professional, ethical and regulatory auditing standards.
D. Must have reached the level of senior manager or partner
of an accounting firm that has among its clients, publicly
listed entities.

Section 9 provides that the Chief Inspector of the QARD should


have at least 10 years of experience in the practice of public
accountancy.
55. Who is responsible for the non-technical aspect of the QAR
program?
A. Chief Inspector
B. Chief of Administration
C. Assistant to the Chief Inspector
D. Chairman of the Executive Committee

56. The QAR program covers all CPAs in


A. Public Accounting
B. Government
C. Commerce and Industry
D. Education/Academe

Section 15 provides that the QAR program covers all CPAs in


public accounting, whether as an individual practitioner, a
firm or a partnership.

57. Which of the following statements concerning enrollment in


QAR program is/are correct?
I. Enrollment in the QAR program under the prescribed
category is a pre-requisite for accreditation or renewal
of accreditation as a CPA in public practice by the
Board of Accountancy.
II. The CPA practitioners registration category should
be stated in the BOA certificate of accreditation.

A. I only
B. II only
C. Both I and II
D. Neither I nor II

58. CPAs in public practice are required to register with the


QAR program under the prescribed category. Those whose
clients are public-interest entities (listed and not-listed
but with public accountability) should register under
A. Category A
B. Category B
C. Category C
D. Category D

Section 19 provides that CPA practitioners should be


registered in accordance with the following categories

Category A registration for CPA practitioners handling


clients that use the full IFRS or their Philippine
equivalents. These would cover CPA practitioners
auditing public-interest entities (listed and not-
listed but with public accountability).

Category B registration for CPA practitioners handling


clients that use the IFRS or their Philippine
equivalents for Small and Medium-sized enterprises.
59. Public-interest entities include publicly-listed entities
and not listed entities but with public accountability. The
following are examples of not-listed public-interest
entities, except
A. Entities which have sold a class of their securities
pursuant to a registration under Section12 of the
Securities Regulation Code.
B. Entities with assets of at least P50 million and having
200 or more holders each holding at least 100 shares of a
class of its equity securities as of the first day of the
issuers fiscal year.
C. Entities which are in the process of filing their financial
statements for the purpose of issuing any class of
instruments in a public market.
D. Entities not in the process of filing their financial
statements for the purpose of issuing any class of
instruments in a public market.

Under Section 19, audit clients are classified as:

I. Public-interest Entities (Entities with Public


Accountability) Using Full IFRS or their Philippine
Equivalents

1. Publicly-listed entities
a. Entities which have issued a class of securities
listed for trading on an Exchange

2. Other Public-interest Entities but not Listed


a. Entities which have a sold a class of their
securities pursuant to a registration under Section
12 of the Securities Regulation Code.
b. Entities with assets of at least P50 million and
having 200 or more holders each holding at least 100
shares of class of its equity securities as of the
first day of the issuers fiscal year.
c. Entities which are in the process of filing their
financial statements for the purpose of issuing any
class of instruments in a public market.
d. Entities that hold assets in a fiduciary capacity
for a board group of outsiders such as a bank (all
types of banks), an investment house, a finance
company, an insurance company, a securities
broker/dealer, a mutual fund and pre-need company or
entities with secondary license.
e. Public utility entities.
f. Entities which are economically significant. These
are entities whose total assets exceed P350 million
or whose total liabilities exceed P250 million. Total
assets and total liabilities are based on the
entitys annual financial statements and on
consolidated totals, if the entity presents
consolidated financial statements. An entity that is
a subsidiary of a parent that is considered to have
public accountability is similarly considered to
have public accountability.

II. Entities Using the IFRS or their Philippine


Equivalent for SMEs
a. Entities with total assets of between P3 million and
P350 million or total liabilities of between P3
million and P250 million.
b. Entities not required to file financial statements
under Part II or Rule 68;
c. Entities not in the process of filing their financial
statements for the purpose of issuing any class of
instruments in a public market;
d. Entities which are not holders of secondary license
issued by a regulatory agency, such as a bank (all
types of banks), an investment house, a finance
company, an insurance company, a securities
broker/dealer, a mutual fund and a pre-need company;
and
e. Entities which are not public utility companies.

60. There are rules to observe in connection with the


registration of CPA practitioners with the QAR program. Which
is incorrect?
A. A CPA practitioner will register in only one category
division.
B. Change in registration category is not allowed.
C. Change in registration sub-category due to the change in
the number of clients should be done upon renewal of
registration within the first two months of each calendar
year.
D. All applications for registration must include information
as to the number of clients classified as to public-
interest entities using full IFRS and small and medium-
sized entities using IFRS for SMEs.

According to the registration riles in Section 19, the change


in registration from Category A to Category B or vice versa
should be done within the year of change.
RULES AND REGULATIONS FOR THE ACCREDITATION OF
ACCOUNTING TEACHERS

BOA Resolution No.88, Series of 2008

61. What is the effectivity date of the BOA Resolution No.88


(Series of 2008) prescribing the rules and regulations for
the accreditation of accounting teachers?
A. June 4, 2008
B. June 20, 2008
C. December 31, 2008
D. June 30,2008

62. Which of the following requirements for the accreditation


of accounting teachers shall not apply to those CPAs already
engaged in teaching as of the effectivity date of the Rules
and Regulations?

I. Possession of relevant Masters degree.


II. Completion of 12 units of relevant education subjects
from CHED recognized schools.
III. A total of 3 years meaningful experience in actual
accounting work either in Public Practice, Commerce &
Industry or Government sector.
IV. Proof that the CPA has undergone Continuing Professional
Education (CPE).

A. I and II only
B. II and III only
C. I and IV only
D. II only

63. The following statements relate to the requirement to


complete 12 units of relevant education subjects for the
accreditation of accounting teachers. Which is incorrect?
A. For purposes of compliance, the 12 units may be earned
from the undergraduate education program or from a graduate
degree program of any Higher Education Institution (HEI)
duly recognized by CHED.
B. The 12 units may be earned from in-service or in-house
training on relevant education subjects offered by schools
or training centers.
C. The 12 units may be a combination of in-service trainings
and units earned in an undergraduate or graduate education
programs.
D. CPAs who have passed the Teachers Board Exams and are
licensed Professional Teachers should earn the 12 units
through in-service or in-house trainings.
64. Which of the following shall be considered proof of
compliance with the CPE requirement for accreditation of
accounting teachers?

I. Certification of CPE units from accredited CPE providers


issued by the PICPA.
II. Certificate of Attendance or other proofs of meaningful
participation in other CPE programs as approved by the
PRC/CPE Council upon recommendation of the PICPA of the
individual CPA of a minimum of sixty (60) credits units
earned for the past immediate 3 years.

A. I only
B. II only
C. Either I or II
D. Neither I nor II

65. What is the required minimum number of CPE credit units


that an accounting teacher shall earn in each year prior to
renewal of accreditation?
A. 15 units
B. 20 units
C. 10 units
D. No minimum number of credit units required

66. The Certificate of Accreditation issued by the PRC to an


accounting teacher shall
A. Be valid for 2 years and renewable every 2 years.
B. Be valid initially for 3 years renewable annually.
C. Remain in full force and effect unless revoked, cancelled
or withdrawn.
D. Be valid for 3 years and renewable every 3 years.

67. According to the Transitory Provisions of the BOA


Resolution No.88, any tenured/full time/full load faculty
member who does not meet the accreditation requirements as of
the effectivity date of the Rules and Regulations may be
issued a provisional Accreditation. Choose the correct
statement.
I. The Provisional Accreditation will be valid for a period
not exceeding 3 years unless earlier withdrawn, revoked,
or cancelled for cause by the BOA.
II. The Provisional Accreditation may be issued only once
and is not renewable.
III. The transitory provision shall also apply to returning
teachers who have not been teaching for the last 5
years.

A. I only
B. I and II only
C. II and III only
D. I, II, and III
B. DIFFERENT GOVERNMENT AGENCIES THAT INFLUENCE THE
PRACTICE OF ACCOUNTANCY

SECURITIES AND EXCHANGE COMMISSION (SEC)

68. Which of the following is responsible for the registration


of corporations and partnerships, as well as monitoring of
compliance with the Corporation Code, Civil Code provisions
on partnerships, Foreign Investments Act, and other related
laws?
A. Bangko Sentral ng Pilipinas (BSP)
B. Securities and Exchange Commission (SEC)
C. Bureau of Internal Revenue (BIR)
D. Philippine Stock Exchange (PSE)

69. Which of the following laws/regulations govern the


registration of corporations and partnerships with the SEC?
A. Civil Code
B. Corporation Code of the Philippines
C. Securities Regulation Code
D. A,B, and C

70. Which of the following entities need not be registered


with the SEC?
A. Sole proprietorships
B. General and limited partnerships
C. Stock corporations
D. Non-stock corporations

71. The SEC is composed of a chairperson and four (4)


commissioners appointed by the President of the Philippines
for a term of
A. 3 years
B. 6 years
C. 7 years
D. 8 years

72. The following statements relate to the qualifications of


the SEC commissioners. Which is incorrect?
A. The commissioners must be natural-born citizen of the
Philippines.
B. The commissioners must be of good moral character, of
unquestionable integrity, of known probity and patriotism,
and with recognized competence in social and economic
disciplines.
C. The majority of the commissioners, including the
Chairperson, shall be the members of the Philippine Bar.
D. The Chairperson and the commissioners must be at least
thirty-five (35) years of age.

The Chairperson must be at least forty (40) years of age and


the commissioners must be at least thirty-five (35) years of
age. (Subsection 4.2, Securities Regulation Code)
73. Subsection 8.1 of the Securities Regulation Code (SRC)
provides that securities shall not be sold or offered for
sale or distribution within Philippines without a
A. Registration Statement
B. Register of Securities
C. Certificate of Accreditation
D. Certificate of Registration

Securities shall not be sold or offered for sale or


distribution within the Philippines without a Registration
Statement duly filed with and approved by the Securities and
Exchange Commission.

74. The following securities are exempted from the


registration requirement under Subsection 8.1 of the
Securities Regulation Code (SRC), except
A. A banks own shares of stock.
B. Any security issued or guaranteed by the Government of the
Philippines.
C. Any security issued by the Government of any country with
which the Philippines maintains diplomatic relations on
the basis of reciprocity.
D. Any security or its derivatives, the sale or transfer of
which by law, is under supervision and regulation of the
Office of the Insurance Commission, Housing and Land Use
Regulatory Board, or the Bureau of Internal Revenue.

Under Subsection 9.1 of the SRC, exempted from the


registration requirement is any security issued by a bank
except its own shares of stock.

75. The following transactions are exempted from the


registration requirement under Subsection 8.1 of the
Securities Regulation Code (SRC), except
A. The distribution by a corporation, actively engaged in the
business authorized by its articles of incorporation, of
securities to its stockholders as a stock dividend.
B. The sale of securities by an issuer to more than twenty
(20) persons in the Philippines during any 12-month period.
C. The issuance of bonds and notes secured by mortgage upon
real estate or tangible personal property, where the entire
mortgage together with the bonds or notes secured thereby
are sold to a single purchaser at a single sale.
D. Brokers transactions, executed upon customers orders, on
any registered Exchange or other trading market.

Under Subsection 10.1 of the SRC, exempted from the


registration requirement I the sale of securities by an issuer
to fewer than twenty (20) persons in the Philippines during
any 12-month period.
76. Rule 68 of the Securities Regulation Code (SRC) prescribes
the requirements applicable to the form and content of
financial statements to be filed by stock corporations except
those whose paid-up capital is less than
A. P500,000
B. P 50,000
C. P250,000
D. P400,000

SRC Rule 68, as amended, prescribes the form and content of


financial statements required to be filed with the Securities
and Exchange Commission (SEC) by corporations which meet the
following thresholds:
a) Stock corporations with paid-up capital stock of P50,000
or more;
b) Non-stock corporations with total assets of P500,000 or
more, or with gross annual receipts of P100,000 or more;
c) Branch offices of stock foreign corporations with
assigned capital in the equivalent amount of P1,000,000
or more;
d) Branch offices of non-stock corporations with total
assets in the equivalent amount of P1,000,000 or more;
e) Regional operating headquarters of foreign corporations
with total revenues in the equivalent amount of
P1,000,000 or more.

77. The audited financial statements to be filed with the SEC


shall be accompanied by a
A. Management Report
B. Registration Statement
C. Statement of Managements Responsibility for Financial
Statements
D. Statement of the Board of Directors Responsibility for
Financial Statements

The SEC requires management of all operations covered by SRC


Rule 68, as amended, to acknowledge their responsibility for
their financial statements. Hence, the financial statements
to be filed with the SEC shall be accompanied by a Statement
of Managements Responsibility (SMR) for Financial
Statements.
78. The Statement of Managements Responsibility to accompany
the financial statements to be filed with the SEC shall be
signed by the

I. Chairman of the Board of Directors


II. Chief Executive Officer
III. Chief Financial officer

A. II and III only


B. I only
C. I and II only
D. I, II, and III

SRC Rule 68, as amended, prescribe the wording of the wording


of the SMR that shall be attached to the financial statements.

Part I, Section B (ii) of the Rule states that the SMR for
Financial Statements that shall be attached to the financial
statements shall read as follows:

STATEMENT OF MANAGEMENTS RESPONSIBILITY


FOR FINANCIAL STATEMENTS

The management of (name of reporting company) is responsible


for the preparation and fair presentation of the financial
statements for the year(s) ended (date), in accordance with
the prescribed financial reporting framework indicated
therein. This responsibility includes designing and
implementing internal controls relevant to the preparation
and fair presentation of financial statements that are free
from material misstatement, whether due to fraud or error,
selecting and applying appropriate accounting policies, and
making accounting estimates that are reasonable in the
circumstances.

The Board of Directors or Trustees reviews and approves the


financial statements and submits the same to the stockholders
or members.

(name of auditing firm), the independent auditors, appointed


by the stockholders has examined the financial statements of
the company in accordance with Philippine Standards on
Auditing, and in its report to the stockholders or members,
has expressed its opinion on the fairness of presentation
upon completion of such examination.
Signature_______________
Printed Name of the Chairman of the
Board________________

Signature_______________
Printed Name of the Chief Executive
Officer______________

Signature_______________
Printed Name of the Chief Financial
Officer______________

The SMR of the entities covered under Part II of the SRC


Rule 68, as amended, shall be signed under oath.

79. In addition to the requirements prescribed under Part I of


the Rule 68, as amended, Part II of the Rule prescribes
special requirements on the financial statements required to
be filed with the SEC by
I. An issuer which has sold a class of its securities
pursuant to a registration under Section 12 of the SRC.
II. An issuer with a class of securities listed for trading
on an Exchange.
III. An issuer with assets of at least P50,000,000 and has
200 or more holders, each holding at least 100 shares
of a class of its equity securities as of the first day
of the issuers fiscal year.

A. I only
B. II only
C. II and III only
D. I,II and III

80. The financial statements to be filed with the SEC shall be


presented in comparative form. The audited balance sheets or
statements of financial position shall be as of the end of
each of the ___ most recently completed fiscal years.
A. 2
B. 3
C. 4
D. 5

81. SRC Rule 68, as amended, requires a regulated entity to


report to the SEC its action on a report of its independent
auditor pertaining to any material findings involving fraud
or error within _____ business days from the date the report
is submitted by the independent auditor.
A. 2
B. 3
C. 4
D. 5
A regulated entity shall report to the SEC its action on a
report of its independent auditor pertaining to the
following:
1) Any material findings involving fraud or error;
2) Losses or potential losses the aggregate of which
amounts to at least 10% of the consolidated total assets
of the company;
3) Any finding to the effect that the consolidated total
assets of the company, on a going concern basis, are no
longer adequate to cover the total claims of creditors;
4) Material internal control weaknesses which may lead to
financial reporting problems.

Rule 68, as amended, prescribed the form of report on the


audit findings to be submitted by the regulated entity to
the SEC within 5 business days from the date the report is
submitted by the independent auditor.

82. If a regulated entity fails to submit the required report


if its action concerning any of the audit findings specified
in Rule 68, as amended, the independent auditor shall, within
____ business days from the submission of his findings to the
entity, file a report to the SEC.
A. 10
B. 15
C. 20
D. 30

83. According to Part II of Rule 68, as amended, the Rule is


considered violated if the auditors opinion on the financial
statements of listed entities or other issuers of securities
to the public is other than unqualified because of
I. Deviation(s) from the required financial reporting
framework.
II. A scope limitation imposed by the company.

A. I only
B. II only
C. Either I or II
D. Neither I nor II

BANGKO SENTRAL NG PILIPINAS (BSP)

84. The primary objective of this government agency is to


maintain price stability conductive to a balanced and
sustainable economic growth. It also aims to promote and
preserve monetary stability and the convertibility of the
peso.
A. Bureau of Internal Revenue (BIR)
B. Securities and Exchange Commission (SEC)
C. Philippine Deposit and Insurance Corporation (PDIC)
D. Bangko Sentral ng Pilipinas (BSP)
85. Which of the following is not a function of the Bangko
Sentral ng Pilipinas (BSP)
A. Recommend measures to improve the efficiency and
effectiveness of government operations.
B. Supervise banks and exercise regulatory powers over non-
bank institutions performing quasi-bank functions.
C. Determine the exchange rate policy of the Philippines.
D. Extend discounts, loans, and receivables to banking
institutions for liquidity purposes.

To recommend measures to improve the efficiency and


effectiveness of government operations is one of the
principal duties of the Commission on Audit (COA).

86. The powers and functions of the BSP shall be exercised by


the
A. Board of Directors
B. Monetary Board
C. Board of Trustees
D. BSP Governor

87. The BSP Monetary Board is composed of seven (7) members


appointed by the President of the Philippines for a term of
A. 6 years
B. 5 years
C. 7 years
D. 3 years

Section 6 of the New Central Bank Act (RA 7653) states that
the powers and functions of the BSP shall be exercised by the
BSP Monetary Board composed of seven (7) members appointed by
the President of the Philippines for a term of six (6) years.

88. The seven (7) members of the Monetary Board are


A. The BSP governor, a member of the cabinet to be designated
by the President of the Philippines, and five (5) members
from the private sector.
B. The BSP governor and six (6) members from the private
sector.
C. A cabinet member and six (6) members from the private
sector.
D. The BSP governor, 2 members of the cabinet to be designated
by the President of the Philippines, and four (4) members
from the private sector.
89. The internal auditor of a universal or commercial bank

I. Should be a CPA.
II. Must have at least five (5) years of experience in the
regular audit (internal or external) of a universal or
commercial bank as auditor-in-charge, senior auditor,
or audit manager.

A. I only
B. II only
C. Neither I nor II
D. Both I and II

The internal auditor of a universal or commercial bank must


be a CPA and must have at least five (5) years of experience
in the regular audit (internal or external) of a universal or
commercial bank as auditor-in-charge, senior, auditor, or
audit manager. He/she must be competent to examine all areas
in which the institution operates. (BSP Circular No. 499,
Series of 2005)

90. In its Circular No. 425, Series of 2000, the BSP requires
that only external auditors accredited by the BSP shall be
engaged by banks for regular audit or special engagements.
Which of the following is not an accreditation requirement
for external auditors?
A. No external auditor may be engaged by a bank if he/she has
or was committed to acquire any direct or material indirect
financial interest in the bank, or if his/her independence
is considered impaired under the circumstances specified
in the Code of Professional Ethics for CPAs.
B. The external auditor and the members of the audit team do
not have/shall not have outstanding loans or any credit
accommodations (including credit card obligations) with
any bank to be audited / being audited at the time of
signing the engagement and during the engagement.
C. The external auditor should have at least five (5) years
track record in conducting external audit.
D. The external auditor and members of the audit team adhere
to the highest standards of professional conduct,
including integrity and objectivity.

The requirement that the external auditor and the members of


the audit team do not have / shall not have outstanding loans
or may credit accommodations with any bank to be audited /
being audited does not include credit card obligations.

The other accreditation requirements are as follows:

a. The external auditor must have at least twenty (20)


existing corporate clients with resources of P50 million
each; and
b. The external auditor must have at least one (1) existing
client commercial bank in the regular audit or in lieu
thereof, the external auditor and the auditor who will
head the team must have at least five (5) years of
experience in the bank audit.

91. One of the documents that should be submitted by an auditor


who is applying for BSP accreditation is a Certification from
the Professional Regulation Commission (PRC) that the
external auditor and the members of the audit team have no
derogatory information, previous conviction or any pending
investigation. However, in the event that the certification
cannot be obtained because of the pendency of a case, the BSP
may dispense with this requirement if the case
A. Involves purely legal question
B. Does not, in any way, negate the auditors adherence to
the highest standards of professional conduct and degrade
his/her integrity and objectivity.
C. Either A or B
D. Neither A nor B

Section 3(i) of BSP Circular No. 245 dated May 25, 2000, was
amended by Circular No.318 (Series of 2002) which states that
in the event the PRC Certification cannot be obtained because
of the pendency of a case, the BSP may dispense with this
requirement upon determination by the Monetary Board that the
case involves purely legal question, or does not, in any way,
negate the auditors adherence to the highest standards of
professional conduct and degrade his/her integrity and
objectivity.

92. Which of the following cases that may be discovered by a


Bangko Sentral ng Pilipinas (BSP) accredited external auditor
during his/her audit fieldwork must be reported to the BSP?
I. Any material finding discovered during the period of
audit involving fraud or dishonesty (except cases that
were resolved during the audit period).
II. Adjustments or potential losses, the aggregate of which
amounts to at least 1% of the capital funds of the bank.
III. Any finding to the effect that the total bank assets,
on a going concern basis, are no longer adequate to
cover the total claims of creditors.

A. I only
B. I and II only
C. II and III only
D. I, II, and III
A BSP accredited external auditor must report to the BSP any
material finding which will reduce the capital funds of the
bank by at least 1%, including those cases that were resolved
during the period of audit.
(Section 4 of BSP Circular No.425, Series of 2000)

93. All banks are required to prepare the Financial Reporting


Package (FRP). The FRP, composed of the balance sheet, income
statement, and supporting schedules, shall be prepared on a
solo and consolidated basis. Solo basis shall refer to the
combined financial statements of the
A. Head office and subsidiaries.
B. Head office and branches/other offices.
C. Parent bank, head office, and subsidiaries.
D. Parent bank and subsidiaries.

Solo basis refers to the combined financial statements of the


head office and branches/other offices.

Consolidated basis refers to the combined financial


statements of parent bank and subsidiaries.

94. Within how many banking days after the end of the reference
quarter should banks submit their consolidated FRP to the
BSP?
A. 10
B. 20
C. 15
D. 30

The solo and consolidated FRP shall be prepared on a quarterly


basis, except for the solo balance sheet which shall be
prepared on a monthly basis.

The consolidated FRP shall be submitted within 30 banking


days after the end of the reference quarter.

The solo FRP shall be submitted within 15 banking days after


the end of the reference quarter. The solo balance sheet shall
be submitted within 15 banking days after the end of the
reference month.

95. Within how many banking days after the end of the reference
month or quarter should banks submit their solo balance sheet
and solo income statement to the Bangko Sentral ng Pilipinas?
A. 10
B. 20
C. 15
D. 30

COMMISSION ON AUDIT (COA)

96. This Constitutional Commission has the power, authority,


and duty to examine, audit, and settle all accounts pertaining
to the revenue and receipts of, and expenditures or uses of
funds and property, owned or held in trust by, or pertaining
to, the government, or any of its subdivisions, agencies, or
instrumentalities, including government-owned or controlled
corporations and recommend measures to improve the efficiency
and effectiveness of government operations.
A. Securities and Exchange Commission
B. Insurance Commission
C. Commission on Audit
D. Bureau of Internal Revenue

97. The COA is composed of


A. A chairman and two (2) commissioners.
B. A chairman and three (3) commissioners.
C. A chairman and four (4) commissioners.
D. A chairman and five (5) commissioners.

The COA is composed of a chairman and two (2) commissioners.


Together they are called the Commission Proper

98. The COA Chairman and the two (2) commissioners shall be
appointed by the President of the Philippines with consent of
the Commission on Appointments for a term of
A. 5 years
B. 6 years
C. 7 years
D. 9 years

99. Which of the following is not one of the principal duties


of the COA?
A. Keep the general accounts of the government and preserve
the vouchers and supporting papers pertaining thereto.
B. Maintain price stability conducive to a balanced and
sustainable economic growth.
C. Promulgate accounting and auditing rules and regulations
including those for the prevention and disallowance of
irregular, unnecessary, excessive, extravagant or
unconscionable expenditures, or uses of government funds
and properties.
D. Submit to the President of the Philippines and Congress,
within the time fixed by law, an annual report covering
the financial condition and operation of the government.

To maintain price stability conducive to a balanced and


sustainable economic growth is the primary objective of the
Bangko Sentral ng Pilipinas.
100. The COA conducts a comprehensive audit that includes
A. Financial and compliance audits
B. Compliance and performance audits.
C. Financial and financial-related audits.
D. Financial, compliance, and performance audits.
KEY ANSWERS

1. C 36. D 71. C
2. D 37. A 72. D
3. D 38. A 73. A
4. B 39. D 74. A
5. A 40. B 75. B
6. A 41. B 76. B
7. A 42. B 77. C
8. C 43. B 78. D
9. C 44. C 79. D
10. B 45. A 80. A
11. D 46. A 81. D
12. B 47. C 82. D
13. A 48. C 83. C
14. D 49. A 84. D
15. D 50. A 85. A
16. B 51. D 86. B
17. B 52. C 87. A
18. A 53. A 88. A
19. A 54. A 89. D
20. A 55. B 90. B
21. A 56. A 91. C
22. C 57. C 92. C
23. D 58. A 93. B
24. D 59. D 94. D
25. B 60. B 95. C
26. B 61. B 96. C
27. B 62. B 97. A
28. D 63. D 98. C
29. D 64. C 99. B
30. C 65. A 100. D
31. A 66. D
32. B 67. D
33. B 68. B
34. A 69. D
35. B 70. A
Chapter 3
The CPAs Professional Responsibilities

1. The Code of Ethics for Professional Accountants in the


Philippines consists of three parts. Part A

A. Applies to professional accountants in public practice.


B. Establishes the fundamental principles for professional
accountants.
C. Applies to professional accountants in business.
D. Provides a conceptual framework for the application of
fundamental principles and illustrates how the framework is
to be applied in specific situations.

The Code of Ethics for Professional Accountants in the


Philippines consists of three parts. Part A establishes
the fundamental principles for professional accountants and
provides a conceptual framework for their application.
Parts B and C illustrate how the conceptual framework is
to be applied in specific situations. Part B applies to
professional accountants in public practice. Part C applies
to professional accountants in business.

2. Which part of the Code of Ethics applies to professional


accountants in public practice?

A. Part A C.
Part C
B. Part B D.
Part D

3. Which of the following fundamental ethical principles requires


a professional accountant to be straightforward and honest
in all professional and business relationships?

A. Objectivity
B. Professional behavior
C. Professional competence and due care
D. Integrity

Part A of the Code establishes the following fundamental


ethical principles:

1. Professional Behavior
- A professional accountant should comply with relevant
laws and regulations and should avoid any action that
discredits the profession.

2. Integrity
- A professional accountant should be straightforward and
honest in all professional and business relationships.

3. Confidentiality
- A professional accountant should respect the
confidentiality of information acquired as a result of
professional and business relationships. Such information
should not be disclosed to third parties without proper
and specific authority unless there is a legal or
professional right or duty to disclose. Also, it should not
be used for the personal advantage of the professional
accountant or third parties.

4. Objectivity
- A professional accountant should not allow bias,
conflict of interest or undue influence of others to
override professional or business judgments.

5. Professional Competence and Due Care


- A professional accountant has a continuing duty to
maintain professional knowledge and skill at the level
required to ensure that a client or employer receives
competent professional service based on current
developments in practice, legislation and techniques.
When rendering professional services, a professional
accountant should act diligently and in accordance with
applicable technical and professional standards.

4. The following statements relate to the fundamental principles


of professional ethics:

A B C
D
Integrity implies fair dealing
And truthfulness. True
True False False
The principle of objectivity
imposes an obligation on
all professional accountants
to maintain professional
knowledge and skill at the
level required. True
False True False

The principles of professional


behavior requires all pro-
fessional accountants to act
diligently and in accordance
with applicable technical
and professional standards
when rendering professional
Services. False
False True True

5. Competence as a certifies public accountant includes all the


following except

A. Having the technical qualifications to perform an


engagement.
B. Possessing the ability to supervise and to evaluate the
quality of staff work.
C. Warranting the infallibility of the work performed.
D. Consulting others if additional technical information is
needed.

6. Which of the following fundamental ethical principles


prohibits association of professional accountants with
reports, returns, communications or other information that
is believed to contain a materially false or misleading
statement?

A. Integrity
B. Objectivity
C. Professional competence and due care
D. COnfidentiality

Under the principle of integrity, professional accountants


are required to be straightforward and honest in professional
and business relationships.

A professional accountant shall not knowingly be associated


with reports, returns, communications or other information
where the professional accountant believes that the
information:

1. Contains a materially false or misleading statement;


2. Contains statements or information furnished
recklessly; or
3. Omits or obscures required information where such
omission or obscurity would be misleading.

There will be no violation of the above provision if a


modified report is issued in respect of a matter described in
the foregoing paragraph.

7. The principle of professional competence and due care imposes


which of the following obligations on professional
accountants?

A. To maintain professional knowledge and skill at the level


required to ensure that clients or employers receive
competent professional service.
B. To refrain from disclosing confidential information
obtained as a result of professional and business
relationships without proper and specific authority
unless there is a legal or professional right or duty
to disclose.
C. To comply with relevant laws and regulations and avoid any
situation that may bring discredit to the profession.
D. Not to compromise professional or business judgment
because of bias, conflict of interest or undue
influence of others.

8. According to the Code of Ethics, professional competence may


be divided into two phases: attainment of professional
competence and maintenance of professional competence. The
attainment of professional competence requires the
following, except
A. A high standard of general education
B. Specific education, training and examination in
professionally relevant subjects.
C. Whether prescribed or not, a period of work experience.
D. A continuing awareness and an understanding of relevant
technical professional and business developments.
According to the Code, The maintenance of professional
competence requires a continuing awareness and an
understanding of relevant technical professional and business
developments. Continuing professional developments enables a
professional accountant to develop and maintain the
capabilities to perform competently within the professional
environment.

Answers A, B and C are the requirements to attain professional


competence.

9. The Code of Ethics provides a Conceptual Framework for


applying the fundamental ethical principles. This framework
requires a professional accountant to

I. Identify threats to compliance with the fundamental


principles.
II. Evaluate the significance of the identified threats.
III. Apply safeguards to eliminate the threats or reduce them
to an acceptable level.

A. I and II only
B. I and III only
C. II and III only
D. I, II, and II

The Code of Ethics provides a conceptual framework to


identify, evaluate, and respond to threats to compliance
with the fundamental ethical principles.

10. Which of the following threats to compliance with the


fundamental principles may occur as a result of the financial
or other interests of a professional accountant or of an
immediate or close family member?
A. Self-interest
B. Self-review
C. Advocacy
D. Familiarity

The Code of Ethics identifies the following threats to


compliance with the fundamental principles:

1. Self-interest threat
- The threat that a financial or other interest will
inappropriately influence the professional accountants
judgment or behavior.

2. Self-review threat
- the threat that a professional accountant will not
appropriately evaluate the results of a previous judgment
made or service performed by the professional accountant,
or by another individual within the professional
accountants firm or employing organization, on which the
accountant will rely when forming a judgment as part of
providing current service.

3. Advocacy threat
- The threat that a professional accountant will promote
clients or employers position to the point that the
professional accountants objectivity is compromised.

4. Familiarity threat
- The threat that due to a long or close relationship with
a client or employer, a professional accountant will be
too sympathetic to their interests or too accepting of
their work.

5. Intimidation threat
- The threat that a professional accountant will be
deterred from acting objectively because of actual or
perceived pressures, including attempts to exercise undue
influence over the professional accountant.

11. Which of the following may be considered by a professional


accountant to eliminate or reduce identified threats to an
acceptable level?
I. Safeguards created by the profession, legislation or
regulation.
II. Safeguards in the work environment.
III. Resign from the client or the employer.
IV. Decline or discontinue the professional engagement.

A. I and II only
B. III and IV only
C. I and IV only
D. II and III only

The Code states that when a professional accountant


identifies threats to compliance with the fundamental
principles and, based on an evaluation of those threats,
determines that they are not an acceptable level, the
professional accountant shall determine whether appropriate
safeguards are available and can be applied to eliminate the
threats or reduce them to an acceptable level.

The Code states further that in making determination, the


professional accountant shall exercise professional
judgment and take into account whether a reasonable and
informed third party, weighing all the specific facts and
circumstances available to the professional accountant at the
time, would be likely to conclude that the threats would be
eliminated or reduced to an acceptable level by the
application of the safeguards, such that compliance with the
fundamental principles is not compromised.

Safeguards are categorized into:


1. Safeguards created by profession, legislation and
regulations; and
2. Safeguards in the work experience.

If appropriate safeguards are not available or cannot be


applied, a professional accountant shall:
1. Decline or discontinue the specific professional
service involved; or
2. When necessary, resign from the engagement (if in
public practice) or the employing organization (if in
business).
12. The Code of Ethics allows an auditor to perform which of the
following services for an audit client that is not a public
interest entity?

A. Performance of bookkeeping services for the client.


B. Authorization of transactions for the client.
C. Preparation of client source documents.
D. Preparation and posting of journal entries without the
clients approval.

The Code states that a CPA may render services related to


the preparation of accounting records and financial statements
to an audit client that is not a public interest entity where
the services are of a routine or mechanical nature, provided
that any self-review threat created is reduced to an acceptable
level.

The following are examples of such services:


Providing payroll services based on client-prepared
data.
Recording transactions for which the client has
determined or approved the appropriate account
classification.
Posting transactions coded by the client to the
general ledger.
Posting client-approved entries to the trial balance;
and
Preparing financial statements based on information in
the trial balance.

13. A CPA provides audit services to a large company. Almost


eighty-five percent of a CPAs revenues come from this
client. Which statement is most likely to be true?

A. Appearance of independence may be lacking.


B. The CPA firm does not have the competence to perform the
audit.
C. The situation is satisfactory if the auditor exercises due
skeptical negative assurance care in the audit.
D. The auditor should provide an other matter paragraph to
his/her audit report adequately disclosing this
information and then it may issue an unmodified
opinion.

14. Safeguards created by the profession, legislation or


regulation include the following, except
A. Continuing professional development requirements.
B. Professional standards.
C. Firm-wide and engagement specific safeguards.
D. Educational, training and experience requirements for
entry into the profession.

Firm-wide and specific safeguards are safeguards in the work


environment.

According to the Code of Ethics, safeguards created by the


profession, legislation or regulation include:

1. Educational, training and experience requirements for


entry into the profession.
2. Continuing professional development requirements.
3. Corporate governance regulations.
4. Professional standards.
5. Professional or regulatory monitoring and disciplinary
procedures.
6. External review by a legally empowered third party of the
reports, returns, communications or information
produced by a professional accountant.

15. Which of the following circumstances may create self-interest


threat for a professional accountant in public practice?

A. A member of the assurance team having a direct financial


interest in the assurance client.
B. Performing a service for an assurance client that directly
affects the subject matter information of the
assurance engagement.
C. Being threatened with litigation by the client.
D. Acting as an advocate on behalf of an audit client in
litigation or disputes with third parties.

The Code of Ethics gives the following examples of


circumstances that may create self-interest threats for a
professional accountant in public practice:

1. A member of the assurance team having a direct financial


interest in the assurance client.
Financial interest is defined in the Code as an interest
in an equity or other security, debenture, loan or other
debt instrument of an entity, including rights and
obligations to acquire such an interest and derivatives
directly related to such interest.

2. A firm having undue dependence on total fees from a client.

3. A member of the assurance team having a significant close


business relationship with an assurance client.

4. A firm being concerned about the possibility of losing


significant client.

5. A member of the audit team entering into employment


negotiations with the audit client.

6. A firm entering into a contingent fee arrangement relating


to an assurance engagement.

7. A professional accountant discovering a significant error


when evaluating the results of previous professional
service performed by a member of the professional
accountants firm.

As defined in the Code, a contingent fee is a fee calculated


on a predetermined basis relating to the outcome of a
transaction or the result of the services performed by the
firm. A fee that is established by a court or other public
authority is not a contingent fee.

In an assurance engagement, a professional accountant in


public practice expresses a conclusion designed to enhance
the degree of confidence of the intended users other than the
responsible party about the outcome of the evaluation or
measurement of a subject matter against criteria.

Performing a service for an assurance client that directly


affects the subject matter information of the assurance
engagement (Answer B) may create self-review threat.
Being the threatened with litigation by the client (Answer
C) may create intimidation threat.

Acting as an advocate on behalf of an audit client in


litigation or disputes with third parties (Answer D) may create
advocacy threat.

16. The following are examples of circumstances that may create


familiarity threat, except

A. The firm promoting shares in an audit client.


B. Long association of senior personnel with the assurance
client.
C. A member of the engagement team having a close or
immediate family member who is a director or officer of
the client.
D. A director or officer of the client or an employee in a
position to exert significant influence over the subject
matter of the engagement having recently served as the
engagement partner.

In addition to the circumstances described in Answers B, C,


and D, the following may also create familiarity threat:

A member of the engagement team having a close or


immediate family member who is an employee of the
client who is in a position to exert significant
influence over the subject matter of the engagement.

A professional accountant accepting gifts or


preferential treatment from a client, unless the value
is trivial or inconsequential.

As defined in the Code, immediate family refers to a


spouse (or equivalent) or dependent. Close family refers
to a parent, child or sibling who is not an immediate
family member.

Promoting shares in an audit client may create advocacy


threat.
17. The following circumstances may create intimidation threat,
except

A. A firm being threatened with dismissal from a client


engagement.
B. A firm being pressured to reduce inappropriately the extent
of work performed in order to reduce fees.
C. A firm being threatened with litigation by the client.
D. A member of the assurance team being, or having recently
been, a director or officer of the client.

The following are examples of circumstances that may create


self-review threat:

1. The discovery of a significant error during a


reevaluation of the work of the professional
accountant in public practice.
2. A firm issuing an assurance report on the effectiveness
of the operation of financial systems after designing
or implementing the systems.
3. A firm having prepared the original data used to
generate records that are the subject matter of
the assurance engagement.
4. A member of the assurance team being, or having recently
been, director or officer of the client.
5. A member of the assurance team being, or having recently
been, employed by the client in a position to exert
significant influences over the subject matter of
the engagement.
6. The firm performing a service for an assurance client
that directly affects the subject matter information
of the assurance engagement.

18. On which of the following safeguards a professional


accountant in public practice cannot rely solely to reduce
threats to an acceptable level?

A. Safeguards created by profession, legislation or


regulation.
B. Firm-wide safeguards.
C. Engagement specific safeguards.
D. Safeguards within the clients systems and procedures.
Safeguards to eliminate or reduce threats to an acceptable
level are categorized into:
1. Safeguards created by the profession, legislation or
regulation; and
2. Safeguards in the work environment which include firm-
Wide safeguards and engagement specific safeguards.

The Code provides that, depending on the nature of the


engagement, a professional accountant in public practice
may also be able to rely on safeguards within the clients
systems and procedures. However, a professional accountant
cannot rely solely on such safeguards to reduce threats to an
acceptable level.

The Code gives the following examples of safeguards within


the clients systems and procedures:

1. The client requires persons other than management to ratify


or approve the appointment of a firm to perform an
engagement.
2. The client has competent employees with experience and
seniority to make managerial decisions.
3. The client has implemented internal procedures that ensure
objective choices in commissioning non-assurance
engagements.
4. The client has a corporate governance structure that
provides appropriate oversight and communications
regarding the firms services.

19. Which of the following is an example of engagement-specific


safeguards in the work environment?

A. Advising partners and professional staff of those


assurance clients and related entities from which they
must be independent.
B. Disclosing to those charged with governance of the client
the nature of services provided the extent of fees
charged.
C. A disciplinary mechanism to promote compliance with the
firms policies and procedures.
D. Published policies and procedures to encourage and empower
staff to communicate to senior levels within the firm
any issue relating to compliance with the fundamental
principles that concerns them.
In addition to the examples in Answers A, C, and D, firm-
wide safeguards in the work environment may include:
1. Leadership of the firm that stresses the importance of
compliance with the fundamental principles.

2. Leadership of the firm that establishes the expectation


that members of an assurance team will act in the
public interest.

3. Policies and procedures to implement and monitor


quality control of engagements.

4. Documented policies regarding the need to identify


threats to compliance with the fundamental principles,
evaluate the significance of those threats, and apply
safeguards to eliminate or reduce the threats to an
acceptable level or, when appropriate safeguards are not
available or cannot be applied, terminate or decline the
relevant engagement.

5. Documented internal policies and procedures requiring


compliance with the fundamental principles.

6. Policies and procedures that will enable the


identification of interests or relationships between
the firm or members of engagement teams and
clients.

7. Policies and procedures to monitor and, if necessary,


manage the reliance on revenue received from a single
client.

8. Using different partners and engagement teams with


separate reporting lines for the provision of non-
assurance services to an assurance client.

9. Policies and procedures to prohibit individuals who are


not members of an engagement team from
inappropriately influencing the outcome of the
engagement.

10. Timely communication of the firms policies and


procedures, including any changes to them, to all
partners and professional staff, and appropriate
training and education on such policies and procedures.

11. Designating a member of senior management to be


responsible for overseeing the adequate functioning
of the firms quality and control system.

12. Advising partners and professional staff of assurance


clients and related entities from which independence
is required.

13. A disciplinary mechanism to promote compliance with


policies and procedures.
14. Published policies and procedures to encourage and
empower staff to communicate to senior levels within the
firm any issue relating to compliance with the
fundamental principles that concerns them.

Engagement-specific safeguards in the work environment


may include:
1. Having a professional accountant who was not
involved with the non-assurance service review the
non-assurance work performed or otherwise advise as
necessary.

2. Having a professional accountant who was not a


member of the assurance team review the
assurance work performed or otherwise advise as
necessary.

3. Consulting an independent third party, such as a


committee of independent directors, a professional
regulatory body or another professional accountant.

4. Discussing ethical issues with those charged with


governance of the client.

5. Disclosing to those charged with governance of the


client the nature of services provided and
extent of fees charged.

6. Involving another firm to operate or re-perform part


of the engagement.

7. Rotating senior assurance team personnel.


20. If the fee quoted for a professional service is so low, it
may be difficult for the CPA to perform the engagement in
accordance with applicable technical and professional standards
for that price. This situation may create a self-interest
threat to

A. Professional competence and due care


B. Objectivity
C. Integrity
D. Professional behavior

21. According to Section 240 of the Code of Ethics, fees charged


for assurance engagements should be fair reflection of the
value of the work involved. In determining professional
fees, the following should be taken into account, except

A. The time necessarily occupied by each person engaged on


the work.
B. The outcome or result of a transaction or the result of
the work performed.
C. The skill and knowledge required for the type of work
involved.

D. The level of training and experience of the persons


necessarily engaged on the work.

A fee computed on a predetermined basis relating to the


outcome or result of a transaction or the result of the work
performed is a contingent fee. Contingent fees may create
threats to compliance with the fundamental ethical principles,
for example, a self-interest threat to objectivity.

22. Which of the following is not a contingent fee?

A. A fee that is dependent upon the approval of the assurance


clients loan application.
B. An audit fee that is based on 5% of the clients adjusted
net income for the current year.
C. A fee that is fixed by the court or other public authority.
D. An arrangement whereby no fee will be charged unless a
specified finding or result is attained.

According to the Code of Ethics, a fee established by a court


or other public authority is not a contingent fee.
23. The Code of Ethics, requires that members of assurance teams,
firms and, when applicable, network firms be independent
of assurance clients. Independence requires

A. Independence of mind only


B. Independence in appearance only
C. Both independence of mind and independence in appearance
D. Either independence of mind or independence in appearance

24. Which of the following most completely describes how


independence has been defined by the accountancy
profession?

A. Possessing the ability to act with integrity, and exercise


objectivity and professional skepticism.
B. Accepting responsibility to act professionally and in
accordance with laws and regulations.
C. Avoiding the appearance of significant interests in the
affairs of an assurance client.
D. Performing an assurance service from the viewpoint of the
public.

Independence, as stated in the Code of Ethics, requires:

Independence of mind - the state of mind that permits the


expression of a conclusion without being affected by
influences that compromise professional judgment, thereby
allowing an individual to act with integrity, and exercise
objectivity and professional skepticism.

Independence in appearance - the avoidance of facts and


circumstances that are so significant that a reasonable
and informed third party, would be likely to conclude, weighing
all the specific facts and circumstances, that a firms, or a
member of the audit teams, integrity, objectivity or
professional skepticism had been compromised.

Answer B is incorrect because ones acceptance of


responsibility to act professionally and in accordance with
laws and regulations does not necessarily require
independence.

Answer C is incorrect because to be independent, one should


be independent both in mind and in appearance.
Answer D is incorrect because professional standards require
objectivity, not the adoption of any viewpoint.

25. Which of the following is a misunderstanding created by the


use of the word independence?

A. Possessing the ability to act with integrity and


objectivity.
B. Independence precludes relationships that may appear to
impair objectivity in rendering assurance services.
C. A person exercising professional judgment should be free
from all economic, financial and other relationships.
D. Possessing the ability to express a conclusion without
being affected by influences that compromise
professional judgment.

It is impossible for a person exercising professional


judgment to be free from all economic, financial and other
relationships as every member of society has relationships
with others. The significance of economic, financial and other
relationships should be evaluated to identify threats that
may exist.

26. Which of the following would not, in itself, create a


network?

A. A larger structure where the entities within the structure


share costs that are limited only to those costs
related to development of audit methodologies, manuals,
or training courses.
B. A larger structure that is aimed at cooperation and the
entities within the structure share common ownership,
control or management.
C. A larger structure that is aimed at cooperation and the
entities within the structure share common quality control
policies and procedures.
D. A larger structure that is aimed at cooperation and it is
clearly aimed at profit or cost sharing among the
entities within the structure.

The Code of Ethics defines a network as a larger structure


that is:
Aimed at cooperation, and
Clearly aimed at profit or cost sharing or shares
common ownership, control or management, common
quality control policies and procedures, common
business strategy, the use of a common brand name, or
a significant part of professional resources.

27. The Code of Ethics provides that where the larger structure
is aimed at cooperation and the entities within the structure
share a significant part of professional resources, it is
considered to be a network. Professional resources include the
following, except

A. Audit methodology or audit manuals


B. Training courses and facilities
C. Brand name
D. Partners and staff

According to the Code, professional resources include:


Common systems that enable firms to exchange information
such as client data, billing and time records;
Partners and staff;
Technical departments to consult on technical or industry
specific issues, transactions or events for assurance
engagements;
Audit methodology or audit manuals; and
Training courses and facilities.

28. In cases when the threat to independence is significant and


no safeguards are available to reduce it to an acceptable
level, which of the following actions should be taken?

I. Eliminating the activities or interests creating the


threat.
II. Refusing to accept or continue the assurance engagement.

A. I only
B. II only
C. Neither I nor II
D. Either I or II

29. When identified threats to independence are significant and


the firm decides to accept or continue the assurance
engagement, the decision should be documented. The firms
documentation should include

I. A description of the threats identified.


II. The safeguards applied to eliminate or reduce threats to
an acceptable level.

A. I only
B. II only
C. Neither I nor II
D. Both I and II

30. Which of the following threats to independence would most


likely be created by a financial interest in an assurance
client?

A. Self-interest threat
B. Self-review threat
C. Familiarity threat
D. Intimidation threat

31. A self-interest threat may be created when a member of the


assurance team knows that his close family member has a
direct financial interest or a material indirect financial
interest in the assurance client. Which of the following
should be considered in evaluating the significance of the
identified threat to independence?

I. The nature of the relationship between the member of the


assurance team and the close family member.
II. The materiality of the financial interest.

A. I only
B. II only
C. Neither I nor II
D. Both I and II

32. In which of the following circumstances would a CPA be


considered independent when performing the audit of the
financial statements of a new client for the year ended
December 31, 20X5?
A. The CPA resigned on January 17, 20X5 from the board of
directors of the client, prior to accepting the new audit
engagement.
B. The CPA continues to hold an immaterial indirect financial
interest in the client.
C. The CPA continues to serve as a trustee for the clients
pension plan and has the authority to make investment
decisions.
D. The CPAs spouse owns an immaterial amount of ordinary
shares in the client.

33. A loan, or guarantee of a loan, to the firm from an assurance


client that is a bank or similar institution, would not
create a threat to independence provided.

I. The loan, or guarantee, is made under normal lending


procedures, terms and requirements.
II. The loan is immaterial to both the firm and the assurance
client.

A. I only
B. II only
C. Neither I nor II
D. Both I and II

34. A loan, or a guarantee of a loan, from an assurance client


that is a bank or a similar institution, to a member of the
assurance team or his immediate family, would not create a
threat to independence provided the loan, or guarantee, is

A. Material to the member of the assurance team or his


immediate family.
B. Material to the assurance client.
C. Material to both the member of assurance team or his
immediate family and the assurance client.
D. Made under normal lending procedures, terms and
requirements.

According to the Code of Ethics, a loan, or guarantee of a


loan, from an assurance client that is a bank or a similar
institution, to a member of the assurance team or his
immediate family would not create a threat to independence
provided the loan, or guarantee, is made under normal lending
procedures, terms and requirements. Examples of such loans
include credit card obligations which are normally available
to other credit card holders and fully secured car loans and
housing loans which are not past due.

35. Which of the following would not create a threat to


independence?

A. A loan, or a guarantee of a loan, from an assurance client


that is a bank or a similar institution and the loan
or guarantee is made under normal lending procedures,
terms and requirements and it is material to the
assurance client or firm receiving the loan.
B. A loan, or a guarantee of a loan, from an assurance client
that is a bank or a similar institution and the loan
or guarantee is material to both the firm and the
assurance client.
C. A deposit made by the firm or a member of the assurance
team with an assurance client that is a bank and such
deposit is held under normal commercial terms.
D. A loan, or a guarantee of a loan, from an assurance client
that is a bank or a similar institution and the loan
or guarantee is not made under normal lending
procedures.

The Code of Ethics states that if a firm or a member of the


assurance team, or a member of that individuals immediate
family, has deposits or a brokerage account with an
assurance client that is a bank, broker, or similar
institution, a threat to independence is not created if the
deposit or account is held under normal commercial terms.

36. A self-interest threat would be created if the firm, or a


member of the assurance team, makes a loan to an assurance
client that is not a bank or similar institution, or
guarantees such an assurance clients borrowing. The self-
interest threat created would be so significant that no
safeguard could reduce the threat to an acceptable level unless
the loan or guarantee is

A. Made under normal lending terms, procedures and


requirements.
B. Immaterial to the firm or the member of the assurance team.
C. Immaterial to both the firm and the member of the assurance
team and the assurance client.
D. Made under normal lending terms, procedures and
requirements and the loan or guarantee is immaterial
to both the firm and the member of the assurance team
and the assurance client.
37. A self-interest threat that would be created if the firm, or
a member of the assurance team, accepts a loan from, or has
borrowing guaranteed by, an assurance client that is not a bank
or similar institution. The self-interest threat created would
be so significant that no safeguard could reduce the threat to
an acceptable level unless the loan or guarantee is

A. Made under normal lending terms, procedures and


requirements.
B. Immaterial to the firm or the member of the assurance team.
C. Immaterial to both the firm and the member of the assurance
team and the assurance client.
D. Made under normal lending terms, procedures and
requirements and the loan or guarantee is immaterial
to both the firm and the member of the assurance team
and the assurance client.

38. A close business relationship between a firm or a member of


the assurance team and the assurance client or its management,
or between a firm, a network firm and financial statement
audit client may create

A. Self-interest and intimidation threats


B. Self-review and familiarity threats
C. Advocacy and self-review threats
D. Self-interest and self-review threats

The Code of Ethics states that a close business relationship


between a firm and a member of the assurance team and the
assurance client or its management, or between the firm, a
network firm and a financial statement audit client, will
involve a commercial or common financial interest and may
create self-interest and intimidation threats.

The Code gives the following examples of such relationships:

Having a material financial interest in a joint


venture with the assurance client or a controlling
owner, director, officer or other individual who
performs senior managerial functions for that client.
Arrangements to combine one or more services or
products of the firm with one or more services or
products of the assurance client and to market the
package with reference to both parties.
Distribution or marketing arrangements under which the
firm acts as a distributor or marketer of the assurance
clients products or services, or the assurance client
acts as the distributor or marketer of the products
or services of the firm.

39. Which of the following threats to independence may be created


by family and personal relationships between a member of
the assurance team and a director, an officer, or an employee
of an assurance client in a position to exert direct and
significant influence over the subject matter information
of the assurance engagement?

A. Self-interest, familiarity or intimidation threats


B. Self-review, familiarity or advocacy threats
C. Advocacy, familiarity or intimidation threats
D. Self-interest, advocacy or self-review threats

40. when an immediate family member of a member of the assurance


team is a director, an officer, or an employee of the
assurance client in a position to exert direct and significant
influence over the subject matter information of the assurance
engagement, or was in such a position during the period
covered by the engagement, the threats to independence can
only be reduced to an acceptable level by

A. Where possible, structuring the responsibilities of the


assurance team so that the professional does not deal with
matters that are within the responsibility of the
immediate family member.
B. Withdrawing from the assurance engagement.
C. Removing the individual from the assurance team.
D. Discussing the issue with those charged with governance,
such as the audit committee.

According to the Code of Ethics, the only safeguard that can


reduce the threats to independence is by removing the
individual from the assurance team. If this safeguard is not
used, the only course of action is to withdraw from the
assurance engagement.

41. When a close family member of a member of the assurance team


is a director, an officer, or an employee of the assurance
client in a position to exert direct and significant
influence over the subject matter information of the assurance
engagement, threats to independence may be created. If the
threats are other than clearly insignificant, which of the
following safeguards can be applied to reduce the threats to
an acceptable level?

I. Removing the individual from the assurance team


II. Where possible, structuring the responsibility of the
assurance team so that the professional does not deal
with matters that are within the responsibility of the
close family member.

A. I only
B. II only
C. Either I or II
D. Neither I nor II

42. Which of the following threats to independence is created


when a member of the assurance team participates in the
assurance engagement while knowing, or having reason to
believe, that he is to, or may, join the assurance client
sometime in the future?

A. Intimidation threat
B. Self-interest threat
C. Self-review threat
D. Familiarity threat

According to the Code of Ethics, firm policies and procedures


shall require members of an assurance team to notify the
firm when entering employment negotiations with the client. The
significance of the threat shall be evaluated and
safeguards applied when necessary to eliminate the threat or
reduce it to an acceptable level. Examples of such
safeguards include:

1. Removing the individual from the assurance team; or


2. A review of any significant judgments made by that
individual while on the team.

43. Using the same engagement partner or the same individual for
the engagement quality control review on a financial
statement audit over a prolonged period may create a

A. Self-review threat
B. Intimidation threat
C. Familiarity threat
D. Self-interest threat

44. CPAs may provide bookkeeping services to their non-


public-interest audit clients, but there are a number of
conditions that must be satisfied if the auditor is to
maintain independence. Which of the following conditions is
not necessary?

A. The CPA must not assume a management role or function.


B. The client must hire an external CPA to approve all of the
journal entries prepared by the auditor.
C. The auditor must comply with GAAS when auditing work
prepared by his/her firm.
D. The client must accept responsibility for the financial
statements.

45. Several months after an unmodified audit report was issued,


the auditor discovers the financial statements were materially
misstated. The clients CEO agrees that there are
misstatements, but refuses to correct them. She claims that
confidentiality prohibits the CPA from informing anyone.
Is the CEO correct?
A. Yes. The auditor must maintain confidentiality.
B. No. But because the audit report has been issued it is too
late.
C. Yes. But to be ethically correct, the auditor should
violate the confidentiality rule and disclose the
error.
D. No. The auditor has an obligation to issue a revised audit
report, even if the CEO will not correct the
financial statements.

46. Which of the following would not generally create a threat


to independence?

A. The purchase of goods and services from an assurance client


by the firm (or from a financial statement audit
client by a network firm) or a member of the assurance
team provided that the transaction is in the normal
course of business and on an arms length basis.
B. A partner or employee of the firm or a network firm serves
as Company Secretary for a financial statement audit
client.
C. Determining which recommendations of the firm should be
implemented.
D. Reporting, in a management role, to those charged with
governance.

Serving as the Company Secretary for a financial statement


audit client may create self-review and advocacy threats
(Answer B).

Determining which recommendations of the firm should be


implemented (Answer C) and reporting in a management role,
to those charged with governance (Answer D) would generally
create self-interest and advocacy threats.

47. The following activities may create self-interest or self-


review threats, except

A. Preparing source documents or originating data evidencing


the occurrence of a transaction.
B. Supervising assurance client employees in the performance
of their normal recurring activities.
C. Having custody of an assurance clients assets.
D. Using the same senior personnel on an assurance engagement
over a long period of time.

Using the same engagement partner on an assurance engagement


over a long period of time may create a familiarity threat.
In evaluating the significance of the threat, the following
factors are to be considered:
The length of time that the individual has been a
member of the assurance team;
The role of the individual on the assurance team;
The structure of the firm; and
The nature of the assurance engagement.

48. The following forms of assistance to a financial statement


audit client do not generally threaten the firms
independence, except

A. Analyzing and accumulating information for regulatory


reporting.
B. Assisting in resolving account reconciliation problems.
C. Authorizing or approving transactions.
D. Assisting in the preparation of consolidated financial
statements.

The provision to audit clients of technical assistance and


advice on accounting principles is an appropriate means to
promote the fair presentation of the financial statements
and does not generally threaten the firms independence. The
following services are considered to be a normal part of
the audit process, and, under normal circumstances, do not
threaten the firms independence:
Assisting an audit client in resolving account
reconciliation problems.
Analyzing and accumulating information for regulatory
reporting.
Assisting in the preparation of consolidated financial
statements.
Drafting disclosure items.
Proposing adjusting journal entries.
Providing assistance and advice in preparation of
local statutory accounts of subsidiary entities.

49. As defined in the Code, a valuation comprises the making of


assumptions with regard to future developments, the
application of certain methodologies and techniques, and
the combination of both in order to compute a certain value,
or range of values, for an asset, a liability or for a
business as a whole. which of the following threats may be
created when a firm or a network firm performs valuation for
an audit client that is to be incorporated in the clients
financial statements?

A. Advocacy threat
B. Familiarity threat
C. Self-review threat
D. Intimidation threat

50. A firm provides valuation services to an audit client. The


service involves valuation of matters material to the
financial statements and involves a significant degree of
subjectivity. Which of the following safeguards should be
applied to eliminate the self-review threat created, or reduce
it to an acceptable level?

A. Confirming with the audit client their understanding of


the underlying assumptions of the valuation and the
methodology to be used and obtaining approval for
their use.
B. Obtaining the audit clients acknowledgement of
responsibility for the results of the work performed by
the firm.
C. Making arrangements so that personnel providing such
services do not participate in the audit engagement.
D. The self-review threat created could not be reduced to an
acceptable level by the application of any safeguard.

The Code states, If the valuation services involves the


valuation of matters material to the financial statements
and the valuation involves a significant degree of
subjectivity, the self-review threat could not be reduced to
an acceptable level by the application of any safeguard.
Accordingly, such valuation services should not be provided
or, alternatively, the only course of action could be to
withdraw from the audit engagement.
If valuation services (either separately or in the aggregate)
are not material to the financial statements, or do not
involve a significant degree of subjectivity, the self-
review threat created could not be reduced to an acceptable
level by the application of the safeguards described in
answers A, B, and C.

51. The following statements relate to the provision of taxation,


internal audit or IT Systems services to audit clients.
Which is false?

A. Tax return preparation services may create a self-review


threat.
B. A self-review threat may be created when a firm, or network
firm, provides internal audit services to an audit
client.
C. The provision of services by a firm or network firm to an
audit client that involve the design and
implementation of financial information technology
systems that are used to generate information forming
part of a clients financial statements may create a
self-review threat.
D. The provision of services in connection with the
assessment, design, and implementation of internal
accounting controls and risk management controls does
not create a threat to independence provided that
firm or network firm personnel do not perform
management functions.

52. Litigation support services include the following


activities, except

A. Acting as an expert witness.


B. Providing assistance to an audit clients internal legal
department.
C. Calculating estimated damages or amounts that might become
receivable or payable as the result of litigation or
other legal dispute.
D. Assistance with document management and retrieval in
relation to a dispute or litigation.

The provision of assistance to an audit clients internal


legal department is a legal service, not a litigation
support service.

53. What threat to independence is created when the litigation


support services provided to an audit client include the
estimation of the possible outcome and thereby affects the
amounts or disclosure to be reflected in the financial
statements?

A. Self-review threat
B. Advocacy threat
C. Intimidation threat
D. Familiarity threat

54. According to the Code, legal services encompasses a wide and


diversified range of areas including both corporate and
commercial services to clients - such as contract support;
litigation, mergers, and acquisition advice and support,
and the provision of assistance to a clients internal legal
department. The provision of legal services by a firm, or
network firm, to an audit client ay create

A. Self-interest threat
B. Self-review and advocacy threat
C. Advocacy and intimidation threat
D. Familiarity and intimidation threat

55. The following statements refer to the provision of legal


services to an audit client. Which is incorrect?

A. The provision of legal services to an audit client


involving matters that would not be expected to have a
material effect on the financial statements may create a
self-review threat.
B. Legal services to support an audit client in the execution
of a transaction (e.g., contract support) may create an s
elf-review threat.
C. Acting for an audit client in the resolution of a dispute
or litigation in such circumstances when the amounts
involved are material in relation to the financial
statements of the audit client would create advocacy and
self-review threats that are so significant no safeguards
could reduce the threats to an acceptable level.
D. The appointment of a partner or an employee of the firm or
network firm as General Counsel for legal affairs to an
audit client would create self-review and advocacy threats
that are so significant no safeguards could reduce the
threats to an acceptable level.

According to the Code, the provision of legal services to


an audit client involving matters that would not be
expected to have a material effect on the financial
statements is not considered to create an unacceptable
threat to independence.

56. The recruitment of senior management for an audit client may


create the following current or future threats to
independence, except

A. Self-interest threat
B. Familiarity threat
C. Intimidation threat
D. Self-review threat

57. The provision of corporate finance services, advice or


assistance to an audit client may create

A. Self-interest threat
B. Self-interest and intimidation threats
C. Advocacy and self-review threats
D. Advocacy and intimidation threats

58. When the total fees generated by an assurance client


represent a large portion of a firms total fees, the
dependence on that client or client group and concern about
the possibility of losing the client may create a/an

A. Self-interest threat
B. Self-review threat
C. Intimidation threat
D. Advocacy threat

59. What threat to independence may be created when the fees


generated by the assurance client represent a large portion
of the revenue of an individual of the firm?

A. Self-review threat
B. Familiarity threat
C. Self-interest threat
D. Advocacy threat

60. What threat to independence may create when fees due from an
assurance client for professional services remain unpaid
for a long time, especially if a significant part is not paid
before the issue of the assurance report for the following
year?
A. Advocacy threat
B. Self-interest threat
C. Intimidation threat
D. Self-review threat

61. These are fees calculated on a predetermined basis relating


to the outcome or result of a transaction or the result of
the work performed.

A. Contingent fees
B. Fixed fees
C. Predetermined fees
D. Commissions

62. What threats to independence are created when a contingent


fee is charged by a firm in respect of an assurance engagement?

A. Self-review and intimidation threats


B. Self-interest and advocacy threats
C. Familiarity and intimidation threats
D. Self-interest and self-review threats

63. Accepting gifts or hospitality (unless inconsequential or


trivial) may create

A. Self-interest and familiarity threats


B. Advocacy and intimidation threats
C. Familiarity and self-review threats
D. Self-interest and self-review threats

64. Which of the following threats to independence may be created


when litigation takes place, or appears literally, between
the firm or a member of the assurance team and the assurance
engagement?

A. Self-interest or advocacy threat


B. Advocacy or intimidation threat
C. Self-interest or intimidation threat
D. Familiarity or self-review threat

65. After evaluating the significance of the threat created by


an actual or threatened litigation, the following safeguards
should be applied to reduce the threat to an acceptable
level, except
A. Disclosing to the audit committee, or others charged with
governance, the extent and nature of the litigation.
B. If the litigation involves a member of the assurance team,
removing that individual from the assurance team.
C. Involving an additional professional accountant in the
firm who was not a member of the assurance team to review
the work or otherwise advice as necessary.
D. Withdraw from, or refuse to accept, the assurance
engagement.

66. Which of the following is not a factor to consider in


determining the professional fee of a professional
accountant in public practice?

A. The skill and knowledge required for the type of


professional services involved.
B. The result of the assurance work.
C. The level of training and experience of the persons
necessarily engaged in performing the professional
services.
D. The time necessarily occupied by each person engaged in
performing the professional services.

An assurance engagement should not be performed for a fee


that is contingent on the result of the assurance work or on
items that are the subject matter of the assurance
engagement.

In addition to factors mentioned in answers A, C and D, the


professional accountant should also take into account the
degree of responsibility the performance of the
professional services entails.

67. Janus De Belen, CPA, was offered the engagement to audit the
financial statements of ABC Company for the year ended
December 31, 2015. Janus had served as a director of ABC
Company until December 31, 2013, and his spouse currently
owns 1,000 of 200,000 outstanding shares of ABC Company. Janus
disassociated form ABC Company prior to being offered the
engagement. Moreover, the engagement does not cover any
period that includes Janus association or employment with ABC
Company. Under the Code of Ethics, Janus should

A. Decline the engagement because of his spouses stock


ownership.
B. Accept the engagement.
C. Decline the engagement because he had served as director.
D. Accept the engagement because his spouses stock ownership
is an indirect financial interest.

68. Under the Code of Ethics,

A. An immediate family member of a professional accountant,


whether or not in public practice, may not accept a gift from
a client.
B. A close relative of a professional accountant not in public
practice may not accept a gift from a client.
C. A professional accountant in public practice may accept an
inconsequential gift from a client.
D. A professional accountant, whether or not in public
practice, may not accept a gift from a client.

69. A client company has not paid its 20X5 audit fees. According
to the Code of Professional Ethics, in order for the
auditor to be considered independent with respect to the 20X6
audit, the 20X5 audit fees must be paid before the:

A. 20X5 report is issued


B. 20X6 fieldwork is started
C. 20X6 report is issued
D. 20X7 fieldwork is started

70. As defined in the Code of Ethics, ______________ is the


communication to the public of information as to the
services or skills provided by professional accountants in
public practice with a view to procuring professional
business.

A. Advertising
B. Publicity
C. Solicitation
D. Marketing professional services

71. As defined in the Code of Ethics, ______________ is the


communication to the public of facts about a professional
accountant which are not designed for the deliberate
promotion of that professional accountant.
A. Advertising
B. Publicity
C. Solicitation
D. Marketing professional services

72. The following statements relate to the provisions of the Code


of Ethics that deal with the professional accountants
marketing of professional services. Which is false?

A. When a professional accountant in public practice solicits


new work through advertising or other forms of
marketing, a self-interest threat to compliance with the
principle of professional behavior may be created.
B. The professional accountant should be honest and truthful
when marketing professional services.
C. Advertising and publicity are generally unacceptable.
D. When marketing professional services, the professional
accountant should not make exaggerated claims for services
offered, qualifications possessed or experience
gained.

73. The Board of Accountancy Resolution No. 126, Series of 2008


(Adoption of the Rules and Regulations on Advertising for
the Philippine Accountancy Profession) states that, generally,
advertising and publicity in any medium are acceptable,
provided:

a) It has as its objective the notification to the public


or such sectors of the public as are concerned, of
matters of fact (e.g., name, address, contact numbers,
services offered) in a manner that is not false,
misleading or deceptive;
b) It is in good taste;
c) It is professionally dignified; and
d) It avoids frequent repetition of, and any undue
prominence being given to, the name of the firm or
professional accountant in public practice.
B. Other Professional Responsibilities

74. A firm should establish and maintain a system of quality


control to provide it with reasonable assurance that:
I. The firm and its personnel comply with
professional standards and applicable legal and
regulatory requirements.
II. Reports issued by the firm or engagement partners
are appropriate in the circumstances.
A. I only
B. II only
C. Both I and II
D. Neither I nor II
75. The firm's system of quality control should include policies
and procedures that address each of the following
elements, except
A. Monitoring
B. Control environment
C. Relevant ethical requirements
D. Human resources
According to PSQC 1, the firm should establish and maintain a
system of quality control that includes policies and
procedures that address each of the following elements:

a) Leadership responsibilities for quality within the


firm.
b) Relevant ethical requirements,
c) Acceptance and continuance of client relationships
and specific engagements.
d) Human resources.
e) Engagement performance.
f) Monitoring.
Control environment is a component of an entity's
internal control system.

76. Which of the following are elements of a CPA firm's


quality control that should be considered in
establishing its quality control policies and
procedures?
Ethical Human Engagement
Requirements Resources Performance
No Yes No
Yes No No
c. Yes Yes Yes
No No Yes
77. Which of the following is an element of a CPA firm's quality
control system that should be considered in establishing its
quality control policies and procedures?
A. Considering audit risk and materiality.
B. Managing human resources.
C. Using statistical sampling techniques.
D. Complying with laws and regulations.

78. Which of the following quality control elements is most


closely associated with the requirement to promote a culture
of quality?
A. Monitoring
B. Leadership responsibilities for quality within the
firm
C. Engagement performance
D. Human resources

79. The statement, "Quality control policies and procedures


should be relevant, adequate, effective, and complied with."
is most closely associated with what quality control element?
A. Engagement performance
B. Leadership responsibilities for quality within the firm
C. Monitoring
D. Relevant ethical requirements
80. This quality control element requires a firm to establish
policies and procedures to provide it with reasonable
assurance that engagements are performed in accordance with
professional standards and regulatory and legal requirements,
and that the firm or the engagement partner issue reports
that are appropriate in the circumstances.
A. Ethical requirements
B. Engagement performance
C. Monitoring
D. Human resources
81. In pursuing a firm's quality control objectives, a firm
should adopt policies and procedures to enable it to identify
and evaluate circumstances and relationships that create
threats to independence, and to take appropriate action to
eliminate those threats or reduce them to an acceptable level
by applying safeguards, or, if considered appropriate, to
withdraw from the engagement. Which quality control element
would this be most likely to satisfy?
A. Ethical requirements
B. Monitoring
C. Human resources
D. Leadership responsibilities for quality within the
firm
82. The primary purpose of establishing quality control
policies and procedures for deciding whether to accept a new
client is to
A. Anticipate before performing any fieldwork whether an
unmodified opinion can be expressed.
B. Enable the CPA firm to attest to the reliability of the
client.
C. Satisfy the CPA firm's duty to the public concerning
the acceptance of new clients.
D. Minimize the likelihood of association with clients
whose management lacks integrity.
According to PSQC l, a firm should establish policies and
procedures for the acceptance and continuance of client
relationships and specific engagements, designed to provide
it with reasonable assurance that it will only undertake or
continue relationships and engagements where the firm:
a. Has considered the integrity of the client and
does not have information that would lead it to
conclude that the client lacks integrity;
b. Is competent to perform the engagement and has the
capabilities, time and resources to do so; and
c. Can comply with ethical requirements.
83. As defined in PSQC 1, is a process comprising an ongoing
consideration and evaluation of the firm's system of quality
control, including a periodic inspection of a selection of
completed engagements, designed to provide the firm with
reasonable assurance that its system of quality control is
operating effectively.
A. Monitoring
B. Inspection
C. Engagement quality control review
D. Supervision
84. The firm shall obtain written confirmation of compliance
with its policies and procedures on independence from all
firm personnel required to be independent by relevant ethical
requirements
A. At least annually
B. At least monthly
C. At least semi-annually
D. At the completion of each engagement

85. Which element of a system of quality control is addressed


by the establishment of policies and procedures designed to
provide the firm with reasonable assurance that it has
sufficient personnel with the competence, capabilities, and
commitment to ethical principles?
A. Monitoring
B. Leadership responsibilities for quality within the
firm
C. Human resources
D. Engagement performance
86. The firm shall establish policies and procedures designed
to provide it with reasonable assurance that the firm and its
personnel comply with relevant ethical requirements. The Code
of Ethics for Professional Accountants in the Philippines
establishes the fundamental principles of professional ethics
which include the following, except
A. Integrity
B. Objectivity
C. Relevance
D. Professional behavior
The Code of Ethics for Professional Accountants in the
Philippines establishes the fundamental principles of
professional ethics, which include:
a. Integrity;
b. Objectivity;
c. Professional competence and due care:
d. Confidentiality; and
e. Professional behavior.
87. The audit work performed by each assistant should be
reviewed by personnel of at least equal competence to
determine whether it was adequately performed and to evaluate
whether the
A. Firm's system of quality control has been maintained
at a high level.
B. Work performed and the results obtained have been
adequately documented.
C. Audit procedures performed are approved in the
professional standards.
D. Audit procedures performed are in accordance with
Philippine Standards on Auditing (PSAs).
The work performed by each assistant should be reviewed
to consider whether:
a) The work has been performed in accordance with the audit
program;
b) The work performed and the results obtained have been
adequately documented;
c) All significant audit matters have been resolved or are
reflected in audit conclusions;
d) The objectives of the audit procedures have been
achieved; and
e) The conclusions expressed are consistent with the results
of the work performed and support the audit opinion.
88. The nature, timing, and extent of an audit firm's quality
control policies and procedures depend on
The Nature Appropriate
The CPA of the CPA Cost-Benefit
Firm's Size Firm's Practice Considerations
A. Yes Yes No
B. Yes Yes Yes
C. No No No
D. Yes No Yes
The nature, timing, and extent of an audit firm's
quality control policies and procedures depend on a number of
factors such as the size and nature of its practice, its
geographic dispersion, its organization, and appropriate
cost/benefit considerations.
89. An audit firm should implement quality control policies and
procedures designed to ensure that all audits are conducted
in accordance with PSAs or relevant national standards or
practices. These policies and procedures should be
implemented
A. At the audit firm level only.
B. On individual audits only.
C. Either at the audit firm level or on individual
audits.
D. Both at the audit firm level and on individual
audits.
90. For audits of financial statements of listed entities, the
engagement partner should not issue the auditor's report
until the completion of the
A. Engagement Quality Control Review
B. Management Review
C. Engagement Team Review
D. Engagement Partner Review
Engagement quality control review, as
defined in the Standard, is "a process designed to provide
an objective evaluation, on or before the date of the report,
of the significant judgments the engagement team made and the
conclusions it reached in formulating the report." The
Standard provides further that and engagement quality control
review is for audits of financial statements of listed
entities, and those other engagements, if any, for which the
firm has determined an engagement quality control review is
required.
91. The following statements relate to the engagement partner's
responsibility to conduct timely reviews of the audit
documentation to be satisfied that sufficient appropriate
evidence has been obtained to support the conclusions reached
and for the auditor's report to be issued. Which is false?
A. The engagement partner's review of the audit
documentation allows significant matters to be
resolved on a timely basis to his/her satisfaction
before the auditor's report is issued.
B. The engagement partner should review all audit
documentation.
C. The engagement partner should document the extent
and timing of the reviews.
D. The reviews cover critical areas of judgment,
especially those relating to difficult or
contentious matters identified during the course of
the engagement, significant risks, and other areas
the engagement partner considers important.
The engagement partner is not required to
review all audit documentation.
92. The engagement partner should be satisfied that appropriate
procedures regarding the acceptance and continuance of client
relationships and specific audit engagements have been
followed, and that conclusions reached in this regard are
appropriate and have been documented. Acceptance and
continuance of client relationships and specific audit
engagements include considering:
I. The integrity of the principal owners, key
management, and those charged with governance of
the entity.
II. Whether the engagement team is competent to perform
the audit engagement and has the necessary time
and resources.
III. Whether the firm and the engagement team can comply
with ethical requirements.
A. I only
B. I and II only
C. II and III only
D. I, II, and III
93. The engagement partner should take responsibility for the
direction, supervision, and performance of the audit
engagement in compliance with professional standards and
regulatory and legal requirements, and for the auditor's
report that is issued to be appropriate in the circumstances.
Supervision includes the following except,
A. Tracking the progress of the audit engagement.
B. Addressing significant issues arising during the
audit engagement, considering their significance,
and modifying the planned approach appropriately.
C. Informing the members of the engagement team of their
responsibilities.
D. Identifying matters for consultation or
consideration by more experienced engagement team
members during the audit engagement.
The engagement partner directs the audit engagement by
informing the members of the engagement team of their
responsibilities; the nature of the entity's business; risk-
related issues; problems that may arise; and the detailed
approach to the performance of the engagement.
In addition to those activities described in answers A,
B, and D, supervision also includes considering the
capabilities and competence of individual members of the
engagement team, whether they have significant time to carry
out their work, whether they understand their instructions,
and whether the work is being carried out in accordance with
the planned approach to the audit engagement.
94. Who should take responsibility for the overall quality on
each audit engagement?
A. Engagement quality control reviewer
B. Engagement partner
C. Engagement team
D. CPA firm
PSA 220 provides that the engagement partner should take
responsibility for the overall quality on each audit
engagement to which that partner is assigned. As defined in
this Standard, the engagement partner is "the partner or the
person in the firm who is responsible for the audit engagement
and its performance, and for the auditor's report that is
issued on behalf of the firm, and who, where required, has
the appropriate authority from a professional, legal or
regulatory body,"
95. The implementation of quality control procedures that are
applicable to the individual audit engagement is the
responsibility

A. CPA firm
B. Engagement quality control reviewer
C. Engagement team
D. Expert contracted by the firm in connection with
the audit engagement

PSA 220 states that the engagement team should


implement quality control procedures that are
applicable to the individual audit engagement. The
engagement team is composed of all personnel (partners
and staff) performing an audit engagement, including
any experts contracted by the firm in connection with
that audit engagement.

A CPA firm has an obligation to establish a system of


quality control designed to provide it with reasonable
assurance that the firm and its personnel comply with
professional standards and regulatory and legal
requirements, and that the auditors' reports issued by
the firm or engagement partners are appropriate in the
circumstances. (PSQC l)

THE AUDITOR'S RESPONSIBILITIES RELATING TO FRAUD IN AN


AUDIT OF FINANCIAL STATEMENTS

96. Misstatements in the financial statements can arise from


fraud or error. The distinguishing factor between fraud and
error is whether the underlying action that results in the
misstatement of the financial statements is
A. Simple or complex
B. Intentional or unintentional
C. Voluntary or involuntary
D. Planned or unplanned
The term "error" refers to an unintentional
misstatement in financial statements. The term
"fraud" refers to an intentional act by one or more
individuals among management, those charged with
governance, employees, or third parties, involving
the use of deception to obtain an unjust or illegal
advantage.

97. "Error" includes


A. Engaging in complex transactions that are
structured to misrepresent the financial
position or financial performance of the entity.
B. Concealing, or not disclosing, facts that could
affect the amounts recorded in the financial
statements.
C. An incorrect accounting estimate arising from
oversight or misinterpretation of facts.
D. Intentional misapplication of accounting
policies relating to amounts, classification,
manner of presentation, or disclosure.

The term "error" refers to an unintentional


misstatement in financial statements, including the
omission of an amount or a disclosure, such as the
following:
A mistake in gathering or processing data from
which financial statements are prepared.
An incorrect accounting estimate arising from
oversight or misinterpretation of facts.
A mistake in the application of accounting
principles relating to measurement,
recognition, classification, presentation, or
disclosure.

98. Fraud involving one or more members of management or those


charged with governance is referred to as
A. Management fraud
B. Employee fraud
C. Fraudulent financial reporting
D. Misappropriation of assets
99. The auditor is concernned with fraud that causes a
material misstatement in the financial statements. There
are two types of intentional misstatements that are
relevant to the auditor: misstatements resulting from
fraudulent financial reporting and misstatements
resulting from
A. Management fraud
B. Employee fraud
C. Misappropriation of assets
D. Collusion within the entity or with third parties

100. Fraudulent financial reporting involves intentional


misstatements including omissions of amounts or
disclosures in financial statements to deceive financial
statement users. It may be accomplished in a number of
ways, including
A. Embezzling receipts.
B. Stealing physical assets or intellectual property.
C. Using an entities assets for personal use.
D. Manipulation, falsification, or alteration of
accounting records or supporting documentation
from which the financial statements are prepared

Fraudulent financial reporting may be accomplished by


the following:
Manipulation, falsification (including
forgery), or alteration of accounting
records or supporting documentation from
which the financial statements are prepared.

Misrepresentation in, or intentional


omission from, the financial statements of
events, transactions, or other siB1ificant
information.
Intentional misapplication of accounting
principles relating to amounts,
classification, manner of presentation, or
disclosure.

The answers in A, B, and C involve misappropriation of


assets.

101. Which of the following conditions are generally present


when misstatements due to fraud occur?
I. Incentive or pressure
II. Perceived opportunity
III. Rationalization
A. I and II only
B. II and only
C. I and III only
D. I, II, and III
Fraud involves the incentive or pressure to commit
fraud, a perceived opportunity to do so, and some
rationalization of the act.
102. Individuals may have an incentive or be under pressure to
commit fraud, or circumstances may provide an opportunity.
Also, certain individuals may have an attitude, character,
or set of values that allow them to rationalize fraud. The
auditor's concern about the risk of material misstatement is
least likely to be increased if management
A. Is interested in inappropriate means of minimizing
reported earnings for tax purposes.
B. Commits to unduly aggressive forecasts,
C. Operating and financing decisions are made by
numerous individuals.
D. Has an excessive interest in increasing the entitys
share price through the application of unduly
aggressive accounting practices.
103. Three conditions are generally present when fraud occurs.
Which of the following is not one of them?
A. Attitude or rationalization about the act of fraud.
B. Opportunity to commit fraud.
C. Professional skepticism about the likelihood of
fraud.
D. Incentive or pressure to commit fraud.

104. Which of the following is a required audit planning


procedure concerning potential fraud?
A. Consider whether estimates prepared and recorded by
management could indicate a biased reporting.
B. Consider the nature of journal entries, particularly
those made near the end of the reporting period.
C. Document the results of procedures used to address
the risk of fraud.
D. Conduct discussions among the members of the audit
team regarding the risks of material misstatement
due to fraud or error.

105. Which of the following is a false statement concerning


fraud?
A. Fraud generally involves incentive or pressure to
commit fraud, a perceived opportunity to do so, and
some rationalization of the act.
B. Two types of misstatements relevant to the auditor
include material misstatements arising from
fraudulent financial reporting and material
misstatements arising from misappropriation of
assets.
C. Fraud involves actions of management but excludes
the actions of employees or third parties.
D. An audit rarely involves the authentication of
document ; thus, fraud may go undetected by the
auditor.

106. Why is computer fraud often much more difficult to detect


than other types of fraud?
A. Perpetrators can commit a fraud and leave little or
no evidence.
B. Perpetrators usually only steal very small amounts
of money at a time, thus requiring a long period of
time to have elapsed before they are discovered.
C. Most computer criminals are older and are considered
to be more cunning when committing such a fraud.
D. Most perpetrators invest their illegal income rather
than spend it, thus concealing key evidence.

107. A classification of fraud where the perpetrator causes a


company to pay too much for ordered goods, or to pay for
goods never ordered is called
A. Payroll fraud
B. Disbursement fraud
C. Cash receipts fraud
D. Inventory fraud

108. In payroll fraud, funds can be stolen by


I. Paying a fictitious or ghost employee.
Il. Increasing pay rates without permission.
Ill. Keeping a real but terminated employee on the
payroll.
A. I and II only
B. I and III only
C. II and III only
D. I, II, and III
109. Stealing a master list of customers and selling it to a
competitor is an example of what classification of fraud?
A. Output theft
B. Data theft
C. Disbursement fraud
D. Cash receipt fraud

110. The primary responsibility for the prevention and


detection of fraud rests with
A. Those charged with governance of the entity.
B. Management of the entity.
C. Both those charged with governance of the
entity and management.
D. The auditor.

111. The following are examples of misappropriation of assets,


except

A. The treasurer diverts customer payments to his


personal due, concealing his actions by debiting an
expense account, thus overstating expenses.
B. An employee steals inventory and the "shrinkage" is
recorded in cost of goods sold.
C. An employee steals small tools from the company and
neglects to return them; the cost is reported as a
miscellaneous operating expense.
D. Company management changes inventory count tags and
overstates ending inventory, while understating
cost of goods sold.

Changing inventory count tags resulting in misstatement


of inventory and cost of goods sold is an example of
fraudulent financial reporting.

112. Which of the following statements best describes an


auditor's responsibility regarding misstatements?
A. An auditor should obtain reasonable assurance that
the financial statements taken as a whole are free
from material misstatement, whether caused by fraud
or error.
B. An auditor should obtain absolute assurance that
material misstatements in the financial statements
will be detected.
C. An auditor is responsible to detect material errors
but has no responsibility to detect material fraud
that is concealed through employee collusion or
management override of internal control.
D. An auditor's failure to detect a material
misstatement resulting from fraud is an indication
of noncompliance with the requirements of the
Philippine Standards on Auditing (PSAs).
PSA 240 states, "An audit conducted in accordance with
PSAs is designed to provide reasonable assurance that
the financial statements taken as a whole are free from
material misstatement, whether caused by fraud or
error."

113. When obtaining an understanding of the entity and its


environment, including its internal control, the auditor
may identify events or conditions that indicate an incentive
or pressure to commit fraud or provide an opportunity to
commit fraud. Such events or conditions are referred to as
A. Fraud conditions
B. Fraud risk factors
C. Fraudulent activities
D. Fraud environment

114. The following are examples of fraud risk factors relating


to misstatements arising from misappropriation of assets,
except
A. Recurring negative cash flows from operating
activities while reporting earnings and earnings
growth.
B. Inadequate physical safeguards over cash, investments,
inventory, or fixed assets.
C. Inadequate segregation of duties or independent
checks.
D. Adverse relationship the entity and employees with
access to cash or other assets susceptible to theft
created by recent changes made to employee
compensation or benefit plans.

Recurring negative cash flows while reporting earnings


and earnings growth is a fraud risk factor relating to
fraudulent financial reporting.

115. Opportunities to misappropriate assets increase when there


are
A. Known or anticipated future employee layoffs.
B. Promotions, compensation, or other rewards
inconsistent with expectations.
C. Recent or anticipated changes to employee
compensation or benefit plans.
D. Inventory items that are small in size, of high
value, or in high demand.
Fraud risk factors are classified according to the three
conditions generally present when fraud exists: (1)
incentives/pressures, (2) opportunities, and (3)
attitudes/rationalizations.

Opportunities to misappropriate assets increase when


there are:
Inventory items that are small in size, of high
value, or in high demand.
Large amounts of cash on hand or processed.
Easily convertible assets, such as bearer bonds,
computer chips, or diamonds.
Fixed assets which are small in size, marketable,
or lacking observable identification of
ownership.

Answers A, B, and C are incorrect because they may create


adverse relationships between the entity and employees
that may motivate them to misappropriate the entity's
assets.

116. Which of the following conditions or events may create


incentives/pressures to commit fraud?
A. Inadequate system of authorization and approval of
transactions.
B. Lack of mandatory vacations for employees performing
key control functions.
C. Excessive pressure on management or operating
personnel to meet financial targets established by
those charged with governance, including sales or
profitability incentive goals.
D. Inadequate access controls over automated records.

Answers A, B, and D are incorrect because they create


opportunities to commit fraud.

117. Because of the risk of material misstatement, an audit of


financial statements in accordance with PSAs should be
planned and performed with an attitude of
A. Impartial conservatism
B. Objective judgment
C. Independent integrity
D. Professional skepticism

As required by PSA 200, the auditor plans and performs


the audit with an attitude of professional skepticism
recognizing that circumstances may exist that cause the
financial statements to be materially misstated.
Professional skepticism includes a questioning -mind and
critical assessment of audit evidence.

118. Which of the following statements describes why an audit


that is properly planned and performed in accordance with
PSAs may not detect a material misstatement resulting from
fraud?
A. Fraud may involve carefully laid out plans of
concealment.
B. The auditor did not consider audit risk factors for
accounts having pervasive effects on the financial
statements.
C. An audit is designed to provide reasonable
assurance of detecting misstatements arising from
errors, but there is no similar responsibility
concerning material misstatements resulting from
fraud.
D. The risk of the auditor not detecting a material
misstatement resulting from employee fraud is
greater than for management fraud.
PSA 240 states, "The risk of not detecting a material
misstatement resulting from fraud is higher than the risk
of not detecting a material misstatement resulting from
error because fraud may involve sophisticated and
carefully organized schemes to conceal it, such as
forgery, deliberate failure to record transactions, or
intentional misrepresentations being made to the auditor.
Such attempts at concealment may be even more difficult to
detect when accompanied by collusion. Collusion may cause
the auditor to believe that evidence is persuasive when it
is, in fact, false."
119. When planning the audit, the auditor should make inquiries
of management. Such inquiries should address the following,
except
A. Management's assessment of the risk that the
financial statements may be misstated due to fraud.
B. Management's process for identifying and responding
to the risks of fraud in the entity.
C. Management's consideration of how an element of
unpredictability will be incorporated into the
nature, timing, and extent of the audit procedures
to be performed.
D. Management's communication, if any, to those charged
with governance regarding its processes for
identifying and responding to the risks of fraud in
the entity.

The consideration of how an element of unpredictability


will be incorporated into the nature, timing, and extent
of the audit procedures to be performed is a matter to
be discussed by the auditor with other members of the
audit team.

120. Which of the following circumstances most likely would cause


an auditor to consider whether material misstatements exist
in an entity's financial statements?
A. Those charged with governance exercise oversight of
management's processes for identifying and
responding to the risks of fraud in the entity and
the internal control that management has established
to mitigate these risks.
B. Significant, unusual, or highly complex
transactions, especially those close to an entity's
financial year-end that pose difficult "substance
over form" questions.
C. Operating profits making the threat of bankruptcy,
foreclosure, or hostile takeover remote.
Low vulnerability to changes in technology, product
obsolescence, or interest rates.
Significant, unusual, or highly complex year-end
transactions may provide opportunities to engage in
fraudulent financial reporting.
121. Which of the following characteristics most likely would
heighten an auditor's concern about the risk of material
misstatement arising from fraudulent financial reporting?
A. Excessive interest by management in increasing stock
price or earnings trend through aggressive
accounting practices.
B. Effective accounting and internal control systems.
C. Low turnover of senior management, legal counsel, or
those charged with governance.
D. Management is dominated by a single person or a small
group with compensating controls such as effective
oversight by those charged with governance.
122. When the auditor encounters circumstances that may
indicate that there is a material misstatement in the
financial statements resulting from fraud or error, the
auditor should perform procedures to determine whether the
financial statements are materially misstated. The nature,
timing, and extent of the procedures to be performed depend
on the auditor's judgment as to the
A B C
D
Type of fraud or error indicated No Yes Yes No
Likelihood of occurrence Yes No Yes No
Likelihood that a particular
type of fraud or error could
have a material effect
on the financial statements Yes No Yes No
123. When the auditor identifies a misstatement in the financial
statements, the auditor should consider whether such a
misstatement may be indicative of fraud and if there is such
an indication, the auditor should
A. Consider the implications of the misstatement in
relation to other aspects of the audit.
B. Withdraw from the
C. Communicate information to regulatory and
enforcement authorities.
D. Report the matter to the person or persons who made
the audit appointment.

PSA 240 requires the auditor to consider the implications


of the misstatement in relation to other aspects of the
audit, particularly the reliability of management
representations.

124. PSA 230 (Audit Documentation) requires the auditor to


document matters which are important in providing evidence
to support the audit opinion, and states that working papers
include the auditor's reasoning all significant matters
which require the auditor's judgment, together with the
auditor's conclusion thereon. Which of the following should
be documented by the auditor?
A. Fraud risk factors identified as being present during
auditor's risk assessment
B. Auditor's to fraud risk factors
C. Both fraud risk factors identified as being present
during the auditor's risk assessment and the auditor's
response to any such factors.
D. The standard does require documentation of the
identified fraud risk factors and the auditor's
responses to them,
PSA 240 states that because of the importance of fraud
risk factors in the assessment of the inherent or control
risk of material misstatement, the auditor documents
fraud risk factors identified and the response considered
appropriate by the auditor.
125. Because of the nature of fraud and the difficulties
encountered by auditors in detecting material misstatements
in the financial statements resulting from fraud, the
auditor should obtain written representations from
management. The following should be confirmed by management
in its written representations, except
A. It is not responsible for the implementation
and operations of internal control that is designed
to prevent and detect fraud.
B. It has disclosed to the auditor its knowledge
of any allegations of fraud or suspected fraud,
affecting the entity's financial statements
communicated by employees, former employees,
analysts, regulators, or others.
C. It has disclosed to the auditor its knowledge of
fraud or suspected fraud affecting the entity.
D. It has disclosed to the auditor the results of its
assessment of the risk that the financial statements
may be materially misstated as a result of fraud.

The standard states that in addition to acknowledging its


responsibility for the financial statements, it is
important that, irrespective of the size of the entity,
management acknowledges its responsibility for internal
control designed and implemented to prevent and detect
fraud.

126. Which of the following statements concerning the auditor's


responsibility to detect conditions relating to financial
stress of employees or adverse relationships between a
company and its employees is correct?
A. The auditor is required to plan the audit to detect
these conditions whenever they may result in
misstatements.
B. The auditor is required to plan the audit to detect
these conditions on all audits.
C. These conditions relate to fraudulent financial
reporting, and an auditor is required to plan the
audit to detect these conditions when the client is
exposed to a risk of misappropriation of assets
D. The auditor is not required to plan the audit to
discover these conditions, but should consider them
if he/she becomes aware of them during the audit.
The financial stress of employees or adverse
relationships between a company and its employees are
conditions that indicate an incentive or pressure to
commit fraud. PSA 240 states that using the auditor's
knowledge of the business, the auditor may identify
events or conditions that provide an opportunity, a
motive, or a means to commit fraud, or indicate that
fraud may already have occurred. Such events or
conditions are referred to as "fraud risk factors."

In designing the substantive procedures, the auditor


should address the fraud risk factors that he/she has
identified as being present. The standard, therefore,
does not specifically require the auditor to plan the
audit to discover fraud risk factors, but should
consider them if he/she becomes aware of them during
the audit.

127. The following statements relate to communication of


misstatements resulting from fraud to management and to
those charged with governance. Which is false?
A. The auditor need not bring to the attention of
those charged with governance any material
weaknesses in internal control related to the
prevention and detection of fraud.
B. If the auditor has identified a fraud, whether or
not it re suits in a material misstatement in the
financial statements, the auditor should
communicate these matters to the appropriate level
of management on a timely basis, and consider the
need to report such matters to those charged with
governance.
C. If the auditor has obtained evidence that indicates
that fraud may exist (even if the potential effect
on the financial statements would not be
material), the auditor should communicate these
matters to the appropriate level of management on
a timely basis, and consider the need to report
such matters to those charged with governance.
D. The auditor's communication with those charged
with governance may be made orally or in writing.
The standard states that the auditor should make
those charged with governance and management aware, as
soon as practicable, and at the appropriate level of
responsibility, of material weaknesses in the design
and implementation of internal control to prevent and
detect fraud which may have come to the auditor's
attention.

CONSIDERATION OF LAWS AND REGULATIONS IN AN AUDIT OF FlNANCIAL


STATEMENTS

128. As used in PSA 250, this term refers to acts of omission


or commission by the entity being audited, either
intentional or unintentional, which are contrary to
prevailing laws or regulations.
A. Noncompliance
B. Illegal acts
C. Deplorable acts
D. Unforgivable acts
129. According to PSA 250, the term "noncompliance" as used
in the standard refers to acts of omission or
commission by the entity being audited, either
intentional or unintentional, which are contrary to
the prevailing laws or regulations. Such acts do not
include
A. Transactions entered into by the entity.
B. Transactions entered into in the name of the
entity.
C. Transactions entered into on the entity's
behalf by its management or employees.
D. Personal misconduct (unrelated to the entity's
business activities) by the entity's management
or employees.
130. The responsibility for the prevention and detection of
noncompliance rests with
A. The auditor
B. Management
C. The auditor's lawyer
D. The client's lawyer

PSA 250 states that it is management's responsibility


to ensure that the entity's operations are conducted in
accordance with laws and regulations.

131. Which of the following statements best describes why the


auditor's examination cannot reasonably be expected to
bring all acts of noncompliance with existing laws and
regulations by the client to the auditor's attention?
A. Acts of noncompliance by clients often relates to
accounting, aspects rather than operating aspects.
B. Noncompliance may involve conduct designed to
conceal it, such as collusion, forgery, deliberate
failure to record transactions, senior management
override of controls, or intentional
misrepresentations being made to the auditor.
C. Noncompliance may be perpetrated by the only person
in the client's organization with access to both
assets and the accounting records.
D. The client's internal control may be so strong that
the auditor performs only minimal substantive
testing.

132. PSA 250 states that in order to plan the audit, the auditor
should obtain a general understanding of the legal and
regulatory framework applicable to the entity and the
industry and how the entity is complying with that
framework. To obtain this understanding, the following
procedures would ordinarily be considered by the auditor,
except
A. Use the existing understanding of the entity's
industry, regulatory, and other external factors.
B. Inquire of management concerning the entity's
policies and procedures regarding compliance with
laws and regulations.
C. Inquire of management to the laws and regulations
that may be expected to have a fundamental effect
on the operations of the entity.
D. Inspect correspondence with relevant licensing or
regulatory authorities.
Inspecting correspondence with the relevant licensing
or regulatory authorities is a procedure to identify
instances of noncompliance with laws and regulations.

133. When the auditor becomes aware of information concerning a


possible instance of noncompliance, the auditor should
obtain an understanding of A B
C D
The nature of the act No Yes Yes No
The circumstances in which it has
occurred No Yes No Yes
Sufficient other information to
evaluate the possible effect
on
financial statements Yes Yes No No
134. Which of the following statements is incorrect concerning
reporting of noncompliance?
A. The auditor, as soon as practicable, either
communicate with those charged with governance, or
obtain evidence that they are appropriately
informed, regarding noncompliance that comes to the
auditor's attention.
B. If the auditor suspects that members of senior
management, including members of the board of
directors, are involved in noncompliance, the
auditor should report the matter to the next higher
level of authority at the entity, if it exists, such
as an audit committee or a supervisory board.
C. The auditor should, as soon as practicable,
communicate with those charged with governance
regarding noncompliance, including matters that are
clearly inconsequential or trivial.
D. If in the auditors judgement, the noncompliance is
believed to be intentional and material, the auditor
should communicate the finding without delay.
135. If the auditor concludes that the noncompliance has a
material effect on the financial statements, and has not
been properly reflected in the financial statements, the
auditor should express
A. A qualified or an adverse opinion
B. A qualified opinion or a disclaimer of opinion
C. A disclaimer of opinion
D. A qualified opinion
136. If the auditor is precluded by the entity from obtaining
sufficient appropriate audit evidence to evaluate whether
noncompliance that may be material to the financial
statements, has, or is likely to have, occurred, the auditor
should express
A. A qualified opinion or an adverse opinion
B. A qualified opinion or a disclaimer of opinion
C. An adverse opinion
D. An adverse opinion or a disclaimer of opinion
137. Under which of the circumstances below would the auditor
conclude that withdrawal from the engagement is necessary?
A. The auditor concludes that the noncompliance has a
material effect on the financial statements and has
not been properly reflected in the financial
statements.
B. The auditor is precluded by the entity from obtaining
sufficient appropriate audit evidence to evaluate
whether noncompliance that may be material to the
financial statements, has, or is likely to have,
occurred.
C. The auditor is unable to determine whether
noncompliance has occurred because of limitations
imposed by the circumstances rather than by the
entity.
D. The entity does not take the remedial action that
the auditor considers necessary in the
circumstances.
According to PSA 250, the auditor may consider that
withdrawal from the engagement is necessary when the
entity does not take the remedial action that the auditor
considers necessary in the circumstances, even when the
noncompliance is not material to the financial
statements.

COMMUNICATION WITH THOSE CHARGED WITH GOVERNANCE


138. Under PSA 260, this term is used to describe the role of
persons entrusted with the supervision, control, and
direction of an entity.
A. Oversight
B. Governance
C. Direction
D. Control
PSA 260 states that for the purposes of this PSA,
"governance" is the term used to describe the role of
persons entrusted with the supervision, control, and
direction of an entity. It states further that those
charged with governance ordinarily are accountable for
ensuring that the entity achieves its objectives with
regard to reliability of financial reporting,
effectiveness and efficiency of operations, compliance
with applicable laws, and reporting to interested
parties.
139. According to PSA 260, those matters that arise from the
audit of financial statements and, in the opinion of the
auditor, are both important and relevant to those charged
with governance in overseeing the financial reporting and
disclosure process are catted
A. Audit matters of governance interest
B. Significant audit matters
C. Auditor's findings
D. Material misstatements in the financial
statements
140. Which the following statements relating to communication of
audit matters of governance interest is incorrect?
A. Audit matters of governance interest include only
those matters that have come to the attention of the
auditor as a result of the performance of the audit.
B. In an audit in accordance with PSAs, the auditor
should design audit procedures for the specific
purpose of identifying matters of governance
interest.
C. The auditor should identify relevant persons who are
charged with governance and with whom audit matters
of governance interest are to be communicated.
D. The auditor's communications with those charged with
governance may be made orally or in writing.
PSA 260 states that audit matters of governance interest
include only those matters that have come to the attention
of the auditor as a result of the performance of the audit.
The auditor is not required, in an audit in accordance with
PSAs, to design audit procedures for the specific purpose
of identifying matters of governance interest.

141. Audit matters of governance interest to be


communicated to those charged with governance ordinarily
include

I. Audit adjustments, whether or not recorded by the


entity that have, or could have, a material effect
on its financial statements.
II. Expected modifications to the auditor's report.
III. Material uncertainties related to events and
conditions that may cast significant doubt on the
entity's ability to continue as a going concern.
A. I only
B. II and III only

C. l and IIII only


D. I, II, and III
In addition to those described in answers A, B, and C,
audit matters of governance interest include the
following:
o The general approach and overall scope of the audit,
including any expected limitations thereon, or any
additional requirements.
o The selection of, or changes in, significant
accounting policies and practices that have, or could
have, a material effect on the entity's financial
statements.
o The potential effect on the financial statements of
any material risks and exposures, such as pending
litigation, that are required to be disclosed in the
financial statements.
o Disagreements with management about matters that,
individually or in aggregate, could be significant to
the entity's financial statements or the auditor's
report. These communications include consideration of
whether the matter has, or has not, been resolved and
the significance of the matter.
o Other matters warranting attention by those charged
with governance, such as material weaknesses in
internal control, questions regarding management
integrity, and fraud involving management.
o Any other matters agreed upon in the terms of the
audit engagement.
142. PSA 260 requires the auditor to determine the
relevant persons who are charged with governance and with
whom audit matters of governance interest are communicated,
for corporations covered by the SEC Code of Corporate
Governance, which of the following is primarily responsible
for corporate governance?
A. President
B. Controller
C. Board of Directors
D. Management
The Code of Corporate Governance promulgated by the
Securities and Exchange Commission (SEC) states, "The
Board of Directors is primarily responsible for the
governance of the corporation."
143. The auditor shall communicate with those charged with
governance his/her responsibilities in relation to the
audit of the entity's financial statements, including that

I.The auditor is responsible for forming and


expressing an opinion on the financial statements.
Il. The audit of the financial statements does not
relieve management or those charged with
governance of their responsibilities.
A. I only
B. II only
C. Neither I nor II
D. Both I and II

144. Which of the following matters will an auditor most likely


communicate to those charged with governance?
A. The level of responsibility assumed by management for
the preparation of the financial statements.
B. The effects of significant accounting policies
adopted by management in emerging areas for which
there is no authoritative guidance.
C. A list of negative trends that may lead to working
capital deficiencies and adverse financial ratios.
D. Difficulties encountered in achieving a satisfactory
response rate from the entity's customers in
confirming accounts receivables.

145. Which of the following matters is an auditor required to


communicate to those charged with governance?
I. Disagreements with management about matters significant
to the entity's financial statements that have been
satisfactorily resolved.
II. Material weaknesses in internal control.
A. I only Both
B. II only
C. I and II
D. Neither I nor II

TRUE OR FALSE

1. Familiarity threat is a threat that a CPA will promote a


clients or employers position to the point that his/her
objectivity is compromised.

2. Safeguards fall into two broad categories: safeguards in the


work environment and firm-wide safeguards.

3. The principle of professional behavior imposes an obligation


on all CPAs to comply with relevant laws and regulations and
avoid any action that may discredit the profession.

4. When identified threats are not an acceptable level, a CPA


may rely solely on safeguards that his/her client has
implemented.

5. A primary purpose for establishing a code of professional


ethics is to demonstrate acceptance of responsibility to
the interests of those served by the profession.

6. Safeguards implemented by the firm, including policies and


procedures to implement professional and regulatory
requirements, may be applied to mitigate or eliminate
threats to independence and other ethical principles.

7. The auditor must be independent of the audit client unless


the lack of independence does not influence his or her
professional judgment.

8. With regard to detecting fraud, auditing standards require


auditors to issue an unmodified opinion only when the
auditor is satisfied that no instances of fraud have
occurred.

9. Due professional care requires auditors to plan and perform


their duties with the skill and care that is commonly
expected of accounting professionals.

10. Fraud is either an intentional or unintentional misstatement


of the financial statements, depending on the materiality
and consistency.

11. A factor that relates to opportunities to commit fraudulent


financial reporting is lack of controls relating to the
calculation and approval of accounting estimates.

12. When determining whether independence is impaired because of


an ownership interest in a client company, materiality will
affect ownership only for direct ownership.

13. A direct financial interest violates independence when close


relatives such as a brother, sister, or in-laws are
employed by the client.

14. If an entity asks a CPA to perform a review engagement, and


the CPA has an immaterial direct financial interest in the
entity, the CPA is not independent and, therefore, may not
issue a review report.
KEY ANSWERS 41. C 83. A 125. A
42. B 84. C 126. D
1. B 43. C 85. C 127. A
2. B 44. B 86. B 128. A
3. D 45. D 87. B 129. D
4. B 46. A 88. D 130. B
5. C 47. D 89. A 131. B
6. A 48. C 90. B 132. D
7. A 49. C 91. D 133. B
8. D 50. D 92. C 134. C
9. D 51. A 93. B 135. A
10. A 52. B 94. C 136. B
11. A 53. A 95. B 137. D
12. A 54. B 96. C 138. B
13. A 55. A 97. A 139. A
14. C 56. D 98. C 140. B
15. A 57. C 99. D 141. D
16. A 58. A 100. D 142. C
17. D 59. C 101. C 143. D
18. D 60. B 102. C 144. B
19. B 61. A 103. D 145. C
20. A 62. B 104. C
21. B 63. A 105. A
22. C 64. C 106. A TRUE OR FALSE
23. C 65. D 107. B
24. A 66. B 108. D
25. C 67. A 109. B 1. False
26. A 68. C 110. C 2. False
27. C 69. C 111. D 3. True
28. D 70. A 112. A 4. False
29. D 71. B 113. B 5. True
30. A 72. C 114. A 6. True
31. D 73. C 115. D 7. False
32. B 74. B 116. C 8. False
33. D 75. C 117. D 9. True
34. D 76. B 118. A 10. False
35. C 77. B 119. C 11. True
36. C 78. C 120. B 12. False
37. C 79. B 121. A 13. False
38. A 80. A 122. C 14. True
39. A 81. D 123. A
40. C 82. A 124. C
CHAPTER 4
The Financial Statement Audit: Client Acceptance and Planning

1. In assessing whether to accept a client for an audit


engagement, an auditor should consider the
I. Clients business risk
II. Auditors business risk
A. I only
B. II only
C. Both I and II
D. Neither I nor II

2. Which of the following factors most likely would cause an


auditor to decline a new audit engagement?

A. Conducting that the entitys engagement probably lacks


integrity.
B. An inability to perform preliminary analytical procedures
before assessing control risk.
C. An inadequate understanding of the entitys internal control.
D. The close proximity to the end of the entitys reporting
period.

PSQC 1 requires a firm (including sole practitioner) to establish


policies and procedures for the acceptance and continuance of
client relationships and specific engagements, designed to
provide reasonable assurance that it will only accept
engagements if it:
1.) Is competent to perform the engagement;
2.) Can comply with relevant ethical requirements; and
3.) Has considered the integrity of the client, and does
not have information to conclude that the entitys management
lacks integrity.
Answer B is incorrect because the auditor performs analytical
procedures after accepting engagement.
Answer C is incorrect because the auditor obtains an
understanding of the entitys internal control system subsequent
to the acceptance of the engagement.
Answer D is incorrect because an auditor may accept an engagement
near or after the end of the reporting period.

3. Before accepting an engagement to audit a new client, an


auditor is required to
A. Obtain a copy of the clients financial statements.
B. Prepare a memorandum setting forth the staffing requirements
and documenting the preliminary audit plan.
C. Make of the predecessor auditor after obtaining the consent
of the prospective client.
D. Discuss the management representation letter with the
clients audit committee.

PSA 300 (Planning an Audit of Financial Statements) states that


the auditor shall undertake the following activities prior to
an initial audit:
1.) Performing procedures required by PSA 220 (Quality
Control for an Audit of Financial Statements) regarding the
acceptance of the client relationship and the specific audit
engagement; and
2.) Communicating with he predecessor auditor, where there
has been a change of auditors, in compliance with relevant
ethical requirements.
Answer A is incorrect because the entitys annual financial
statements will be prepared after the end of its reporting period
and the auditors acceptance of the engagement will likely be
prior to that time.
Answers B and D are incorrect because the procedures described
will be performed only after the acceptance of engagement.

4. Which of the following conditions most likely would pose the


greatest risk in accepting a new audit engagement?
A. There will be a client-imposed scope limitation.
B. The clients financial reporting system has been in place for
10 years.
C. The firm will have to hire an expert in one audit area.
D. Staff will need to be rescheduled to cover this new client.

According to PSA 210 (Agreeing the Terms of Audit Engagements),


the auditor shall not accept the engagement if management or
those charged with governance impose the limitation on the scope
of the auditors work in the terms of a proposed audit engagement
such that the auditor believes the limitation will result in the
auditor disclaiming an opinion on the financial statements.

5. Which of the following circumstances would permit an


independent auditor to accept an engagement after the end of the
reporting period?
A. Expectation of the operating effectiveness of controls.
B. Issuance of a disclaimer opinion as a result of inability to
conduct certain tests required by PSAs due to the timing of
acceptance of the engagement.
C. Remedy the limitations resulting from accepting the
engagement after the end of the reporting period, such as those
relating to the existence of physical inventory.
D. Receipt of an assertion from the predecessor auditor that the
entity will be able to continue as a going concern.
Prior to accepting apropos audit engagement subsequent to the
end of the entitys reporting period, the auditor should
determine whether the circumstances permit an audit with
accordance with PSAs and expression of an unmodified opinion.
Otherwise, the auditor should discuss with the prospective
client the possibility of rendering a qualified opinion or a
disclaimer of opinion. However in some cases, the auditor may
remedy the audit limitations uch as by observing another physical
count of inventories.

Answer A is incorrect because sufficient appropriate evidence


regarding the operating effectiveness of the entity internal
control during the year should be available after the year-end.
Answer B is incorrect because a disclaimer of opinion is
appropriate only if the auditor cannot resolve an issue by
performing alternative procedures.
Answer D is incorrect because there is no need to obtain
representations regarding the prospective clients ability to
continue as going concern from the predecessor auditor.

6. In an audit based on Philippine Standards of Auditing (PSAs),


a successor auditor would normally become satisfied with opening
balances by
A. Performing analytic review procedures.
B. Reviewing the predecessors working paper.
C. Auditing the previous years working papers.
D. Interviewing client personnel.

7. A predecessor withdrew from the engagement after discovering


that the clients financial statements are materially misstated
that it would not revise. If asked by the successor auditor
about the termination of the engagement, the predecessor should

A. Suggest that the successor auditor should obtain the clients


consent to discuss the reasons.
B. Indicate that there was a misunderstanding.
C. State that the audit revealed material misstatement that the
client would not revise.
D. Suggest that the successor auditor ask the client.

8. Which of the following is not correct regarding the


communications between successor/incoming and
predecessor/previous auditors?
A. The burden of initiating the communication rests with
predecessor auditor.
B. The burden of initiating the communication rests with
successor auditor.
C. The predecessor auditor may choose to a limited response to
a successor auditor.
D. The predecessor auditor must received his/her former clients
permission prior to disclosing client information to the
auditor.
9. The auditor may accept or continue an audit engagement only
when the basis upon which it is to be performed has been agreed,
through

I. Establishing whether the preconditions for audit are present.


II. Confirming that there is a common understanding between the
auditor and management and, where appropriate those charge with
governance of the terms of the audit engagement.
A. I only
B. II only
C. Both I and II
D. Neither I nor II

As defined in PSA 210 (Agreeing the Terms of Audit Engagements),


preconditions for an audit refers to:
a.) The use of management of an acceptable financial
reporting framework in the preparation of the financial
statements; and
b.) The agreement of management and, where appropriate
those charged with governance to the premise of which
audit is conducted.

The auditor establishes whether the preconditions for audit are


present by:
1.) Determining whether the financial reporting
framework to be applied in preparation of financial
statements is acceptable; and
2.) Obtaining managements agreement that it
acknowledges and understands its responsibilities that are
fundamental to the conduct of an audit n accordance with
PSAs.

10. An audit is conducted on the premise that management and,


where appropriate, those charged with governance have
acknowledge and understand that they have responsibilities that
are fundamental to the conduct of an audit in accordance with
PSAs. Which of the following is not one of those
responsibilities?
A. The preparation of financial statements with accordance with
relevant pronouncements issued by the AASC.
B. The establishment and maintenance of an adequate internal
control system that is necessary to enable the preparation of
financial statements that is free from material misstatement,
whether due to fraud or error.
C. To provide the auditor with access to all information that
is relevant to the preparation of the financial statements such
as records, documentation and other matters.
D. To provide the auditor with unrestricted access to persons
within the entity from which the auditor determines it necessary
to obtain audit evidence.

Management is responsible for the preparation of the financial


statement in accordance with the applicable financial reporting
framework, including where relevant their fair presentation.
11. The auditor shall agree the terms of the audit engagement
with management or those charged with governance, as
appropriate. The agreed terms shall be recorded in a/an
A. Engagement letter
B. Letter of audit inquiry
C. Management representation letter
D. Confirmation letter

The standard states that it is in the interest of both the entity


and the auditor that the auditor sends an audit engagement letter
before the commencement of the audit to help avoid
misunderstandings with respect to the audit.
The engagement letter shall include:
a.) The objective and scope of the audit of the financial
statement;
b.) The responsibilities of the auditor;
c.) The responsibilities of the management;
d.) Identification of applicable financial reporting
framework for the preparation of financial statements;
and
e.) Reference to the expected form and content of any
reports to be issued by the auditor and a statement that
there may be circumstances in which a report may differ
from its expected form and content.

12. The following matters are generally included in the auditor


engagement letter, except
A. The factors to be considered in determining the overall
materiality.
B. The fact that because of the test nature and other inherent
limitations of an audit, together with the inherent limitations
of internal control, there is an unavoidable risk that even some
material misstatements may remain undiscovered.
C. The scope of the audit
D. Managements responsibility for the financial statements.

13. The following are usually included in an auditors engagement


letter, except
A. List of audit procedures to be used in inventory observation.
B. The financial statements are the responsibility of the
companys management.
C. A reference to PFRS.
D. A reference to PSAs.

14. Which of the following statements would least likely appear


an auditors engagement letter?

A. Our audit will be made with the objective of our expressing


an opinion on the financial statements.
B. We remind you that the responsibility for the preparation of
financial statements including adequate disclosure is that of
the management of the entity.
C. After performing our preliminary analytical procedures, we
will discuss with you the other procedures we consider necessary
to complete the engagement.
D. Our fees, which will be billed as work progresses, are based
on the time required by individuals assigned to the engagement
plus out-of-the-pocket expenses.

15. An auditor is required to establish an understanding with a


client regarding the services to be performed for each
engagement. This understanding generally includes

A. The auditors responsibility to apply the concept of


materiality in planning and performing the audit.
B. Managements responsibility for providing the auditor with
an assessment of the risk of material misstatement due to fraud.
C. The auditors responsibility for ensuring that those charged
with governance are aware of any significant deficiencies in
internal control that may come to auditors attention.
D. Managements responsibility for errors and the illegal
activities of employees that may cause material misstatement.

An auditors engagement letter shall include, among the other


things, the auditors responsibility to communicate to those
charged with governance of the client significant internal
control deficiencies that may be discovered during the audit.

Answer A is incorrect because determining materiality to


planning and performing an audit is a responsibility that the
auditor is not required to share with the client.
Answer B is incorrect because the auditor assesses the risk of
material misstatement.
Answer D is incorrect because management is not responsible for
the errors and illegal activities of employees.
16. An auditors engagement letter most likely will include
A. A request for permission to contact the clients lawyer for
assistance in identifying litigation, claims and assessments.
B. A reminder that management is responsible for illegal acts
committed by employees.
C. The auditor preliminary assessment of the risk factors
relating to misstatements arising from fraudulent financial
reporting.
D. Managements acknowledgement of its responsibility for such
internal control as it determines is necessary to enable the
preparation of financial statements that are free from material
misstatements.
Among the matters addressed in an engagement letter is
managements responsibility for:
a.) The preparation and fair presentation of the
financial statements in accordance with Philippine
Financial Reporting Standards.
b.) Such internal control as it determines is necessary
to enable the preparation of financial statements that are
free from material misstatements, whether due to fraud or
error.

Answer A is incorrect because management is responsible for


adopting policies and procedures to identify, evaluate and
account for litigation, claims and assessment.
Answer B is incorrect because management is responsible for
ensuring that the entitys operation are conducted accordance
with laws and regulations. However, it is not responsible for
the illegal acts of employees that are unrelated to the entitys
business activity.
Answer C in incorrect because an auditor submit an engagement
before the commencement of an audit. Prior to performing
procedures, the auditor does not assess the risk factors relating
to misstatements that may arise from fraudulent financial
reporting.
17. The auditor of a parent entity is also the auditor of its
component. Which of the following factors may influence the
auditors decision whether to send a separate engagement letter
to entitys component?
A. Whether a separate auditors report is to be issued on the
component.
B. The components management does not accept its
responsibilities that are fundamental to the conduct of an audit.
C. The financial reporting framework use by the component is
unacceptable.
D. The preconditions for an audit of the component financial
statement are not present.

The following factors may influence the auditors decision


whether to send a separate audit engagement letter to the
component:

1.) Who appoints he component auditor;


2.) Whether a separate auditors report is to be issued on the
component;
3.) Legal requirements in relation to audit appointments;
4.) Degree of ownership by parent; and
5.) Degree of independence of the component management from
the parent entity.

18. On recurring audits, the auditor may decide not to send a


new engagement letter to each period. Which of the following
factors may make it appropriate to send a new engagement
letter?
A B C D
Any revised or special terms of
Yes Yes
the audit engagement
A recent change of senior
Yes
management
A change in legal or regulatory
Yes Yes Yes Yes
requirements
A significant change in nature or
Yes Yes Yes Yes
size of the entitys business

According to the standard, the following factors may make it


appropriate to revise the terms of the audit engagement or to
remind the entity of existing terms:

Any indication that the entity misunderstands the


objective and scope of the audit.
Any revised or special terms of the audit
A recent change in senior management.
A significant change of ownership.
A significant change in nature or size of the entity
business.
A change on legal and regulatory requirements.
A change in the financial reporting framework adopted in
the preparation of the financial statements.

19. The auditor shall not agree to a request from the entity to
change the terms of the audit engagement or to change the audit
engagement to an engagement that conveys a lower level of
assurance when there is no reasonable justification for doing
so. Which of the following may be considered reasonable
justifications for the change in audit engagement?
I. A change in circumstance affecting the need for the service.
II. A misunderstanding as to the nature of an audit as originally
requested.
III. A restriction on the scope of the engagement, whether
imposed by management or caused by other circumstances.
A. I and II only
B. I and III only
C. II and III only
D. I, II and III

PSA 210 states that a change in circumstances that affects the


entitys requirements or a misunderstanding concerning the
nature of the service originally requested may be considered
reasonable basis for requesting a change in the audit engagement.
A restriction on the scope of the audit would not be considered
a reasonable justification for change. For example, a change
would not be considered reasonable if the auditor is unable to
obtain sufficient appropriate audit evidence regarding
receivable and the client requests that the audit engagement be
changed into a review engagement to avoid a qualified opinion
or a disclaimer of opinion.

20. Before the completion of the audit engagement, an auditor


is requested to change the engagement to one that provides a
lower level of assurance. If the auditor concludes that there
is a reasonable justification for the change of engagement, the
report to be issued would
A. Be that appropriate for the revised terms of engagement.
B. Include reference to the original engagement.
C. Include reference to any procedures that may have been
performed in the original engagement.
D. Not include reference to any procedures that may have been
performed, particularly when the new engagement is to undertake
agreed-upon procedures.

PSA 210 states that if the auditor concludes that there is a


reasonable justification to change the engagement and if the
audit work performed complies with the PSAs applicable to the
change engagement, the report issued would be that appropriate
for the revised terms of engagement. In order avoid confusing
the reader, the report would not include the reference to:
a.) The original engagement; or
b.) Any procedures that may have been performed in the
original engagement, except where the engagement is
changed to an engagement to undertake agreed-upon
procedures. And thus, reference to the procedures
performed is normal part of the report.

21. If the auditor is unable to agree to a change of the


engagement and is not permitted to continue the original
engagement, the auditor should
A. Insist on continuing the original engagement.
B. Express a qualified opinion.
C. Express an adverse opinion.
D. Withdraw from the engagement.

Under PSA 210, if the auditor is unable to agree to a change of


the engagement and is not permitted to continue the original
engagement, the auditor should withdraw and considered whether
there is any obligation, either contractual or otherwise, to
report to other parties, such as the board of directors or
shareholders, the circumstances necessitating the withdrawal.
22. Planning an audit involves
I. Establishing the overall audit strategy for the engagement.
II. Developing an audit plan.
A. I only
B. II only
C. Both I and II
D. Neither I nor II

23. Which of the following activities should be performed by the


auditor at the beginning f the current audit engagement?
I. Perform procedures regarding th continuance of the client
relationship and the specific audit engagement.
II. Evaluate compliance with relevant ethical requirements,
including independence.
III. Establish an understanding of the terms of the engagement.
A. I and II only
B. II and III only
C. I and III only
D. I, II and III

24. Initial audit planning involves the following matters,


except
A. Identify the clients reason for the engagement.
B. Schedule engagement staff and auditors experts.
C. Develop an overall audit strategy.
D. Bequest that blank balances be confirmed.

25. Adequate planning helps to ensure that


A B C D
Appropriate attention is devoted to
No Yes Yes No
important areas of the audit
Potential problems are identified and
Yes Yes No No
resolved on a timely basis
The audit engagement is properly
Yes Yes No No
organized and managed

The benefit of adequate planning includes the following:

Helping the auditor to devote appropriate attention to


important areas of the audit.
Helping the auditor to identify and resolve potential
problems on a timely basis.
Helping the auditor properly organize and manage the audit
engagement so that it is performed in an effective and
efficient manner.
Assisting in the selection of engagement team members with
appropriate levels of capabilities and competence to
respond to anticipated risk, and the proper assignment of
work to them.
Facilitating the direction and supervision of engagement
team members and the review of their work.
Assisting, where applicable, in coordination of work done
by auditors of components and experts.

26. Which of the following statements concerning audit planning


is incorrect?
A. Planning is a discrete phase of an audit.
B. Planning is a continual and iterative process.
C. In recurring audit, planning often begins shortly after (or
in connection with) the completion of the previous audit and
continuous until the completion of the current audit engagement.
D. In planning audit, the auditor considers the timing of certain
planning activities and audit procedures that are to be completed
prior to the performance of further audit procedures.

The standard states, Planning is not a discrete phase of an


audit but rather a continual and iterative process that begins
shortly after (or in connection with) the completion of the
previous audit and continuous until the completion of the current
audit engagement. Planning, however, includes consideration of
a timing of certain activities and audit procedures that need
to be completed prior to performance of further audit
procedures.

27. In performing an audit of financial statements, the auditor


should obtain a sufficient knowledge of a clients business and
industry to
A. Develop an attitude of professional skepticism concerning
managements financial statements assertions.
B. Make constructive suggestions concerning improvements to the
clients internal control.
C. Evaluate whether the aggregation of known misstatements
causes the financial statements taken as a whole to be materially
misstated.
D. Understand the events and transactions that may have an effect
on the clients financial statements.

The auditor is required to have or obtain a sufficient knowledge


of the clients business and industry to identify and understand
the events, transactions, and practices that, in the auditors
judgment, may have a significant effect on the financial
statements.
28. Which of the following is least likely procedure to be
performed in planning financial statement audit?
A. Selecting a sample of sales invoices for comparison with
shipping documents.
B. Coordinating the assistance of entity personnel in data
preparation.
C. Reading the current years interim financial statements.
D. Discussing matters that may affect the audit with firm
personnel responsible for non-audit services to the entity.

Selecting a sample of sales invoices for comparison with shipping


documents is substantive test procedure. Substantive tests are
performed to accumulate sufficient appropriate audit evidence,
not to plan the audit.

Answers B, C, and D are all planning procedures.

29. The establishment of an overall audit strategy involves


I. Determining the characteristics of the engagement that define
its scope.
II. Ascertaining the reporting objectives of the engagement to
plan the timing of the audit and the nature of the communications
required.
III. Considering the important factors that will determine the
focus of the engagement teams efforts.
A. I and II only
B. II and III only
C. I and III only
D. I, II and III

PSA 300 (Planning an Audit of Financial Statements) requires the


auditor to establish an overall audit strategy that sets the
scope, timing and direction of the audit, and that guides the
development of the audit plan.
In establishing the overall audit strategy, the auditor is
required to
a.) Identify the characteristics of the engagement that
define its scope.
b.) Ascertain the reporting objectives of the engagement
to plan the timing of the audit and the nature of the
communication required.
c.) Consider the factors that, in the auditors
professional judgment, are significant in directing the
engagement teams effort.
d.) Consider the results of preliminary engagement
activities and, where applicable, whether knowledge gain
on other engagement performed by the engagement partner
for the entity is relevant, and
e.) Ascertain the nature, timing and extent of resources
necessary to perform the engagement.

30. Which of the following should be included in the audit plan?


I. The nature, timing and extent of planned risk assessment
procedures.
II.The nature, timing and extent of planned further audit
procedures at the assertion level.
A. I only
B. II only
C. Both I and II
D. Neither I nor II

The audit plan shall include a description of


a.) The nature, timing and extent of planned risk assessment
procedures as determined under PSA 315 (Identifying and
Assessing the Risks of Material Misstatement through
Understanding the entity and its Environment).
b.) The nature, timing and extent of planned further audit
procedures at the assertion level, as determined under PSA
330 (The Auditors Responses to Assessed Risks).
31. Which of the following matters would an auditor least likely
consider when setting the direction of the audit?
A. The selection of the engagement team and the assignment of
the audit work to the team members.
B The engagement budget which includes consideration of the
appropriate amount of time to allot for areas where there may
be higher risk for material misstatement.
C. The availability of client personnel and data.
D. The manner in which the auditor emphasizes to engagement team
members the need to maintain a questioning mind and to exercise
professional skepticism in the gathering and evaluation of audit
evidence.

The availability of client personnel and data is a matter to


consider when establishing the scope of the audit.

32. Which of the following matters would an auditor most likely


consider when establishing the scope of the audit?
A. The expected audit coverage, including the number and location
of the entitys components to be included.
B. The entitys timetable for the reporting, such as at interim
and final stages.
C. The discussion with the entitys management concerning the
expected communications on the status of the audit work
throughout the engagement and the expected deliverables
resulting from the audit procedures.
D. Audit areas where there is a higher risk of material
misstatement.

Answers B and C are matters to consider when determining the


reporting objectives of the engagement, the timing of the audit,
and the nature of communication required.
Answer D is a matter to consider when setting the directions of
the audit.

33. In the planning stage of an audit engagement, the auditor


is required to perform audit procedures to obtain an
understanding of the entity and its environment, including its
internal control. These procedures are called
A. Risk assessment procedures
B. Substantive tests
C. Tests of controls
D. Dual-purpose tests

The audit procedures to obtain an understanding of the entity


and its environment, including its internal control are referred
to as risk assessment procedures because some of the
information obtained through these procedures may be used as
audit evidence to support assessments of the risks of material
misstatement.
34. In planning the audit engagement, the auditor should consider
each of the following, except
A. The type of opinion that is likely to be expressed.
B. The entitys accounting principles and procedures.
C. Matters relating to the entitys business and industry in
which it operates.
D. Materiality level and audit risk.

The nature of the report expected to be issued (e.g. a report


on consolidated financial statements) should be considered in
planning an audit engagement. The type of opinion to be rendered
is determined after the performance of audit procedures.

35. Audit programs are modified to suit the circumstances of


particular engagements. A complete audit program usually should
be developed
A. When the engagement letter is prepared.
B. After obtaining an understanding of the control environment
and control activities component of the entitys internal
control.
C. After the auditor has obtained an understanding of the entity
and its environment, including its internal control and assessed
the risk of material misstatement.
D. Prior to beginning the actual audit work.

Only after obtaining knowledge of the entity and its environment,


including its internal control and assessed the risk of material
misstatement can be the auditor determine the nature, timing,
and extent of substantive tests of financial statement
assertion.

36. In designing written audit programs, an auditor should


establish specific audit objectives that can relate primarily
to the
A. Selected audit techniques.
B. Cost-benefit of gathering audit evidence.
C. Timing of audit procedures.
D. Financial statement assertions.

The audit procedures included in the audit program should enable


the auditor to gather sufficient appropriate audit evidence
about management presentations embodied in the entitys
financial statements. Such management representations are called
management assertions. Hence, in designing written audit
programs, the auditor develops specific audit objectives in
light of those management assertions.

37. An audit program should be designed for each individual and


should incorporate steps and procedure to
A. Detect and eliminate fraud of any type.
B. Gather sufficient amount of management information available.
C. Provide assurances that the objectives of the audit are
satisfied.
D. Insure that only material items are audited.

A written audit program sets forth, in reasonable detail, the


specific audit procedures that in the auditors judgment are
necessary to satisfy the specific audit engagement.

38. Which of the following is an aspect of scheduling and


controlling of the audit engagement?
A. Including in the engagement letter an estimate of the minimum
and maximum audit fee.
B. Writing a conclusion in individual working papers indicating
how the results of the audit will affect the auditors report.
C. Performing audit work only after the entitys books have been
closed for the period under audit.
D. Including in the audit program a column for budgeted and
actual time.

The audit program/plan serves as set of instructions to


assistants involved in the audit and as a means to control and
record the proper execution of the work.
By including a column for estimated and actual time for each
audit step, the audit program can be a useful tool for
controlling and scheduling audit work.

39. In connection with the planning phase of an audit engagement,


which of the following statements is always correct?
A. Final staffing decisions must be made prior to completion of
planning stage.
B. Observation of inventory count should be performed at year-
end.
C. A portion of the audit of a continuing audit client can be
performed at interim dates.
D. An engagement should not be accepted after the clients
financial year-end.

40. The auditor shall undertake which of the following activities


prior to starting an initial audit?
I. Performing procedures required by PSA 220 (Quality Control
for an audit of Financial Statements) regarding the acceptance
of the client relationship and the specific audit engagement.
II. Communicating with the predecessor auditor, where there has
been a change of auditors, in compliance with relevant ethical
requirements.
A. I only
B. II only
C. Either I or II
D. Neither I nor II

41. Before accepting an audit engagement, a proposed


(successor/incoming) auditor should make inquiries of the
previous (predecessor) auditor regarding the previous auditors
A. Evaluation of all matters of continuing accounting
significance.
B. Understanding as to the reasons for the change of auditors.
C. Awareness of the consistency in the application of PAS/PFRS
between periods.
D. Opinion on any subsequent events occurring since the previous
auditors report was issued.

The standard requires the proposed auditor to communicate


directly with the previous auditor before accepting the
engagement. The proposed auditor should initiate the
communication, although both must obtain client permission to
communicate. The proposed auditor should inquire about reasons
for the change in auditors, disagreement with management
concerning accounting principles and auditing procedures, and
facts bearing on the integrity of the entitys management.

42. The auditor is required to determine three different levels


of materiality: (1) materiality for the financial statements as
a whole, (2) performance materiality, and (3)
A. Overall materiality
B. Planning materiality
C. General materiality
D. Specific materiality

The auditor is required to determine three different levels of


materiality. These are
1. Materiality for the financial statements as a whole ( also
called overall materiality, general materiality, or
tolerable misstatement)
2. Performance materiality (also called planning materiality
or scoping materiality)
3. Materiality applied to specific classes of transaction,
account balances or disclosures (also called specific
materiality or individual materiality)

43. What materiality level would be considered by the auditor


to determine whether the proposed adjustments are significant
or not?
A. Overall materiality
B. Scoping materiality
C. Specific materiality
D. Performance materiality
AASC Bulletin, Series 001 of 2010 states, Materiality for the
financial statements as a whole (herein referred to as the
overall materiality) is the materiality determined at the
overall financial statement level. This materiality level helps
the auditor determine whether the proposed audit adjustments are
significant or not,. I the audit adjustments exceed this level,
the auditor may need to adjust the financial statements.

44. What materiality level is used by the auditor in determining


which line items in the financial statements are to be tested?
A. Overall materiality
B. Performance materiality
C. Specific materiality
D. Individual materiality

Performance materiality, calculated as a certain percentage of


overall materiality, is used in scoping of financial statement
line items to be tested by the auditor. This will ensure that
significant accounts in the financial statements are covered by
audit testing.

45. _____________________ is the amount set by the auditor for


particular classes of transactions, account balances or
disclosures for which misstatements, well though lower than
overall materiality could reasonably be expected to influence
be expected to influence the economic decisions of users of the
financial statements.
A. Performance materiality
B. Planning materiality
C. Specific materiality
D. General materiality

46. Which of the required materiality levels is calculated by


multiplying a certain percentage by the appropriate benchmark,
which is either an element or component of an entitys financial
statements?

A. Overall materiality
B. Planning materiality
C. Scoping materiality
D. Specific materiality

47. Which of the following factors are normally considered by


the auditor in determining the appropriate benchmark for the
purpose of calculating overall materiality?
I. Components of the entitys financial statements
II. Laws and regulations
II. Nature of the entity
A. I and II only
B. I and III only
C. II and III only
D. I, II and III

The following factors are normally considered in choosing the


appropriate benchmark:

Components of the entitys financial statements


Focus of the users of the financial statements
Nature of the entity
Ownership structure of the entity
Volatility of the benchmark identified
Laws and regulation (e.g SEC)

48. Which of the following statements concerning materiality is


not correct?
A. When establishing the overall audit strategy, the auditor
shall determine materiality for the financial statements as a
whole.
B. If, in the specific circumstances of the entity, there is one
or more particular classes of transactions, account balances or
disclosures for which misstatements of lesser amounts than
materiality for the financial statements as a whole could
reasonable expected to influence the economic decisions of users
taken on the basis of the financial statements, the auditor
shall also determine the materiality level or levels to those
particular classes of transactions, account balances or
disclosures.
C. Determining materiality involves the exercise of professional
judgment.
D. The materiality level for the financial statements as a whole
determined in the planning stage of the audit should not be
affected by changes in the circumstances of the engagement.

PSA 320 (Materiality in Planning and Performing an Audit) states


that materiality for the financial statements as a whole (and
if applicable, the materiality level or levels for particular
classes of transactions, account balances or disclosures) may
need to be revised as a result of:
A change in circumstances that occurred during the audit
(e.g. a decision to dispose of a major part of the entitys
business);
New information; or
A change in the auditor understands of the entity and its
operations as a result of performing further audit
procedures.
49. Which of the following would an auditor most likely use in
determining a preliminary judgment about materiality?
A. The content of the management representation letter.
B. The anticipated sample size of the planned substantive tests.
C. The entitys annualized interim financial statements.
D. The results of the internal control questionnaire.
The measurement of a preliminary materiality level usually
relates to an annual figure (e.g. net income)

50. An auditor shall consider materiality when


I. Determining the nature, timing and extent of audit procedures.
II. Evaluating the effect of misstatements.
A. I only
B. II only
C. Both I and II
D. Neither I nor II

The auditor considers materiality when determining the nature,


timing and extent of audit procedures and evaluating the effect
of misstatements.

51. It is an appraisal activity established within the entity.


Its functions include, among other things, examining, evaluating
and monitoring the adequacy and effectiveness of the accounting
and internal control systems.
A. External auditing
B. Internal auditing
C. Governmental auditing
D. Internal auditing

52. Which is not a similarity between external and internal


auditors?

A. Both consider materiality and risk in their work.


B. Both use similar methodologies in performing their work.
C. Both must be competent.
D. Both must be independent of the company.

53. The external auditor should obtain a sufficient


understanding of the internal audit function because

A. The understanding of the internal audit function is an


important substantive test to be performed by the external
auditor.
B. The audit programs, working papers, and reports of internal
auditors may often be used as a substitute for the work of the
external auditors staff.
C. The procedures performed by the internal audit staff may
eliminate the external auditors need for considering internal
control.
D. The work performed by the internal auditors may be a factor
in determining the nature, timing and extent of the external
auditors procedures.

The standard requires the external auditor to obtain a sufficient


understanding of internal audit activities to identify and
assess the risks of material misstatement in the financial
statements and to design and perform further audit procedures.
Thus, an internal audit function is one of the many factors that
influence the nature, timing and extent of the external auditors
procedures.

54. Internal auditing can affect the scope of the external


auditors audit of financial statements by
A. Decreasing the external auditors need to perform detailed
tests.
B. Eliminating the need to observe the physical inventory taking.
C. Allowing the external auditor to limit his/her audit to the
performance of substantive test procedures.
D. Limiting direct testing by the external auditor to management
assertions not directly tested by internal auditing.

55. Which of the following is incorrect statement concerning the


relationship of the internal auditor and the scope of the
external audit of an entitys financial statements?

A. The external auditor is not required to give consideration


to the internal audit function beyond obtaining a sufficient
understanding to identify and assess the risks of material
misstatement of the financial statements and to design and
perform further audit procedures.
B. The internal auditors may determine the extent to which audit
procedures should be employed by the external auditor.
C. Under certain circumstances, the internal auditors may assist
the external auditor in performing substantive tests and tests
controls.
D. The nature, timing and extent of the external auditors
substantive tests may be affected by the work of the internal
auditors.
The standard categorically states that the external auditor has
sole responsibility for the audit opinion expressed, and the
responsibility is not reduced by any use made of internal
auditing. All judgments relating to the audit of an entitys
financial statements are those of the external auditor.

56. In determining whether the work of the internal auditors is


likely to be adequate for purposes of the audit, the external
auditor shall evaluate the internal auditors
A. Efficiency and experience
B. Independence and review skills
C. Training and supervisory skills
D. Competence and objectivity

The external auditor shall evaluate

The objectivity of the internal audit function


The technical competence of the internal auditors;
Whether the work of the internal auditors is likely to
carried out with due professional care; and
Whether there is likely to be effective communication
between the internal auditors and external auditor.

57. In assessing the technical competence of an internal auditor


an external auditor most likely would obtain information about
the
A. Quality of working paper documentation, reports, and
recommendations.
B. Organizational level to which the internal auditor reports.
C. Influence of management on the internal auditors duties.
D. Entitys commitment to integrity and ethical values.

In assessing the competence of an internal auditor, the external


auditor should consider the following factors, such as the
following:
Educational level and professional experience.
Professional certification and continuing education.
Audit policies, programs and procedures.
Supervision and review of the internal auditors
activities.
Departmental practices regarding assignments.
Quality of working papers documentation, reports, and
recommendations.
Evaluation of the internal auditors performance.

Answers B, C and D are incorrect because they all relate to


objectivity rather than technical competence.

58. Which of the following is s false statement about the use


of the internal auditors work by the external auditor?
A. The PSAs do not allow the external auditor to use to work of
the internal auditor.
B. PSAs do not allow the external auditor to substitute the work
of the internal auditor for the work of the external auditor in
critical judgments.
C. The PSAs state that, when specific work of the internal
auditor is to be used, it should be evaluated and tested.
D. PSAs states that, when considering whether to use the work
of the internal auditor, the external auditor should consider
the internal auditors competence and objectivity.

59. The coordination between internal and external auditors

A. Eliminates duplication of audit efforts


B. Includes the exchange of audit reports and management letters.
C. Prevents external auditors from having access to the programs
used by the internal auditors.
D. Prohibits the internal auditor from using the same audit
techniques as external auditors and vise versa.
Coordination of internal and external audit efforts helps to
ensure adequate audit coverage and to minimize duplication of
audit efforts. Coordination involves:
Meetings at appropriate intervals during the period
discuss matters of mutual interest.
Access to each others audit programs and working papers.
Exchange of audit reports and management letters.

60. Which of the following are included in the activities of the


internal audit function?
I. Monitoring of internal control.
II. Examination of financial and operating information.
III. Review of operating activities.
A. I and II only
B. I and III only
C. II and III only
D. I, II and III

The activities of the internal audit function may include one


or more of the following:

Monitoring of internal control.


Examination of financial and operating information.
Review of operating activities.
Review of compliance with laws and regulation.
Risk management
Governance

61. ______________ is an individual or organization possessing


expertise in a field other than accounting or auditing, whose
work in that field is used by the auditor to assist the auditor
in obtaining sufficient appropriate audit evidence.
A. Auditors expert
B. Managements expert
C. Expert
D. Specialist

62. ________________ is an individual or organization possessing


expertise in a field other than accounting or auditing, whose
work in that field is used by the entity to assist the entity
in preparing financial statements.
A. Auditors expert
B. Managements expert
C. Expert
D. Specialist

63. When planning to use the work of an expert, the auditor


should evaluate the experts
I. Professional competence
II. Objectivity
A. I only
B. II only
C. Both I and II
D. Neither I nor II

64. Which of the following statements is correct concerning the


auditors use of the work of an expert?
A. The auditor is required to perform substantive test procedures
to verify the experts assumptions and findings.
B. The auditor should obtain understanding of the methods and
assumptions used by the expert.
C. The entity should not have an understanding of the nature of
the work to be performed by the expert.
D. The expert should not have an understanding of the nature of
the auditors corroborative use of the experts findings.

The expert is responsible for the appropriateness and


reasonableness of assumptions and methods used. Because the
auditor does not have the same expertise, he/she is not in the
position to challenge the experts methods and assumptions.
However, the standard states that the auditor will need to obtain
an understanding of the methods and assumptions used and consider
whether they are appropriate and reasonable, based on his/her
knowledge of the entitys business and the results of other
procedures.

Answer A is incorrect because, if the experts findings


corroborative the financial statement assertions being
considered, the auditor may reasonably conclude that sufficient
appropriate evidence has been obtained.
Answers C and D are incorrect because the auditor, the entity
and the expert should have an understanding about the nature,
scope, and objective of the work to be performed.

65. Which of the following is not an expert upon whose work an


auditor may rely?
A. An actuary.
B. An individual with expertise in complex modeling for the
purpose of valuing financial instruments.
C. An expert in taxation law.
D. An individual with expertise in applying methods of accounting
for deferred income tax.

66. If the results of the experts work do not provide sufficient


appropriate audit evidence or are not consistent with other audit
evidence, the auditor should

A. Report the matter to the appropriate regulatory agency of the


government.
B. Resolve the matter.
C. Withdraw from the engagement.
D. Express an unmodified opinion with reference to the work of
the expert.

If the results of the experts work do not provide sufficient


appropriate audit evidence or are not consistent with other audit
evidence, the auditor should resolve the matter. The auditor may
need to discuss the matter with the entity and the expert, apply
additional audit procedures, possibly engage another expert, or
modify the auditors report.
67. When issuing an unmodified auditors report, the auditor
A. May refer to the work of an expert.
B. Should refer to the work of an expert to indicate a division
of responsibility.
C. Should include in the auditors report the identity of the
expert and the extent of the experts involvement.
D. Should not refer to the experts work.

The standard prohibits the auditor to refer to the experts work


when issuing an unmodified auditors report because such a
reference might be misunderstood to be a qualification of the
auditors opinion or a division of responsibility, neither of
which is intended.

68. In using the work of an expert, an auditor referred to the


experts finding in the auditors report. This is an appropriate
practice if the
A. Auditor, as a result of the experts work, decides to
indicate a division of responsibility with the expert.
B. Expert is aware that his/her work will be used to evaluate
the assertions in the financial statements.
C. Auditor, as a result of the experts work, issues a report
that an unmodified opinion.
D. Auditor, as a result of the experts work, adds an emphasis-
of-matter paragraph I his/her unmodified auditors report.

An auditor shall not refer to the experts work in an auditors


report containing an unmodified opinion. However, if as a result
of the experts work, the auditor decides to express a modified
opinion, it may be appropriatein explaining the nature of the
modificationto refer to or describe the experts work

69. As used in PSA 600, Special Considerations Audits of


Group, Financial Statements (Including the Work of Component
Auditor) , ______________ is an entity or business activity
for which group or component management prepares financial
information that should be included in the group financial
statements.
A. Component
B. Group
C. Significant component
D. Group management

70. As used in PSA 600, financial statements that include the


financial information of more than one component are called

A. Component financial statements


B. Group financial statements
C. Consolidated financial statements
D. Common financial statements

71. The __________________ is the partner or other person in the


firm who is responsible for the group audit management and its
performance, and for the auditors report on the group financial
statements that is issued on behalf of the firm.

A. Engagement partner
B. Component engagement partner
C. Principal auditor
D. Group engagement partner

72. The group engagement team shall obtain an understanding that


is sufficient to

I. Confirm or revise its initial identification of components


that are likely to be significant.
II. Assess the risks of material misstatement of the group
financial statements, whether due to fraud or error.
A. I only
B. II only
C. Both I and II
D. Neither I nor II

73. If the group engagement team plans to request a component,


auditor to perform work on the financial information of a
component, the group engagement team shall obtain an
understanding of

I. Whether the component auditor understands and will comply


with the ethical requirements that are relevant to the group
audit and, in particular, is independent.
II. The component auditors professional competence.
A. I only
B. II only
C. Both I and II
D. Neither I nor II

74. Which of the following statements concerning the group audits


is incorrect?

A. The group engagement team has the responsibility to establish


an overall group audit strategy and audit plan.
B. The group engagement team shall determine the materiality for
the group financial statements as a whole when establishing the
overall group audit strategy.
C. The component engagement partner shall review the overall
group audit strategy and audit plan.
D. The group engagement team shall agree on the terms of the
group audit engagement in accordance with PSA 210.

75. An auditor who, at the request of the group engagement team,


performs work on financial information related to a component
for the group audit is a

A. Group auditor
B. Component auditor
C. Component engagement team
D. Group engagement team
KEY ANSWERS
1. C 26. A 51. B

2. A 27. D 52. D

3. C 28. A 53. D

4. A 29. D 54. A

5. C 30. C 55. B

6. B 32. C 56. D

7. A 32. A 57. A

8. A 33. A 58. A

9. C 34. A 59. B

10. A 35. C 60. D

11. A 36. D 61. A

12. A 37. C 62. B

13. A 38. D 63. C

14. C 39. C 64. B

15. C 40. D 65. D

16. D 41. B 66. B

17. A 42. D 67. D

18. A 43. A 68. C

19. A 44. B 69. A

20. A 45. C 70. B

21. D 46. A 71. D

22. C 47. D 72. C

23. D 48. D 73. C

24. D 49. C 74. C

25. B 50. C 75. B


CHAPTER 5
RISK ASSESSMENTS AND INTERNAL COTROL

1. PSA 135 (Identifying and Assessing the Risks of Material


Misstatement through Understanding the Entity and its
Environment) requires the auditor to perform risk assessment
procedures at
A. The financial statement level only.
B. The assertion level only.
C. The financial statement level and the assertion level for
classes of transactions, account balances and disclosures.
D. Either the financial statement or assertion level.

Risk assessment procedures are performed to obtain an


understanding of the entity and its environment, including
the entitys internal control, to identify and assess the
risks of material misstatement, whether due to fraud or
error, at the financial statement and assertion levels.

2. The auditors risk assessment procedures should always


include the following, except
A. Inquiries of management and of others within the entity.
B. Analytical procedures.
C. Observation and inspection.
D. Substantive test procedures and tests of controls.

The auditors risk assessment procedures shall include:


1. Inquiries of management and of others within the entity
who, in the auditors judgment, may have information that
Is likely to assist in identifying risks of material mis-
Statement due to fraud or error.

2. Analytical procedures.

3. Observation and inspection.

Substantive test procedures are audit procedures designed to


detect material misstatements at the assertion level. Tests
of controls are audit procedures designed to evaluate the
effectiveness of controls in preventing, or detecting and
correcting, material misstatements at the assertion level.

3. The auditors risk assessment procedures


A. By themselves, do not provide sufficient appropriate audit
evidence on which to base the audit opinion.
B. Should not consider information obtained from the
auditors previous experience with the entity.
C. Are designed to detect material misstatements at the
assertion level for classes of transactions, account balances
and disclosures.
D. Are designed to test the effectiveness of the entitys
controls.

4. The auditor should obtain an understanding of the entitys


objectives and strategies, and those business risks that may
result in risks of material misstatement. Which of the
following statements concerning the entitys business risk
is incorrect?
A. Business risk is broader than the risk of material
misstatement of the financial statements, though it
includes the latter.
B. An understanding of the business risks facing the entity
increases the likelihood of identifying risks of material
misstatement.
C. The auditor has a responsibility to identify or assess
all business risks.
D. Business risk may arise from the development of new
products or services that may fail.

As defined in PSA 315, business risk is a risk resulting


from significant conditions, events, circumstances, actions
or inactions that could adversely affect an entitys ability
to achieve its objectives and execute its strategies, or form
the setting of inappropriate objectives and strategies.

The auditor does not have a responsibility to identify or


assess all business risks facing the entity because not all
business risks give rise to risks of material misstatement.

5. The risk that the auditor may give an inappropriate opinion


when the financial statements are materially misstated is
called
A. Detection risk
B. Business risk
C. Audit risk
D. Inherent risk

6. Audit risk has three components: inherent risk, control risk,


and detection risk. Which is correct?
A. Detection risk is a function of the efficiency of an
auditing procedure.
B. Cash is more susceptible to theft than an inventory of coal
because it has a greater inherent risk.
C. The risk that material misstatements will not be prevented
or detected on a timely basis by internal control can be
reduced to zero by effective controls.
D. The existing levels of inherent risk, control risk, and
detection risk can be changed at the discretion of the
auditor.
The three components of audit risk are:

1. Inherent risk is the susceptibility of an account


balance or class of transactions to misstatement that could
be material, individually or when aggregated with
misstatements in other balances or classes, assuming that
there were no related internal controls.

2. Control risk is the risk that a misstatement that


could occur in an account balance or class of transactions
that could be material, individually or when aggregated with
other misstatements in other balances or classes, will not
be prevented, or detected and corrected, on a timely basis,
by the accounting and internal control systems.

3. Detection risk is the risk that an auditors


substantive procedures will not detect a misstatement that
exists in an account balance or class of transactions that
could be material, individually or when aggregated with
misstatements in other balances or classes.

Cash has a greater inherent risk than an inventory of coal.

Answer A is incorrect because detection risk is a function


of the effectiveness of an auditing procedure, not its
efficiency.

Answer C is incorrect because even the most effective


internal control has inherent limitations. Thus, control risk
cannot be reduced to zero.

Answer D is incorrect because inherent risk and control risk


exist independently of the audit and, therefore, cannot be
changed at the discretion of the auditor.

The acceptable level of detection risk is a function of the


desired level of overall audit risk and the assessed levels
of inherent risk and control risk. Hence, detection risk can
be changed at the discretion of the auditor. However, it
should be emphasized that the auditors preliminary
assessments of inherent risk and control risk may change as
the audit work continues.

7. The risk that an auditors substantive procedures will lead


to the conclusion that a material misstatement does not exist
in an account balance or transaction class when, in fact,
such misstatement does exist is
A. Control risk
B. Inherent risk
C. Audit risk
D. Detection risk

8. The existence of audit risk is recognized by the statement


in the auditors report that the
A. Financial statements are presented fairly, in all material
respects, in accordance with Philippine Financial
Reporting Standards.
B. Audit includes examining, on a test basis, evidence
supporting the amounts and disclosures in the financial
statements.
C. Auditor obtains reasonable assurance about whether the
financial statements are free of material misstatement.
D. Auditor is responsible for expressing an opinion on the
financial statements, which are managements
responsibility.

An audit conducted in accordance with PSAs provides only


reasonable, not absolute, assurance that the financial
statements are free of material misstatement, whether caused
by error or fraud.

9. Which of the following audit risk components may be assessed


I quantitative terms?
Inherent Risk Control Risk Detection Risk
A. Yes No Yes
B. Yes Yes Yes
C. No No No
D. No No Yes

Audit risk and its components may be assessed in quantitative


or non-quantitative terms.

10. Some account balances, such as those for retirement benefits


and finance leases, are the results of complex calculations.
The susceptibility to material misstatements in these types
is referred to as
A. Audit risk
B. Detection risk
C. Inherent risk
D. Control risk

Inherent risk, which is the susceptibility of an assertion


to material misstatement in the absence of related controls,
exists independently of the audit. Some assertions and
related account balances and classes of transactions have
greater level of inherent risk than others. For example,
account balances resulting from complex calculations such as
those for retirement benefits and finance leases have a
higher risk of misstatement.
11. There is an inverse relationship that exists between the
acceptable level of detection risk and the
A. Risk of failing to discover material misstatements.
B. Assurance provided by substantive tests.
C. Preliminary judgments about materiality levels.
D. Risk of misapplying audit procedures.

An auditor assesses control risk by considering internal


control. There exists an inverse relationship between control
risk and detection risk that is, the greater (lower) the
assessed level of control risk, the lower (greater) the
acceptable level of detection risk.

The acceptable level of detection risk affects substantive


testing. As the acceptable level of detection risk decreases,
the auditor changes the nature, timing, and extent of
substantive tests to increase the assurance they provide.
Therefore, there is an inverse relationship between the
acceptable level of detection risk and substantive testing.

12. Which of the following statements concerning audit risk and


its components is incorrect?
A. Regardless of the assessed levels of inherent and control
risks, the auditor should always perform some substantive
procedures for material account balances and classes of
transactions.
B. The higher the assessment of inherent and control risks,
the more evidence the auditor should obtain from the
performance of substantive procedures.
C. The assessed level of inherent risk need not be considered
in determining the nature, timing, and extent of
substantive procedures required to reduce audit risk to
an acceptably low level.
D. After obtaining an understanding of the accounting and
internal control systems, the auditor should make a
preliminary assessment of control risk, at the assertion
level, for each material account balance or class of
transactions.

According to the standard, the auditor should consider the


assessed levels of inherent and control risks in determining
the nature, timing, and extent of substantive procedures
required to reduce audit risk to an acceptable level.

13. Because the concepts of audit risk and materiality are


interrelated, they must be considered together by the
auditor. Which of the following statements is correct?
A. The phrase in the auditors report present fairly, in
all material respects, in accordance with Philippine
Financial Reporting Standards indicates the auditors
belief that the financial statements taken as a whole are
not materially misstated.
B. If misstatements are not individually material, but are
material when aggregated with other misstatements, the
concept of materiality does not apply.
C. Audit risk is the risk that an auditor may unknowingly
modify his/her opinion when, in fact, the financial
statements are fairly presented.
D. Only material errors cause financial statements to be
materially misstated.

The opinion paragraph of the auditors report explicitly


refers to materiality. By stating that the financial
statements are presented fairly, in all material respects,
in accordance with an applicable financial reporting
framework, the auditor is of the opinion that the financial
statements are not materially misstated.

Answer B is incorrect because the concept of materiality


recognizes that some misstatements, either individually or
when aggregated with other misstatements, are important for
the fair presentation of financial statements.

Answer C is incorrect because audit risk is the probability


that an auditor may give an inappropriate opinion when the
financial statements are materially misstated.

Answer D is incorrect because material misstatements can


result from errors or fraud.

14. The statements below describe the interrelationship of audit


risk components. Which is false?
A. There is an inverse relationship between detection risk
and the combined level of inherent and control risks.
B. When inherent and control risks are high, the acceptable
level of detection risk needs to be low to reduce audit
risk to an acceptably low level.
C. When inherent and control risks are low, an auditor can
accept a higher detection risk and still reduce audit
risk to an acceptably low level.
D. The assessed level of inherent and control risks can be
sufficiently low to eliminate the need for the auditor to
perform any substantive procedures.

The standard states that regardless of the assessed levels


of inherent and control risks, the auditor should perform
some substantive procedures for material account balances
and classes of transactions.
15. Based on audit evidence gathered and evaluated, an auditor
decides to increase the assessed level of control risk from
that originally planned. To achieve an overall audit risk
level that is substantially the same as the planned audit
risk level, the auditor would
A. Increase materiality levels.
B. Decrease detection risk.
C. Decrease substantive testing.
D. Increase inherent risk.

16. A basic premise underlying analytical procedures is that


A. Plausible relationships among data may reasonably be
expected to exist and continue in the absence of known
conditions to the contrary.
B. These procedures cannot replace tests of details of
transactions and balances.
C. Statistical tests of financial information may lead to
the detection of material misstatements in the financial
statements.
D. The study of financial ratios is an acceptable alternative
to the investigation of unusual fluctuations.

Analytical procedures as defined in the standard means


evaluations of financial information made by a study of
plausible relationships among both financial and
nonfinancial data.

A basic premise underlying the application of analytical


procedures is that plausible relationships among data may
reasonably be expected to exist in the absence of known
conditions to the contrary.

Answer B is incorrect because for some assertions, the


application of analytical procedures alone may provide the
auditor with the level of assurance he/she desires.

Answer C is incorrect because the use of analytical


procedures (e.g., simple comparisons) does not necessarily
require statistical testing.

Answer D is incorrect because analytical procedures, such as


ratio analysis, enable an auditor to identify significant
differences that should be evaluated and investigated.

17. For all audits of financial statements conducted in


accordance with PSAs, the use of analytical procedures is
required to some extent
A B C D
As a risk assessment procedure
in the planning phase Yes Yes Yes No
As a substantive test procedure Yes No No Yes
In the overall review of the
financial statements Yes Yes No Yes

Analytical procedures are used for the following purposes:

1. As risk assessment procedures to obtain an understanding


of the entity and its environment.

2. As substantive test procedures when their application


is, based on the auditors judgment, more effective and
efficient than tests of details in reducing the risk
of material misstatement.

3. As an overall review of the financial statements at the


completion stage of the audit engagement.

18. Which of the following statements concerning analytical


procedures is true?
A. Analytical procedures are more efficient, but not more
effective, than tests of details of transactions.
B. Analytical procedures used as risk assessment procedures
use data aggregated at a high level.
C. Analytical procedures can replace tests of controls in
gathering audit evidence to support the assessed level of
control risk.
D. Analytical procedures usually involve comparisons of
ratios developed from recorded amounts with ratios
developed by management.

Analytical procedures applied as risk assessment procedures


focus on enhancing the understanding of the entitys
business, identifying unusual transactions and events,
identifying areas that may represent specific audit risks,
and most often use data aggregated at a high level.

Answer A is incorrect because for many assertions, tests of


details are still more effective or efficient in providing
the level of assurance desired by the auditor.

Answer C is incorrect because analytical procedures can be


used as substantive tests, not as tests of controls.

Answer D is incorrect because analytical procedures involve


comparisons of recorded amounts or ratios developed from
recorded amounts with expectations developed by the auditor
not with assertions developed by management.

19. Analytical procedures used in planning an audit should focus


on
A. Enhancing the auditors understanding of the clients
business.
B. Evaluating the adequacy of evidence gathered concerning
unusual balances.
C. Identifying significant deficiencies in internal control.
D. Testing individual account balances that depend on
accounting estimates.

20. Analytical procedures performed in the


A. Planning stage of the audit should assist in assessing
the validity of the conclusions reached.
B. Planning stage of the audit should address the risk of
material misstatement of revenue due to fraudulent
financial reporting.
C. Final review stage of the audit should achieve audit
objectives related to specific assertions in the
financial statements.
D. Final review stage of the audit should focus on
identifying specific risks.

21. An auditor obtains an understanding of the nature of the


entity to
A. Make constructive suggestions concerning improvements in
the clients internal control.
B. Understand the account balances, transactions, and
disclosures in the financial statements.
C. Develop an attitude of professional skepticism concerning
managements financial statement assertions.
D. Evaluate whether the aggregation of known misstatements
causes the financial statements taken as a whole to be
materially misstated.

22. It is the process designed, implemented and maintained by


those charged with governance, management, and other
personnel to provide reasonable assurance about the
achievement of the entitys objectives.
A. Internal auditing
B. Internal control
C. Business strategy
D. Accounting process

PSA 315 states that internal control is designed and


implemented to achieve the entitys objectives with regard
to:
Reliability of financial reporting;
Effectiveness and efficiency of operations; and
Compliance with applicable laws and regulations.

23. Which of the following statements concerning internal


control is incorrect?
A. Internal control is effected by people.
B. Internal control is expected to provide absolute
assurance.
C. Internal control is a process.
D. Internal control is geared to achieving objectives in the
overlapping categories of financial reporting, compliance
and operations.

24. The inherent limitations in an entitys internal control


system include
I. Collusion among individuals
II. Breakdowns in established controls.
III. Mistakes in judgment.

A. I and III only C. II and III only


B. I and II only D. I, II and III

25. The procedures performed to obtain an understanding of the


internal control system provide an auditor with
A. Sufficient appropriate evidence to use in forming an
overall opinion on the entitys financial statements.
B. Enough understanding to design procedures to gather
sufficient appropriate audit evidence.
C. Enough understanding to express an opinion on the
effectiveness of the entitys internal control system.
D. Audit evidence to use in forming an overall opinion on
the entity.

26. Some inherent limitations in an entitys internal control


system include
I. The possibility of management overriding the
internal control in place for illegitimate
purposes.
II. Collusion among employees to conceal fraud.
III. Mistakes in judgment as the result of inadequate
information or time pressures.

A. I and II only C. II and III only


B. I and III only D. I, II and III

27. When considering an entitys internal control system, an


auditor is not required to
A. Search for significant deficiencies in the operation of
the internal control system.
B. understand the components of the entitys internal control
system.
C. Determine whether relevant controls have been placed in
operation.
D. Perform procedures to understand the design of the
internal control system policies.

28. The following are components of internal control, except


A. Control activities
B. The entitys risk assessment process
C. Control environment
D. Business risk

The five components of internal control are:


1. The control environment.
2. The entitys risk assessment process.
3. The information system, including the related business
processes relevant to financial reporting, and
communication.
4. Control activities.
5. Monitoring of controls.

29. An entitys internal control system contains manual elements


and often contains automated elements. Manual elements in
internal control may be less reliable than automated elements
because
A. Manual control elements can be more easily bypassed,
ignored, or overridden and they are also more prone to
simple errors and mistakes.
B. Manual control elements facilitate the additional analysis
of information.
C. Consistency of application of manual control elements can
always be assumed.
D. Manual control elements include reliance on systems or
programs that are inaccurately processing data,
processing inaccurate data, or both.

30. IT benefits an entitys internal control by enabling an


entity to
A. Reduce the risk that controls will be circumvented.
B. Process large, unusual or non-recurring transactions.
C. Rely on systems or programs that are inaccurately
processing data.
D. Allowing unauthorized changes to data in master files.

The use of IT allows an entity to:

Consistently apply predefined business rules and


perform complex calculations in processing large
volumes of transactions and data;
Enhance the timeliness, availability, and accuracy of
information;
Facilitate the additional analysis of information;
Enhance the ability to monitor the performance of the
entitys activities and its policies and procedures;
Reduce the risk that control will be circumvented; and
Enhance the ability to achieve effective segregation of
duties by implementing security controls in
applications, databases, and operating systems.

31. When obtaining an understanding of controls that are relevant


to the audit, the auditor is required to
A. Evaluate the design of those controls.
B. Determine whether those controls have been implemented.
C. Evaluate the design of those controls and determine
whether they have been implemented.
D. Evaluate the design of those controls and determine
whether they have been implemented by performing tests of
controls.

PSA 315 states that when obtaining an understanding of


controls that are relevant to the audit, the auditor shall
evaluate the design of those controls and determine whether
they have been implemented.

Evaluating the design of a control involves considering


whether the control, individually or in combination with
other controls, is capable of effectively preventing, or
detecting and correcting, material misstatements.
Implementation of a control means that the control exists
and that the entity is using it.

Risk assessment procedures to obtain an understanding of


controls relevant to the audit include of the following:

Inquiring of entity personnel.


Observing the application of specific controls.
Inspecting documents and reports.
Tracing transactions through the information system to
financial reporting.

As defined in the standard, tests of controls are audit


procedures designed to evaluate the operating
effectiveness of controls in preventing, or detecting and
correcting, material misstatements at the assertion level.

32. This internal control component is the foundation for all


other components. It sets the tone of the organization,
provides discipline and structure, and influences the control
consciousness of employees.
A. Control activities
B. Monitoring of controls
C. Control environment
D. The entitys risk assessment process

PSA 315 states that the control environment includes the


governance and management functions and the attitudes,
awareness, and actions of those charged with governance and
management concerning the entitys internal control and its
importance in the entity. The control environment sets the
tone of the organization, influencing the control
consciousness of its people.

33. Which of the following are considered control environment


elements?
Commitment Detection Organizational
To Competence Risk Structure
A. No Yes No
B. Yes Yes Yes
C. Yes No Yes
D. No No Yes

The control environment components of internal control


encompasses the following elements:

a. Communication and enforcement of integrity and ethical


values.
b. Commitment to competence.
c. Participations by those charged with governance.
d. Managements philosophy and operating style.
e. Organizational structure.
f. Assignment of authority and responsibility.
g. Human resource policies and practices.

Detection risk is a component of audit risk.

34. Which of the following elements are included in an entitys


control environment?
Integrity and Assignment of
Ethical Values Authority and Responsibility
A. Yes Yes
B. Yes No
C. No Yes
D. No No

35. Which of the following components of an entitys internal


control includes development and use of training policies
that communicate prospective roles and responsibilities to
employees?
A. Monitoring of controls
B. Control activities
C. Control environment
D. Information and communication

The control environment component of internal control


includes human resource policies and practices relative to
recruitment, orientation, training, evaluating, counseling,
promoting, compensating, and remedial actions.

36. Which of the following statements concerning the relevance


of various types of controls to a financial statement audit
is correct?
A. All controls are ordinarily relevant to a financial
statement audit.
B. Controls over safeguarding of assets and liabilities are
of primary importance, while controls over the
reliability of financial reporting may also be relevant.
C. Controls over the reliability of financial reporting are
ordinarily most directly relevant to a financial
statement audit, but other controls may also be a
relevant.
D. An auditor may ordinarily ignore a consideration of
controls when a substantive audit approach is taken.

Controls that are relevant to a financial statement audit


pertain to the entitys objective of preparing financial
statements for external purpose and the management of risk
that may give rise to a material misstatement in those
financial statements.

37. Which of the following internal control components relates


to an entitys process for identifying and responding to
business risks?
A. Control activities
B. Information and communication
C. Risk assessment
D. Monitoring of controls

PSA 315 states that the entitys risk assessment process


forms the basis for how management determines the risks to
be managed.

For financial reporting purposes, an entitys risk assessment


process includes how management identifies risks relevant to
the preparation of financial statements, estimates their
significance, assesses the likelihood of their occurrence,
and decides upon actions to manage them.
38. The following are subcomponents of the control environment,
except
A. Managements philosophy and operating style.
B. Adequate separation of duties.
C. Organizational structure.
D. Commitment to competence.

39. The purpose of an entitys accounting information and


communication system is to
Record and
Monitor process Initiate
transactions transaction transactions
A. No No No
B. Yes Yes Yes
C. Yes No No
D. No Yes Yes

40. Which of the following statements concerning physical


controls is incorrect?
A. Access to computer hardware should not be limited to
authorized personnel.
B. Physical controls limit access to assets and important
records.
C. In systems with online entry of data, many users may have
access through remote input devices.
D. Access often extends beyond the entitys employees to
customers and suppliers through remote terminals.

41. Inherent risk is ______ related to detection risk and ______


related to the amount of audit evidence.
A. directly, inversely
B. directly, directly
C. inversely, inversely
D. inversely, directly

42. An auditors flowchart of an entitys information system is


a diagrammatic representation that depicts the auditors
A. Documentation of the study and evaluation of the internal
control system.
B. Understanding of the system.
C. Understanding of the types of fraud that are probable,
given the present system.
D. A program for tests of controls.
43. Which of the following statements about auditor
documentation of the clients internal controls is correct?
A. Documentation must include flowcharts.
B. Documentation must include procedural write-ups.
C. Documentation is desirable but not necessary.
D. No one particular form of documentation is necessary.

44. Which of the following are the two key issues that an auditor
considers when obtaining an understanding of a clients
internal controls?
A. The effectiveness and efficiency of the controls.
B. The implementation and efficiency of the controls.
C. The design and utilization of the controls.
D. The frequency and effectiveness of the controls.

45. The auditor documents his or her understanding of the


internal control system to substantiate
A. The fairness of presentation of the financial statements.
B. Adherence to the requirements of management.
C. Compliance with PSAs.
D. Conformity of the accounting records with the applicable
financial reporting framework.

46. Which of the following is not an advantage of an IT system


used as part of the entitys internal control system?
A. A reduced amount of information is required to be reviewed
manually.
B. Detection of potential problems only if the computer is
programmed to do so.
C. The delivery, timeliness and accuracy of information are
maintained as the volume of transactions increases.
D. Potentially fewer errors as individual calculation
discrepancies are reduced.

47. The best way for an auditor to obtain evidence about


segregation of duties is to
A. Observe personnel performing their duties.
B. Reperform the task.
C. Inspect documents and records to ensure an independent
check has been performed.
D. Discuss the performance of duties within the company with
the internal auditor.

48. An auditor must assess control risk as high when he has


A. Decided not to test the internal control system.
B. Decided on a high level of detection risk.
C. Assessed inherent risk to be high.
D. Found no significant deficiencies in the internal control
system.

49. Which of the following describes narrative memoranda?


A. A schematic diagram that uses standardized symbols,
interconnecting flow lines and annotations to portray the
steps involved in processing information through the
information system.
B. Series of questions about accounting and control policies
and procedures that an auditor considers necessary to
prevent material misstatements in the financial
statements.
C. A form of documentation that summarizes the auditors
overall understanding of the information system or
specific control policies or procedures.
D. A written report on the effectiveness of the entitys
internal control system.

50. ______________ are groups of related business activities


such as the acquisition of merchandise and payment of
vendors.
A. Transactions
B. Transaction cycles
C. Economic cycles
D. Business events

51. The following are transaction cycles, except


A. General ledger and reporting
B. Human resources
C. Expenditure/Disbursement
D. Revenue/Receipt

52. The collection of job time tickets or time sheets is part


of which of the following transaction cycles?
A. Revenue/Receipt
B. Human resources
C. Production
D. Expenditure/Disbursement

53. Which of the following best explains why many modern


accounting software packages offer separate transaction
cycle modules?
A. A properly designed Accounting Information System does
not use the concept of separate business transaction
cycles to process transactions.
B. The nature of a given transaction cycle is the same
irrespective of the type of organization.
C. Most businesses do not need the revenue cycle module as
part of their Accounting Information System.
D. Every organization does not need to implement all of the
available transaction cycle modules.

54. One advantage of pre-numbering source documents is that it


helps to verify that
A. All cash has been collected.
B. No inventory has been misplaced.
C. All transactions have been recorded since the numerical
sequence serves as a control.
D. Documents have been used in order.

Use the following flowchart symbols to answer the questions


below:
55. Which symbol would be used to represent a computer process?
A. #1 C. #4
B. #2 D. #12

56. Which symbol would be used to represent a decision?


A. #9 C. #8
B. #11 D. #5

57. Which symbol would be used to represent an invoice sent to


a customer?
A. #2 C. #1
B. #5 D. #9

58. Which symbol would be used to represent a general ledger?


A. #2 C. #3
B. #1 D. #4

59. Which symbol would be used to represent a manual process?


A. #4 C. #9
B. #5 D. #10

60. Which symbol would be used to represent a connection to


another part of the flowchart on the same page?
A. #4 C. #10
B. #9 D. #11

61. Which symbol would be used to represent a connection to


another part of the flowchart on a different page?
A. #4 C. #10
B. #9 D. #11

62. Which symbol would be used to represent a file of paper


documents?
A. #6 C. #8
B. #7 D. #10

63. Which symbol would be used to represent the display of a


report on a computer screen?
A. #1 C. #3
B. #2 D. #10
64. Which symbol would be used to represent a beginning, an
ending, or a connection to another procedure?
A. #8 C. #11
B. #10 D. #12

65. Duplicate checking of calculations is an example of a


corrective control.

Adherence to appropriate procedures to resubmit rejected


transactions is an example of a detective control.

A. True; False C. False; True


B. True; True D. False; False

66. General IT controls are policies and procedures that relate


to many applications and support the effective functioning
of application controls. General IT controls commonly include
controls over the following, except
A. Data center and network operations
B. Manual follow-up of exception reports
C. Access security
D. Program change

Manual follow-up of exception reports is an example of


application controls. These controls relate to procedures
used to initiate, record, process and report transactions or
other financial data.

67. Application controls are


A. Manual or automated procedures that typically operate at
a business process level and apply to the processing of
transactions by individual applications.
B. Policies and procedures that relate to many applications.
C. Controls that maintain the integrity of information and
security of data such as controls over system software
acquisition, change and maintenance.
D. Controls that relate to many applications and support the
effective functioning of general controls.

68. Under PSA 315, monitoring of controls is an internal control


component that involves a process of assessing the quality
of internal control performance over time. It involves
assessing the design and operation of controls on a timely
basis and taking necessary corrective actions. Monitoring of
controls is accomplished through ongoing monitoring
activities, separate evaluations, or a combination of the
two. An entitys ongoing monitoring activities often include
A. Periodic reporting by the entitys internal auditors about
the functioning of internal control.
B. Reviewing the purchasing function.
C. Periodic audits by the audit committee.
D. The audit of the annual financial statements.

According to the standard, ongoing monitoring activities are


built into the normal recurring activities of an entity and
include regular management supervisory activities, such as
reviewing the purchasing function.

Answer A is incorrect because regularly providing information


about the functioning of internal control is monitoring of
controls by the entitys internal auditors through separate
evaluations.

Answer C is incorrect because the audit committee (a


subcommittee of the board of directors that is composed of
board members who are not offices of the entity) does not
ordinarily perform periodic audits.

Answer D is incorrect because the annual audit of financial


statements is not a monitoring activity.

69. Control activities relate to the following, except


A. Segregation of duties
B. Performance reviews
C. An internal audit function
D. Authorization

Control activities are the policies and procedures that help


ensure that management directives are carried out. They are
intended to ensure that necessary actions are taken to
address risks that threaten the achievements of the entitys
objectives. Control activities have various objectives and
are applied at various organizational and functional levels.
Specific control activities include those that relate to:
Authorization
Performance reviews
Information processing
Physical controls
Segregation of duties

An internal audit function is part of the monitoring


component of internal control.
70. The following are examples of monitoring activities, except
A. Information Technology (IT) management regularly
generates reports for unusual transactions or volumes
of transactions and follows up with investigation as
to causes.
B. Management has asked internal auditing to perform
regular audits of the controls over cash processing.
C. Management regularly compares divisional performance
with budgets for the division.
D. IT management regularly reconciles batch control totals
for items processed with batch controls for items
submitted.

Reconciling batch control totals being a processing control


is not part of the internal controls monitoring component.

71. Which of the following is a reason to establish internal


control?
A. To provide reasonable assurance that the entitys
objectives are achieved.
B. To safeguard the resources of the organization.
C. To encourage compliance with the organizational
objectives.
D. To ensure the accuracy, reliability, and timeliness of
information.

According to PSA 315, Internal control is the process


designed, implemented and maintained by those charged with
governance, management, and other personnel to provide
reasonable assurance about the achievement of the entitys
objectives with regard to reliability of financial reporting,
effectiveness and efficiency of operations, and compliance
with applicable laws and regulations.

Answers B, C, and D are incorrect because they are subsumed


under the overall purpose of providing reasonable assurance
that the entitys objectives are achieved.

72. An internal control system that is working effectively


A. Eliminates risk and potential loss to the entity.
B. Cannot be circumvented by management.
C. Reduces the need for management to review exception
reports on a day-to-day basis.
D. Is unaffected by changing circumstances and conditions
encountered by the entity.

An effective internal control system reduces the need for


management to spend tie reviewing exception reports on a day-
to-day basis. An entitys internal control system, if working
effectively, should prevent as well as detect and correct
exceptions.

Answer A is incorrect because some risks are unavoidable and


others are too costly to eliminate.

Answer B is incorrect because a basic limitation of internal


control is the potential for management override.

Answer D is incorrect because changes in circumstances and


conditions may require modification of internal control.

73. Because of inherent limitations, internal control cannot be


designed to provide reasonable assurance regarding the
achievement of objectives concerning
A. Effectiveness and efficiency of operations.
B. Elimination of all fraud.
C. Reliability of financial reporting.
D. Compliance with applicable laws and regulations.

74. When considering an entitys system of internal control, one


of the auditors major concerns is to ascertain whether
internal control is designed to provide reasonable assurance
that
A. Financial statements are fairly presented.
B. The accounting manager reviews all accounting
transactions.
C. Profit margins are maximized, and operational efficiency
is optimized.
D. Corporate morale problems are addressed immediately and
effectively.

Controls that are relevant to an audit pertain to the


entitys objective of preparing financial statements for
external purposes that are fairly presented in accordance
with an applicable financial reporting framework.

75. When an entitys internal control is functioning


effectively, management can expect various benefits. The
benefit least likely to occur is
A. Some assurance of compliance with laws and regulations.
B. Availability of reliable data for decision-making
purposes.
C. Reduced cost of an external audit.
D. Elimination of employee fraud.
Because of inherent limitations of any system of internal
control, even the most effective internal control cannot
guarantee the elimination of employee fraud.

76. Internal controls should be designed to provide reasonable


assurance that
A. Managements planning, organizing, and directing
processes are properly evaluated.
B. Managements plans have not been circumvented by employee
collusion.
C. Material errors or fraud will be prevented, or detected
and corrected, within a timely period by employees in the
course of performing their assigned duties.
D. The internal auditing departments guidance and oversight
of managements performance is accomplished economically
and efficiently.

Internal controls should be designed to prevent, or detect


and correct, material errors or fraud within a timely period
by employees in the normal course of their assigned duties.

Answer A is incorrect because it is the entitys internal


auditing department that evaluates management processes.

Answer B is incorrect because collusion is an inherent


limitation of internal control.

Answer D is incorrect because those charged with governance


are responsible for the guidance and oversight of management.

77. Which of the following is a correct statement about internal


control?
A. The cost-benefit relationship is a primary criterion that
should be considered in designing internal control.
B. The auditor can eliminate substantive tests on significant
account balances and classes of transactions for an
entity that has exceptionally strong internal control.
C. The internal auditor has the responsibility to establish
and maintain internal control.
D. Properly maintained internal control reasonably ensures
that collusion among employees cannot occur.

The cost-benefit relationship is a primary criterion in


designing internal control that is, the cost of a control
should not exceed its benefits. Because it is impossible to
precisely measure the costs and benefits of internal control,
both quantitative and qualitative estimates and judgments
are used by management to evaluate the relationship.
78. When obtaining knowledge about an entitys internal control,
it is important for the auditor to consider the competence
of its employees, because their competence bears directly
and importantly upon the
A. Cost-benefit relationship of internal control.
B. Comparison of recorded accountability with assets.
C. Achievement of the objectives of internal control.
D. Timing of substantive tests to be performed.

An important element of the control environment component of


internal control is human resource policies and practices.
Human resource policies and practices concern hiring,
orientation, training, evaluating, counseling, compensating,
promoting, and remedial actions. The objectives of internal
control cannot be achieved without sufficient competent
personnel who will operate the system.

Answer A is incorrect because the cost-benefit relationship


is a basic concept of internal control.

Answer B is incorrect because although comparison of recorded


accountability with assets is an important characteristic of
internal control, it is not directly related to the
competence of employees.

Answer D is incorrect because the timing of substantive tests


to be performed depends on a number of factors such as the
assessed level of risk of material misstatement.

79. Control activities are the policies and procedures that help
ensure that management directives are carried out. These
include activities relating to authorization, performance
reviews, information processing, physical controls, and
segregation of duties. There is proper segregation of duties
when an individual who
A. Records a transaction does not compare the accounting
record of the asset with the asset itself.
B. Authorizes a transaction records it.
C. Authorizes a transaction maintains custody of the asset
that resulted from the transaction.
D. Maintains custody of an asset has access to the accounting
records for the asset.

A proper segregation of duties requires that one person


should not be responsible for all phases of a transaction:
authorization, recording, and custodianship of the related
assets. Separate individuals should perform these
incompatible duties to reduce the opportunities for any
person to be in a position to both perpetrate and conceal
errors or fraud in the normal course of his/her duties.

80. Proper segregation of duties reduces the opportunities for


persons to be in positions to both
A. Establish internal control and authorize transactions.
B. Perpetrate and conceal errors or fraud.
C. Record cash receipts and cash disbursements.
D. Record transactions and prepare financial statements.

81. Control activities include those that relate to physical


controls over access to and use of assets and records. A
departure from the purpose of physical controls is that
A. The mail room clerk compiles a list of the checks received
in the incoming mail.
B. Access to safe-deposit box requires two officers.
C. Only warehouse personnel and production supervisors have
access to raw materials storeroom.
D. Only sales personnel use sales department vehicles.

82. Which of the following most likely would not be considered


an inherent limitation of internal control?
A. Management override
B. Incompatible functions
C. Mistakes in judgment
D. Collusion among employees

The performance of incompatible functions is not an inherent


limitation of internal control but a failure to segregate
functional responsibilities properly.

Answers A, C, and D are inherent limitations of internal


control.

83. A basic concept of internal control is the concept of


reasonable assurance, which recognizes that
A. Establishing and maintaining internal control is an
important responsibility of management.
B. Internal control may be ineffective due to collusion among
employees.
C. The cost of an entitys internal control should not exceed
the benefits expected to be derived.
D. Adequate safeguards over access to assets and records
should permit an entity to maintain proper
accountability.
84. An auditor should obtain sufficient knowledge of an entitys
information system, including the related business processes
relevant to financial reporting, to understand the
A. Policies used to detect the concealment of fraud.
B. Process used to prepare significant accounting estimates.
C. Safeguards used to limit access to computer facilities.
D. Procedures used to assure proper authorizations of
transactions.

The auditor should obtain an understanding of the information s


ystem relevant to financial reporting to understand:

a) The classes of transactions which are significant to th


e financial statements.
b) The procedures within both IT (Information Technology)
and manual systems by which those transactions are init
iated, recorded, processed, and reported in the financial
statements.
c) The accounting records and supporting documents for those
transactions.
d) How the information system captures events and conditions
, other than transactions, that are significant to the fi
nancial statements.
e) The financial reporting process used to prepare the entit
ys financial statements, including significant accountin
g estimates and disclosures.

Answers A, C, and D are incorrect because they describe


activities outside the accounting system relevant to
financial reporting.

85. The primary objective of procedures performed to obtain an


understanding of internal control is to provide an auditor
with
A. Knowledge necessary to plan the audit.
B. A basis for modifying tests of controls.
C. Information necessary to prepare flowcharts.
D. Evidence to use in reducing detection risk.

Understanding the entitys internal control system is a


matter that should be considered by an auditor in developing
overall strategy.

Answer B is incorrect because the auditors understanding


is of internal control is the basis for determining the
nature, timing, and extent of both substantive tests and
tests of controls.

Answer C is incorrect because flowcharts are but one form


of internal control documentation.

Answer D is incorrect because substantive tests are formed


to gather evidence in order to reduce detection risk to an
acceptable level.

86. In a financial statement audit, the auditor is required to


obtain an understanding of the entitys internal control to
assess the risk of material misstatement in the financial
statements. The result of the understanding

A. Bear no relationship to the nature, timing, and extent of


further audit procedures to be performed.
B. Must be reported to the stockholders and the SEC.
C. Are not reported to client management.
D. May be used as the basis for withdrawing from an audit
engagement.

PSA 315 states that the auditors understanding of internal


control may raise doubts about the auditability of an
entitys financial statements.

Concerns about the integrity of the entitys management may


be so serious as to cause the auditor to conclude that the
risk of management representation in the financial
statements is such that an audit cannot be conducted.

Also, concerns about the condition and reliability of an


entitys records may cause the auditor to conclude that it
is unlikely that sufficient appropriate audit evidence will
be available to support an unmodified opinion on the
financial statements. In such circumstances, the auditor
considers a qualification or disclaimer of opinion, but in
some cases, the auditors only recourse may be to withdraw
from the engagement.

87. In obtaining an understanding of internal control relevant


to the audit, an auditor is required to obtain knowledge
about the
A. Effectiveness of controls that have been implemented.
B. Consistency in which controls are currently being applied.
C. Design of the controls pertaining to internal control
components.
D. Controls related to each class of transactions and account
balance.

According to PSA 315, obtaining an understanding of internal


control involves evaluating the design of a control and
determining whether it has been implemented.

Evaluating the design of a control means determining whether


the control individually or in combination with other
controls capable of effectively preventing, or detecting
and correcting, material misstatements.

Implementation of a control means that the control exists


and that the entity is using it.
Answers A and B are incorrect because in some cases, the
auditor may determine that performing only substantive
procedure is appropriate for specific assertions and,
therefore, may choose not to obtain information about the
operating effectiveness of controls and their consistency
of application.

Answer D is incorrect because the standard does not


explicitly require obtaining knowledge about controls
related to each transaction class and account balance.

88. The auditor uses the understanding of internal control to

I. Identify types of potential misstatements.


II. Consider factors that affect the risks of material
misstatement.
III. Design the nature, timing, and extent of further audit
procedures.

A. I and II only
B. I and III only
C. II and III only
D. I, II, and III

89. In obtaining an understanding of internal control relevant


to the audit, the auditor may trace several transactions
through the system, including how the transaction interface
with any service organizations whose services are part of
the entitys information system. The primary objective of
this procedure is to
A. Evaluate the design of the internal control and determine
whether it has been implemented.
B. Determine the effectiveness of internal control.
C. Detect fraud.
D. Replace substantive tests.

90. Information about segregation of duties ordinarily is best


obtained by
A. Performing tests of transactions that corroborate
managements financial statement assertions.
B. Developing audit objectives that reduce control risk.
C. Observing employees as they apply specific controls.
D. Obtaining a flowchart of activities performed by entity
personnel.

According to PSA 315, risk assessment procedures to obtain


audit evidence about the design and implementation of
relevant controls may include:

Inquire entity personnel.


Observing the application of the controls.
Inspecting documents and reporting.
Tracing transactions through the information system
relevant to financial reporting

91. In obtaining an entitys internal control in a financial


statement audit, an auditor is not required to
A. Determine whether the controls have been implemented.
B. Perform procedures to evaluate the design of controls.
C. Document the understanding of the entitys internal
controls components.
D. Search for significant deficiencies in the operation of
internal control.

The auditor should obtain a sufficient understanding of


internal control to assess the risks of material
misstatement. The understanding includes knowledge about the
design of relevant controls and whether they have been
implemented, i.e., whether they have been placed in
operation. Though the auditor may become aware of material
weaknesses of internal control, he/she is not required to
search for such internal control weaknesses or deficiencies.
A financial statement audit is not designed to determine the
adequacy of internal control for management purposes.

92. In a financial statement audit, the auditor is required to


perform tests of controls when

I. The auditor's risk assessment includes an expectation of


the operating effectiveness of controls.
II. When substantive procedures alone do not provide
sufficient appropriate audit evidence at the assertion
level.

A. I only
B. II only
C. Either I or II
D. Neither I nor II

Under PSA 330 (The Auditor' Responses to Assessed Risks),


the auditor should perform tests of controls when his/her
assessment of risks of material misstatements at the
assertion level includes an expectation that controls are
operating effectively or when substantive procedures alone
do not provide sufficient appropriate evidence to reduce the
risk of material misstatement at the assertion level.

93. Which of the following determines the extent of the auditor's


tests of control?
A. Auditor's knowledge
B. Auditor's initial/planned assessment control of risk
C. Resources available to the auditor
D. Management's desire to help the auditor

94. After obtaining an understanding of internal control


relevant to the audit, the auditor may attempt to assess
control risk at below the maximum level. In turn, the auditor
will (1) identify specific controls that are likely to
prevent or detect material misstatements in the relevant
financial statement assertions and (2) perform tests of
controls. The purpose of the tests of controls is to
A. Evaluate inherent risk.
B. Assure that the auditor has a sufficient understanding
of internal control.
C. Evaluate the effectiveness of such controls.
D. Provide recommendations to management to improve internal
control.

The auditor performs tests of controls to obtain assurance


about the operating effectiveness of controls.

95. Tests of controls are concerned primarily with each of the


following questions, except
A. By whom were the controls applied?
B. Were the necessary controls consistently performed?
C. How were the controls applied?
D. Why were the controls applied?

Testing the operating effectiveness of controls includes


obtaining audit evidence about (1) how controls were applied
at relevant times during the period under audit, (2) the
consistency with which they were applied, and (3) by whom
or by what means they were applied. Determining why controls
were applied relates more to obtaining an understanding of
internal control than testing the effectiveness of controls.

96. In an audit of financial statements in accordance with PSAs,


an auditor is required to
A. Identify specific controls relevant to managements
financial statement assertions.
B. Determine whether controls are suitably designed to
prevent or detect material misstatement.
C. Perform tests, of controls to evaluate the operating
effectiveness of controls
D. Document the auditor's understanding of the entity's
internal control.

The auditor is required to document in the audit working


papers the understanding obtained of the entity's internal
control and the assessment of control risk. When control
risk is assessed at less than the maximum, the auditor should
also document the basis for the conclusion.

Answers A, B, and C are incorrect because these procedures


are to be performed only when the auditor wishes to assess
control risk at less than the maximum.

97. In performing a walk-through test, the


A. Auditor traces one or several transactions within each
major class through the transaction trail to confirm his
or her understanding of the information system and control
procedures.
B. Auditor performs a physical, review of the entity's
property, plant and equipment to confirm the physical
location of major assets.
C. Senior auditor "walks-through" the processing of the
entity's transactions with other members of the audit team
focusing on the control process which aim to prevent
misstatements in the financial statements.
D. Auditor traces one or several transactions within each
major class through the transaction trail to confirm the
cut-off period.

98. In evaluating the effectiveness of the design of an entity's


internal control system in order to make a preliminary
assessment of control risk for an assertion the auditor has
to

I. Identify potential misstatements that could occur in the


entity's assertion.
II. Identify the necessary controls that would be likely to
prevent or detect the material misstatements.
III. Evaluate the evidence and makes the assessments.

A. I and II only
B. I and III only
C. II and III only
D. I, II, and III

99. The ultimate purpose of assessing control risk is to


contribute the auditor's evaluation of the risk that
A. Material misstatements may exist in the financial
statements.
B. Specified controls requiring segregation of duties may
be circumvented by collusion.
C. Entity's control may be overridden by management.
D. Tests of controls may fail to identify procedures
relevant to assertions.

The ultimate purpose of assessing control risk at the


assertion level for each material account balance or class
of transactions is to contribute to the auditor's evaluation
of the risk that material misstatements exist in the
financial statements.

100. An auditor may decide to assess control risk at a high


level for some or all assertions because the auditor
believes
A. The entity's internal control system is not effective.
B. More emphasis on tests of controls than substantive tests
is warranted.
C. Sufficient appropriate audit evidence to support the
assertions is likely to believe.
D. The entity's internal control components are
interrelated.

The auditor ordinarily assesses control risk at a high level


for some or all assertions when:
a) the entity's internal control system is not
effective; or
b) evaluating the operating effectiveness of the
entity's control would not be efficient.

101. An auditor assesses control risk in terms of


A. Types of potential fraud.
B. Financial statement assertions.
C. Specific control activities.
D. Control environment elements.

The assessment of control risk is the process of evaluating


the effectiveness of an entity's internal control in
preventing or detecting and correcting material
misstatements. Control risk is assessed in terms of
financial statements assertions.

102. The preliminary assessment of control risk for a financial


statement assertion should be high unless the auditor
I. Is able to identify controls relevant to the assertion
which are likely to prevent, or detect and correct,
material misstatements.
II. Plans to perform tests of controls to support the
assessment.

A. I only
B. II only
C. Either I or II
D. Neither I nor II

103. In performing tests of the operating effectiveness of an


entity's controls, an auditor selects from a variety of
techniques, including
A. Reperformance and observation.
B. Inquiry and analytical procedures.
C. Comparison and confirmation.
D. Inspection and verification.

Tests of controls ordinarily include inquiry, observation,


inspection, and reperformance of a policy or procedure that
pertains to an assertion.

Answer B, C, and D are incorrect because they include


procedures which are more closely associated with
substantive testing namely, analytical procedures,
comparison, confirmation, and verification.

104. An auditor intends to perform tests of control on a client's


cash disbursements procedures. If the control procedures
leave no audit trail of documentary evidence, the auditor
most likely will test the procedures by
A. Inquiry and analytical procedures.
B. Inquiry and observation.
C. Analytical procedures and confirmation.
D. Confirmation and observation.

According to PSA 330, documentation of operation of controls


may not exist for some factors in the control environment,
such as assignment of authority and responsibility, or for
some types of control activities, such as those performed
by a computer. In such cases, audit evidence about operating
effectiveness may be obtained through inquiry in combination
with other procedures like observations and the use of
computer-assisted audit techniques (CAATs).

105. Which of the following procedures concerning accounts


receivable would an auditor most likely perform to obtain
audit evidence in support of an assessed level of control
risk that is less than high?

A. Comparing an entity's bad debt expense to actual bad debt


losses.
B. Inspecting an entity's analysis of accounts receivable
for unusual balances.
C. Observing an entity's staff prepare the schedule of past
due accounts receivable.
D. Sending confirmation requests to an entity's principal
customers to verify the existence of accounts receivable.

An auditor who wishes to assess control risk at less than


high should perform tests of operating effectiveness of
relevant controls. Tests of controls include inquiry,
observation, inspection, and reperformance of a control by
the auditor.

106. The following statements relate to an auditor's assessment


of control risk. Which is true?
A. The lower the assessed level of control risk, the less
assurance the audit evidence must provide about operating
effectiveness of controls.
B. The basis for an auditor's conclusion about the assessed
level of control risk need not be documented unless control
risk is assessed at the maximum level.
C. Assessing control risk and obtaining an understanding of
an entity's internal control may be performed
concurrently.
D. When assessing control risk, an auditor should not consider
evidence obtained in prior audits about the operation of
controls.

For efficiency reasons, an auditor may plan to perform tests


of controls when evaluating the design of controls and
obtaining evidence of their implementation.
107. The following statements relate to the use of audit evidence
when testing the operating effectiveness of relevant
controls. Which is false?
A. An auditor who obtains sufficient appropriate audit
evidence about the operating effectiveness of controls
during the interim period should no longer obtain
additional evidence of operating effectiveness for the
remaining period.
B. An auditor may plan to use audit evidence about the
operating effectiveness of controls obtained in the prior
audits.
C. If an auditor plans to rely on controls that have changed
since they were last tested, the auditor should test the
operating effectiveness of such controls in the current
audit.
D. Audit evidence pertaining only to a point in time may be
sufficient for the auditor's purpose, for example, when
testing controls over an entity's physical count of
inventories at year-end.

PSA 330 states that when the auditor obtains audit evidence
about the operating effectiveness of controls during an
interim period, the auditor should determine what additional
audit evidence should be obtained for the remaining period.
In addition, the auditor is required to obtain audit
evidence about the nature and extent of any significant
changes in internal control, including changes in the
information system, processes, and personnel that occur
subsequent to the interim period.

Answers B and C are correct statements. PSA 330 states that


if the auditor plans to use audit evidence about the
operating effectiveness of controls obtained in prior
audits, the auditor should obtain audit evidence about
whether changes in those specific controls have occurred
subsequent to the prior audit. In this regard, the auditor
may perform inquiry in combination with observation or
inspection. If the auditor plans to rely on controls that
have changed since they were last tested, the auditor is
required to test the operating effectiveness of such
controls in the current audit.

Answer D is a correct statement. According to PSA 330, the


timing of tests of controls depends on the auditor's
objective and determines the period of reliance on those
controls.

Tests of controls at a particular time provide evidence of


operating effectiveness at that time, while tests controls
throughout a period provide evidence of operating
effectiveness for that period.

Audit evidence of operating effectiveness at a point in time


may be sufficient for the auditor's purpose, such as tests
of controls over physical inventory at year-end.

108. According to PSA 330, an auditor who plans to rely on


controls that have not changed since they were last tested
should test the operating effectiveness of such controls at
least once in every
A. Second audit
B. Third audit
C. Fourth audit
D. Fifth audit

An auditor may plan to rely on controls that have not changed


since they were last tested. The length of time period
between retesting such controls is a matter of professional
judgment but cannot exceed two years. Therefore, the
operating effectiveness of such controls should be tested
at least once in every third audit.

109. Which of the following procedures are performed by an


auditor who wants to detect material misstatements at the
assertion level?
A. Compliance tests
B. Tests of controls
C. Substantive tests
D. Dual-purpose tests

Substantive tests are procedures performed in order to


detect material misstatements at the assertion level,
including tests of details of account balances, classes of
transactions, and disclosures, and substantive analytical
procedures.

110. After gaining an understanding of internal control and


assessing the risks of material misstatement, an auditor
decided to perform tests of controls. The auditor most
likely decided that
A. Additional evidence to support a further reduction in
control risk is not available.
B. It is not possible or practicable to reduce the risks of
material misstatements at the assertion level to an
acceptably low level with audit evidence obtained only
from substantive test procedures.
C. There were many internal control weaknesses that could
allow misstatements to enter the accounting system.
D. An increase in the assessed level of control risk is
justified for certain financial statement assertions.

An auditor shall perform tests of controls when:

1. The auditor's risk assessment includes an expectation


of the operating effectiveness of controls; or
2. Substantive procedures alone do not provide
sufficient appropriate audit evidence at the
assertion level.

Answer A is incorrect because tests of controls are


performed to obtained sufficient appropriate audit evidence
to support a further reduction in control risk.

Answer C is incorrect because the auditor should not perform


tests of controls when, as a result of his/her assessment,
there is no expectation of the operating effectiveness
control, i.e., if there are many internal control
weaknesses.

Answer D is incorrect because tests of controls are designed


to decrease the auditor's preliminary assessment of control
risk, not increase it.

111. In conducting an audit in accordance with PSAs, the auditor


is required to identify and assess the risk of material
misstatement at financial statement level, and at the
assertion level for classes of transactions, account
balances, and disclosures. Some of these risks, in the
auditor's judgment, require special audit consideration,
such as those involve fraud or complex transactions. Such
risks are called
A. Business risk
B. Audit risk
C. Significant risk
D. Material risk

112. As a result of obtaining an understanding of the entity's


internal control system, the auditor may become aware of
material weaknesses in the design or implementation of
internal control. The auditor is required to report this
mater to
A. Those charged with governance or management
B. Chief executive office
C. Securities and Exchange Commission
D. Board of Accountancy

PSA 315 states, "The auditor shall communicate material


weaknesses in internal control identified during the audit
on a timely basis to management at an appropriate level of
responsibility, and, as required by PSA 260, (Communication
with Those Charged with Governance), with those charged with
governance (unless all of those charged with governance are
involved in managing the entity.)"

113. The auditor's final assessment of control risk for financial


statement assertions is based on
A. Evaluating the evidence obtained from the entity's
management.
B. Evaluating the evidence obtained from third parties.
C. Evaluating the evidence obtained from an understanding
of relevant internal control system components and
related tests of controls.
D. Evaluating the evidence obtained from an understanding
of relevant internal control system components.

REVENUE/RECEIPT CYCLE

114. Which of the following is not a source document for the


revenue cycle?
A. Sales order
B. Receiving report
C. Credit memo
D. Delivery receipt

115. Which of the following activities is not part of the revenue


cycle?
A. Sales order entry
B. Receiving
C. Billing
D. Shipping

116. In the revenue cycle, a customer places an order for a


certain product. What step should be taken before the order
is checked for inventory availability?
A. A packing list should be generated for the warehouse.
B. The customer's credit should be checked or a sale on
account.
C. The sales order should created and written to a file.
D. The shipping department should be notified of an order
in process.

117. ________________ is the maximum allowable account balance


for a given customer.
A. Credit limit
B. Reorder point
C. Credit checkpoint
D. Backorder point

118. An essential part of the revenue cycle is filling customer


orders and shipping goods to customer. Automating warehouse
systems cut costs, improves efficiency, and enables more
customer-responsive shipments. Which of the following
elements is required for an automated perpetual inventory
system?
A. Forklifts
B. Conveyor belts
C. Bar-code scanners
D. VAN

119. The basic document created in the billing process is the


A. Bill of lading
B. Purchase order
C. Sales invoice
D. Packing list
120. To ensure proper segregation of duties, who should make
decisions concerning issuance of credit memos?
A. Warehouse manager
B. Credit manager
C. Cashier
D. Accounts receivable supervisor

121. Which of the following is not an objective of the


revenue/receipt cycle?
A. Received goods are counted and inspected or quality.
B. Custody over assets resulting from the revenue/receipt is
properly maintained.
C. Transactions related to the revenue/receipt cycle are
properly authorized, executed, and recorded.
D. Approved sales orders are shipped on a timely basis and
in accordance with customer specifications.

The revenue/receipt cycle encompasses the activities


pertaining to the sale of goods or services and the
collection of cash for the goods or services provided.
Counting and inspecting received goods for quality is an
objective of the expenditure/disbursement cycle and is only
indirectly related to the revenue/receipt cycle.

122. The following are directly involved in revenue/receipt


cycle, except?
A. Treasurer and controller
B. Receiving department clerk
C. Billing clerk
D. Sales manager and the credit manager

The receiving department clerk counts and inspects the goods


received from suppliers and prepares receiving reports which
serve as partial authorization for invoice payment. The
activity relates more directly to expenditure/disbursement
cycle.

Answer A is incorrect because the treasurer has custody of


cash receipts from customers and the controller maintains
records such as those relating to sales and cash receipts.

Answer C is incorrect because the billing clerk is


responsible for the billing process, including the
preparation of the sales invoices.

Answer D is incorrect because the sales manager executes


sales transactions and the credit manager is responsible for
authorizing sales.

123. Which of the following controls most likely would provide


reasonable assurance that all credit sales transactions of
an entity are recorded?
A. The accounting department supervisor controls the mailing
of monthly statements to customers and investigates any
differences reported by customers.
B. The accounting department supervisor independently
reconciles, on a monthly basis, the account receivable
subsidiary ledger to the accounts receivable control
account.
C. The billing department supervisor matches prenumbered
shipping documents with entries in the sales journal.
D. The billing department supervisor sends copies of
approved sales orders to the credit department for
comparison to authorized credit limits and current
customer account balances.

Comparing prenumbered shipping documents with entries in the


sales journal will detect unrecorded sales transactions.

Answer A is incorrect because unrecorded credit sales


transactions result to understatement of receivables.
Customers are unlikely to report understatement of their
accounts.

Answer B is incorrect because the reconciliation will not


detect unrecorded sales transactions.

Answer D is incorrect because credit approval does not


provide assurance that all sales transactions have been
recorded.

124. For effective internal control, employees maintaining the


accounts receivable subsidiary ledger should not also
approve
A. Cash disbursements
B. Write-offs of customer accounts
C. Granting of credit customers
D. Employee overtime wages

Segregating the functions of authorization, record-keeping,


and custody of assets provides reasonable assurance that
perpetration and concealment of fraud or error by the same
employee will be minimized. Therefore, the authorization
function of approving write-offs of receivables should not
be done by the same employee who performs a record-keeping
function of maintaining the accounts receivable subsidiary
ledger.

Answer A and D are incorrect because authorizing cash


disbursements and employee overtime wages are not related
to receivables.

Answer C is incorrect because although granting of credit


authorizes a debit to receivable, combining this function
with maintaining the accounts receivable subsidiary ledger
is less likely to result in fraud.

125. A sound internal control procedure should require that


defective merchandise returned by the customers be
presented initially to the
A. Receiving clerk
B. Accounts receivable supervisor
C. Billing clerk
D. Shipping department supervisor

Sound internal control procedures dictate that all receipts


of merchandise should be initially handled by the receiving
department personnel. Receiving reports must be prepared
for all goods received.

126. An employee misappropriates cash receipts from sales on


account. Which of the following acts would conceal this
defalcation and be least likely to be detected by an
auditor?
A. Understating the cash receipts journal.
B. Overstating the accounts receivable subsidiary ledger.
C. Overstating the accounts receivable control account.
D. Understating the sales journal.

The most effective way of concealing theft of cash receipt


is by not recording sales on account. The procedure that
will be applied in the accounting records will not detect
the defalcation because the accounts remain balanced, yet
incomplete. In addition, customers are unlikely to report
the irregularity to the auditor because unrecorded sales
will not be billed and unrecorded receivables will not be
confirmed.

Questions 127 through 130 are based on the following information:

In connection with your audit of the financial statements of ABC


Company, you gathered the following information about its sales
procedures:

Customer orders are received by the sales-order department. A


clerk computes the peso amount of the order and sends it to the
credit department for approval. Credit approval is stamped on
the order and returned to sales order department. A sales invoice
is prepared, and the sales order is filled in the customer order
file.

The customer copy of the sales invoice is sent to the billing


department and held in the pending file awaiting notification
that the order has been shipped.

The shipping copy of the sales invoice is routed to warehouse


and the shipping department as authority for the respective
departments to release and ship the goods.

Shipping department personnel pack the order and prepare a three-


copy bill of lading:
Original copy - mailed to the customer
Second copy - sent with the shipment
Third copy - filed in sequence in the bill of
lading file

The shipping copy of the sales invoice is sent to the billing


department.

The billing clerk matches the received shipping copy of the


sales invoice with the customer copy from the pending file. Both
copies of the invoice are priced, extended, and footed. The
customer copy is then mailed to the customer, and the shipping
copy is sent to the accounts receivable clerk.

The accounts receivable clerk enters the sales invoice data in


the sales journal, posts the customer's account in the accounts
receivable subsidiary ledger, and files the shipping copy in the
sales invoice file. The sales invoices are numbered and filed
in sequence.

127. To obtain evidence, concerning the proper credit approval


of sales, the auditor would select a sample of transaction
document from the population represented by the
A. Bill of lading file
B. Accounting receivable subsidiary ledger
C. Sales invoice file
D. Customer order file

As described, the entity's procedure is to send the customer


order to the credit department for approval. Approval is
stamped on the customer order which is then returned to the
sales department and filed. Thus, the only documentation for
credit approvals is the customer order file.

128. In determining whether the internal control operated


effectively to minimize errors of failure to post sales
invoices to the accounts receivable subsidiary ledger, the
auditor should select a sample of transactions from the
population represented by the
A. Sales invoice file
B. Accounts receivable subsidiary ledger
C. Customer order file
D. Bill of lading file

A sample of sales invoices should be traced to the accounts


receivable subsidiary ledger. Sales invoices without
corresponding entries in the accounts receivable subsidiary
ledger represent transaction not posted.

129. To effectively determine whether the entity's control


activities minimize errors of failure to invoice goods that
have been shipped, the auditor should select a sample of
transactions from the population represented by the
A. Sales invoice file
B. Bill of lading
C. Accounts receivable subsidiary ledger
D. Customer order file

To determine if goods that have been shipped were not


invoiced, an effective audit procedure is to match a sample
from the bill of lading file to sales invoices.

130. To obtain evidence that uncollected items in customer's


accounts represented valid trade receivables, the auditor
should select a sample of items from the population
represented by the
A. Bill of lading
B. Customer order file
C. Accounts receivable subsidiary ledger
D. Sales invoice file

Confirmation directly with customers of a sample of open


accounts from the accounts receivable subsidiary ledger
would provide an auditor with evidence that the receivables
are valid assets.

131. Which of the following most likely would be the result of


ineffective controls in the revenue/receipt cycle?
A. Omission of shipping documents could go undetected,
causing an understatement of inventories.
B. Irregularities in recording transactions in the
subsidiary accounts could result in a delay in goods
shipped.
C. Final authorization of credit memos by sales department
personnel could permit an employee defalcation scheme.
D. Fictitious transactions could be recorded causing an
understatement of revenues and an overstatement of
receivables.

Final approval of credit memos in the sales department may


allow sales department personnel to commit irregularities.

Answer A is incorrect because if shipping documents are


omitted, shipped goods may not be credited to inventory,
causing the account to be overstated, not understated.

Answer B is incorrect because posting in the subsidiary


accounts takes place only after shipment of goods.

Answer D is incorrect because recording fictitious sales


would overstate revenues.

132. At which point in an ordinary sales transaction of a


wholesaling business is a lack of specific authorization of
least concern to the auditor in performing risk assessment
procedure?
A. Granting of credit
B. Determination of discounts
C. Selling of goods for cash
D. Shipment of goods

Selling of goods for cash would likely be covered by a


general authorization.

Answers A, B, and D are incorrect because granting of credit,


determination of discounts, and shipment of goods may
require specific authorization.

133. EFG Company uses its sales invoice for posting perpetual
inventory records. Inadequate internal control over the
invoicing function allows good to be shipped but not
invoiced. The inadequate controls could cause what type of
misstatement in each of the following accounts?

Revenues Receivables Inventories


A. Understatement Understatement Understatement
B. Overstatement Overstatement Understatement
C. Understatement Understatement Overstatement
D. Overstatement Overstatement Overstatement

If goods are shipped but are not invoiced, there will be no


documentation for sales, thus understanding revenues and
receivables. Moreover, inventory will not be credited
thereby overstating the account.

134. Which of the following controls is often lacking in a retail


cash sales environment?
A. Segregation of functions
B. Competent personnel
C. Monitoring
D. Access to assets limited to authorized personnel.

In a retail cash sales environment, the sales clerk


ordinarily authorizes and records the transactions and has
custody of assets. But the apparent lack of proper
segregation o functions is compensated by controls such as
close supervision of the cash registers that limits access
to assets, and an effective internal recording function that
maintains control over cash receipts.

135. An auditor's risk assessment procedures disclosed that the


accounts receivable clerk approves credit memos and has
access to cash. Which of the following controls would be
most effective in offsetting these weaknesses?
A. The controller reconciles accounts to the amount shown
in the ledger.
B. The owner reviews credit memos after they are recorded.
C. The controller receives the monthly bank statement
directly and reconciles the cash accounts.
D. The owner reviews errors in billings to customers and
postings to the subsidiary ledger.
The clerk is holding incompatible duties and thus, is an
position to both perpetrate and conceal a fraud in the normal
course of his/her duties.

The clerk has custody of cash, authorizes credit memos, and


performs the record-keeping function for accounts
receivable. Hence, the clerk can easily misappropriate cash
received from customers and conceal such defalcation by
recording fictitious sales returns.

In a small business, it may not be practicable to institute


formal control activities. In this situation, the active
involvement of the owner may compensate for the absence of
some control activities such as proper segregation of
duties. The owner may consider reviewing credit memos for
authenticity.

Answer A is incorrect because there will be no discrepancy


between the subsidiary ledger and the control account even
if improper credits to accounts receivable are posted.

Answer C is incorrect because reconciliation of cash


accounts will not reveal misappropriation of cash collected
from customers concealed by recording improper credit memos.

Answer D is incorrect because the absence of errors is


billings and postings to the subsidiary ledger would not
preclude the clerk from defalcating cash collected from
customers.

136. The following controls are ordinarily found in an entity's


revenue/receipt cycle. Which controls most likely would be
effective in offsetting the tendency of sales personnel to
maximize sales volume at the expense of high bad debt write-
offs?
A. Employees responsible for authorizing sales and bad debt
write-offs are denied access to cash.
B. Subsidiary account receivable ledgers are reconciled to
the control account by an employee independent of the
authorization of credit.
C. Employees involved in the credit-granting function are
separated from the sales function.
D. Shipping documents and sales invoices are matched by an
employee who does not authorize write-offs of customers'
accounts.

The sales department's function should be confined to the


generation of sales and provision of services to customers.
The credit approval process should be handled by the credit
department.

The problem is the incompatibility of the sales and the


creditgranting functions. The controls described in answers
A, B, and D do not address the issue.
137. Which of the following control activities in an entity's
revenue/receipt cycle would provide reasonable assurance
that all billed sales are correctly posts to the accounts
receivable ledger?
A. Each shipment of goods on credit is supposed by a
prenumbered sales invoice.
B. The accounts receivable subsidiary ledger is reconciled
daily to the accounts receivable control account in the
general ledger.
C. Daily sales summaries are compared to daily postings to
the accounts receivable ledger.
D. Each sales invoice is supported by a prenumbered shipping
document.

Daily sales summaries include information about billed


sales. Reconciliation of these summaries to the accounts
receivables ledger would provide reasonable assurance that
all billed sales were posted.

Answer A is not incorrect because matching shipment with


sales invoices would provide reasonable assurance that goods
shipped were billed.

Answer B is incorrect because reconciling the subsidiary


ledger with the control account would only prove that the
aggregate amount debited to the subsidiary ledger agrees
with the amount debited to the control account in the general
ledger.

Answer D is incorrect because if sales invoices are


supported by shipping documents, it would assure that billed
sales were actually shipped.

138. Which of the following tests of controls would most likely


be performed by an auditor to obtain evidence about
management's assertion concerning the completeness of sakes
transactions?
A. Inquiries about the entity's credit granting policies and
whether credit checks are consistently applied.
B. Inspect the entity's reports of prenumbered shipping
documents that have not been recorded in the sales
journal.
C. Compare prices on prenumbered sales invoices to the
entity's authorized price list.
D. Verify that extensions and footings on sales invoices and
monthly statements of customers' accounts have been
checked.

Inspecting shipping that has not been recorded in the sales


journal will possibly disclose items that have been sold as
evidence by the shipping documents but were not recorded as
sales.
139. An online sales order processing system most likely would
have an advantage over a batch sales order processing system
by
A. Enabling shipment of customer orders to be initiated as
the orders are received.
B. Maintaining more accurate records of accounts receivables
and inventory.
C. Maintaining backup copies of the database.
D. Detecting errors in the data entry process by the use of
edit checks.

In an online system, terminals installed in different


locations are directly connected to the computer network
that makes it possible to process transactions as they are
entered. Hence, shipments of customer orders can be
initiated instantaneously as they are received. On the other
hand, batch processing involves processing of transactions
in batches on a delayed basis.

140. When an office supply company is unable to fill an order


completely, it marks the out-of-stock items as back ordered
on the customer's order file that forms and enters these
items in a back order file that can be viewed or printed by
management. Customers are becoming dissatisfied with the
company because of its failure to keep track of and ship
out-of-stock items as soon are they are available. Which of
the following is the best approach to ensure prompt delivery
of out-of-stock items?
A. Increase inventory levels to minimize the occurrence of
stock outs.
B. Match the back order file to goods received daily.
C. Reconcile the sum of filled and back orders with the
total of orders of all orders placed daily.
D. Implement electronic data interchange (EDI) with supply
vendors to decrease the time to replenish inventory.

The inability to keep track of and ship out-of-stock items


as soon as they become available may be addressed by matching
the back order file to goods received daily. Once
identified, appropriate action on unfilled orders could then
be made.

Answers A, C, and D, are incorrect because the approaches


described do not ensure prompt delivery of items that are
out of stocks.

EXPEDITURES/DISBURSEMENT CYCLE

141. Which of the following is a source document that would be


found in the expenditure cycle?
A. Journal voucher
B. Purchase order
C. Time card
D. Delivery report
142. ____________ is a standing order to purchase specified items
at a designated price, from a particular supplier for a set
period of time.
A. Set order
B. Blanket purchase order
C. Purchasing order
D. Commodity order

143. Which of the following is probably the most effective


control or the prevention of kickbacks to purchasing agents?
A. Review a vendor performance.
B. A corporate policy to prohibit purchasing agents from
accepting kickbacks.
C. Purchasing from approved vendors.
D. Good supervision in the purchasing area.

144. The easiest way to prevent the acceptance of unordered


goods is to
A. Always require that a valid purchase order exists before
goods can be accepted at the time of delivery.
B. Order only from approved vendors.
C. Have an appropriate conflict of interest policy in place.
D. Require receiving department personnel to call the vendor
before accepting any goods.

145. The auditor's primary objective in obtaining an


understanding of the client's control over the purchasing
function is to
A. Investigate the recording of unusual transactions
regarding raw materials.
B. Determine the reliability of financial reporting by the
purchasing function.
C. Observe the annual physical count.
D. Ascertain that raw materials paid for are on hands.

Controls are the relevant of to the audit are those that


relate to the reliability of financial reporting.

Answer A is incorrect because investigation of the recording


of unusual transactions is an audit procedure, not an audit
objective.

Answer C is incorrect because observation of the annual


physical count of inventories relates more to the production
cycle of the entity.

Answer D is incorrect because materials paid for need not


be on hand materials may have been sold or used in
production.

146. In a well-designed internal control system, employees in


the same department approves purchase orders and also
A. Negotiate terms with vendors
B. Authorize requisitions of goods.
C. Inspect and count goods upon receipt
D. Reconcile the open invoice file,

In a well-designed internal control system, the entity's


purchasing department approves purchase orders and
negotiates terms with vendors as part of the authorization
process.

Answers B, C, and D are incorrect because authorizing the


requisition of goods, inspecting and counting goods
received, and reconciling the open invoice file are
functions incompatible with the authorization performed by
the purchasing department.

147. Effective controls relevant to purchasing of raw materials


should usually include all of the following, except
A. Determining the need for the raw materials prior to
preparing the purchase orders
B. Systematic reporting of product changes that will affect
the raw materials.
C. Obtaining financial approval prior to making a
commitment.
D. Obtaining third-party written quality and quantity
reports prior to payment for the raw materials.

Third-party written quality and quantity reports are not


ordinarily sought before making the payment. The entity
receiving department personnel usually have sufficient
knowledge about the purchased goods. Only in exceptional
cases when an outside may be considered.

Answer A is incorrect because determining the need for the


raw materials before preparing the purchase order will
ensure that only needed materials will be ordered.

Answer B is incorrect because reporting of product changes


affecting raw materials ensures that obsolete materials will
not be ordered.

Answer C is incorrect because obtaining financial approval


before making a commitment is important to ensure that
sufficient funds will be available on payment date.

148. As part of its purchasing system, entity's receiving


department receives copies of purchase orders for use in
identifying and recording goods received. The purchase
orders list the name of the vendor and the quantities of
material ordered. A possible error that this system could
allow is
A. Payment for unauthorized purchases
B. Payment to unauthorized vendors
C. Overpayment of partial deliveries
D. Delay in recording purchases.
If the quantities of the materials ordered are include in
the purchase order copy that is sent to the receiving
department, receiving department personnel may be tempted
to just copy the quantities ordered from the purchase order
when they prepare receiving reports. Hence, a possible error
that this system could allow is overpayment for partial
deliveries.

From the standpoint of a good internal control, a "blind


copy" of the purchase order-that is, no indication of the
quantities ordered-should be sent to the receiving
department. That will ensure that its personnel will conduct
an actual count and inspection of goods received.

149. Which of the following controls is not usually performed


in the accounts payable department?
A. Indicating on the vouchers the affected asset and expense
to be debited.
B. Approving vouchers for payment by having authorized
employee sign the vouchers.
C. Accounting for unused prenumbered purchase orders and
receiving reports.
D. Matching the vendor's invoice with the related purchase
requisition, purchase order, and receiving report.

The accounts payable department processes vendor invoices


for payment. Employees in this department should not perform
functions related to purchasing and receiving goods.

Accounting for unused prenumbered purchase orders and


receiving reports should be performed by employees in the
purchasing and receiving departments, respectively.

Answers A, B, and D are incorrect because the


responsibilities described are all functions of accounts
payable department.

150. Which of the following procedures would best discourage the


resubmission of vendor invoices after they have been paid?
A. The mailing of payments directly to payees by accounts
payable department personnel.
B. A requirement for double endorsement of checks.
C. The cancellation of vouchers by accounting personnel.
D. The cancellation of vouchers by treasurer personnel.

Canceling the voucher package voucher and supporting


documents upon payment prevents duplicate payment of a
voucher. This should be done in the treasurer's office to
ensure that the document will not be recycled for duplicate
payments.

Answer A is incorrect because mailing payments directly


to payees does not prevent recycling of invoices by
unscrupulous individuals.

Answer B is incorrect because a single endorsement is not


a control weakness provided that proper supporting
documents are required before the check is signed and the
signatory does not have incompatible duties.

Answer A is incorrect because the voucher package should


not be canceled before payment.

151. A university does not have a centralized receiving


function for departmental purchases of books, supplies,
and equipment. Which of the following controls would most
effectively prevent payment for goods not received, if
performed prior to invoice payment?
A. Vendor invoices should be approved by a departmental
supervisor other than the employee ordering the goods.
B. Invoices for a specified amount should be approved by
the vice president of finance.
C. Names and addresses on vendor invoices should be
compared to a list of department-authorized vendors.
D. Vendor invoices should be matched with department
purchase orders.

The departmental supervisors would most likely be aware


f all goods received by their departments. Thus, approval
of vendor invoices by departmental supervisors would most
effectively prevent payment for goods not received.

Answer B is incorrect because the vice president of


finance does not receive incoming goods.

Answer C and D is incorrect because comparing invoices


with lists of authorized vendors and matching invoices
with purchase orders do not provide evidence that goods
were actually received.

152. To minimize the risk that purchasing agents will use their
positions for personal gain, an entity should

A. Direct the purchasing department to maintain records on


purchase prices paid, to be reviewed every 6 months.
B. Request internal auditors to send confirmation requests
to selected vendors.
C. Require competitive building.
D. Specify that all items purchased must pass quality
control tests.

The purchase of goods through competitive bidding will reduce


costs and the possibility that purchasing agents will have
side agreements with vendors.
The controls described in answers A,B and D are detective, not
preventive controls.

153. The following are appropriate questions on an internal


control questionnaire concerning purchase transactions,
except

A. All are goods received in a centralized receiving


department counted, inspected, and compared with purchase
orders on receipt?
B. Are intact cash receipts deposited daily in the bank?
C. Are pre-numbered purchase orders and receiving reports
used and accounted for?
D. Are an approved purchase requisition and a signed
purchase order required for each purchase?

The question about the daily deposit of intact cash receipts


relates to the revenue/receipt cycle, not the expenditure/
disbursement cycle.

154. The following questions ordinarily appear in an internal


control questionnaire on cash disbursements, except

A. Are pre-listings made of all cash receipts?


B. Is each check supported by an approved voucher?
C. Are imprinted and pre-numbered checks used and is a
check provision device used in printing the check amount?
D. Are all disbursements except for petty cash made by
check?

Pre-listing of cash receipts is part of the revenue/receipt


cycle.

155. In a well-designed internal control, the same employee may be


permitted to

A. Prepare receiving reports and also approve purchase


orders.
B. Approve vouchers for payment and also have access to
unused purchased orders.
C. Mail signed checks and also cancel supporting documents
D. Mail signed checks and also prepare bank reconciliations.

In a typical cash disbursement system, approved check vouchers


and supporting documents are forwarded to the cash
disbursements department. This department, having a custodial
function, is responsible for signing checks, cancelling
supporting documents, and mailing signed checks.

Answer A is incorrect because approving purchase orders is a


function of the purchasing department. The receiving
department, responsible for the preparation of receiving
reports, should not know the quantity ordered.
Answer B is incorrect because vouchers are approved in the
accounts payable department, and only the purchasing
department should be allowed access to unused purchase orders.

Answer D is incorrect because the bank reconciliation should


be performed by an employee who does not have a custodial
responsibility.

156. Which of the following is of least concern to an auditor in


assessing the risks of material misstatement?

A. Signed checks are distributed by the controller to


approved payees.
B. Checks are signed by one person.
C. Cash receipts are not deposited intact daily.
D. Treasurer does not verify the names and addresses of
check payees.

Answer A is incorrect because the controller, who performs a


record-keeping function, should not have access to signed
checks.

Answer C is incorrect because daily deposit of intact cash


receipts should be required to minimize defalcation.

Answer D is incorrect because the treasurer should sign checks


only after verification of supporting documentation has been
performed.

157. Under which of the following circumstances would an auditor be


most likely to intensify an audit of a P100,000 petty cash
fund?

A. Petty cash vouchers are not pre-numbered.


B. The custodian endorses reimbursement checks.
C. Reimbursement occurs twice each week.
D. The custodian occasionally uses the petty cash fund to
cash employee checks.

Frequent reimbursement of petty cash fund (for example, twice


each week) suggests that the fund is not functioning as
intended. In this case, the auditor may need to intensify the
audit of the petty cash fund.

Answer A and D are incorrect because although failure to pre-


number vouchers and occasional use of the fund to cash employee
checks are not preferable practices, they do not necessarily
lead to commission of errors or fraud.

Answer B is incorrect because the custodians endorsement


is required to cash a replenishment check.

158. In ABC Companys accounting system, the quantities counted by


the receiving department and entered at a terminal are
transmitted to the computer, which immediately transmits the
amounts back to the terminal for display to enable the
operator to
A. Verify that the amount was entered accurately.
B. Establish the validity of the account number.
C. Verify the authorization of the disbursement.
D. Prevent the overpayment of the account.

The display of the amounts entered is called closed-loop


verification-an effective control to verify the accuracy of
data input.

159. What document is prepared to authorize the removal of the


necessary quantity of raw materials from storeroom to factory?

A. Production order
B. Materials requisition
C. Movie ticket
D. Purchase invoice

160. Which of the following is an essential control procedure to


ensure the accuracy of the recorded inventory quantities?

A. Calculating unit costs and valuing obsolete or damaged


inventory items in accordance with inventory policy.
B. Testing inventory extensions.
C. Performing a gross profit test.
D. Establishing a cutoff for goods received and shipped.

Establishing a cutoff for goods received and shipped would


ensure that only goods owned by the entity are included
in inventory.

Answers A, B, and C are incorrect because the procedures


described relate more directly to inventory valuation rather
than inventory quantities.

161. The following questions are appropriate for an internal control


questionnaire concerning inventory except

A. Are goods stored in locked storage areas?


B. Is access to the storage area limited to authorized
personnel?
C. Are disbursement vouchers approved before payment?
D. Are there independent, periodic comparisons of inventory
records with goods on hand?

Vouchers for all disbursements (not only for inventory


purchases) must be approved before payment. Therefore, a
question on voucher approval would be more appropriately
included in the expenditure/disbursement cycle questionnaire.

162. Effective internal controls over inventories are designed and


implemented for the following reasons, except

A. Inventories typically represent a large component of an


entitys current assets.
B. Inventories are the most liquid asset.
C. Inventories directly affect the financial performance of
an entity.
D. Inventories typically represent a large portion of an
entitys total assets.

Cash, not inventories, is the most liquid asset and has the
greatest inherent risk.

Answers A and D are incorrect because inventories typically


represent a material component of an entitys current assets
and total assets.

Answer C is incorrect because once sold, inventories become


cost of goods sold and is a determinant of an entitys
financial performance for a given period.

163. ABC Manufacturing Corporation mass produces ten different


products. The companys controller is interested in
strengthening internal control over the accounting for
materials used in production. He/she would most likely design
and implement

A. An economic order quantity (EOQ) system.


B. A perpetual inventory system.
C. A separation of duties among production personnel.
D. A job-order cost accounting system.

The entity can easily keep track of materials usage by


maintaining perpetual inventory records.

Answer A is incorrect because EOQ system will ensure cost-


effective recording of materials but will not strengthen the
control over accounting for material usage.

Answer C is incorrect because segregation of duties among


production personnel does not strengthen control over
accounting for materials used in production unless the
authorization, custodial, and record-keeping functions are
likewise separated.

Answer D is incorrect because a process-cost system, not a


job-order cost system, is appropriate for goods that are mass
produced.

164. Your client, a merchandising concern, has annual sales of


P150,000,000 and a 40& gross profit rate. Tests revealed that
2% of the peso amount of purchases do not get into inventory
because of breakage and inventory pilferage by employees. The
company estimates that these losses could be reduced to 0.5 of
purchases by designing and implementing certain controls
costing approximately P1,750,000. Should the contols be
designed and implemented?

A. Yes, regardless of cost-benefit considerations, because


the situation involves employee theft.
B. Yes, because the ideal system of internal control is the
most expensive one.
C. No, because the cost of designing and implementing the
added controls exceeds the projected savings.
D. Yes, because the expected benefits to be derived
exceed the cost of the added controls.
A basic concept of internal control is the concept of
reasonable assurance, which recognizes that the cost of
internal control should not exceed the benefits expected
to be derived.
The additional controls should not be considered because
the cost is P1,750,000, but the estimated saving is only
P1,350,000 {(2%-0.5%)x(P150 million sales x 60% cost of
sales ratio)}
165. The objectives of internal control for a production cycle
are to provide assurance that transactions are properly
executed and recorded, and that
A. Production orders are pre-numbered and signed by a
supervisor.
B. Custody of work-in-process and of finished goods is
properly maintained.
C. Independent internal verification of activity reports
is established.
D. Transfers to finished goods are documented by a
completed production report and a quality control
report.
A primary objective of internal control in the production
cycle is to safeguard inventories from misuse and theft.
The inventories should be in the custody of a storekeeper,
and inventory movements should be properly documented and
recorded to establish accountability.
Answers A, C and D are incorrect because the use of pre-
numbered production orders signed by a supervisor,
independent internal verification of activity reports, and
documenting inventory transfers are control activities-an
internal control component, not control objectives.
166. Which of the following controls most likely would be
implemented to achieve the production cycle control
objective of maintaining accurate inventory records.
A. Periodic inventory counts are used to adjust the
perpetual inventory records.
B. A just-in-time inventory ordering system keeps
inventory levels to a desired minimum.
C. Perpetual inventory records are periodically compared
with the net realizable value of individual inventory
items.
D. Purchase requisitions, receiving reports, purchase
orders, and vendor invoices are independently matched
before payment is approved.
A well-designed internal control system should require
comparison of the recorded accountability for assets with
existing assets at reasonable intervals.
In the production cycle, periodic inventory counts should
be reconciled to the perpetual inventory records. This
control will provide reasonable assurance about the
accuracy of inventory records.
Answer B is incorrect because just-in-time ordering a
system provides assurance that desired inventory levels are
maintained but does not ensure that accurate inventory
records are maintained.
Answer C is incorrect because periodic comparison of
perpetual inventory records with net realizable value
relates more to inventory valuation rather than the
accuracy of inventory records.
Answer D is incorrect because matching of purchase
documents provides assurance that payments are made only
for goods authorized and received but does not ensure that
the accuracy of perpetual inventory records.
167. Which of the following questions would an auditor most
likely include in the production cycle internal control
questionnaire?
A. Are details of individual disbursements for raw
materials compared to the total for posting to the
general ledger?
B. Are vendor invoices for raw materials approved before
payment?
C. Are all issuances of raw materials to production based
on approved requisition forms?
D. Are signed checks for the purchase of raw materials
sent directly to intended payees after signing,
without being returned to the person who authorized the
invoice processing?
Answers A, B, and D are incorrect because comparing
individual disbursements for raw materials to totals,
approving vendor invoices before payment, and sending
checks directly to payees after signing pertain more
directly to the expenditure/disbursement cycle.
168. Which of the following is the most likely procedure an
auditor would perform in obtaining an understanding of a
manufacturing entitys internal control for inventory
balances?
A. Perform test counts of inventory when observing the
entitys physical count.
B. Perform analytical procedures designed to identify
significant cost variances.
C. Analyze the liquidity and turnover ratio of the
inventory.
D. Review the entitys description of inventory policies
and procedures.
The auditor will review the entitys description of
inventory policies and procedures to evaluate the design
of controls and determine whether they have been
implemented.
169. A properly designed internal control should require that
defective merchandise returned by customers be presented
initially to the
A. Receiving clerk
B. Purchasing clerk
C. Billing clerk
D. Inventory control clerk
All incoming goods, including defective merchandise
returned by customers, should be received by the receiving
clerk.
INVESTING CYCLE
170. The following controls are appropriate for property, plant,
and equipment (PPE) except
A. Written policies for capitalization and expenditure
and review of application of depreciation methods.
B. Disposal of fully depreciated PPE items.
C. Proper authority for acquisition and retirement of
PPE items.
D. Detailed PPE records and physical controls over PPE
items.
Fully depreciated PPE items need not be disposed of because
they may still be useful in the business. These assets
should remain on the books until disposal.
Answers A, C, and D are incorrect because written policies
for capitalization and expenditure and review of
depreciation methods, proper authority for acquisition and
retirement, and detailed PPE records and physical controls
are proper controls over PPE.
171. The question that an auditor would least likely include on
an internal control questionnaire concerning the initiation
and execution of equipment transactions is
A. Are requests for purchases of equipment reviewed for
consideration of soliciting competitive bids?
B. Are requests for major repairs approved at a higher
level than the department initiating the request?
C. Are procedures in place to monitor and properly
restrict access to equipment?
D. Are pre-numbered purchase orders used for equipment
and periodically accounted for?
The issue is the initiation and execution of equipment
transactions. Restricting access to equipment to authorized
personnel only does not address the issue because it deals
more with proper custody of assets.
Answers A, B and D are incorrect because competitive
bidding, approval of major repairs, and use of pre-numbered
purchase orders are related to the issue concerning
initiation and execution of equipment transactions.
172. Which of the following controls would most likely detect
equipment acquisitions that are misclassified as
maintenance expense?
A. Segregation of duties of employees in the accounts
payable department.
B. Authorization by the board of directors of significant
equipment acquisitions.
C. Independent verification of invoices for disbursements
recorded as equipment acquisitions.
D. Investigation of variances within a formal budgeting
system.
If equipment acquisitions are misclassified as maintenance
expense, an entitys formal budgeting system that includes
estimates of maintenance expense will report a significant
variance. The misclassification may be detected by
investigating the variance.
Answer A is incorrect because the accounts payable
department that processes payment transactions is unlikely
to question the classification of expenditures that are
based on proper documentation.
Answer B is incorrect because verification of invoices from
the population of recorded equipment acquisitions will not
disclose items misclassified as maintenance expense.
Answer C is incorrect because verification of invoices from
the population of recorded equipment acquisitions will not
disclose items misclassified as maintenance expense.
173. Which of the following control activities is most likely
to prevent the improper disposition of equipment?
A. A periodic analysis of the scrap sales and the repairs
and maintenance accounts.
B. Periodic comparison of removal work orders with
authorizing documentation.
C. The use of serial numbers to identify equipment that
could be sold.
D. A separation of duties between those authorized to
dispose of equipment and those authorized to approve
removal work orders.
There should be proper segregation of duties to reduce the
opportunity for an individual to both perpetrate and
conceal errors of fraud.
The functions of authorization, recording, and asset
custody should be separated. Accordingly, the authorization
to dispose of equipment by approving removal work orders
(authorization) and the disposal of equipment (asset
custody) should not be assigned to the same person.
Answers A, B and C are incorrect because the control
activities described are detective, not preventive, in
nature.
174. Which of the following control activities most likely would
justify a reduced level of control risk concerning
property, plant, and equipment (PPE) acquisitions?
A. Periodic physical inspection of PPE by the internal
audit
staff.
B. Approval of periodic depreciation entries by a
supervisor independent of the accounting department.
C. The review of pre-numbered purchase orders to detect
unrecorded trade-ins.
D. Comparison of current-year PPE account balances with
prior-year figures.
A periodic physical inspection by an objective and
competent internal audit staff is the best way to verify
the existence of PPE. This will reduce the possibility of
recording fictitious PPE acquisitions and other fraudulent
acts. The implementation of this control activity would
justify a lower assessed level of control risk.
Answer B is incorrect because depreciation is computed
based on recorded amounts of PPE. Depreciation will be
misstated if the basis used in calculating it is also
misstated.
Answer C is incorrect because reviewing purchase orders to
detect unrecorded trade-ins is not as effective as directly
inspecting the assets.
Answer D is incorrect because comparing recorded PPE
account balances may not be effective for detecting non-
existent PPE.
175. An internal control objective concerning property, plant,
and equipment (PPE) acquisitions is that they be recorded
at the correct amounts and in the proper period, and
properly classified. In which of the following conditions
would an auditor most likely assess a high level of risk
of material misstatement?
A. All material acquisitions of PPE are required to be
approved by the board of directors.
B. Most additions are self-constructed by the entity.
C. Recently acquired loans include covenants that
preclude further plant acquisitions for 5 years.
D. Gross PPE increased 30% during the current period.
If an entity has on-going in-house construction projects,
labor and overhead costs should be allocated between its
inventories and in-house construction projects. Moreover,
capitalization of borrowing costs would involve complex
calculations. Therefore, the inherent risk of misstatement
for self-constructed assets is high.
Answer A is incorrect because the required approval of the
entitys board of directors for all material PPE additions
relates to authorization, not recording.
Answer C is incorrect because the loan covenant that
precludes further plant additions for 5 years is most likely
to decrease risk.
Answer D is incorrect because the significant increase in
PPE does not necessarily increase the risk of misstatement.
For example, the significant increase may be due to material
acquisitions of PPE from outside vendors. In this case, the
cost of PPE can easily be determined by just referring to
amounts on vendor invoices.
176. Why is property, plant, and equipment (PPE) typically
considered to be one of the accounts least susceptible to
fraud?
A. Internal control on this account is inherently
effective.
B. The depreciated values are always smaller than cost.
C. The inherent risk of PPE is usually low.
D. For most companies, the recorded amounts of PPE
are immaterial.
PPE is one of the accounts that is least susceptible to
misstatement in the absence of related controls-that is,
its inherent risk is low. This is because of the
infrequency of transactions in the account and the
relative ease in verifying its existence.
Answer A is incorrect because inherent risk, not control
risk, pertains to the susceptibility of an account to
misstatement.
Answer B is incorrect because not all PPE items are
depreciable, for example, land.
Answer D is incorrect because although transactions
affecting PPE are infrequent, the amounts involved are
usually large.
177. Which of the following misstatements or questionable
practices may be uncovered if an auditor tours an entitys
production facility?
A. Insurance coverage on the facility has lapsed.
B. Overhead has been over applied.
C. Depreciation expense on fully depreciated machinery
has been recognized.
D. Necessary facility maintenance has not been
performed.
The auditors tour of an entitys production facility would
involve direct observation of the asset condition. Hence,
it is likely that the auditor will discover that necessary
facility maintenance has not been performed during the
year.
Answer A is incorrect because inspection of insurance
contracts, not a tour of the plant facility, will detect
lapsed insurance coverage.
Answer B is incorrect because comparison of the actual
overhead incurred with applied overhead will determine if
overhead has been over applied.
Answer C is incorrect because the auditor should examine
depreciation records to determine if depreciation has been
provided on fully depreciated machinery.
178. Which of the following controls would an entity most likely
use in safeguarding against the loss of trading securities?
A. The independent auditor traces all purchases and sales
of trading securities through the subsidiary ledgers
to general ledger.
B. An independent trust company that has no direct
contact with the employees who have record-keeping
responsibilities has possession of the securities.
C. The internal auditor inspects the trading securities
in the entitys safe each year on the balance sheet
date.
D. A designated member of each board of directors
controls the securities in a bank safe-deposit box.
Engaging an independent trust company for the custody of
an entitys trading securities may be considered the best
way to safeguard such securities against loss. Such an
institution normally has very strict controls over assets
under its custody, including access to its vaults.
Answer A is incorrect because an independent auditors
tracing of trading securities transactions is an audit
test, not a control.
Answer C is incorrect because the internal auditors
inspection of trading securities in the entitys safe each
year on the balance sheet date does not assure that no
securities have been removed between inspections dates.
Answer D is incorrect because a better control is to require
the presence of two authorized persons to access a safe
deposit box.
179. Which of the following controls would a company most likely
use to safeguard marketable securities when such securities
are not in the custody of an independent trust agent?
A. The chairman of the board of directors verifies the
marketable securities, which are kept in a bank safe-
deposit box, each year on the balance sheet date.
B. The internal auditor and the controller independently
trace all purchases and sales of marketable securities
from the subsidiary ledgers to the general ledger.
C. Two company officials have joint control of marketable
securities, which are kept in a bank safe-deposit box.
D. The investment committee of the board of directors
periodically reviews the instrument decisions
delegated to the treasurer.
A physical control to safeguard an entitys assets such
as marketable securities is to keep them in a bank safe-
deposit box requiring two signatures to gain access.
Answer A is incorrect because verification of marketable
securities once a year is unlikely to provide adequate
control.
Answer B is incorrect because tracing of marketable
securities transactions to the accounting records
assures proper recording but does not physically
safeguard the asset.
Answer D is incorrect because periodic review of the
investment decisions delegated to the treasurer does not
physically safeguard the asset.
180. Which of the following internal control activities would
an entity most likely implement to assist in satisfying the
completeness assertion related to noncurrent investments?
A. The internal auditor compares the securities in the
bank safe-deposit box with recorded investments.
B. Senior management verifies that securities in the
bank safe-deposit box are registered in the entitys
name.
C. The controller compares the current market prices of
recorded investments with the brokers advices on
file.
D. The treasurer vouches the acquisition of securities by
comparing brokers advices with canceled checks.
Comparison of securities in bank safe-deposit box with the
accounting records assures that the investment balance is
complete, that is, all investment securities have been
reflected in the account.
Answer B is incorrect because senior managements
verification that securities are registered in the entitys
name relates to the right assertion.
Answer C is incorrect because comparing market prices with
brokers advices pertains to the valuation assertion.
Answer D is incorrect because vouching securities purchased
by comparing brokers advices with canceled checks relates
to the rights assertion.
181. Which of the following is not a proper control over
investment securities?
A. Separation of custodial and treasury functions.
B. Employing an independent trust agent.
C. Proper authorization of transactions.
D. Storage in a safe-deposit box.
An entitys treasury function should include custody of
cash and securities.
Answers B, C, and D are incorrect because employing an
independent trust agent, proper authorization of
transactions, and storage of investment securities in a
safe-deposit box are proper controls over investment
securities.
182. The following controls are designed to protect investment
securities, except
A. Investment securities should be properly controlled
physically in order to prevent unauthorized usage.
B. Custody over investment securities should be limited
to personnel having record-keeping responsibility over
the securities.
C. Securities should be registered in the entitys name.
D. Access to securities should be vested in two
individuals.
As with other assets, the custody of investment securities
should be delegated to individuals who do not have recording
responsibility over securities.

FINANCING CYCLE
183. Which of the following questions is most likely to be
included by an auditor on an internal control questionnaire
for notes payable?
A. Are direct borrowings on notes payable authorized by
the board of directors?
B. Are assets that collateralize notes payable critically
needed for the entitys continued existence?
C. Are two or more authorized signatures required on
checks that repay notes payable?
D. Are the proceeds from notes payable used for the
purchase of noncurrent assets?
Most companies are ordinarily require that direct
borrowings on notes payable be authorized by the board of
directors. Accordingly, an auditor should verify if such
control has been properly implemented.
184. The audit program for long-term debt should include steps
that require the
A. Verification of the existence of the bondholders.
B. Examination of any bond trust indenture.
C. Inspection of the accounts payable master file.
D. Investigation of credits to the bond interest income
account.
185. During the year under audit, a company has completed a
private placement of a substantial amount of bonds. Which
of the following is the most important step in the
auditors program for the audit of bonds payable?
A. Confirming the amount issued with the bond trustee.
B. Tracing the cash received from the issue to the
accounting records.
C. Examining the bond records maintained by the transfer
agent.
D. Recomputing the annual interest cost and the effective
yield.
186. Several years ago, ABC, Inc., secured a conventional real
estate mortgage loan. Which of the following audit
procedures would be least likely to be performed by an
auditor auditing the mortgage balance?
A. Examine the current years cancelled checks.
B. Review the mortgage amortization schedule.
C. Inspect public records of lien balances.
D. Recompute mortgage interest expense.
187. During an audit of publicly held company, the auditor should
obtain written confirmation regarding debenture
transactions from the
A. Debenture holders
B. Clients authority
C. Internal auditors
D. Trustee
188. An audit program for the audit of retained earnings account
should include a step that requires verification of
A. Market value used to charge retained earnings to
account for a 2-for-1 stock split.
B. Approval of the adjustment to the beginning balance
as a result of a write-down of an account receivable.
C. Authorization for both cash and stock dividends.
D. Gain or loss resulting from disposition of treasury
shares.
189. Where no independent stock transfer agents are employed and
the corporation issues its own stocks and maintains stock
records, cancelled stock certificates should
A. Be defaced to prevent reissuance and attached to their
corresponding stubs.
B. Not be defaced, but segregated from other stock
certificates and retained in a cancelled certificates
file.
C. Be destroyed to prevent fraudulent reissuance.
D. Be defaced and sent to the Secretary of the Department
of Finance.

HUMAN RESOURCES CYCLE


190. A potential threat to the payroll processing activity is
theft or fraudulent distribution of payroll checks. One
control that can be implemented to help prevent paychecks
being issued to ghost employees is
A. Use of payroll clearing account.
B. Paychecks should be physically distributed by someone
who does not authorize or record payroll.
C. Periodic reconciliation of the payroll bank account.
D. The cashier should sign all payroll checks.
191. Effective controls over the payroll function may include
A. Custody of rate authorization records by the
supervisor of the personnel department.
B. Preparation of payroll transaction journal entries
by an employee who reports to the supervisor of the
personnel department.
C. Verification of agreement of job time tickets with
employee clock card hours by a payroll department
employee.
D. Reconciliation of totals on job time tickets with
job reports by employees responsible for those
specific jobs.
Comparison of the job time tickets, which show the total
time spent on jobs, and time cards will provide an
independent check of the accuracy of time indicated on time
cards.
Answer A is incorrect because the pay rates used in the
calculation of payroll should be authorized by the
personnel department.
Answer B is incorrect because the payroll department, not
the personnel department, prepares the payroll transaction
journal entries. This is to segregate the authorization and
record-keeping functions.
Answer D is incorrect because an independent party, not the
employees involved, should reconcile totals on job time
tickets with job reports.
192. Employees of a manufacturing entity are often required to
use time cards and job time tickets. Which of the following
statements concerning the use of these documents is
incorrect?
A. Time reported on job time tickets should be reconciled
to time cards.
B. Payroll should be calculated based on job time
tickets.
C. Each employee should have only one time card.
D. An employees may have one or many job time tickets
in a day.
Time cards are the official records of time worked by
employees and should be the basis for payroll preparation.
193. Organizational independence in the processing of payroll
can be achieved by segregating the functions of
authorization, record-keeping, and custody of assets. Which
one of the following functional separations is not required
for internal control purposes?
A. Separation of payroll preparation and paycheck
distribution.
B. Separation of personnel function from payroll
preparation.
C. Separation of timekeeping from payroll preparation.
D. Separation of payroll preparation and maintenance of
year-to-date records.
194. Which of the following controls is most effective in
providing reasonable assurance that salary, wage, and
benefit expenses are incurred only for worked performed?
A. The payroll register is used as the source document
for posting employees benefit costs to the general
ledger.
B. All time cards and reports are reviewed and approved
in writing by immediate line supervisors who do not
have responsibilities for paycheck distribution.
C. Actual payroll amounts are regularly compared against
budgeted amounts by management, with all material
budget variances being investigated.
D. The accuracy of extensions of hours worked and pay
rates is rechecked by an independent party, and pay
rate and other key payroll information is changed
only upon the receipt of a written authorization
from the personnel department.
An appropriate control is the review and approval of time
cards and reports by line supervisors because they have
direct knowledge about whether work has been performed.
And because they have no responsibilities for paycheck
distribution, they are not in a position to misappropriate
paychecks.
Answer A is incorrect because using the payroll register
as the source document for posting to the general ledger
controls recording of employee benefit costs, not the
propriety of the reported time worked.
Answer C is incorrect because investigating material
variances between actual payroll and budgeted amounts may
probably not reveal specific improprieties. It is,
therefore, less effective than the review of time cards
and reports by line supervisors/
Answer D is incorrect because although an arithmetic check
of payroll calculations and personnel authorization of pay
rate changes are effective controls over payroll
processing, they do not assure that employee benefit costs
are incurred only for work performed.
195. The purpose of segregating the duties of hiring personnel
and distributing payroll checks is to separate the
A. Authorization of transactions from the custody of
related assets.
B. Operational responsibility from the record-keeping
responsibility.
C. Human resource function from the controllership
function.
D. Administrative controls from internal accounting
controls.
A well-designed internal control provides for proper
segregation of authorization, recording, and asset custody
functions.
An entitys personnel department provides authorization for
hiring, pay rates, and deductions; the production
department provides authorization for hours worked. In
turn, the payroll department calculates and records the
payroll based on these authorizations. Based on these
calculations, the treasurer signs and distributes paychecks
to employees-an asset custody function.

196. Which of the following departments most likely approved


changes in pay rates and deductions from employee salaries?
A. Payroll
B. Personnel
C. Controller
D. Treasurer
An entitys personnel department provides authorization
for hiring, termination, deductions, and changes in pay
rates.
197. Which of the following situations represents an internal
control weakness in the payroll department?
A. The timekeeping function is independent of the payroll
department.
B. Payroll records are periodically reconciled with tax
reports.
C. Paychecks are distributed by the employees immediate
supervisor.
D. Payroll department personnel are rotated in their
duties.
A sound internal control dictates that there should be
proper segregation of functional responsibilities to
minimize the risk that errors or fraud will be committed
and concealed by the same person.
The functions of authorization, recording, and asset
custody should be separated. If the employees immediate
supervisor also distributes paychecks, paychecks of
fictitious employees can be diverted if that supervisor
also has access to personnel records.
198. Which of the following personnel department procedures
reduces the risk of payroll fraud and represents an
appropriate responsibility for the department>
A. Authorizing the addition or deletion of employees
from the payroll.
B. Authorizing overtime hours.
C. Collection and retention of unclaimed paychecks.
D. Distributing paychecks.
The personnel department has the responsibility of
authorizing employee transactions such as hiring,
terminating, and changes in pay rates and deductions.
199. Each Saturday afternoon, paychecks are distributed by the
production departments shift supervisor. The companys
production department is so large and the turnover of
factory workers is so great that the supervisor does not
know many of the workers. Unclaimed paychecks are return
claim them at some later time. The payroll clerk routinely
continues the payroll record for workers one week after
their departure from the company and ultimately diverts the
unclaimed paychecks. Which of the following controls would
most likely prevent this misappropriation?
A. Require the treasurers office to prepare checks only
on the basis of supporting documentation from both the
timekeeper and payroll accounting.
B. Periodically rotate the shift supervisor.
C. Require the timekeeper to compute weekly pay of each
factory worker and to make distribution of the checks
received from the treasurers office.
D. Require the shift supervisor to know all the workers
by name.
The payroll accounting should calculate and record the
payroll based on authorized pay rates and deductions
and each employees timecard. Paychecks should be
prepared based on this documentation. The payroll
department, having a record-keeping responsibility,
should not have custody of unclaimed paychecks.
Answer B is incorrect because the identity of the shift
supervisor is not an element of the payroll clerks fraud
Answer C is incorrect because the calculation of payroll
distribution of paychecks are incompatible functions and
should not be performed by the timekeeper.
Answer D is incorrect because it would be very difficult
to implement the control of knowing all factory workers
by name.
200. Proper internal control over the cash payroll function
mandates which of the following?
A. A separate checking account for payroll should be
maintained.
B. Each employee should be asked to sign a receipt.
C. The payroll clerk should fill the envelopes with
cash and a computation of the net pay.
D. Unclaimed pay envelopes should be retained by the
paymaster.
In a cash payroll system, employees must be required to
sign a receipt that will serve as documentation for the
payment.
Answer A is incorrect because a separate checking
account need not be maintained.
Answer C is incorrect because the payroll clerk who

has a recording responsibility should not be assigned


to fill the envelopes with cash.
Answer D is incorrect because unclaimed cash payroll
should be deposited in the bank for proper custody.

TRUE OR FALSE
1. As the risk of material misstatement increases, detection
risk should increase.
2. If planned detection risk is reduced, the amount of
evidence the auditor accumulates will decrease.
3. Inherent risk and control risk are inversely related to
each other.
4. The risk of material misstatement refers to the combination
of inherent risk and control risk.
5. In a financial statement audit, inherent risk is evaluated
to help an auditor assess the susceptibility of a financial
statement assertion to a material misstatement assuming
there are no related controls.
6. Inherent risk is inversely related to the amount of audit
evidence whereas detection risk is directly related to the
amount of audit evidence required.
7. Inherent risk is directly related to the amount of
evidence whereas detection risk is inversely related to
the amount audit evidence required.
8. Inherent risk and control risk are assessed by the audit
and function independently of the financial statement
audit.
9. As the acceptable level of detection risk increases, an
auditor may change the timing of the tests on controls
by performing them throughout the year rather than at one
time.
10. As the acceptable level of detection rick increases, an
auditor may change the timing of substantive tests by
performing them at an interim date rather than year end.
11. The risk that an auditor will conclude, based on
substantive tests, that a material misstatement does not
exist in account balance, when, in fact, such misstatement
does exist is referred to as detection risk.
12. When an auditor increases the assessed level of risk of
material misstatement because certain control procedures
were determined to be ineffective, the auditor would most
likely increase the level of detection risk.
13. The risk of material includes non-sampling risk.
14. As the acceptable level of detection risk decreases, an
auditor may change the nature of substantive procedures
from less effective to more effective procedures.
15. As the acceptable level of detection risk decreases, the
assurance directly provided from substantive procedures
should increase.
16. Materiality judgements are made in light of surrounding
circumstances and necessarily involve both quantitative
and qualitative judgements.
17. The documentation of an auditors understanding of
internal controls must include flowcharts.
18. To obtain evidential matter about control risk, an auditor
selects tests from a variety techniques including
confirmation.
19. Reports on service organizations typically provide
reasonable assurance that their financial statements are
free of material misstatements.
20. Where computer is processing is used in significant
accounting applications, internal control activities may
be defined by classifying control activities into two
types: general and application.
21. In evaluating internal control, the auditor is basically
concerned that the system provides reasonable assurance
that operational efficiency has been achieved in
accordance with management plans.
22. The maintenance of the system of internal control is an
important responsibility of the internal auditor.
23. Because of the cost/benefit relationship, tests of
controls may be applied on a test basis in some
circumstances.
24. The concept of reasonable assurance in the context of an
entitys internal controls recognizes that auditors may
fail to detect material misstatements.
25. An effective control environment guarantees that all
controls are followed as prescribed.
26. The risk assessment component of internal controls refers
to the auditors assessment of control risk.
27. Assessing control risk at below the maximum would involve
identifying specific internal controls relevant to
specific assertions.
28. In the audit of financial statements, an auditors primary
consideration regarding an internal control policy or
procedure is whether the policy or procedure affects
managements financial statement assertions.
29. As part of gaining an initial understanding of internal
control, an auditor is required to obtain knowledge about
the operating effectiveness of the internal control.
30. Assessing control risk below maximum involves concluding
that controls are ineffective.
31. After the auditor has prepared a flowchart of the internal
controls surrounding sales and evaluated the design of the
system, the auditor would perform tests of controls on all
control activities documented in the flowchart.
32. Observation is a procedure that would most likely be used
by an auditor in performing tests of control activities
that involve segregation of functions and that leave no
transaction trail.
33. Tracing bills of lading to sales invoices provides evidence
that billed sales were shipped.
34. Tracing copies of sales invoices to shipping documents
will provide evidence that all shipments to customers were
billed.
35. The accounts payable department receives the purchase
order form to ensure that the goods had been received by
the party requesting the goods.
36. The authority to accept incoming goods in receiving should
be based on approved purchase order.
37. For effective internal control purposes, the vouchers
payable department generally should stamp, perforate, or
otherwise cancel supporting documentation after payment
is mailed.
38. Vendors statements and vendors invoices are both
relatively reliable evidence because they originate from
a third party.
39. Credit memos are normally issued to adjust the customers
balance to the amount owed to the company.
40. The document that the accounting staff will use as the
primary basis for recording sales transactions and
updating the customers accounts receivable subsidiary
ledger is the sales invoice.

KEY ANSWERS

1. C 21. B 41. D 61. D 81. C

2. D 22. B 42. B 62. C 82. B

3. A 23. B 43. D 63. C 83. C

4. C 24. D 44. C 64. D 84. B

5. C 25. B 45. C 65. D 85. A

6. B 26. D 46. B 66. D 86. A

7. D 27. A 47. A 67. B 87. C

8. C 28. D 48. A 68. B 88. D


9. B 29. A 49. A 69. C 89. A

10. C 30. A 50. B 70. D 90. C

11. B 31. C 51. A 71. A 91. D

12. C 32. C 52. B 72. A 92. C

13. A 33. C 53. D 73. B 93. B

14. D 34. A 54. C 74. A 94. C

15. B 35. C 55. D 75. D 95. D

16. A 36. C 56. A 76. C 96. D

17. B 37. C 57. C 77. A 97. A

18. B 38. B 58. A 78. C 98. D

19. A 39. B 59. B 79. A 99. A

20. B 40. A 60. C 80. B 100. A

101. B 121. A 141. B 161. C 181. A

102. C 122. B 142. B 162. B 182. B

103. A 123. C 143. B 163. B 183. A

104. B 124. B 144. A 164. C 184. B

105. C 125. D 145. B 165. B 185. A

106. C 126. D 146. A 166. A 186. C

107. A 127. D 147. D 167. C 187. D

108. B 128. A 148. C 168. D 188. C

109. C 129. B 149. C 169. A 189. A

110. B 130. C 150. D 170. B 190. B

111. C 131. C 151. A 171. C 191. C


112. A 132. C 152. C 172. D 192. B

113. C 133. C 153. B 173. D 193. D

114. B 134. A 154. A 174. A 194. B

115. B 135. B 155. C 175. B 195. A

116. B 136. C 156. B 176. C 196. B

117. A 137. C 157. C 177. D 197. C

118. C 138. B 158. A 178. B 198. A

119. C 139. A 159. B 179. C 199. A

120. B 140. B 160. D 180. A 200. B

TRUE OR FALSE

1. True 9. False 17. False 25. False 33. False

2. False 10. True 18. False 26. False 34. False

3. False 11. True 19. False 27. True 35. False

4. True 12. False 20. True 28. True 36. True

5. True 13. False 21. False 29. False 37. False

6. False 14. True 22. False 30. False 38. True

7. True 15. True 23. True 31. False 39. True

8. True 16. True 24. False 32. True 40. True


CHAPTER 6
AUDITING IN A COMPUTER INFORMATION SYSTEMS (CIS) OR INFORMATION
TECHNOLOGY (IT) ENVIRONMENT

1. IT has several significant effects on an entity. Which of the


following would be important from an auditing perspective?

I. The potential for material misstatement.


II. The visibility of information.
III. Changes in the organizational structure.

A. I and II only C. II and III only


B. I and III only D. I, II, and III

2. The use of a computer changes the processing, storage, and


communication of financial information. A CIS environment may
affect the following, except

A. The accounting and internal control systems of the entity.


B. The overall objective and scope of an audit.
C. The auditors design and performance of tests of control and
substantive procedures to satisfy the audit objectives.
D. The specific procedures to obtain knowledge of the entitys
accounting and internal control systems.

A CIS environment does not affect the overall objective and


scope of an audit.

3. The following are benefits of using IT-based controls, except

A. Ability to process large volume of transactions.


B. Over-reliance on computer-generated reports.
C. Ability to replace manual controls with computer-based
controls.
D. Reduction in misstatements due to consistent processing of
transactions.

4. Which of the following statements concerning the Internet is


incorrect?

A. The Internet is a shared public network that enables


communication with other entities and individuals around the
world.
B. The Internet is a private network that only allows access to
authorized persons or entities.
C. The Internet is interoperable, which means that any computer
connected to the Internet can communication.
D. The Internet is a worldwide network that allows entities to
engage in e-commerce/e-business activities.
5. In planning the portions of the audit which may be affected
by the clients CIS environment, the auditor should obtain an
understanding of the significance and complexity of the CIS
activities and the availability of data for use in the audit.
The following relate to the complexity of CIS activities except
when

A. Transactions are exchanged electronically with other


organizations (for example, in electronic data interchange
systems [EDI])
B. Complicated computations of financial information are
performed by the computer and/or material transactions or
entries are generated automatically without independent
validation.
C. Material financial statement assertions are affected by the
computer processing.
D. The volume of transactions is such that users would find it
difficult to identify and correct errors in processing.

The materiality of the financial statement assertions affected


by the CIS relates to the significance, not the complexity of
computer processing.

6. The auditor shall consider the entitys CIS environment in


designing audit procedures to reduce risk to an acceptably low
level. Which of the following statements is incorrect?

A. The auditors specific audit objectives do not change whether


financial information is processed manually by computer.
B. The methods of applying audit procedures to gather audit
evidence are not influenced by the methods of computer
processing.
C. The auditor may use either manual audit procedures, computer-
assisted audit techniques (CAATs), or a combination of both to
obtain sufficient appropriate audit evidence.
D. In some CIS environments, it may be difficult or impossible
for the auditor to obtain certain data for inspection, inquiry,
or confirmation without the aid of a computer.

The methods of applying audit procedures to gather audit evidence


may be influenced by the methods of computer processing.

7. Regardless of the nature of an entitys information system,


the auditor must consider internal control. In a CIS environment,
the auditor must, at a minimum, have

A. A background in programming procedures.


B. An expertise in computer systems analysis.
C. A sufficient knowledge of the computers operating system.
D. A sufficient knowledge of the computer information system.
The auditor should have a sufficient knowledge of the CIS to
plan, direct, supervise, and review the work performed.

Answers A and B are incorrect because an auditor need not have


expertise in programming and computer systems analysis. If
specialized CIS skills are needed in the audit, the auditor may
seek the assistance of an auditors expert.

Answer C is incorrect because the auditor should have sufficient


knowledge of the entire CIS not only of the computers operating
systems.

8. Who is ultimately responsible for the design and


implementation of cost-effective controls in a CIS environment?

A. The internal audit manager


B. The entitys management
C. The CIS manager
D. The control group in the CIS environment

An entitys management is ultimately responsible or designing


and implementing systems that will provide reasonable assurance
that the entitys objectives will be achieved.

9. Are the following risks greater in CIS than in manual systems?

A B C D
Erroneous data conversion Yes Yes Yes Yes
Erroneous source document
preparation Yes Yes Yes No
Repetition of errors No No Yes Yes
Concentration of data Yes No Yes Yes

The preparation of source documents wither precedes or is not


done at all in a computer information systems. Thus, the risk
of erroneous source document preparation in a CIS environment
may be equal to or less than the equivalent risk in a manual
system.

In a CIS environment, the computer converts data to machine-


readable form prior to processing of transactions. This will
increase that risk of input error. In addition, the computers
ability to uniformly process like transactions with the same
processing instructions will ordinarily result in all
transactions being processed incorrectly if there are
programming errors (or other systematic errors in hardware aor
software). Also, the concentration of data stored on magnetic
disk increases the risk of loss of valuable financial information
from damage or theft.
10. Which of the following is not a hardware element in an IT
environment?

A. Scanners
B. CD-ROM drive
C. Application programs
D. Modems

An IT environment consists of hardware and software components.


Computer hardware consists of the computer and all other physical
equipment. The software component consists of computer programs
that are either purchased from a software vendor or developed
in-house by the entity.

Application software a type of computer software performs


desired processing tasks such as payroll processing.

Answers A, B, and D are incorrect because optical scanners, CD-


ROM drive, and modems are elements of computer hardware.

11. Which of the following computer hardware elements is not


associated with data input?

A. Touch screen
B. Printer
C. Mouse
D. Optical scanner

A printer is an output device that procedures a hard copy of


computer processing results.

Answers A, C, and D are incorrect because a touch screen, a


mouse, and an optical scanner can be used for data input.

12. A hardware element that takes the computers digital


information and transforms it into signals that can be sent,
over ordinary telephone lines is a/an

A. Intelligent terminal
B. Point-of-sale terminal
C. Terminal emulator
D. Modem

A modem converts data in digital form into analog or wave form


(the process is called modulation) so that data can be sent to
remote locations through the telephone system.

The modem at the receiving end of the transmission path converts


the analog or wave form back to the digital form (the process
is called demodulation) used by the terminal or CPU.
13. Uninterruptible power supplies are used in computer
facilities to minimize the risk of

A. Crashing disk drive read-write heads


B. Dropping bits in data transmission
C. Failing to control concurrent access to data
D. Losing data stored in main memory

An uninterruptible power source such a s a generator or battery


backup used in a computer facility will reduce the likelihood
of losing data stored in the computers main memory in the event
of an electrical failure such as a power outage or voltage
fluctuation.

14. In a computer system, the parts of the operating system


program and language translator program are stored in the

A. Read only memory (ROM).


B. Random access memory (RAM).
C. Magnetic tape drive.
D. Magnetic disk drive.

ROM consists of semiconductor chips that can be read from (but


not written to) and are used as permanent storage of the
operating system and language translator.

Answers B, C, and D are incorrect because RAM and magnetic tape


and disk drives are temporary storage devices.

15. A characteristic that distinguishes computer processing from


manual processing is

A. The potential for systematic error is ordinarily greater in


manual processing than in computerized processing.
B. Errors or fraud in computer processing will be detected soon
after their occurrences.
C. Most computer systems are designed so that transaction trails
useful for audit purposes do not exist.
D. Computer processing virtually eliminates the occurrence of
computational errors normally associated with manual processing.

Computational or clerical errors are virtually eliminated in


computer processing because of the computers capability to
uniformly process like transactions with the same processing
instructions.

Answer A is incorrect because the risk of systematic or


programming error is greater in computer processing than is
manual processing.
The computers ability to subject like transactions to uniform
processing will result in all transactions being processed
incorrectly if there are errors embedded in the program logic.

Answer B is incorrect because errors or fraud in computer


processing may remain undetected for long periods of time, or
worse, may never be detected at all.

The potential for observing errors or fraud is reduced in


computer processing because of decreased human involvement in
handling transactions processed by CIS.

Answer c is incorrect because CIS are designed to include


transaction trails. However, some transaction trails in computer
processing may exist for only a short period of time or only in
computer-readable form.

16. An affordable yet powerful self-contained general purpose


computer which consists typically of a central processing unit
(CPU), monitor, keyboard, disk drives, printer cables, and
modems is a /an

A. Personal computer
B. Mainframe
C. On-line computer
D. Terminal

17. A CIS where two or more personal computers are linked


together through the use of special software and communication
lines and allows the sharing of application software, data files,
and computer peripherals, such as printers and optical scanners
is a/an

A. Local area network (LAN)


B. On-line system
C. Batch processing system
D. Wide area network (WAN)

Each personal computer linked to a LAN is called a workstation


that can access data, software, and other resources through a
file server a linked PC that manages the network.

A LaN is usually confined to a small geographic location such


as a building or two or more adjacent buildings.

Two or more LANs can be linked together to form a wide area


network (WAN).

18. A file server in a local area network (LAN) is


A. A workstation that is dedicated to a single user on the LAN.
B. A computer that stores programs and data files for users of
the LAN.
C. The cabling that physically interconnects the nodes of the
LAN.
D. A device that connects the LAN to other networks.

Common resources such as programs and data shared by LAN nodes


are stored and managed by special-purposed computers called file
servers.

Answer A is incorrect because a workstation or node in a LAN is


called a client.

Answer C is incorrect because the cabling that physically


interconnects the nodes of the LAN is communications link.

Answer D is incorrect because bridges and gateways are used to


link networks together. Bridges connect LANs of the same type
while gateways connect LANs of different types.

19. Audit team members can use the same database and programs
when their PCs share a hard disk and printer in a LAN. Which of
the following communication devices enables a PC to connect to
a LAN?

A. A network interface card (NIC) that plugs into the


motherboard.
B. a fax modem that send signals through telephone lines.
C. An internal modem that plugs into the motherboard.
D. An external modem with a cable connection to a serial port.

A workstations physical connection to the LAN is achieved


through a network interface card (NIC) which plugs into one of
the expansion slots on the PC.

Answers B, C, and D are incorrect because modems connect PCs to


ordinary telephone lines.

20. A computer information system that allows individual users


to develop and execute application programs, enter and process
data, and generate reports in a decentralized manner is called
a/an

A. Online system
B. Batch processing system
C. End-user computing
D. Networking
In end-user computing, management empowers individual users to
develop and execute application programs, enter and process
data, and generate computer processing results. This system is
an example of decentralized processing and usually involves the
use of PCs.

21. Which the following statements most likely represents a


disadvantage for an entity that maintains data files on personal
computers (PCs) rather than manually prepared files?

A. It is usually more difficult to compare recorded


accountability with the physical count of assets.
B. Random error associated with processing similar transactions
in different ways is usually greater.
C. Attention is focused on the accuracy of the programming
process rather than errors in individual transactions.
D. It is usually easier for unauthorized persons to access and
alter the files.

In a PC environment, unauthorized individuals can easily gain


access to and change data files without visible evidence.

Answer A is incorrect because an advantage of CIS is the


computers ability to process like transactions in the same way.

Answer C is incorrect because focusing on the accuracy of the


programming process is an advantage of CIS.

22. The following are risks specific to IT environments, except

A. Reduced segregation of duties.


B. Loss of data due to insufficient backup.
C. Increased human involvement.
D. Reliance on the functioning capabilities of hardware and
software.

23. Most personal computers have both a CD-ROM drive and a hard
disk drive. The major difference between the two types of storage
is that a hard disk.

A. Is suitable for an online system, whereas a CD-ROM is not.


B. Provides an automatic audit trail, whereas a CD-ROM does not.
C. Has a much larger storage capacity than a CD-ROM.
D. Is a direct-access storage medium, whereas a CD-ROM is a
sequential-access storage medium.

24. What type of online computer system is characterized by data


that are assembled from more than one location and records that
are updated immediately?
A. Online, batch processing system
B. Online, real-time processing system
C. Online, inquiry system
D. Online, downloading/uploading system

In an online processing system, individual transactions are


entered through workstations or terminals that are connected to
the mainframe.

A type of online system is online, real-processing system that


involves immediate validation and processing of data input to
update related computer files that allows users to receive the
output soon enough to affect a current decision to be made.

Answer A is incorrect because in an online, batch processing


system, individual transactions are entered through remote
terminals, subjected to certain validation routines and added
to a transaction file containing other transactions entered
during the period.

The transaction file is to be subjected to further validation


checks and then used in updating the relevant master file in the
subsequent processing cycle.

Answer is incorrect because in an online, inquiry system, users


are restricted to making inquiries of master files (for example,
inquiry of a customer account balance).

Answer D is incorrect because online, uploading/downloading


system involves the transfer of data between the mainframe and
workstations.

25. Misstatements in a batch computer system caused by incorrect


programs or data may not be detected immediately because

A. The processing of transactions in a batch system is not


uniform.
B. There are time delays in processing transactions in a batch
system.
C. The identification of errors in input data typically is not
part of the program.
D. Errors in some transactions may cause rejection of other
transactions in the batch.

In a batch processing system, similar transactions are processed


in groups or batches periodically for example, daily, weekly,
or even monthly. Hence, errors in a given batch may be detected
only after the lapse of consideration time from the initiation
of the transactions.
Answer A is incorrect because like transactions are processed
uniformly in a batch system.

Answer C is incorrect because data validation routines may be


embedded in the computer program.

Answer D is incorrect because although similar transactions are


processed together in batches, individual transactions are not
dependent upon one another.

26. Which of the following features is least likely to be found


in an online, real-time processing system?

A. Turnaround documents
B. User manuals
C. Preformatted screens
D. Automatic error correction

A turnaround document is a source document generated by the


computer system as output and then later used as input for
subsequent processing. Turnaround documents are least likely to
be found in an online, real-time processing system because it
normally does not use source documents.

Answer B is incorrect because user manuals provide explanations


on the proper use of the system, making them an important
component of the real-time system.

Answer C is incorrect because users usually interact with the


mainframe through preformatted screens of remotes terminals.

Answer D is incorrect because automatic error correction is a


principal advantage of real-time systems that is, errors are
immediately detected and corrected.

27. Which of the following is usually not a factor to consider


in designing and implementing an online, real-time system?

A. Priority allocation
B. Queues
C. Interrupts
D. Hardware diagnostics

Computers are designed to include hardware diagnostic routines


that allow identification of hardware problems such as a party
check to determine if the integrity of the bot structure of each
character has been destroyed during the internal transaction of
data within the system.
Hardware diagnostic routines are applicable to all systems, not
only to online, real-time systems.

Answers A and B are incorrect because priority allocation and


queues are important factors in real-time systems. Both of them
relate to deciding which jobs should be given priority in
processing.

Answer C is incorrect because interrupts allow high priority


jobs to get immediate action. In a multiprogramming environment,
work on one program is interrupted so the CPU may attend to
another.

28. Workstations or terminals are an integral component of online


computer systems. Which of the following statements concerning
workstations is incorrect?

A. Workstations may be located either locally or at remote sites.


B. Both local and remote workstations require the use of
telecommunications to link them to the main computer.
C. Local workstations are connected directly to the main computer
through cables.
D. Workstations may be used by different users, for different
purposes, in different locations, all at the same time.

Only remote workstations require the use of telecommunications


to link them to the main computer. Local workstations are linked
through cables.

29. Online computer systems use workstations or terminals that


are located either locally or at remote sites. There are two
types of workstations: general purpose terminals and special
purpose terminals. General purpose terminals include the
following, except

A. Basic keyboard and monitor


B. Point of sale devices
C. intelligent terminal
D. Personal computers

General purpose terminals include:

Basic keyboard and monitor used for entering data without


any validation checks; the monitor displays data from the
computer system.
Intelligent terminal performs the functions of the basic
keyboard and monitor with the additional functions of
validating data within the terminal, maintaining
transaction logs, and performing other local processing.
Personal computers perform all the functions of an
intelligent terminal with the additional local processing
and storage capabilities.

Special purpose terminals include:

Point of sale devices used to record sales


transactions as they occur and to transmit them to the
main computer such as electronic cash registers and
optical scanners.
Automated teller machines (ATMs) used to initiate,
validate, record, transmit, and complete various
banking transactions.

30. The test data approach

A. Involves reprocessing actual entity data using the entitys


computer software.
B. Involves reprocessing actual entity data using the auditors
computer software.
C. Is where dummy transactions are prepared by the auditor and
processed under the auditors control using the entitys
computer software.
D. Is where actual transactions are prepared by the auditor.

31. Which of the following is a primary example of source data


automation?

A. A subsidiary ledger
B. A utility bill
C. Point-of-sale (POS) scanners in malls
D. A bill of lading

32. Express Padala, Inc. stated in one of its mission statements


that positive control of each package will be maintained by
utilizing . . . electronic tracking and tracing systems. Express
Padala uses what type of IT system?

A. Batch processing which features immediate updating as to the


location of packages.
B. Real-time processing which features updating at fixed time
periods.
C. Batch processing which features updating at fixed time-
periods.
D. Real-time processing which features immediate updating as to
the location of packages.

33. In a file-oriented approach to data and information, data


is maintained in many separate files. This may create problems
for organizations because of
A. Multiple users.
B. Multiple transaction files.
C. Multiple master files which may contain redundant data.
D. A lack of sophisticated file maintenance software.

34. __________________ refers to the combination of the


database, the Database Management System (DBMS), and the
application programs that access the database through the DBMS.

A. Data warehouse
B. Data administrator
C. Database system
D. Database manager

35. Who is the individual responsible for the database?

A. Data coordinator
B. Database master
C. Database administrator
D. Database manager

36. Which feature of many database systems simplifies the


creation of reports by allowing users to specify the data
elements desired and the format of the output?

A. Report generator
B. Report writer
C. Report printer
D. Report creator

37. Which of the following is probably the most significant


effect of database technology on accounting?

A. Quicker access to and greater use of accounting information


in decision-making.
B. Replacement of the double-entry system.
C. Change in the nature of financial reporting.
D. Elimination of traditional records such as journals and
ledgers.

38. An entity should have a disaster recovery plan to ensure


that data processing capacity can be restored as smoothly and
quickly as possible. The following would typically be part of
an adequate disaster recovery plan, except

A. A system upgrade due to operating system software changes.


B. Backup computer and telecommunication facilities.
C. Scheduled electronic vaulting of files.
D. Uninterruptible power systems installed for key system
components.
39. Which of the following statements concerning computer
program modifications is incorrect?

A. After the amended program has received final approval, the


change is implemented by replacing the production version with
the developmental version.
B. During the modification process, the developmental version
of the program must be kept separate from the production version.
C. When a program change is submitted for approval, a list of
all required updates should be compiled and then approved by
management and program users.
D. Only material program changes should be thoroughly tested and
documented.

40. Old and new systems operating simultaneously in all locations


is a test approach known as parallel testing.

Pilot testing involves implementing a new system in one part of


the organization, while other locations continue to use the
current system.

A. True; False C. False; True


B. Both are True D. Both are False

41. A collection of data that is shared and used by a number of


different users for different purposes is a
A. Database
B. Memory
C. File
D. Record

The standard defines database as a collection of data that is


shared and used by a number of users for different purposes.

42. Which of the following computer software is used to create,


maintain, and operate a database?
A. Application software
B. Systems software
C. Database management system (DBMS)
D. Database administrator

The DBMS is used to create, maintain, and operate a data-base.


It facilitates the physical storage of the data, maintains the
interrelationships among the data, and makes the data available
to application programs.

43. Two important characteristics of a database system are


A. The database and the DBMS.
B. Data sharing and data independence
C. The DBMS and data sharing.
D. The DBMS and data independence.

The two important characteristics of a database system are data


sharing and data independence.

Data sharing can be achieved if the database contains data which


are setup with defined relationships and are organized in a
manner that permits several users to access and use the data in
different application programs.

The need for data sharing creates the need for data independence
from application programs. Through the DBMS, data are recorded
only once, for use by different application programs. There will
be true data independence if the structure of data can be changed
without affecting the application programs, and vice versa.

44. To protect the integrity of the database, data sharing by


different users requires organization, coordination, rules, and
guidelines. The individual responsible for managing the database
resource is the
A. Programmer
B. Database administrator
C. User
D. CIS manager

The database administrator is responsible generally for the


definition, structure, security, operational control, and
efficiency of databases, including the definition of the rules
by which data are accessed and stored.

45. An auditor who wishes to trace data through several


application programs should know what programs use the data,
which files contain the data, and which printed reports display
the data. In a database system, the information could be found
in a
A. Decision table
B. Data dictionary
C. Database schema
D. Data encryptor

A software within the DBMS that keeps track of the location of


the data in the database is called the data dictionary.

Answer A is incorrect because a decision table is a matrix


presentation of the decision points and related actions included
in a computer program.
Answer C is incorrect because the database schema describes the
database structure.

Answer D is incorrect because an encryptor encodes messages.

46. Which of the following is the greatest advantage of a data-


base system?
A. Data redundancy can be reduced.
B. Backup and recovery procedures are minimized.
C. Multiple occurrence of data items are useful for consistency
checking.
D. Conversion to a database system is inexpensive and can be
accomplished quickly.

In a database system, data redundancy is kept to a minimum


because the DBMS records the data once, for use by various
application programs. Storage structures are created that make
the application programs independent of the location of the data.

Because each item in the database has a standard definition,


name, and format; and related items are linked by a system of
pointers, the application programs need only to specify the data
name, not the location.

Answer B is incorrect because backup and recovery procedures in


a database system are just as crucial as in a traditional flat-
file system.

Answer C is incorrect because data redundancy-that is, multiple


occurrences of data items-is substantially reduced in a database
system.

Answer D is incorrect because converting large amount of data


to a database is costly and time consuming.

47. The following statements relate to a database management


system (DBMS) application environment. Which is false?
A. Data definition is independent of any one program.
B. The physical structure of the data is independent of user
needs.
C. Data are used concurrently by different users.
D. Data are shared by passing files between programs or systems.

In a database system, application programs share the data in the


common database for different purposes. Thus, there is no need
to pass files between applications.

48. Which of the following is an advantage of a database


management system (DBMS)?
A. A decreased vulnerability as the DBMS has numerous security
controls to prevent disasters.
B. Each organizational unit takes responsibility and control for
its own data.
C. Data independence from application programs.
D. The cost of the CIS department decreases because users are
now responsible for establishing their own data handling
techniques.

An important characteristic of a database system is that


applications are independent of the database structure. This
allows programs to be developed for the users specific needs
without concern for data retrieval problems. Moreover, changes
to the physical or logical structure of the database can be made
without the need to modify any of the application programs that
use the database.

Answer A is incorrect because the DBMS is no safer than any


other computer information systems.

Answer B is incorrect because each organizational unit develops


its application programs that will use the data items in the
common database.

Answer D is incorrect because data handling techniques remain


to be the responsibility of the CIS department.

49. Which of the following is usually a benefit of transmitting


transactions in an electronic data interchange (EDI)
environment?
A. A reduced need to test computer controls related to sales and
collections transactions.
B. A compressed business cycle with lower year-end receivables
balances.
C. No need to rely on third-party service providers to ensure
security.
D. An increased opportunity to apply statistical sampling
techniques to account balances.

Because EDI transactions are transmitted and processed in real


time, delays are eliminated in receiving and processing an order,
shipping goods, and receiving payment. Thus, EDI compresses an
entitys business cycle and results in lower year-end
receivables balances.

Answer A is incorrect because the use of a complex processing


system increases the need to test computer controls.
Answer C is incorrect because an EDI system typically uses a VAN
value added network) as a third-party service provider, and
reliance on VAN controls may be critical.

Answer D is incorrect because all transactions (not just a


sample) may be tested with the aid of computer technology.

50. The internal controls over computer processing include both


manual procedures and procedures designed into computer programs
(programmed control procedures). These manual and programmed
control procedures comprise the general CIS controls and CIS
application controls. The purpose of general CIS controls is to

A. Establish specific control procedures over the accounting


applications in order to provide reasonable assurance that all
transactions are authorized and recorded and are processed
completely, accurately, and on a timely basis.
B. Establish a framework of overall controls over the CIS
activities and to provide a reasonable level of assurance that
the overall objectives of internal control are achieved.
C. Provide reasonable assurance that systems are developed and
maintained in an authorized and efficient manner.
D. Provide reasonable assurance that access to data and computer
programs is restricted to authorized personnel.

The purpose of general CIS Controls is to establish a framework


of overall controls over the CIS activities and to provide a
reasonable level of assurance that the overall objectives of
internal control are achieved.

General CIS controls may include:


Organization and management controls
Application systems development and maintenance controls.
Computer operation controls.
Systems software controls.
Data entry and program controls.

Answer A is incorrect because the establishment of specific


control procedures over the accounting applications is the
purpose of CIS application controls.

Answer C is incorrect because controls designed to provide


reasonable assurance that systems are developed and maintained
in an authorized and efficient manner are application systems
development and maintenance controls.

Answer D is incorrect because controls designed to provide


reasonable assurance that access to data and programs are
restricted to authorized personnel are data entry and program
controls.

51. CIS application controls include the following, except

A. Controls over input.


B. Controls over processing and computer data files.
C. Controls over output.
D. Controls over access to systems software and documentation.

Restricting access to systems software and documentation to


authorized personnel is a general CIS control.

CIS application controls include:

1. Controls over input designed to provide reasonable assurance


that:

Only authorized transactions are submitted for processing.


All authorized transactions are accurately converted into
machine-readable form.
Incorrect transactions are rejected, corrected, and, if
necessary, resubmitted on a timely basis.

2. Controls over processing and computer data files designed


to provide reasonable assurance that:

All transactions are processed as authorized.


No authorized transactions are omitted.
No unauthorized transactions are processed.
Processing errors are identified and corrected on a timely
basis.

3. Controls over output designed to provide reasonable


assurance that:

The results of processing are accurate.


Output is distributed only to authorized users.

52. The auditor is required to consider how an entitys general


CIS controls affect the CIS applications significant to the
audit. Accordingly, the auditor should

A. Review the design of the general CIS controls only.


B. Review the design of the CIS application controls only.
C. Review the design of the general CIS controls before reviewing
the CIS application controls.
D. review the design of the CIS application controls before
reviewing the design of the general CIS controls.
General CIS controls that relate to some or all applications are
typically interdependent controls in that their operation is
often essential to the effectiveness of CIS application
controls. A more efficient approach is to review the design of
the general CIS controls before reviewing the CIS application
controls.

53. The two broad categories of IT controls are general controls


and application controls. General controls include controls

A. For developing, maintaining, and modifying computer programs.


B. That relate to the correction and resubmission of erroneous
data.
C. Designed to provide reasonable assurance that only authorized
users receive output from processing.
D. Designed to provide reasonable assurance that all data
submitted for processing have been properly authorized.

General controls relate to all or many IT activities and often


include organization and management controls, application
systems development and maintenance controls, and data entry and
program controls.

Answers B, C, and D are incorrect because controls over


correction of erroneous input data, output distribution, and
authorization of input data are IT application controls.

54. Which of the following statements concerning application


controls is correct?

A. Application controls relate to the processing of individual


transactions.
B. Application controls relate to the processing of individual
transactions.
C. Application controls relate to various aspects of the IT
function including software and hardware acquisitions.
D. Application controls relate to various aspects of the IT
function including physical security and the processing of
transactions in various cycles.

55. The significance of hardware controls is that they

A. Ensure that run-to-run totals in application systems are


consistent.
B. Reduce the incidence of user input errors in online systems.
C. Ensure correct programming of operating system functions.
D. Assure that machine instructions are executed correctly.

To detect and control errors arising from the use of computer


equipment, hardware controls are built into the equipment by the
manufacturer, such as parity checks, read-after-write checks,
and echo checks.

Answer A is incorrect because run-to-run totals are used to


determine the completeness of update in an online system.
Separate totals are accumulated for all transactions processed
throughout a period and compared with the total of items
submitted for computer processing.

Answer B is incorrect because input controls such as the use of


limit checks, self-checking digits, and input screens can reduce
the incidence of user input errors in online systems.

Answer C is incorrect because computer programmers and/or


systems analysts are responsible for correcting program errors.

56. The following statements relate to internal control in an


electronic data interchange (EDI) environment. Which is the
true?

A. In EDI systems, preventive controls are generally more


important than detective controls.
B. Control objectives for EDI systems generally are different
from the objectives for other computer information systems.
C. Internal controls that relate to the segregation of duties
generally are the most important controls in EDI systems.
D. Internal controls in EDI systems rarely permit control risk
at below the maximum.

In all information system manual and computerized preventive


controls are more important than detective controls because
typically, the benefits exceed the costs. In an EDI environment,
it may be difficult to apply detective controls once a
transaction enters the computer system.

Answer B is incorrect because the basic objectives of internal


control are the same regardless of the nature of data processing.

Answer C is incorrect because adequate segregation of


incompatible functions in a CIS environment may not be feasible.

Answer D is incorrect because control risk in an EDI system may


be assessed at below the maximum level if relevant controls
exist and tests of controls provide evidence that those controls
are functioning effectively.

57. An entity has recently converted its revenue/receipt cycle


from a manual processing to an online, real-time processing
system. Which is the most probable result associated with
conversion to the new computerized processing system?
A. Less segregation of traditional duties.
B. significant increase in processing time.
C. Reduction in the entitys risk exposures.
D. Increase in processing errors.

The basic segregation of functions authorization,


recordkeeping, and asset custody in a manual system is not
usually feasible in a computerized system because of decreased
human involvement in processing financial information.

Answer B is incorrect because processing time is decreased in a


computerized system.

Answer C is incorrect because computer processing does not


necessarily reduce the number of risk exposures.

Answer D is incorrect because processing errors will decrease


as a result of the conversion to a new computerized system.

58. The most important segregation of duties in the organization


of the information systems function is

A. Using different programming personnel to maintain utility


programs from those who maintain the application programs.
B. Having a separate information officer at the top level of the
organization outside of the accounting function.
C. Assuring that those responsible for programming the system
do not have access to data processing operations.
D. Not allowing the data librarian to assist in data processing
operations.

An important general CIS control is segregation of duties.


Although some separation of duties common in a manual system may
not be feasible in a CIS environment, come functions should not
be combined.

The functions of systems analysts and programmers should not be


combined with the functions of computer operators. Programmers
and systems analysts may be able to effect changes in programs,
files, and controls and should therefore have no access to
computer equipment.

Computer operators should have no opportunity to modify programs


and data files, and should not have programming duties or
responsibility for installing new or modifying existing systems.

Answer A is incorrect because computer programmers handle all


types of computer software.
Answer B is incorrect because having a separate information
officer at the top level of the organization outside of the
accounting function would be less critical than separation of
duties between programmers and computer operators.

Answer D is incorrect because computer librarians may assist in


data processing operations. However, because they maintain
control over system and program documentation and data files,
they should not have access to computer equipment.

59. A systems analyst should have access to each of the


following, except

A. Edit criteria
B. Source code
C. Password identification tables
D. User procedures

Unauthorized changes to application programs and data files can


be made by the analyst if he/she has access to password
identification tables.

Answer A, B, and D are incorrect because the systems analyst


needs access to edit criteria, source code, and user procedures.

60. Which of the following would represent an internal control


weakness in an IT environment?

A. The computer librarian maintains custody of computer


application programs and files.
B. The data control group is solely responsible for distributing
computer-generated reports.
C. Computer operators have access to operator instructions and
have the authority to modify application programs.
D. Computer programmers write and modify programs designed by
systems analysts.

Computer operators should have access to operator instructions


so they can perform their duties. However, they should not have
the authority to modify application programs.

Answer A is incorrect because the computer librarian is


responsible for maintaining custody and recordkeeping for
computer application programs and data files.

Answer B is incorrect because an appropriate function of the


data control group is distribution of computer output and othe
reports.
Answer D is incorrect because computer programmers are
responsible for writing and revising programs designed by
systems analysts.

61. The manager of computer operations prepares a weekly schedule


of planned computer processing and sends a copy to the computer
librarian. The control objective of this procedure serves is to

A. Authorize the release of data files to computer operators.


B. Specify the distribution of computer results.
C. Specify file retention and disaster recovery policies.
D. Keep improper and unauthorized transactions from entering the
computer facility.

A computer librarian has in his/her custody data files, programs,


and documentation, all of which are his/her accountability. The
weekly schedule of planned computer processing provides
authorization for release of files to computer operators and a
consequent transfer of accountability.

Answers B and D are incorrect because the data control group


keeps unauthorized and improper transactions from entering the
computer facility and specifies the distribution of computer
results.

Answer C is incorrect because file retention and disaster


recovery policies are specified in the entitys backup and
recovery plan.

62. One of the major problems in a CIS environment is that


incompatible duties may be performed by the same individual. One
compensating control is the use of

A. Computer-generated hash totals


B. A computer log
C. A self-checking digit system
D. Echo checks

Computer and software y=usage is recorded in a computer (console)


log, including operator intervention during computer processing.
A compensating control for the lack of adequate segregation of
duties is by proper monitoring of the computer log. For example,
a computer log may include a list of operator intervention during
computer processing.

Answer A is incorrect because hash totals are control totals


calculated using nonfinancial data (for example, the sum of sales
order numbers) to keep track of the records in a batch.
Answer C is incorrect because a self-checking digit system is
an input control to detect data coding errors. It involves adding
a control digit to a code (for example, a bank account number)
when it is originally designed to allow the codes integrity to
be established during subsequent processing.

Answer D is incorrect because echo check is a hardware control


that involves the receiver of the message returning the message
to the sender to determine if the correct message was received.

63. In the organization of the information systems function, the


most important separation of duties is

A. Using different programming personnel to maintain utility


programs from those who maintain the application programs.
B. Assuring that those responsible for programming the system
do not have access to data processing operations.
C. Not allowing the data librarian to assist in data processing
operations.
D. Having a separate information officer at the top level of the
organization outside of the accounting function.

64. An entity has recently converted its purchasing cycle from


a manual process to an online computer system. Which of the
following is a probable result associated with conversion to the
new IT system?

A. Traditional duties are less separated.


B. Increased processing time.
C. Reduction in the entitys risk exposure.
D. Increased processing errors.

65. An entity should plan the physical location of its computer


facility. Which of the following is the primary consideration
for selecting a computer site?

A. It should be in the basement or on the ground floor.


B. It should maximize the visibility of the computer.
C. It should minimize the distance that data control personnel
must travel to deliver data and reports and be easily accessible
by a majority of company personnel.
D. It should provide security.

The computer and other peripheral pieces of hardware should be


protected from disasters such as fire, flood, sabotage, and
theft. Thus, the primary consideration for selecting a computer
site should be the security of the computer facility.

Answer A is incorrect because the basement or the ground floor


is not always a secured place. For example, installing a computer
facility on the ground floor or in the basement of an old office
building in Malabon City could be disastrous because of frequent
flooding.

Answer B is incorrect because maximizing the visibility of the


computer would be an invitation to burglars and other computer
criminals.

Answer C is incorrect because a majority of entity personnel


need not have an easy access to computer site since only
authorized personnel should be allowed in the computer facility.

66. Which of the following statements regarding security


concerns for notebook computers is false?

A. The primary methods of control usually involve application


controls.
B. Centralized control over the selection and acquisition of
hardware and software is a major concern.
C. Some conventional controls such as segregation of duties may
not be feasible.
D. As their use becomes more sophisticated, the degree of concern
regarding physical increases.

General controls apply to all CIS activities. Given the nature


of notebook computers, general controls to prevent theft of
equipment and data and restrict access to the use of equipment
and data must be the primary concerns.

67. The following are a database administrators


responsibilities, except

A. Develop application programs to access the database.


B. Design the content and organization of the database.
C. Protect the database and its software.
D. Monitor and improve the efficiency of the database.

Systems analysts and programmers, not a database administrator,


have the responsibility of developing application programs to
access the database.

Answe4r B, C, and D are incorrect because designing the content


and organization of the database; protecting the database and
its software; and monitoring and improving the efficiency of the
database are appropriate responsibilities of a database
administrator.

68. Which of the following groups should have the operational


responsibility for the accuracy and completeness of computer-
based information?
A. External auditors
B. Internal auditors
C. Users
D. Top management

Users are in the best position to review the accuracy and


completeness of computer output in relation to the input
provided. Thus, the operational responsibility for the accuracy
and completeness of computer-based information should be placed
on users.

Answer A is incorrect because the primary purpose of external


auditing is the expression of an opinion on an entitys financial
statements.

Answer B is incorrect because internal auditing is an independent


appraisal activity within an organization. Therefore, internal
auditors should not have operational responsibility.

Answer D is incorrect because top management is responsible for


the overall control of the CIS.

69. An inexperienced computer operator mounted an incorrect


version of the accounts receivable master file on a tape drive
during processing. Consequently, the entire processing run had
to be repeated at a prohibitive cost. Which of the following
software controls would be most effective in preventing this
type of operator error from affecting the processing of files?

A. File header and label check


B. Data transmission check
C. Memory isolation protection
D. Unauthorized access protection

An effective control to reduce risk of mounting an incorrect


version of a master file is the use of external, header, and
trailer labels. An external label is a human-readable label
written on a gummed paper to be attached to the file. A header
label is a machine-readable label at the beginning of a file
that identifies it. A trailer label is also a machine-readable
label at the end of a file containing control totals and record
counts.

Answer B is incorrect because only the accuracy of the


communication is verified by a data transmission check.

Answer C is incorrect because memory isolation protection (also


called boundary protection) ensures that while multiple jobs are
running simultaneously, the memory partition allocated to each
job is not changed.

Answer D is incorrect because access controls (for example the


use of personal identification codes such as passwords and PINs)
ensure that unauthorized access to programs and files is
prevented.

70. Which of the following is the best method to prevent


unauthorized alteration of online records?

A. Computer sequence checks


B. Computer matching
C. Database access controls
D. Key verification

Unauthorized access to online records can be prevented by


establishing and implementing access controls to ensure that
only authorized personnel have access to the companys database.

73. Which of the following controls would most likely provide


protection against unauthorized changes in production programs?

A. Restricting programmer access to the computer room

B. Requiring two operators to be present during equipment


operation.

C. Limiting program access solely to operators.

D. Implementing management review of daily run logs.

The risk unauthorized changes will be reduced if systems


analysts, programmer, and others are denied access to the
resident programs. However, computer operators should have
access to the production programs in order to run the programs.

Answers A and B are incorrect because unauthorized changes to


programs can be made by programmers at terminals regardless of
whether they are denied access to the computer room and
regardless of whether two operators are present during equipment
operation.
Answer D is incorrect because management review of computer
(console) logs, not run logs, would be an effective control.

74. Which of the following would most likely indicate that a


computer virus is present?

A. Numerous copyright violations due to unauthorized use of


purchased software.

B. Unexplained losses of or changes of data.

C. Frequent power surges that harm computer equipment.

D. Inadequate backup, recovery, and contingency plans.

A virus is a program that attaches itself to legitimate program


to penetrate the operating system and cause destruction to the
operating system, application programs, and data files. For
example, a virus can simply copy itself a number of times within
the memory to destroy resident programs and data.

Answers A, C, and D are incorrect because copyright violations,


frequent power surges, and inadequate backup, recovery, and
contingency plans are not indicators of a computer virus.

75. Which of the following operating procedures would most likely


increase an entitys exposure to computer viruses?

A. Downloading public-domain software from electronic bulletin


boards.

B. Installing original copies of purchased software on hard disk


drives.

C. Frequent back up of files.


D. Encryption of data files.

Personal computers are major source of virus penetration.


Downloading public-domain software carries a risk that virus-
infected data may enter the system.

Answer B is incorrect because original copies of purchased


software should be virus- free.

Answers C and D are incorrect because viruses are spread through


distribution of infected files, not through encryption or
frequent backup of files.

76. An entity installed antivirus software on all its personal


computers. The software was designed to prevent initial
infections, stop replication attempts, detect infections after
their occurrence, mark affected system components, and remove
viruses from infected components. The major risk in relying on
antivirus software is that it may

A. Consume too many system resources.

B. Interfere with system operations.

C. Not detect certain viruses.

D. Make software installation too complex.

Antiviral programs (also called vaccines) are used to examine


application and operating system programs for the presence of
viruses and remove them from the affected programs. However, a
vaccine works only on known viruses and there is no guarantee
that it will work if virus has been mutated.
Answers A and B are incorrect because antiviral software can be
set to execute at startup so as not to consume too many system
resources.

Answer D is incorrect because installation of antiviral software


is not an overly complex process.

77. The accountant who prepared a spreadsheet model for workload


forecasting left the company, and his successor was unable to
understand how to use the spreadsheet. The best control to permit
new employees to understand internally developed programs is

A. Adequate backups are made for spreadsheet models.

B. Use of end-user computing resources is monitored.

C. End- user computing efforts are consistent with strategic


plans.

D. Documentation standards exist and are followed.

Because of inadequate program documentation, the accountants


successor could not use the spreadsheet model. New employees
will be able to understand internally developed programs if
documentation standards exist and are being followed.

Answer A is incorrect because the accountants successor could


not use the spreadsheet model due to inadequate documentation,
not inadequate backups.

Answer B is incorrect because monitoring means controlling the


use of resources.

Answer C is incorrect because ensuring consistency with


strategic plans relates to the systems effectiveness.
78. What is the appropriate term for the process of monitoring,
evaluating, and modifying a system?

A. Feasibility study

B. Maintenance

C. Implementation

D. Analysis

Systems maintenance means keeping a new system that has been


designed and implemented current wit user needs. This basically
involves revising the systems and application programs to meet
new user needs and to correct designed errors. The responsibility
for systems maintenance is assumed by systems analysts and
programmers.

Answer A is incorrect because a feasibility study is made to


determine the technical, legal, operational, and schedule (i.e.,
the companys ability to implement the project within an
acceptable time) feasibility of a proposed system.

Answer C in incorrect because system implementation involves


data conversion; coding and testing applications; purchased and
installation of equipment; training of employees; system
documentation; and installation of the new system.

Answer D is incorrect because systems analysis involves a survey


of the current system, an analysis of the users needs, and
gathering and evaluation of facts.

79. Program documentation is a control designed primarily to


provide reasonable assurance that

A. Programs are kept up to date and perform as intended.

B. No one uses the computer hardware for personal reasons.


C. Programs are free of syntax and logic errors.

D. Programmers have access to operational materials.

Program documentation provides detailed information about each


application program including the source program, file formats
and record layouts, program flowcharts, written authorizations
for all program changes, and operating transactions. For a
computer system to operate efficiently, adequate and up-to-date
program documentation is necessary.

Answer B is incorrect because program documentation cannot


ensure security of computer hardware.

Answer C is incorrect because debugging should uncover errors


in programs.

Answer D is incorrect because programmers should not have access


to operational materials such as tape library or information on
disk files.

80. An entity updates its account receivable master file weekly


and retains the master files and corresponding update
transactions for the most recent two-week period. The purpose
of this periodic retention of master files and transaction data
is to

A. Validate groups of update transactions for each version.

B. Permit reconstruction of the master file if needed.

C. Verify run-to-run control totals for receivables.

D. Match internal labels to avoid writing on the wrong volume.


The grandparent-parent-child approach (also called grandfather-
father-son approach) is used in sequential file batch systems.
This backup technique begins when the current master file (the
parent) is processed against a transaction file to create a new
updated master file (the child). When a new batch of transactions
is processed, the child becomes the parent (the current master
file), and the parent (the original master file) becomes the
grandparent or backup file.

As described, the grandparent-parent-child backup technique


involves the creation and retention of three generations of
master files enable reconstruction of destroyed or corrupted
master file.

The systems designer is responsible for determining the number


of backup files needed for each application. The designer should
consider the degree of file activity and the financial relevance
of the system in making such decision.

Answer A and D are incorrect because validation routines and


internal labels may prevent data from being destroyed but do not
allow recovery of lost or destroyed data.

Answer C is incorrect because verification of run-to-run totals


ensures completeness of processing, not data recovery.

81. An entitys contingency plans for computer information


systems should include appropriate back up arrangements. Which
of the following arrangements would be considered too vendor-
dependent when vital operations require almost immediate
availability of computer resources?

A. A cold site arrangement


B. A hot site arrangement

C. A cold and hot arrangement

D. Using excess capacity at another data center within the


entity.

A cold site is a backup facility that has all the needed


computer equipment. This backup arrangement is too vendor-
dependent because it relies on the vendors timely delivery of
the needed computer equipment.

Answer B is incorrect because a hot site backup facility has


all the needed resources in place, including the computer
equipment, and is therefore not vendor-dependent.

Answer C is incorrect because a cold and hot site backup


facility has a hot site component that is fully configured and
available for immediate use while the cold site is being
configured, making it not too vendor-dependent.

Answer D is incorrect because having excess capacity at another


data center within an entity means that there are available
resources that can be used.

82. Which of the following is primary objective of security


software?

A. To detect the presence of computer viruses.

B. To monitor the segregation of functional responsibilities


within applications.

C. To prevent installation of unauthorized utility software.

D. To control access to information system resources.


The primary objective of security software is to keep
unauthorized intruders from accessing information system
resources and data files.

Answer A is incorrect because antiviral software, not security


software, detects the presence of computer viruses.

Answer B is incorrect because security software can be used to


establish, not monitor, separation of duties.

Answer C is incorrect because security software can be used to


control the use of utility software not to prevent installation
of unauthorized utility software.

83. All administrative and professional staff in an entitys


legal department prepare documents on terminals connected to a
host LAN file server. Which of the following is the best control
over unauthorized access to sensitive documents in the system?

A. Required entry passwords for access to the system.

B. Required entry passwords for access to individual documents.

C. Physical security for all disks containing document files.

D. Periodic server backup and storage in a secure area.

Effective access controls normally required different passwords


to access the system, to read certain data files, and other
information system resources. The control over unauthorized
access to sensitive documents is required password entry for
access to individual documents.
Answer A is incorrect because required entry of passwords for
access to systems allows all departmental personnel to gain
access to all documents in the system.

Answer C is incorrect because LAN may not use floppy disks.

Answer D is incorrect because although periodic server backup


and storage in a secure area is a good security/ backup control
procedure, it would not prevent intruders from accessing
sensitive documents online.

84. An internal control has just concluded a physical security


audit of data center which is primarily engaged in top-secret
defense contract work. The auditor has recommended biometric
authentication for workers entering the building. The
recommendation might include devices that verify all of the
following, except

A. Fingerprints

B. Password patterns

C. Speech patterns

D. Retina patterns

The use of biometrics devices is considered the ultimate in user


authentication procedures. These devices are used to establish
an individuals identity by measuring various personal
characteristics, fingerprints, voiceprints, retina prints, or
signature characteristics.

85. Which of the following best describes the process called


authentication?
A. The system verifies the identity of the user

B. The user identifies himself/herself to the system

C. The user indicates to the system that the transaction was


processed correctly

D. The system verifies the user is entitled to enter the


transactions request.

Authentication is the process of verifying the identity of the


user. Biometric devices are used to authenticate an individuals
identity using physiological or behavioral traits such as retina
patterns, fingerprints, and speech patterns.

Answer B is incorrect because when the user identifies


himself/herself to the system, it does not necessarily mean that
the system verifies his/her identity.

Answer C is incorrect because this is an application control


that relates to the accuracy of processing transactions.

Answer D is incorrect because authentication does not


necessarily include determining the function of a user whose
identity has been verified.

86. Which of the following assurances is not provided by an


application control?

A. Review and approval procedures for new systems are set by


policy and adhered to

B. Authorized transactions are completely processed once and


only once

C. Transaction data are complete and accurate

D. Processing results are received by the intended users


Answers B, C and D are incorrect because these are the objectives
of application controls.

87. Data processing activities may be classified in terms of


three stages or processes: input, processing, and output. Which
of the following activities is not normally associated with the
input stages?

A. Recording

B. Batching

C. Reporting

D. Verifying

Reporting is normally associated with the output stage. Output


is the result of computer processing, for example, a hard copy
printout of a report, magnetic files, or invoices.

Answers A, B and D are incorrect because recording, batching,


and verifying are normally associated with the input stage.

88. Which of the following is the purpose of input controls?

A. To ensure the authorization of access to data files

B. To ensure the completeness, accuracy, and validity of updating

C. To ensure the completeness, accuracy, and validity of input

D. To ensure the authorization of access to program files.

Input controls are designed to provide reasonable assurance that


the data received for computer processing are complete, accurate
and valid.
Answers A and D are incorrect because assuring the authorization
of access to date and program files is the objective of access
control.

Answer B is incorrect because ensuring the completeness,


accuracy and validity of updating is the objective of processing
controls.

89. If a control total were to be computed on each of the


following data items, which would best be identified as a hash
total for a payroll IT application?

A. Employee numbers

B. Total debit and credit amounts

C. Gross wages earned by employees

D. Total hour worked

90. An entity uses the account code 699 for depreciation expense.
However, one of the company data input clerks often codes
depreciation expense as 996. The highest account code in the
companys system is 700. What programmed control procedure would
detect this error?

A. Pre-data input check

B. Sequence check

C. Valid-code test

D. Valid-character test

91. Which of the following provides the most valuable information


for detecting unauthorized input from a terminal?

A. User error report


B. Transaction log

C. Error file

D. Console log printout

A transaction log is a permanent record of all completely


validated transactions received for the computer processing.
Subsequent comparison of the transaction log with authorized
transactions such as authorized source documents will detect
unauthorized input from a terminal.

Answer A is incorrect because a user error report only lists


input that fails the validation tests.

Answer C is incorrect because an error file is used to store and


correct records detected during validation.

Answer D is incorrect because a console log is a record of


computer and software usage. It does not record individual
transactions transmitted from the terminal.

92. Many customers, managers, employees, and suppliers have


blamed the computer for making errors. In reality, computers
make very few mechanical errors. Which of the following is the
most likely source of errors in fully operational computer-based
system?

A. Systems analysis and programming

B. Operator error

C. Processing

D. Input
It is garbage-in, garbage-out in computer processing---
erroneous input results in erroneous output.

Answer A is incorrect because proper design and implementation


of computer programs would eliminate most syntax and logic errors
or bugs.

Answer B is incorrect because operator (run) manuals which


describe how to run the system, decrease the chance of operator
error.

Answer C is incorrect because, once a program has been thoroughly


tested (for example, by creating hypothetical master files and
transaction files to be processed by the program being tested),
the processing of appropriate data does not result in errors.

93. Data conversion is the transcription of transaction data


from source documents to magnetic tape or disk suitable for
computer processing. Which of the following data conversion
methods is most difficult to audit?

A. Keying data to disk for online processing

B. Keying data to disk for batch processing

C. Reading source data using optical character recognition

D. Keying data to source documents for magnetic ink character


recognition

Data conversion in online systems is difficult to audit because


there is usually no visible audit trail. Transactions are
transmitted directly from terminal and hard copy source
documents are often lacking.
Answer B is incorrect because keying data to disk for batch
processing creates records that can be readily tested.

Answer C is incorrect because hard copy source documents are


retained in optical character recognition. Moreover, this method
reduces the risks of conversion error.

Answer D is incorrect because magnetic ink character recognition


provides hard copy source documents that can be used for audit
purposes.

94. Which of the following best describes the online data


processing control called preformatting?

A. The display of a document with blanks for data items to be


entered by the terminal operator.

B. a program initiated prior to regular input to discover errors


in data before entry so that the errors can be corrected.

C. A series of request for required input data that requires an


acceptable response to each request before a subsequent request
is made.

D. A check to determine if all data items for a transaction have


been entered by the terminal operator.

A preformatted screen approach may be used in online systems to


avoid data entry error. Under this approach, blanks for specified
data items will be displayed on the monitor. This is most
appropriate when data entry is from a source document. Moreover,
the screen format may even be in the form of a transaction
document.

Answer B is in correct because an edit/validation routine is a


program initiated prior to regular input to discover errors
before entry so that errors can be corrected.
Answer C is incorrect because the dialogue approach is another
screen prompting method that is most appropriate for data
received orally, e.g., by phone.

Answer D is incorrect because a check to determine if all data


items for a transaction have been entered by the terminal
operator is called completeness check.

95. When erroneous data are detected by the computer program


controls, such data may be excluded from processing and printed
on an error report. Who should review and follow up this error
report?

A. System analyst

B. Data control group

C. Computer operator

D. Computer programmer

Many entities have a data control group (independent of the


computer processing operation) that acts as liaison between the
end user and data processing.

The data control group is responsible for receiving from users,


transaction documents for processing; and controlling the
distribution of computer output such as documents and reports.
It is responsible for the following up error reports to ensure
that the erroneous records are corrected by the users and
reprocessed by the computer center.

Answers A, C, and D are incorrect because systems analysts,


computer operators, and computer programmers are not independent
of computer operations.
96. If a payroll system continues to pay employees who have been
terminated, control weakness most likely exist because

A. Input file label checking routines built into the program


were ignored by the operator.

B. Programmed controls such limit checks should have been built


into the system.

C. Procedures were not implemented to verify and control the


receipt by the computer processing department of all
transactions prior to processing.

D. There were inadequate manual controls maintained outside the


computer system.

In a payroll system, the authorization to pay employees should


come from the personnel department, which is external to the
computer processing department. Hence, inadequate controls
maintained outside the computer system are likely to allow the
payments to terminated employees to continue without being
detected.

Answers A, B, and C are incorrect because the use of input file


labels, limit checks, and batch total will not detect
unauthorized transactions.

97. In the accounting system of ABC Company, the amounts of cash


disbursements entered at a computer terminal are transmitted to
the computer, which immediately transmits the amounts back to
the terminal for display on the terminal screen. This display
enables the operator to

A. Establish the validity of the account number

B. Prevent the overpayment of the account


C. Verify the accuracy of the amount entered

D. Verify the authorization of the disbursement

Displaying the amounts entered on the terminal screen allows the


terminal operator to visually verify the accuracy of the amounts
entered.

98. Which of the following input validation checks is least


likely to be appropriate in an online, real-time system?

A. Sign check

B. Sequence check

C. Reasonableness check

D. Redundant data check

The sequence check control is appropriate only in systems that


uses sequential master files. This control determines if the
records are in the proper order by comparing the sequence of
records are in the batch with the previous record. Because
records are not processed sequentially in an online, real-time
system, this control is not likely to be appropriate.

Answers A, C, and D are appropriate in an online, real-time


system.

A sign check tests data to determine if they have appropriate


arithmetic sign.

A reasonableness check determines if an amount falls within


predefined limits. For example, the number of hours worked in a
single day should be neither less than zero nor more than 12.
A redundancy check assures that an application processes each
record only once.

99. A receiving clerk keyed in a shipment from a remote terminal


and inadvertently omitted the purchase order number. Which of
the following controls would most likely detect this error?

A. Completeness check

B. Compatibility check

C. Sequence check

D. Reasonableness test

A completeness test identifies missing data within a single


transaction record (for example, missing purchase order number
on the shipping document) or records within a batch of
transaction data.

Answer B is incorrect because a compatibility check (also called


field test) determines whether a field contains proper
characters.

Answer C is incorrect because a sequence check determines if


records have been properly sorted.

Answer D is incorrect because a reasonableness test determines


if the value is within predetermined limits.

100. A wholesaler of automotive parts has a computerized billing


system. Because of a clerical error while entering information
from the sales order, one of its customer was billed for only
three of the five items ordered and received. Which of the
following controls could have prevented or promptly detected
this clerical error?
A. Periodic comparison of total accounts receivable per accounts
receivable master file with total accounts receivable per
accounts receivable control account.

B. A completeness check that does not allow a sales invoice to


be processed if key fields are blank.

C. Prenumbered shipping documents together with a procedure for


follow up anytime there is not a one-to-one relationship between
shipping documents and sales invoices.

D. Matching line control counts produced by the computer with


predetermined line control counts.

A line control count could have prevented or promptly detected


the clerical error. This control technique involves a count of
individual line items on a document. Missing lines can be
detected by simply comparing these counts with predetermined
line control counts for each document.

Answer A is incorrect because the three-item sales invoice would


be the basis for updating both the accounts receivable master
file and control account. Hence, no discrepancy would be
disclosed by the comparison.

Answer B is incorrect because a completeness check would not


detect the billing error because other sales invoices may
properly contain three or fewer lines.

Answer C is incorrect because although the sales invoice has


missing lines, it exist and can be matched with the shipping
document.

101. Which of the following computerized control procedures


would most likely provide reasonable assurance that data
uploaded from personal computers to a mainframe are complete and
that no additional data are added?

A. Field-level edit controls that test each field for


alphanumerical integrity.

B. Self-checking digits to ensure that only authorized part


numbers are added to the database.

C. Batch control details, including financial totals and hash


totals.

D. Passwords that effectively limit access to only those


authorized to upload the data to the mainframe.

Batch totals which consist of record counts, financial or control


totals, and hash totals can be used to ensure the accuracy and
completeness of data uploaded from personal computers to
mainframe. After the uploading process, these totals are
reconciled with predetermined totals to test if the data have
been completely transferred.

A record count (also called item count) is the total number of


records in a batch.

A financial or control total is the peso value of a financial


field, for example, the total sales invoice amounts.

A hash total is the total of a unique nonfinancial field, for


example, the total of purchased order numbers in a batch.

Answers A, B, and D are incorrect because they do not provide


assurance about the completeness of data upload.

102. An entitys labor distribution report requires extensive


corrections each month because of labor hours charged to inactive
jobs. Which of the following data processing input controls
appears to be missing?

A. Validity check

B. Limit check

C. Missing data check

D. Control total

Validity checks compare actual values in a field (for example,


a transaction code) against acceptable (valid) values in the
master file. If the value in the field does not match one of the
acceptable values. The record is considered to be in error. If
the computer checks first for validity of the jobs, labor hours
would not be erroneously assigned to inactive jobs.

Answer B is incorrect because a limit check determines if the


value in the field exceeds a predetermined limit.

Answer C is incorrect because missing data checks are used to


determine if a field contains blank spaces. The computer
considers a record in error if blanks are detected where data
values are expected.

Answer D is incorrect because control totals are used to


reconcile computer input with processing result.

103. If, in reviewing an application system, it is noted that


the batch controls are not used, which of the following
statements by the user of the system is acceptable as a
compensating control?

A. The volume of transactions prohibits batching.

B. We do a 100% physical review of the input document to the


output document.
C. We do a 100% key verification of all data input.

D. The supervisor must approve all inputs.

A 100% physical review of the input document to the output


document will provide evidence that all records are completely
and accurately processed. Thus, this procedure will compensate
for the lack of batch control totals.

Answer A is incorrect because the use of batch control totals


is most appropriate in managing high volumes of transaction data.

Answer C is incorrect because a 100% key verification does not


assure that all records submitted for processing were
keypunched.

Answer D is incorrect because the supervisors approval of all


inputs does not assure that all approved inputs were processed.

104. A mail-order retailer of low-cost novelty items is receiving


an increasing number of complaints from customers about the wrong
merchandise being shipped. The order code for items has the
format WWXXYYZZ, which has the following meaning:

WW- major category

XX- minor category

YY- identifies the item

ZZ- identifies the catalog

In many cases, the wrong merchandise was sent because adjacent


characters in the order code had been transposed. The most
effective control to prevent this erroneous input is to

A. Use a master file reference for all codes to verify the


existence of items.
B. Separate the parts of the order code with hyphens to make the
characters easier to read.

C. Add check digits to the order codes and verify them for each
order.

D. Require customers to specify the name of each item they order.

Transposition errors can corrupt data codes and cause serious


data processing problems if they go undetected.

An effective control to detect data coding errors is by adding


a check digit (or digits) to a data code.

The check digits is the result of the mathematical calculation


done based on the original data code (the simplest form is to
add all the digits in the code). During the input process, the
system recalculates the check digits for each input and compares
the result with the check digit attached to the data code
entered.

Answer A is incorrect because order codes containing transposed


characters may match other items in the file. Thus, the use of
a master file reference code would not detect erroneous order
codes.

Answer B is incorrect because the use of hyphens would make the


order code easier to read, but would not detect order codes with
transposed characters.

Answer D is incorrect because requiring customers to specify the


name for each item they order would generally not allow detection
of erroneous codes.

105. Which of the following is the major purpose of the auditors


study and evaluation of the companys computer processing
operations?
A. Ensure the exercise of due professional care

B. Evaluate the reliability and integrity of financial


information

C. Become familiar with the companys means of identifying,


measuring, classifying, and reporting information

D. Evaluate the competence of computer processing operating


personnel

The auditor studies and evaluates information systems primarily


to ascertain whether financial data are accurate, reliable,
timely, and complete.

Answer A is incorrect because auditors are required to exercise


due care in all audits.

Answer B is incorrect because becoming familiar with the


companys information system is a means to achieve the auditors
principal objective.

Answer D is incorrect because evaluating the competence of


computer processing operating personnel is not the auditors
primary purpose of evaluating the companys information system.

106. When the auditor chooses to use only the non-IT segment of
a clients control to asses control risk, it is referred to as
auditing around the computer. Which one of the following
conditions need not be present to apply this audit approach?

A. The output must be listed in sufficient detail to enable the


auditor to trace individual transactions.

B. The source documents must be filed in a manner that makes it


possible to locate them.
C. The source documents must be available in a non-machine
language.

D. Computer programs must be available in English.

107. The following statements relate to the auditors assessment


of control risk in an entitys computer environment. Which is
correct?

A. The auditor usually can ignore the computer system if he/she


can obtain an understanding of the controls outside the computer
information system.

B. If the general controls are ineffective, the auditor


ordinarily can assess control risk at a low level if the
application controls are effective.

C. The auditors objectives with respect to the assessment of


control risk are the same as in a manual system.

D. The auditor must obtain an understanding of the internal


control and test controls in computer environments.

The overall objective and scope of an audit does not change in


a CIS environment. Regardless of the information system used by
the entity--- manual or computerized, the auditor is required
to obtain an understanding of internal control and assess control
risk to plan the audit.

Answer A is incorrect because, when an entitys computer


information system is significant (i.e., it has a material effect
on financial statement assertions), the auditors is required to
obtain an understanding of the CIS environment and determine
whether it may influence the assessment of inherent and control
risk.
Answer B is incorrect because, if general controls are
ineffective, the auditor is unlikely to assess control risk at
a low level, regardless of whether application controls have
been designed and implemented for each significant accounting
application.

Answer D is incorrect because tests of controls should be


performed only when the auditors risk assessment includes an
expectation of the operating effectiveness of controls (i.e.,
control risk is assessed at below the maximum), or when
substantive procedures alone do not provide sufficient
appropriate audit evidence at the assertion level.

108. Computer programs and data that the auditor may us as part
of the audit procedures to process data of audit significance
contained in an entitys information system are called

A. CAATs

B. DOOGs

C. BIIKs

D. BIIRDs

Computer-assisted audit techniques (CAATs) are computer programs


and data that the auditor may use in performing various audit
procedures, including the following:

Tests of details of transactions and balances


Analytical review procedures
Tests of general and application controls
Sampling programs to extract data for audit testing
Re-performance of calculations performed by the entitys
accounting system
Answer B, C, and D are incorrect because DOOGs, BIIKs, and BIIRDs
are not used in information technology (IT).

109. One common type of CAAT is the use of audit software to


process data audit significance from the entitys information
system. An audit software that has widespread popularity because
it is easy to use and requires little computer background on the
part of the auditor; it can be used on both mainframe and PC
systems; it allows the auditor to perform hi/her tests
independent of the entitys computer processing personnel; and
it can be used to audit the data in most file formats and
structures is called a

A. Customized program

B. Purpose-written program

C. Utility program

D. Package or Generalized audit software (GAS)

The easy-to-use and flexibility features of generalized audit


software (GAS) make it very popular to auditors in the audit
information technology (IT) environments. This audit software
is designed to perform common audit tasks or standardized data
processing functions, such as the following:

Reading data files


Selecting and analyzing information
Summarizing and totaling files
Performing or verifying calculations
Creating data files
Providing totals of unusual items
Reporting in an auditor-specified format
Answer A and B are incorrect because customized or purpose-
written programs are designed to perform audit tasks in specific
circumstances. These programs are used when an entitys computer
information system is so unique or complex that any GAS is deemed
unsuitable.

Answer C is incorrect because utility programs are part of the


operating system and security software packages that are
provided by computer manufacturers and software vendors. This
software performs routine data processing functions, such as
sorting, copying, creating, merging, erasing, and printing
files. It is not generally designed for audit purposes and may
not contain audit features, such as record counts or control
totals.

110. Customized or purpose-written programs perform audit tasks


in specific circumstances where package audits software is
deemed unsuitable usually because system constraints make it
difficult or impossible to use. A purpose-written program may
be developed by

A B C D

The auditor no yes yes no

The entity being audited yes yes no no

An outside programmer

hired by the auditor yes yes no no

111. These computer programs are enhanced productivity tools


that are typically part of a sophisticated operating systems
environment, for example, data retrieval software or code
comparison software

A. Purpose-written programs
B. System management programs

C. Utility programs

D. Generalized audit software

112. Embedded audit routines are sometimes built into an entitys


computer information system to provide data for later use by the
auditor. One technique involves embedding audit software modules
within an application system to provide continuous monitoring
of entitys transactions. These audit modules are used to create
logs that collect transaction information for subsequent review
by the auditor. These logs are called

A. System control audit review files (SCARFs)

B. Console logs

C. Computer logs

D. IT logs

113. When an accounting application is processed by computed,


an auditor cannot verify the reliable operation of programmed
controls by

A. Periodically submitting auditor-prepared test data to same


computer process and evaluating the results.

B. Constructing a processing system for accounting applications


and processing actual data from throughout the period through
both the clients program and the auditors program.

C. Manually comparing detail transaction files used by an edit


program with the programs generated error listings to determine
that errors were properly identified by the edit program.

D. Manually re-performing, as of a moment in time, the processing


of input data and comparing the stimulated results with the
actual results.
The effectiveness of programmed controls may not be tested if
auditing around the computer (also called the black box approach)
is to be applied. This involves manual comparison of the input
data with the computer output.

Because programmed controls are built into the computer program,


the auditor should instead apply the white box approach. This
means that the auditor should have an in depth understanding of
how the programmed control function and should consider using
CAATs in testing their effectiveness.

Answer A is incorrect because the use of the test data approach


is an effective method of evaluating the reliability of
programmed control procedures.

Answer B is incorrect because parallel simulation is also an


effective method of evaluating the reliability of programmed
controls.

Answer C is incorrect because manually comparing the output of


an auditors edit program with the error listing generated by
the clients program would provide evidence about the
reliability of programmed controls.

114. Auditing through the computer must be used when

A. Generalized audit software is not available.

B. Processing is primarily online and updating is real-time.

C. Input transactions are batched and system logic is


straightforward.

D. Processing primarily consists of sorting the input data and


updating the master files sequentially.
Auditing through the computer involves an in-depth understanding
of the computer programs logic. This approach is appropriate
when a complex and significant application is involved and
evidence external to the computer system is unlikely to be
available--for example, in an online, real-time system.

Answer A is incorrect because, in deciding on what audit approach


is appropriateauditing through or around the computer, the
auditor determines whether evidence external to the computer is
available, not whether generalized audit software is available.

Answer C is incorrect because, in a simple batch system, auditing


around the computer (the black box approach) is appropriate
because evidence external to the computer, such as printouts and
source documents, can be directly examined by the auditor.

Answer D is incorrect because, when processing is simple (for


example, when files are stored and processed sequentially),
evidence outside the computer is likely to be available.

115. When an auditor tests a computer information system, which


of the following is true of the test data approach?

A. Test data are processed by the clients computer programs


under the auditors control.

B. Several transactions of each type must be tested.

C. Test data must consist of all possible valid and invalid


conditions.

D. The program tested is different from the program used


throughout the year by the entity.

Under the test data approach, the auditor processes a specially


prepared set of input data containing possible valid and invalid
conditions using the clients application program.
The result of each test are compared with predetermined results,
based on the auditors understanding of the programmed controls.
This approach will allow the auditor to make an objective
evaluation of the program logic and the effectiveness of
programmed controls.

Answer B is incorrect because only one of each transaction type


needs to be tested and evaluated.

Answer C is incorrect because the auditor tests only those


controls that are relevant to the financial statement audit.

Answer D is incorrect because, if the program to be used for


testing is different from the program used throughout the year
by the client, no assurance can be obtained about the
effectiveness of programmed controls.

116. An auditor who is testing IT controls in a payroll system


would most likely use test data that contain conditions such as

A. Payroll checks with unauthorized signatures

B. Deductions not authorized by employees

C. Time tickets with invalid job numbers

D. Overtime not approved by supervisors

117. Auditors have learned that increased computerization has


created more opportunities for computer fraud but has also led
to the development of computer audits techniques to detect
frauds. A type of fraud that has occurred in the banking industry
is programming fraud in which the programmer designs a program
to calculate daily interest on savings accounts to four decimal
points. The programmer then truncates the last two digits adds
it to his account balance. Which of the following CAATs would
be most effective in detecting this type of fraud?
A. Generalized audit software that select account balances for
confirmation with the depositor

B. Snapshot

C. Parallel simulation

D. SCARF (Systems Control and Audit Review File)

In parallel simulation, the auditor uses a specially prepared


computer program that simulates key features or processes of the
application program to be tested.

Program logic and controls are evaluated by comparing the results


of processing actual data using the simulation program with the
results of processing the same actual data using the clients
application program.

Parallel simulation is the most effective CAAT application


because the amounts credited to the depositors accounts can be
compared with the amounts calculated by the auditors simulation
program.

Answer A is incorrect because confirmation of a depositors


account balance may fail to detect errors involving a very
insignificant amount (i.e., less than one centavo daily).

Answer B and D are incorrect because SCARFs and snapshots will


not detect the computer fraud prescribed.

118. To obtain evidence that online access controls are properly


functioning, an auditor is most likely to

A. Vouch a random sample of processed transactions to assure


proper authorization.

B. Create checkpoints at periodic intervals after live data


processing to test for unauthorized use of the system.
C. Enter invalid identification numbers or passwords to
ascertain whether the system rejects them.

D. Examine the transaction log to discover whether any


transactions were lost or entered twice because of a system
malfunction.

The auditor can directly test whether online access controls are
properly functioning by attempting to gain access to the system
by using invalid identification numbers or passwords.

Answer A is incorrect because unauthorized transactions may be


entered by any intruder who know valid identification numbers
or passwords.

Answer B is incorrect because, in a batch computer systems,


checkpoints are used as a recovery procedure.

Answer D is incorrect because examining the transaction log to


discover whether any transactions were lost or duplicated would
not determine if any online access controls are functioning
effectively.

119. Which of the following CAATs allows fictitious and real


transactions to be processed together without the knowledge of
client operating personnel?

A. Data entry monitor

B. Integrated test facility (ITF)

C. Parallel simulation

D. Input control matrix

The integrated test facility (ITF) approach enables the auditor


to test a computer programs logic and controls during its normal
operation. Under this approach, fictitious records for dummy
units (for example, a division, a department, or a dummy entity)
are integrated with legitimate records in the database.

During the normal computer processing, test transactions are


merged with actual transactions and processed against the dummy
records in the master database.

Because computer application with ITF can be tested without


intervention of operating personnel, ITF enhances audit
efficiency and increases reliability of the audit evidence.

Answers A and D are incorrect because data entry monitor and


input control matrix are not used by the auditor in testing an
entitys computer information system.

Answers C is incorrect because, in parallel simulation, real


(not fictitious) transactions are reprocessed.

120. In auditing an online perpetual inventory system, an auditor


selected certain file-updating transactions for detailed
testing. The audit technique that will provide a computer trail
of all relevant processing steps applied to a specific
transaction is called

A. Snapshot

B. Simulation

C. Tagging and tracing

D. Code comparison

Tagging and tracing involves selection of specific transactions


to be tagged (by attaching an indicator at input) and traced
through critical control points in the computer information
system.
The computer trail can be printed or stored in a computer file
for the auditors evaluation.

Answers A, B, and D are incorrect because snapshot, simulation,


and code comparison do not provide a trail of all relevant
processing steps.

TRUE OR FALSE

1. A hash total is a numeric value computed to provide assurance


that the original value has not been altered in construction
or transmission.

2. General controls include data validation controls.

3. A limit or reasonableness test is a test to ensure that a


numerical value does not exceed some predetermined value.

4. The control environment component of internal controls


includes access to computer programs.

5. As opposed to a manual control, an automated control should


function consistently in the absence of program changes.

6. The display monitor is a software component of a computer


system.

7. The systems analyst should not be allowed access to programs


listings of application programs.

8. The posting of a transaction, as it occurs, to several files,


without intermediate printouts is a characteristic of a
batch processed computer system.
9. Controls which are built in by the manufacturer to detect
equipment failure are called input controls.

10. Echo checks, data encryption, and parity checks are data
transmission controls.

11. When applying the test data approach, auditors use auditor-
controlled software to do the same operations that the
clients software does, using the same data files.

12. A problem for a CPA associated with advanced IT systems is


that the audit trail is sometimes generated only in machine
readable form.

13. Controls which are designed to assure that the information


processed by the computer is authorized, complete, and
accurate are called input controls.

14. A system in which the end use is responsible for the


development and execution of the computer application that
he or she uses is called decentralized computing.

15. In an IT-intensive environment, most processing controls are


programmed controls.

16. An example of an access control is a check digit.

17. Output controls are designed to assure that the data


generated by the computer are used appropriately by
management.
18. An internal control deficiency occurs when computer
personnel originate changes in customer master files.

19. Auditing through the computer is generally used processing


is primarily on line and updating is real- time.

20. General control have a pervasive effect on the operating


effectiveness of application controls.

21. Random errors are more likely in a batch system than in an


online system.

22. Auditing by testing the input and output of a computer system


instead of the computer program itself will detect all
program errors, regardless of the nature of the output.

23. In an IT system, automated equipment controls or hardware


controls are designed to detect and control errors arising
from the use of equipment.

24. Logging in to the companys information system via password


is an application control.

25. Controls that relate to a specific use of the IT system,


such as the processing of sales or cash receipts, are called
general controls.
KEY ANSWERS

1. D 25. B 49. B 73. C 97. C


2. B 26. A 50. B 74. B 98. B
3. B 27. D 51. D 75. A 99. A
4. B 28. B 52. C 76. C 100. D
5. C 29. B 53. A 77. D 101. C
6. B 30. C 54. B 78. B 102. A
7. D 31. C 55. D 79. A 103. B
8. B 32. D 56. A 80. B 104. C
9. D 33. C 57. A 81. A 105. B
10. C 34. C 58. C 82. D 106. D
11. B 35. C 59. C 83. B 107. C
12. D 36. B 60. C 84. B 108. A
13. D 37. A 61. A 85. A 109. D
14. A 38. A 62. B 86. A 110. B
15. D 39. D 63. B 87. C 111. B
16. A 40. B 64. A 88. C 112. A
17. A 41. A 65. D 89. A 113. D
18. B 42. C 66. A 90. C 114. B
19. A 43. B 67. A 91. B 115. A
20. C 44. B 68. C 92. D 116. C
21. D 45. B 69. A 93. A 117. C
22. C 46. A 70. C 94. A 118. C
23. C 47. D 71. D 95. B 119. B
24. B 48. C 72. A 96. D 120. C

TRUE OR FALSE

1. FALSE 6. FALSE 11. FALSE 16. FALSE 21. FALSE


2. FALSE 7. FALSE 12. TRUE 17. FALSE 22. FALSE
3. TRUE 8. FALSE 13. TRUE 18. TRUE 23. TRUE
4. FALSE 9. FALSE 14. FALSE 19. TRUE 24. FALSE
5. TRUE 10. TRUE 15. TRUE 20. TRUE 25. FALSE
Chapter 7
Audit Objectives and Procedures, Evidence, and Documentation

1. All the information used by the auditor in arriving at the


conclusions on which the audit opinion is based is called
a. Audit information
b. Audit evidence
c. Accounting records
d. Corroborating information
According to PSA 500 (Audit Evidence), audit evidence refers to
information used by the auditor in arriving at the conclusions
on which the auditor's opinion is based. It includes both
information contained in the accounting records underlying the
financial statements and other information.
2. Anentity's accounting records generally include the record
ofinitial entries and supporting records including
e)Confirmations from third parties
f)Information obtained by the auditor from such audit
procedures as inquiry, observation, and inspections.
g) Worksheets and spreadsheets supporting cost allocations
h) Other information developed by, or available to, the
auditor to permit him/her to reach conclusions through
valid reasoning.
According to PSA 500, accounting records include the records of
initial accounting entries and supporting records such as the
following:
f) Checks and records of electronic fund transfer (EFT)
g) Invoices
h) Contracts
i) The general and subsidiary ledgers, journal entries and
other adjustments to the financial statements that are not
reflected in formal journal entries
j) Records such as worksheets and spreadsheets supporting
cost allocations, computations, reconciliations, and
disclosures.
Other information that the auditor may use as audit evidence
includes the following:
A. Minutes of meetings
B. Confirmations received from third parties
C. Analyst's reports
D. Comparable data about competitors (benchmarking)
E. Controls manuals
F. Information obtained by the auditor from such audit
procedures as inquiry, observation, and inspection.
G. Other information developed by, or available to, the
auditor to reach conclusions through valid reasoning.
3. Audit evidence comprises
A. Information that supports and corroborates management
assertions.
B. Any information that contradicts managements assertions.
A. I only
B. II only
C. Neither I nor II
D. Both I and II
PSA 500 states that in some cases, the absence of information
such as managements refusal to provide a requested
representation also constitutes audit evidence that is used by
the auditor.
4. As defined in PSA 500, __________ is an individual or
organization possessing the expertise in a field other than
accounting or auditing, whose work in that field is used by
the entity in preparing the financial statements.
A. Auditors expert
B. Managements expert
C. Auditors internal expert
D. Auditors external expert

5. If a managements experts work is used to prepare the


information to be used as audit evidence, the auditor shall
I. Evaluate the competence, capabilities, and objectivity of
the managements expert.
II. Obtain an understanding of the work of the managements
expert.
III. Evaluate the appropriateness of the managements experts
work as audit evidence for the relevant assertion.

A. I and II only
B. I and III only
C. II and III only
D. I, II, and III only

6. Which of the following statements concerning the managements


experts competence, capabilities, and objectivity is
correct?
A. Objectivity relates to the ability of the managements
expert to exercise the competence in the circumstances.
B. Competence relates to the possible effects that bias,
conflict of interest or the influence of others may have
on the professional or business judgement of the
managements expert.
C. Capability relates to the nature and level of expertise of
the managements expert.
D. The managements experts competence, capabilities, and
objectivity are important factors in relation to the
reliability of any information prepared by the
managements expert.
The standard states that the competence, capabilities, and
the objectivity of a managements expert, and any controls
within the entity over the experts work, are important
factors in relation to the reliability of any information
produced by a managements expert.

Competence relates to the nature and level of expertise of


the managements expert.

Capability relates to the ability of the managements expert


to exercise the competence in the circumstances.

Objectivity relates to the possible effect that bias,


conflict of interest or the influence of others may have on
the professional or business judgement of the managements
expert.

7. Audit evidence is information used to draw reasonable


conclusions on which to base the auditors opinion. Audit
evidence is obtained by performing
I. Risk assessment procedure
II. Further audit procedures

A. I only
B. II only
C. Either I or II
D. Both I and II

Risk assessment procedures includes:


1. Inquiries of management, and of others within the entity who
in the authors judgement may have information that is
likely to assist in identifying risk of material
misstatement due to fraud or error.
2. Analytical procedures.
3. Observation and inspection.

Further audit procedures compromise:


1. Test of controls when required by the PSAs or when the
auditor has chosen to do so; and
2. Substantive procedures which include tests of details and
substantive analytical procedures.

8. Which of the following statements concerning audit evidence


is correct?
A. Appropriateness is the measure of the quantity of audit
evidence.
B. Sufficiency is the measure of the quality of audit
evidence, that is, its relevance and reliability.
C. The quantity of audit evidence needed is affected by its
quality and the risk of misstatement.
D. The sufficiency and appropriateness of audit evidence are
not interrelated
PSA 500 states,The quantity of audit evidence needed is affected
by the risk misstatement (the higher the assessed risks, the
more audit evidence is likely to be required) and also by the
quality of such audit evidence (the higher the quantity, the
less may be required).

Answer A is incorrect because appropriateness is the measure of


the quality of audit evidence.

Answer B is incorrect because sufficiency is the measure of the


quantity of audit evidence.

Answer D is incorrect because the sufficiency and


appropriateness of audit evidence are interrelated.

9. Which of the following statements concerning audit evidence


is correct?
A. An audit usually involves the authentication of
documentation.
B. Agiven set of procedure may provide audit evidence that
is relevant to certain assertions, but not others.
C. Audit evidence obtained from an independent external
source is always reliable.
D. An entitys acconting records can be sufficient audit
evidence to support the financial statements.

A given set of audit procedures may provide audit evidence that


is relevant to certain assertions, but not others. For example,
inspection of records and documents related to the collection
of receivables after the period end may provide audit evidence
regarding both existence and valuation, although not necessarily
the appropriateness of period-end cutoffs.

Answer A is incorrect because an audit rarely involves the


authentication of documentation, nor is the auditor trained or
expected to be an expert in such authentication.

Answer C is incorrect because audit evidence obtained from an


independent source may not be reliable if the source is not
knowledgeable.

Answer D is incorrect because accounting records alone do not


provide sufficient audit evidence in which to base an audit
option.

10. Which of the following generalizations does not relate to


the reliability of audit evidence?
A. Audit evidence is more reliable when it is obtained from
independent sources outside the entity.
B. Audit evidence obtained directly by the auditor is more
reliable than audit evidence obtained indirectly or by in
C. Audit evidence that is generate internally is more reliable
when the related controls imposed by the entity are
effective.
D. An auditors opinion, to be economically useful, is formed
within a reasonable time and based on audit evidence
obtained at a reasonable cost.

Cost-benefit considers relate to the sufficiency, not the


reliability, of audit evidence.

PSA 500 gives the following generalizations about the


reliability of audit evidence:

1. Audit evidence is more reliable when it is obtained from


independent sources outside the entity.
2. Audit evidence that is generated internally is more
reliable when the related controls imposed by the entity
are effective.
3. Audit evidence obtained directly by the auditor (for
example, observation of the application of a control) is
more reliable than audit evidence obtained indirectly or
inference (for example, inquiry about the application of
a control).
4. Audit evidence is more reliable when it exis in documentary
form, whether paper, electronic, or other medium (for
example, a contemporaneously written record of a meeting
is more reliable than a subsequent oral representation of
the matters discussed).
5. Audit evidence provided by original documents is more
reliable than audit evidence provided by photocopies or
facsimiles.
11. Which of the following statements concerning audit
evidence is false?

A. The auditor uses professional judgement and exercises


professional skepticism in evaluating the quantity and
quality of audit evidence, and thus its sufficiency and
appropriateness, to support the audit opinion.
B. The auditor ordinarily finds it necessary to rely on
audit evidence that is persuasive rather than
conclusive.
C. In forming the opinion, the auditor does not examine
all the information available because conclusions
ordinarily can be reached by using sampling approaches
and other means of selecting items for testing.
D. The difficulty and expense of obtaining audit evidence
concerning an account balance is valid basis for
omitting the test

The standard states that the auditor considers the relationship


between the cost of obtaining audit evidence and the usefulness
of the information obtained. However, the matter of difficulty
or expense involved is not itself a valid basis for omitting an
audit procedure for which there is no alternative.
12. In representing that the financial statements are
presented fairly, in all material respects, in accordance
with the applicable financial reporting framework, management
implicitly or explicitly makes regarding the recognition,
measurement, presentation, and disclosure of the various
elements of financial statements and related disclosures.

A. Assertions
B. Allegations
C. Conclusions
D. Assurances

As defined in the standard, assertions are representations by


management, explicit or otherwise, that are embodied in the
financial statements.

13. The auditor is required to use assertions for classes of


transformations, account balances, and presentation and
disclosures of material misstatement and the design and
performance of further audit procedures. Assertions about
classes of transactions include occurrence, completeness,
accuracy, cutoff, and

A. Valuation and allocation


B. Rights and obligations
C. Existence
D. Classification

Assertions about classes of transactions and events for the


period under audit include:
1. Occurrence transactions and events that have been
recorded have occurred and pertain to the entity.
2. Completeness all transactions and events that should
have been recorded.
3. Accuracy amounts and other data relating to recorded
transactions and event have been recorded appropriately.
4. Cutoff transactions and events have been recorded in the
correct accounting period.
5. Classification transactions and events have been
recorded in the proper accounts.

Answer A, B, and C are incorrect because valuation and allocating


rights and obligations and existence are assertions about
account balances at the period end.

14. The following are assertions about account balances at the


period end, except

The cutoff assertion deals with whether transactions and events


have been recorded in the correct accounting period. Thus, it
is an assertion about classes of transactions and events for the
period under audit
Assertions about account balance at the period end include the
following:
1. Existence assets, liabilities, and equity interests
exist.
2. Rights and obligations the entity holds or controls the
rights to assets, and liabilities are the obligations of
the entity.
3. Completeness all assets, liability, and equity interests
that should have been recorded.
4. Valuation and allocation assets, liabilities, and equity
interest are included in the financial statements at
appropriate amounts and any resulting valuation or
allocation adjustments are properly recorded.
15. The following are assertions about presentation and
disclosure.

Existence is an assertion about account balances at the period


end. It deals with whether assets, liabilities, and equity
interest exist.

Assertions about presentation and disclosure include:


1. Occurrence and rights and obligations - disclosure
events, transactions, and other matters have occurred
and pertain to the entity.
2. Completeness all disclosures that should have been
included in the financial statements have been included.
3. Classification and understandability financial
information is appropriately presented and described,
and discloses are clearly expressed.
4. Accuracy and valuation financial and other information
are disclosed fairly and at appropriate mounts.
16. Which of the following statements concerning the auditors
use of assertions is correct?

A. The auditor may combine the assertions about


transactions and events with assertions about
account balances.
B. In every audit engagement, the auditor should use
the assertions as described in PSA 500, i.e., the
assertions should always fall into three categories:
assertions about classes of transactions and events,
account balances, and presentations and disclosure.
C. There should always be a separate assertion related
to cutoff of transactions and events.
D. The completeness assertion deals only with whether
all transactions and events that should have been
recorded.

The auditor may use the assertions as described in PSA 500 or


may express them differently provided all aspects described in
the standard have been covered.
Answer B is incorrect because the auditor mmay choose to combine
the assertions about transactions and events with the
assertions about account balances.
Answer C is incorrect because there may not be a separate
assertion related to cutoff of transactions and events
when the occurrence and completeness assertions include
appropriate consideration of recording transactions in the
correct accounting period.
Answer D is incorrect because the completeness assertions may
relate to classes of transactions and events for the period
under audit, account balances at the period end, and
presentation and disclosure.

17. Which of the following statements concerning audit


objectives is incorrect?

A. The auditor should resolve any substantial doubt about any


of managements material financial statements assertions.
B. Selection of test to meet audit objectives should depend
upon the understanding of internal control.
C. There should be a one-to-one relationship between audit
objectives and procedures.
D. Audit objectives should be developed in light of management
assertions about the financial statement elements.

Audit objectives and procedures need not have a one-to-one


relationship. Some audit procedures may satisfy more than one
specific audit objective. In some cases, a combination of audit
procedures may be needed to achieve a single audit objective.

Answer A is incorrect because the auditor should resolve all


substantial doubts about any of managements material financial
statement assertions before an opinion is rendered

Answer B is incorrect because the auditors understanding of


internal control and the assessed level of control risk influence
the nature, timing, and extent of the substantive test
procedures.

Answer D is incorrect because, in developing specific audit


objectives, the auditor considers the assertions made by
management.

18. The primary difference between an audit of the balance


sheet and an audit of the income statement is that the audit
of the income statement addresses the verification of
A. Cutoffs
B. Authorizations
C. Transactions
D. Costs

Because most income statement elements represent large volumes


of transactions, the auditors focus is on the propriety of the
accounting for transactions and events during the period.
Answer A is incorrect because cutoff procedures apply to both
statements.

Answer B is incorrect because all transactions must be


authorized.

Answer D is incorrect because the auditor is concerned with the


cost reflected in both statements.

19. Which of the following would least likely affect the


appropriateness of evidence available to an auditor?
A. The sampling method employed by the auditor to obtain a
sample of such evidence
B. The relevance of such evidence to the financial statement
assertion being verified.
C. The relationship of the preparer of such evidence to the
entity being audited.
D. The timeliness of such evidence.

Appropriateness is the measure of the quality of audit evidence,


that is, its relevance and reliability. Generally, audit
evidence obtained from independent sources outside the entity,
generated under effective controls, or obtained directly by the
auditor is presumed to be the most reliable. The relevance and
reliability of audit evidence will not be affected by the sample
selection method as long as the sample is representative of the
population.

Answer B is incorrect because appropriate audit evidence must


be relevant and reliable.

Answer C is incorrect because audit evidence obtained from


independent source outside the entity is generally more reliable
than that obtained from the client.

Answer D is incorrect because the timeliness of evidence


influences its relevance and, therefore, its appropriateness.

20. Which of the following forms of documentary evidence would


be considered the most reliable by an auditor?
A. Internally generated
B. Pre-numbered
C. Easily duplicated
D. Authorized by a responsible official

It is generally presumed that externally generated documents are


more reliable than those prepared by the client. However, the
reliability of internally generated documents is enhanced if
they are prepared under satisfactory condition of internal
control. Thus, a document that is authorized by a responsible
official can be considered credible because such authorization
increases the likelihood that the underlying transaction is
authorized and valid.
Answer A and C are incorrect because internally generated and
easily duplicated documents are readily available to employees
who commit fraud.

Answer B is incorrect although the use of pre-numbered documents


is an effective control, such documents may be accessible to
employees who commit fraud.

21. The objective of tests of details of transactions performed


as substantive tests is to
A. Attain assurance about the reliability of the accounting
system.
B. Evaluate whether managements policies and procedures
operated effectively.
C. Detect material misstatements in the financial statements.
D. Comply with Philippine Standards on Auditing (PSAs).

Substantive tests are performed primarily to detect material


misstatements at the assertion level. These include tests of
details of classes of transactions, account balances, and
disclosures and substantive analytical procedures.

Answer A and B are incorrect because test of controls are


performed to test the reliability of the accounting system and
to determine whether managements policies and procedures
operated effectively.

Answer D is incorrect because the auditor may apply a variety


of audit procedures and is not required to perform tests of
details of transactions to comply with PSAs.

22. In testing the existence assertion for an asset, an auditor


ordinarily works from the
A. Potentially unrecorded items to the financial statements.
B. Financial statements to the potentially unrecorded items.
C. Supporting evidence to the accounting records.
D. Accounting records to the supporting evidence.

Answers A, B, and C are incorrect because these procedures relate


to the completeness assertion. The completeness assertion
concerns whether all assets, liabilities, and equity interests
of the entity that should have been recorded.

23. In determining whether transactions have been recorded,


the direction of the audit testing should be from the
A. General journal entries
B. Original source documents
C. General ledger balances
D. Adjusted trial balance

The completeness assertion deals with whether all transactions


and events that should have been recorded. Thus, determining
whether transactions have been recorded is testing the
completeness assertion. In applying this test, the auditor
ordinarily begins with the original source documents and traces
them to the appropriate accounting records to determine whether
the underlying transactions were properly recorded.

Answers A, C, and D are incorrect because beginning with the


accounting records is unlikely to provide evidence about
unrecorded transactions.

24. Which of the following audit procedures consists of looking


at the process or procedure being performed by others?
A. Observation
B. Inspection of records and documents
C. Inspection of tangible assets
D. Inquiry

As stated in PSA 500, observation consist of looking at a process


or procedure being performed by others. An example is observation
of the counting of inventories by the entitys personnel.

Answer B is incorrect because inspection consist of examining


records or documents, whether internal or external, in paper
form, electronic form, or other media.

Answer C is incorrect because inspection of tangible assets


consist of physical examination of the assets.

Answer D is incorrect because inquiry consist of seeking


information of knowledgeable persons, both financial and non-
financial, throughout the entity or outside the entity.

25. Which of the following elements ultimately determines the


specific auditing procedures that are necessary in the
circumstances to afford a reasonable basis for an opinion?
A. Materiality
B. Audit risks
C. Auditor judgement
D. Reasonable assurance

The auditor is required to obtain sufficient appropriate audit


evidence to afford a reasonable basis for an opinion regarding
the financial statements. The auditor uses his/her professional
judgement in determining the nature, timing, and extent of
specific audit procedures to be applied to obtain sufficient
appropriate audit evidence.

Answer A and B are incorrect because material and audit risk are
considered in exercising professional judgement.

Answer D is incorrect because the auditor exercises professional


judgement in determining whether reasonable assurance that the
financial statements are free of material misstatement has been
obtained.
ANALYTIC PROCEDURES

26. The auditor should apply analytical procedures

I. As risk assessment procedures.


II. As substantive procedures.
III. In the overall review at the end of the audit.

A. I and II only
B. II and III only
C. I and III only
D. I, II, and III

The auditor should apply analytical procedures as risk


assessment procedures to obtain an understanding of the entity
and its environment and in the overall review at the end of the
audit. Analytical procedure may also be applied as substantive
test procedures.

27. Analytical procedures include the consideration of


comparisons of the entitys financial information with

I. Comparable information for prior periods.


II. Anticipated result of the entity.
III. Similar industry information.

A. I and II only
B. II and III
C. I and III
D. I, II, and II

Analytical procedures include the consideration of comparison


of the entitys financial information with, for example:

Comparable information of prior


Anticipated result of the entity, such as budgets or
forecast, or expectations of the auditor, such as an
estimation of depreciation.
Similar industry information, such as a comparison of the
entitys ratio of sales to accounts receivable with
industry averages or with other entities of comparable
size in the same industry.

Moreover, analytical procedures may include consideration of


relationship:

Among elements of financial information that would be


expected to conform to a predictable pattern based on the
entitys experience, such as gross profit ratios.
Between financial information and relevant non-financial
information, such as payroll cost to number of employees.
28. Which of the following should be considered by the auditor
when designing and performing analytical procedures as
substantive procedures?

I. Suitability of using substantive analytical procedures


given the assertions.
II. The reliability of the data, whether internal or external,
from which the expectation of recorded amount or ratios is
developed.
III. Whether the expectation is sufficiently precise to
identify a material misstatement at the desired level of
assurance
IV. The amount of any difference of recorded amounts from
expected values that is acceptable.

A. I, II, and III only


B. I III, and IV only
C. II, III, and IV only
D. I, II, III, and IV

29. The following statements relate to the use of analytical


procedures as substantive procedures. Which is false?
A. Substantive analytical procedures are applicable when
there is only a small volume of transactions.
B. The application of substantive analytical procedures is
based on the expectation that relationships among data
exist and continue in the absence of known conditions to
the contrary.
C. The presence of relationships among data provides evidence
as to the completeness, accuracy, and occurrence of
transactions captured in the information produced by the
entitys information system.
D. Reliance on the results of substantive analytical
procedures will depend on the auditors assessment of the
risk that the analytical procedures may identify
relationships as expected when, in fact, a material
misstatement exists.

According to PSA 520 (Analytical Procedures), substantive


analytical procedures are generally more applicable to large
volumes of transactions that tend to be predictable over time.

30. Which of the following should be considered by the auditor


in determining the suitability of substantive analytic
procedures given the assertions?
I. The assessment of the risk of material misstatement.
II. Any test of details directed toward the same assertion.

A. I only
B. II only
C. Both I and II
D. Neither I nor II
The standard states that the auditor considers the understanding
of the entity and its internal control, the materiality and
likelihood of misstatement of the items involved, and the nature
of the assertion in determining the suitability of substantive
analytical procedures. It states further that substantive
analytical procedures may also be considered appropriate when
tests of details are performed on the same assertion.

31. Reliability of data is influenced by its source and by its


nature and is dependent on the circumstances under which it
is obtained. Which of the following should the auditor
consider in determining whether data is reliable for purposes
of designing substantive analytical procedures?
I. Source of the information available.
II. Comparability of the information available.
III. Nature and relevance of the information available.
IV. Controls over the preparation of the information.

A. I, III, and IV only


B. II, III, and IV only
C. I, II, and III only
D. I, II, III, and IV
32. According to PSA 520, when analytical procedures identify
significant fluctuation or relationship that are inconsistent
with other relevant information or that deviate from
predicted amounts, the auditor should investigate and obtain
adequate explanations and appropriate corroborative audit
evidence. The auditors investigation of unusual fluctuations
and relationships ordinarily begins with inquiries of
management, followed by
I. Corroboration of managements responses.
II. Consideration of the need to apply other audit procedures
based on the results of such inquiries, if management is
unable to provide an ex planation or if the explanation is
not considered adequate.
A. I only
B. II only
C. Both I and II
D. Neither I nor II

REVENUE/RECEIPT CYCLE

33. The following are the auditors principal objectives in


the audit of revenues, except
A. To determine whether all cash owned by the entity at the
balance sheet date is included on the balance sheet.
B. To determine whether earned revenue has been recorded and
recorded revenue has been earned.
C. To determine whether revenues are reported in the income
statement at the appropriate amounts.
D. To determine whether revenues are properly classified,
described, and disclosed in the financial statements,
including notes, in conformity with an applicable
financial reporting framework.
Determining whether all cash owned by the entity at the end of
the reporting period is included on the statement of financial
position is not part of the audit of sales and other revenues.
A separate audit program is ordinarily prepared for the audit
of cash, including bank balances.

34. Auditors are often concerned with the possibility of


overstatement of sales and receivables. However, management
may also have reasons for understating these balances. Which
of the following would explain understatement of sales and
receivables?
I. To avoid paying taxes.
II. To window-dress the financial statements.
III. To meet budgets and forecasts.
A. I only
B. II only
C. I and III only
D. I, II, and III

One way of avoiding tax liability is by not recording and


reporting all sales and receivables.

Answers B, C D are incorrect because management may attempt to


overstate sales and receivables to window-dress the financial
statements or meet budgets and forecast.
35. In the audit of which of the following general ledger
accounts will tests of controls be particularly appropriate?
A. Bank charges
B. Equipment
C. Bonds payable
D. Sales

Because of the large volume of sales transactions, test of


controls are particularly appropriate to restrict substantive
testing, provided that the evidence obtained supports an
assessment of control risk at less than the maximum.

Answer A is incorrect because bank charges can be easily verified


by examining the monthly bank statement.

Answer B is incorrect because the small volume of equipment


transactions and the ease of verifying its physical existence
and computing depreciation may make evaluation of the
effectiveness of controls inefficient.

Answer C is incorrect because the infrequency of transactions


the availability of bond contracts may make it inefficient to
perform tests of controls.

36. Which of the following might be detected by an auditors


review of the entitys sales cutoffs?
A. Inflated sales for the year.
B. Lapping of year-end accounts receivable.
C. Unrecorded sales discounts.
D. Excessive goods returned for credit.

An auditors view of the clients sales cutoff involves examining


recorded sales for several days (for example, one week) before
and after the balance sheet date and comparing them with sales
invoices and shipping documents. By performing this substantive
procedure, the auditor may detect the recording of sale in a
period other that in which title passed. The completeness and
cutoff assertions are directly related to the auditors cutoff
test.

Answer B is incorrect. Lapping is the concealment of a cash


shortage resulting from delays in recording cash receipts. This
may be detected by confirming receivable balances and by
comparing bank validated deposit slips to the entries in the
accounts receivables subsidiary ledger.

Answer C is incorrect because unrecorded sales discounts may be


revealed by examining cash collections.

Answer D is incorrect because an auditors review of the clients


sales cutoff does not include examination of sales returns.

37. An auditor most likely would review a clients periodic


accounting for the numerical sequence of shipping documents
and sales invoices to support managements financial
statement assertion of
A. Existence
B. Rights and obligations
C. Completeness
D. Valuation and allocation

An auditors test of the numerical sequence of shipping documents


and sales invoices may reveal omitted items. This procedure
addresses managements completeness assertion that deals with
whether all transactions, assets, liabilities, and equity
interests that should have been recorded.

Answer A is incorrect because existence is an assertion that all


assets, liabilities, and equity interests exist.

Answer B is incorrect because the rights and obligations


assertion addresses whether the entity holds or controls the
rights to assets, and liabilities are the obligations of the
entity.
Answer d is incorrect because the valuations and allocation
assertion concerns whether assets, liabilities, and equity
interests are included in the financial statements at
appropriate amounts and any resulting valuation or allocation
adjustments are appropriately recorded.
38. If the objective of a test of details of transactions is
to detect overstatements of sales, the auditors direction of
testing should be from the
A. Cash receipts journal to the sales journal.
B. Accounting records to the source documents.
C. Source documents to the account records.
D. Sales journal to the cash receipt journal.

The existence of source documents such as sales invoices and


shipping documents indicates that the sales are valid.
Thus, the proper direction of testing should be from the recorded
sales transactions to the source documents. This procedure may
reveal overstatements of sales that are likely to result from
entries in the sales account with no supporting documentation.

Answer A and D are incorrect because the cash receipts journal


and the sales journal are not source documents.

Answer c is incorrect because tracing a sample of source


documents to the accounting records may reveal unrecorded sales,
not overstatements of sales.

39. Cutoff test designed to detect credit sales made before


the end of the year that have been recorded in the subsequent
year provide assurance about managements assertion of
A. Accuracy
B. Classification
C. Rights and obligations
D. Cutoff

The cutoff assertion addresses whether transactions and events


have been recorded in the correct accounting period. This
assertion is tested by examining recorded sales for several days
before and after the balance sheet date.

Answer A is incorrect because accuracy is an assertion that


amounts and other data relating to recorded transactions and
events have been recorded appropriately.

Answer B is incorrect because the classification assertion


addresses whether transactions and events have been recorded in
the proper accounts.

Answer c is incorrect because right and obligations assertion


concerns whether the entity controls or holds the right to
assets, and liabilities are the obligations of the entity.

40. An auditor most likely would limit substantive audit tests


of the sales transactions when control risk is assessed as
low for the occurrence assertion concerning sales
transactions and the auditor has already gathered evidence
supporting

A. Beginning and ending inventory balances.


B. Cash receipt and accounts receivables.
C. Cutoffs of sales and purchases.
D. Shipping and receiving activities.

The occurrence assertion concerns whether transactions and


events that have been recorded have occurred and pertain to the
entity.

Because cash receipt and accounts receivable are directly


related to sales (a cash sale is debited to cash and credited
to sales; a sale on account is debited to accounts receivable
and credited to sales), evidence related to these accounts
provides assurance about the occurrence of sales transactions.

Answer A is incorrect because the beginning and ending inventory


balances do not have a direct relationship with sales
transactions.

Answer C and D are incorrect because cutoffs of purchases and


receiving activities are not related to sales.

41. Tracing bills of lading to sales invoice provides evidence


that
A. Invoice sales were shipped.
B. Recorded sales were shipped.
C. Shipments to customers were recorded as sales
D. Shipments to customers were invoiced.

Tracing shipping documents (such as bills of lading) to sales


invoices provides direct proof that the goods shipped were billed
to customers.

Answers A and B are incorrect because in determining whether


invoiced and recorded sales were shipped, the proper direction
of testing is from the sales records to the shipping documents

Answer C is incorrect because to determine whether shipments


were recorded as sales, the auditor would have to trace the
shipping documents to the sales records.

42. The auditor finds a situation in which one person has the
ability to collect receivables, make deposits, issue credit
memos, and record receipt of payments. Which of the following
audit procedures would be most effective in discovering fraud
in this scenario?
A. A perform a detailed review of debits to sales discounts,
sales returns and allowances, or other debit accounts,
excluding cash posted to the cash receipt journal.
B. Take a sample of bank deposits and trace the detail in
cash bank deposit back deposit back to the entry in the
cash receipts journal.
C. Send negative confirmations to all outstanding accounts
receivable customers.
D. Send positive confirmations to a random selection of
customers.

A theft of cash collections can be concealed by debiting a non-


cash account (for example, sales discounts, sales returns and
allowances, or allowance for bad debts). Thus, the most effective
procedure is to review debits posted to these accounts.

Answer A is incorrect because bank deposits will agree with the


entries in the cash receipts journal. The theft of cash is
concealed by debiting a non-cash account when recording cash
collections

Answers C and D are incorrect because a stolen cash collection


is recorded by debiting a non-cash account and crediting accounts
receivables. Thus, customers would be unaware of the fraudulent
act of the employee because the balance per their records will
agree with the amount provided in the confirmation request.

43. Which of the following most likely would give the most
assurance concerning the valuation and allocation assertion
of accounts receivable?
A. Vouching amounts in the subsidiary ledger to details on
shipping documents.
B. Inquiring about receivables pledged under load agreements.
C. Assessing the allowance for bad debts for reasonableness.
D. Comparing receivable turnover ratios with industry
statistics for reasonableness.

Valuation and allocation is an assertion that assets,


liabilities, and equity interests are included in the financial
statements at appropriate amounts and any resulting valuation
or allocation adjustments are appropriately recorded.

Management asserts that accounts receivable are stated at net


realizable value (gross accounts receivable minus allowance for
bad debts, allowance for sales returns, allowance for sales
discounts, etc.). Hence, assessing the reasonable-valuation of
the accounts receivable.

Answer A is incorrect because vouching amounts in the subsidiary


ledger to details on shipping documents provides evidence about
the occurrence of sales transactions.

Answer B is incorrect because inquiring about receivables


pledged under loan agreements relates to presentation and
disclosure assertion.

Answer D is incorrect because comparing receivable turnover


ratios with industry statistics for reasonableness provides
evidence about the completeness assertion.

44. Which of the following is not a principal objective in


auditing accounts receivable?
A. To determine whether receivables are carried at their net
realizable value.
B. To determine whether receivables are properly classified,
described, and disclosed in the financial statements,
including notes, in accordance with the applicable
financial reporting framework.
C. To determine whether the entity has real claims in all
receivables on the balance sheet.
D. To determine whether the accounts are collected by the
balance sheet date.
Accounts receivable are claims against customer arising from the
sales of goods or services. In auditing this account, the auditor
should not anticipate collections at the balance sheet date.

45. A large university has relatively ineffective internal


control. To obtain assurance that all tuition revenue has
been recorded, the auditor should

A. Confirm a sample of tuition payments with the students.


B. Prepare a year-end bank reconciliation.
C. Compare business office revenue records with registrars
office records of students enrolled.
D. Observe tuition payment procedures on a surprise basis.

An auditor who seeks assurance that all tuition revenue has been
recorded should perform substantive procedures such as test of
details and substantive analytical procedures. For example, the
auditor may obtain analytical evidence by comparing business
office revenue records with the registrars office records of
students enrolled.

Answers A and B are incorrect because confirming tuition payment


with the students and preparing year-end bank reconciliation
will not detect unrecorded stolen cash receipts.

Answer D is incorrect because observing tuition payment


procedures is a test of controls and does not provide evidence
about the completeness of the revenue account.

46. The process of obtaining and evaluating audit evidence


through a direct communication from a third party in response
to a request for information about a particular item affecting
Assertions made by management in the financial statements is
called

A. Re-performance
B. External confirmation
C. Inquiry
D. Recalculation

According to PSA 500, confirmation is a specific type of inquiry


which involves obtaining a representation of information or of
an existing condition directly from a third party.
PSA 505 (External Confirmations) gives the following examples
of situations where external confirmations may be used:
Bank balances and other information from bankers.
Accounts receivable balances.
Inventories held by third parties at bonded warehouse for
processing or on consignment.
Property title deeds held by lawyers or financiers for
safe custody or as security.
Investments purchased from stockbrokers but not delivered
at the balance sheet date.
Loans from ledgers.
Accounts payable balances.

Answer A is incorrect because re-performance is the auditors


independent execution of procedures or controls that were
originally performed as part of the entitys internal control,
either manually or through the use of CAATs. An example is re-
performing the aging of accounts receivable.

Answer C is incorrect because inquiry consists of seeking


information of knowledgeable persons, both financial or non-
financial, throughout the entity or outside the entity.

Answer D is incorrect because recalculation consists of checking


the mathematical accuracy of documents or records. For example,
the auditor may obtain an electronic file from the entity and
check the accuracy of the summarization of the file by using
CAATs.

47. The confirmation of customers accounts receivable rarely


provides reliable evidence about the valuation assertion
because

A. Customers may not be inclined to report understatement


errors in their accounts.
B. Auditors typically select many accounts with low recorded
balances to be confirmed.
C. It is not practicable to ask the customer to confirm
detailed information relating to its ability to pay the
account.
D. Recipients usually respond only if they disagree with the
information on the request.

PSA 505 states that external confirmation does not normally


provide all the necessary audit evidence relating to the
valuation assertion since it is not practicable to ask the debtor
to confirm detailed information relating to its ability to pay
the account.

Answer A is incorrect because non-reporting by customers of


understatement errors in their account does not provide reliable
evidence about the completeness assertion.
Answer B is incorrect because auditors typically select accounts
with material balance since confirmation provides strong
evidence regarding the existence of accounts receivable as at a
certain date.

Answer D is incorrect because a positive external confirmation


request asks the respondent to reply to the auditor in all cases,
that is, whether or not the respondent agrees with the
information provided in the request.

48. Auditors may use positive and/or negative forms of


confirmation requests. An auditor most likely will use

A. The negative form for small balances.


B. The positive form, when the combined assessed level of
inherent and control risk for related assertions is
acceptably low, and the negative form when it is
unacceptably high.
C. The positive form to confirm all balances regardless of
size.
D. A combination of the two forms, with the positive form
used for trade balances and the negative form for other
balances.

According to PSA 505, a negative external confirmation request


ask the respondent to reply only in the event of disagreement
with information provided in the request.

Negative confirmation request may be used when:

The combined assessed level of inherent and control risk


is low.
A large number of small balances is involved.
A substantial number of errors is not expected.
The auditor has no reason to believe that respondents will
disregard theses requests.

Answer B is incorrect because the positive form is used when the


combined assessed level of inherent and control risk is high.

Answer C is incorrect because the negative form is used when a


large number of small balances is involved.

Answer D is incorrect because the type of confirmation request


does not depend on nature of the balances.

49. The following statements relate to the use of negative


confirmation request. Which is true?

A. Negative confirmation request are effective when detection


risk is low.
B. Unreturned negative confirmation request indicate that
alternative procedures are necessary.
C. Unreturned negative confirmation request rarely provide
significant explicit evidence.
D. Negative confirmation request are effective when
understatements of account balances are suspected.

PSA 505 states that, A negative external confirmation request


ask the respondent to reply only in the event of disagreement
with the information provided in the request. However, the
auditor remains aware that there will be no explicit audit
evidence that intended third parties have received the
confirmation request and verified that the information contained
therein is correct.

Answers A and D are incorrect because positive confirmation


request are more effective when detection risk is low and when
understatements of account balances are suspected.

Answer B is incorrect because unreturned negative confirmation


request indicate that the recipients agree with the information
provided in the request, thus alternative procedures may be
unnecessary.

50. In confirming accounts receivable, an auditor decided to


confirm customers account balances rather than individual
invoices. Which of the following most likely would be more
included with the clients confirmation letter?

A. An auditor-prepared letter explaining that nonresponse may


cause an inference that the account balance is correct
B. An auditor-prepared letter requesting the customer to
supply missing and incorrect information directly to the
auditor.
C. A client-prepared letter reminding the customer that a
nonresponse will cause a second request to be sent.
D. A client-prepared statement of account showing the details
of the customers account balance.

A client-prepared statements of account that shows the detail


of the receivable balance would make it easier for the customer
to reconcile the account balance and thus, may increase response
rates.

Answers A and B are incorrect because a confirmation of the


account balance must be requested by the client, not the auditor.

Answer C is incorrect because a confirmation request should not


contain any form of threat to customers.

51. Which of the following statements would an auditor most


likely add to the negative form of confirmations of accounts
receivable to encourage timely consideration by the
recipient?
A. This is not a request for payment; remittances should not
be sent to our auditors in the enclosed envelope
B. If you do not report any differences within 15 days, it
will be assumed that this statement is correct.
C. The following invoices have been selected for
confirmation and represent amounts that are overdue
D. Report any differences on the enclosed statement directly
to our auditors; no reply is necessary if this amount
agrees with your records.

Indicating a time limit on the negative form of confirmation


request may encourage timely consideration by the recipient.

Answer A and D are incorrect because the statements do not


encourage a timely response.

Answer C is incorrect because stating that the amount is overdue


could discourage a customer from responding promptly.

52. An auditor confirms a representative number of open


accounts as of December 31 and investigates respondents
exceptions and comments. By this procedure, the auditor is
most likely to learn of which of the following?

A. One of the cashier has been covering a personal


embezzlement by lapping.
B. The credit manager has misappropriated remittances from
customers whose accounts have been written off.
C. One of the sales clerk has not been preparing charge slips
for credit sales to family and friends.
D. One of the computer processing control clerks has been
removing all sales invoices applicable to his account from
data fil.

Lapping is the concealment of a cash shortage resulting from the


delay in recording cash receipts. For example, the theft of cash
receipt from one customer is concealed by crediting that
customers account when another customer pay his/her account.
Lapped accounts can be detected through confirmation of
receivable balances because the customer will not agree with the
balance provided in the request.

Answer B is incorrect because written off accounts are no longer


open accounts.

Answer C is incorrect because there will be no account balance


to be confirmed if a charge slip has not been prepared by the
sales clerk.

Answer D is incorrect because there will be no account balance


if the sales invoices are not processed.
53. During the process of confirming receivables as of December
31, 200A, a positive confirmation was returned indicating the
balance owed as of December 31 was paid on January 6, 200B.
The auditor will most likely

A. Reconfirm the zero balance as of January 7, 200B


B. Verify that the amount was received.
C. Determine whether a cash discount was taken by the
customer.
D. Determine whether there were any changes in the account
between January 1 and January 6, 200B.

When a response indicates that the balance was already paid, the
auditor should trace the customers remittance to verify that
the payment was actually received by the client.

Answer A is incorrect because there is no requirement to


reconfirm the balance.

Answer C is incorrect because the auditors primary focus is the


customers confirmation of the year-end balance, not whether a
cash discount was taken.

Answer D is incorrect because the auditors objective is to


confirm the year-end receivable balance, not the transactions
subsequent to year-end.

54. A company was computerized sales and cash receipts journal.


The computer programs for these journals have been properly
debugged. The auditors examination of the accounting records
revealed that the total of the accounts receivable subsidiary
accounts differs materially from the accounts receivable
control account. This discrepancy could indicate

A. Credit memoranda being improperly recorded.


B. Statements being intercepted prior to mailing.
C. Lapping of receivables.
D. Receivables not being properly aged.

Based on approved credit memoranda, sales returns and allowance


are recorded by crediting accounts receivable. The discrepancy
between the subsidiary ledger and the control account could be
due to the recording of unauthorized credit memoranda, assuming
sales and cash receipts have been properly recorded.

Answer B is incorrect because, although the interception of


customer statements might be indicative of fraud, such action
would not cause the ledger discrepancy.

Answer C is incorrect because lapping result from delays in


recording cash receipts from customers. It would conceal the
theft of cash but would not cause ledger discrepancy to arise.
Answer D is incorrect because no accounting entries are made
when aging receivables.

55. To reduce the risk associated with accepting fax responses


to request for confirmations of accounts receivable, an
auditor most likely would

A. Inspect the faxes for forgeries or alterations and consider


them to be acceptable if none are noted
B. Consider the faxes to be nonresponses and evaluate them as
unadjusted differences.
C. Verify the source and contents of the faxes in telephone
calls to the senders.
D. Examine the shipping documents that provide evidence for
the existence assertion.

PSA 505 (External Confirmations) states that the auditor should


consider validating the source of replies received in electronic
format (for example, fax or electronic mail). The auditor may
verify the source and contents of a response in a telephone call
to the purported sender. Moreover, the auditor should consider
requesting the purported sender to mail the original
confirmation directly to the auditor.

Answer A is incorrect because faces may not be signed. IN


addition, PSA 500 (Audit evidence) states that an audit rarely
involves the authentication of documentation, nor is the auditor
trained as or expected to be an expert in such authentication.

Answer B is incorrect because replies received in electronic


format are considered valid if the source and contents can be
verified.

Answer D is incorrect because the auditors purpose is to verify


the existence of receivables by direct communication with the
customers.

56. PSA 505 defines confirmation as the process of obtaining


and evaluating audit evidence through representation of
information or an existing condition directly from a third
party in response to a request for information about a
particular item affecting assertions in the financial
statements or related disclosures. Two assertions for which
confirmation of accounts receivable balances provides primary
evidence are
A. Completeness and valuation and allocation.
B. Valuation and allocation and rights and obligations.
C. Rights and obligations and existence.
D. Existence and completeness.

Confirmation of receivable balances directly with customers is


most likely to be effective for the rights and obligations and
existence assertions. It provides reliable evidence that the
receivables exist and that the client has the right to collect
such receivables from customers.

Answers A, B, and D are incorrect because confirmation does not


provide sufficient evidence about the completeness and valuation
and allocation assertions.

57. For accounts receivable, negative confirmation is less


effective than positive confirmation because

A. Some recipients may report incorrect balances that require


extensive follow-up.
B. A majority of recipients usually lack the willingness to
respond objectively.
C. The auditor cannot infer that all nonrespondents have
verified their account information.
D. Negative confirmations do not produce evidence that is
statistically quantifiable.
When negative confirmations are used, unreturned requests
indicate that the intended recipients agree with the information
provided in the requests. However, no explicit evidence is
provided that the intended recipients received their requests
and verified the information.

Answer A is incorrect because inaccurate reporting of balances


by the intended parties is always a possibility under both forms
of confirmation.

Answer B is incorrect because the assumed lack of willingness


to respond objectively would affect both forms of confirmation.

Answer D is incorrect because both forms of confirmation provide


quantifiable audit evidence.

58. Which of the following procedures would an auditor most


likely perform for year-end accounts receivable confirmations
when the auditor did not receive replies to second requests?
A. Inspect the shipping records documenting the merchandise
sold to the debtors.
B. Review the cash receipts journal for the month prior to
year-end.
C. Intensify the study of internal control concerning the
revenue cycle.
D. Increase the assessed level of detection risk for the
existence assertion.

When no response is received to a second positive confirmation


request, the auditor should perform alternative procedures such
as examination of subsequent cash receipts, examination of
shipping documents or other client documentation to provide
audit evidence for the existence assertion, and examination of
sales for several days before and after the balances sheet date
to provide audit evidence for the cutoff assertion.
Answer B is incorrect because the auditor should examine
subsequent cash receipts.

Answer C is incorrect because nonresponses do not necessarily


indicate that the clients internal control is weak.

Answer D is incorrect because the auditor assesses inherent and


control risks, not detection risk. The level of detection risk
acceptable to the auditor depends on the combined assessed level
of inherent and control risks.

59. Which of the following is the most effective procedure for


determining collectability of an account receivable?
A. Confirmation of the account
B. Review of the subsequent cash collections
C. Review of authorization of credit sales to the customer
and the previous history of collections.
D. Examination of the related sales invoice(s).

The most effective procedure of determining collectability is


to obtain evidence that it was subsequently collected.
Collectability relates to the valuation and allocation assertion
and is primarily considered in determining the adequacy of the
allowance for bad debts.

Answer A is incorrect because confirmation provides audit


evidence that the receivable exists and that the customer
acknowledges the liability. However, confirmation does not
provide complete assurance that the amount is collectible.

Answer C is incorrect because while the clients history of


collections may be a good indicator of collectability, it is not
as good as subsequent cash collections.

Answer D is incorrect because examination of the related sale


invoice(s) provides audit evidence about the receivables
validity, not collectability.

60. An auditor reconciles the total of the account receivable


subsidiary ledger to the general ledger control account as of
December 31. By this procedure, the auditor is most likely to
learn which of the following?

A. An account balance is past due and should be written off.


B. A December invoice was improperly computed.
C. A December check from a customer was posted in error to
the account of another customer with a similar name.
D. An opening balance in the subsidiary ledger account was
improperly carried forward from the previous accounting
period.

An auditor's reconciliation of the subsidiary ledger total to


the general ledger will identify erroneous balances in the
subsidiary ledger at the end of the previous period carried
forward to the current period.
Answer A is incorrect because aging receivables will most
probably detect uncollectible accounts.
Answer B is incorrect because reconciliation of the subsidiary
ledger and general ledger control account will not detect an
improper amount posted in both records if such amount is based
on the same erroneous document. The auditor should consider
recalculation of invoice amounts to detect the error.
Answer C because confirmation will most likely detect posting
of customer payment to a wrong account. This error will not
affect the subsidiary ledger and general ledger control
balances.
61. Once a CPA has determined that accounts receivable have
increased because of slow collections in a tight money
environment, the CPA is likely to

d) Review the entity's credit and collection policy.

e) Expand tests collectability.

f) Review the going concern ramifications.

g) Increase the balance in the allowance for bad debts


account.

The auditor should determine the effect of slow collections of


accounts receivable on the allowance for bad debts account. The
auditor should expand his/her test of collectability such as a
review of collections subsequent to the balance sheet date.
Answer A is incorrect because the auditor's review of the credit
and collections policy is just a part of expansion of the tests
of collectability.
Answer C is incorrect because slow collections of accounts
receivable does not necessarily raise doubts about the ability
of the client to continue as going concern.
Answer D is incorrect because the allowance balance need not be
increased because of slow collections if the accounts are
expected to be collectible as in prior periods.
62. All of the following are examples of substantive procedures
to verify the valuation of net accounts receivables, except
the

A. Comparison of the allowance for bad debts with past


records.

B. Re-computation of the allowance for bad debts.

C. Inspection of the aging schedule and credit records of


past due accounts.
D. Inspection of accounts for current versus noncurrent
status in the statement of financial position.

The inspection of accounts for current versus noncurrent status


tests management's assertion about presentation and disclosure,
not valuation.
Answers A, B, and C are incorrect because comparison for
allowance for bad debts with past records, re-computation of the
allowance for bad debts, and inspection of the aging schedule
and credit records of past due accounts are tests of management's
valuation assertion about the account's balance.
63. The most likely reason for the auditor to be concerned
about the valuation of cash is that

A. The proof of cash cannot be reconciled.

B. The client uses a checking account.

C. Both currencies and negotiable securities are on hand.

D. The client has foreign currency accounts.

The audit objective of valuation is normally of minimum concern


during the audit of cash. However, the client's foreign currency
must be converted to Philippine pesos using the current exchange
rate. Changes in the exchange rates and restriction on the
client's foreign accounts may affect the valuation of these
accounts.
Answers A and C because a proof of cash that cannot be reconciled
and currency and negotiable securities that are both on hand
affect the auditor's concern about the existence of cash, not
its valuation.
Answer B is incorrect because most entities maintain checking
accounts for cash disbursements.
64. When counting cash on hand, the auditor must exercise
control over all cash and other negotiable assets to prevent
A. Theft
B. Deposits in transit
C. Substitution
D. Irregular endorsement

There should be simultaneous verification of cash and other


negotiable assets to prevent the likelihood of converting
negotiable assets to cash to conceal a cash shortage.
Answers A and D are incorrect because simultaneous verification
does not directly prevent theft or irregular endorsement except
during the time the auditor performs the verification
procedures.
Answer B is incorrect because deposits in transit (collections
recorded in the client's book but are not yet credited by the
bank) normally arise from cash transactions.
65. The best evidence regarding year-end bank balances is
documented in the
A. Bank reconciliations
B. Interbank transfer schedule
C. Cash in bank lead schedule
D. Cut-off bank statement

A bank reconciliation statement is a schedule that explains any


differences between the bank statement balance and the cash
balance per the company's records. The adjusted cash balance is
the amount of cash reported in the financial statements. Thus,
a bank reconciliation statement provides direct evidence of the
year-end bank balances.
Answer B is incorrect because an interbank transfer schedule
shows only the transfer of funds between banks.
Answer C is incorrect because a cash in bank lead schedule shows
the components of the amount of cash included in the working
trial balance.
Answer D is incorrect because a cutoff bank statements reports
cash transactions for a short period after the balance sheet
date and is used by the auditor to verify year-end reconciling
items such as deposits in transit and outstanding checks.
66. Which of the following sets of information does an auditor
usually confirm on one form?
A. Accounts receivable and accrued interest receivable
B. Cash in bank and collateral for loans
C. Accounts payable and purchase commitments
D. Inventory on consignment and contingent liabilities.

The standard bank confirmation form is used by the auditors to


confirm the cash in bank balance and request loan information
such as balance, maturity date, interest rate, and description
of collateral.
Answers A, C, and D are incorrect because accounts receivable
and accrued interest receivable, accounts payable and purchase
commitments, and inventory on consignments and contingent
liabilities are pairs of information that are not usually
confirmed on one form.
67. An auditor ordinarily sends a standard confirmation
request to all banks with which the client has done business
during the year under audit, regardless of the year-end
balance. The purpose of this procedure is to
A. Detect kiting activities that may otherwise not be
discovered.
B. Provide the data necessary to prepare a proof of cash.
C. Request that a cutoff bank statement and related checks be
sent to the auditor.
D. Seek information about other deposit and loan amounts that
come to the attention of the institution in the process of
completing the confirmation.

The bank confirmation form requests information about cash in


bank and direct liabilities of the entity to the bank.
Answer A is incorrect because kiting can be detected by preparing
an interbank transfer schedule for a few days before and after
the balance sheet date to determine whether fund transfer between
banks have been recorded in the proper periods.
Answer B is incorrect because the cash records and the month-
end bank statement provide data for the preparation of the proof
of cash.
Answer C is incorrect because a separate letter should be
prepared by the client to request for a cutoff bank statement.
68. Which of the following items is not requested on a standard
bank account balance confirmation form?
A. The principal paid on a direct liability
B. Maturity date of a direct liability
C. Description of collateral for a direct liability.
D. The interest rate of a direct liability.

The bank confirmation form requests information about account


number and description, balance, maturity date, interest rate,
date through which interest is paid, and description of
collateral. The auditor is not concerned with the amount paid
on a direct liability.
69. Which of the following is not considered an objective of
the audit of cash?
A. Cash is stated at its realizable value.
B. Compensating cash balances are reported as other current
assets.
C. Cash is properly classified, described, and disclosed in
the financial statements, including notes, in accordance
with the applicable financial reporting framework.
D. The client has ownership rights in the reported cash.

Compensating balance are classified according to the appropriate


classification of the related borrowing, either current or non-
current.
70. The purpose of a proof of cash is to
A. Validate that the client's bank did not make an error
during the period being examined.
B. Confirm that the client has properly separated the custody
function from the recording function with respect to cash.
C. Proof that the client's year-end balance of cash is fairly
stated.
D. Determine whether any unauthorized disbursements or
unrecorded deposits were made for the given time period.
A proof of cash is a four-column schedule that reconciles bank
balances for two successive months (first and last columns) and
cash receipts and disbursements for the period (middle columns).
It can be used to detect unauthorized disbursements or unrecorded
deposits for the period.
Answer A is incorrect because detection of bank errors is not
the primary reason why a proof of cash is prepared.
Answer B is incorrect because the auditor should obtain knowledge
about internal control relevant to cash transactions to
determine whether the custody and the recording functions are
properly segregated.
Answer C is incorrect because a proof of cash reconciles cash
balances and transactions for two successive months of only one
bank account. Other substantive procedures must be performed to
examine other cash accounts (for example, cash on hand) to obtain
audit evidence about the fairness of the cash balance.
71. An internal auditor would be concerned about the
possibility of fraud if
A. Only one person has access to the petty cash fund.
B. Cash receipts, net of the amounts used to pay petty cash-
type expenditures, are deposited in the bank daily.
C. The monthly bank statement reconciliation is performed by
the same employee who maintains the perpetual inventory
records.
D. The accounts receivable subsidiary ledger and accounts
payable subsidiary ledger are maintained by the same
person.

From the standpoint of a good internal control, all cash receipts


should be deposited intact daily. Using cash receipts to pay
petty cash expenditures increases the possibility of making
unauthorized removal of cash before deposit. An imprest petty
cash fund should be maintained for small disbursements.
Answer A is incorrect because accountability for petty cash fund
is established if only one person, designated as the petty cash
custodian, has access to the petty cash fund.
Answers C and D are incorrect because the functions described
are not incompatible and thus, can be performed by the same
person. Incompatible functions are those that will allow a person
both to commit and conceal errors or fraud.

EXPENDITURES/ DISBURSEMENT CYCLE


72. Purchase cutoff procedures test the completeness
assertion. An entity should include goods in inventory if it
A. Has paid for it
B. Holds legal title to the goods
C. Has physical possession of the goods
D. Has sold the goods
A purchase cutoff test is a substantive procedure to obtain
audit evidence about whether purchases are recorded in the
appropriate period. An entity should include goods in the
inventory and recognize a liability in the period in which
acquired title to the goods.
Answer A is incorrect because goods purchased on account need
not be paid for by the cutoff date.
Answer C is incorrect because title to the goods can pass before
actual delivery to the buyer, for example, in transit goods
purchased under FOB shipping point term.
Answer D is incorrect because goods already sold should be
excluded from inventory.
73. In auditing accounts payable, an auditor's procedures will
most likely focus primarily on management's assertion on
A. Existence
B. Valuation and allocation
C. Completeness
D. Presentation and disclosure

The primary audit risk for liabilities, including accounts


payable, is the possible understatement of the account. Thus,
in auditing accounts payable, the auditor will most likely focus
on the completeness assertion.
Answer A is incorrect because the risk that reported liabilities
do not exist is generally lower than the risk that the balance
is understated.
Answer B is incorrect because the risk that accounts payable are
not reported at appropriate amounts is less than the risk that
the balance is understated.
Answer D is incorrect because the risk that the accounts payable
are not properly presented and disclosed in the financial
statement is not as high as the risk that they are materially
understated.
74. Which of the following primary audit test to determine if
accounts payable are valued properly?
A. Vouching accounts payable in supporting documentation.
B. An analytical procedure.
C. Verification that the accounts payable are recorded as
current liability in the balance sheet.
D. Examination of cash disbursements subsequent to year-end.

To determine if accounts payable are properly valued, the auditor


will vouch a sample of recorded accounts payable to the
supporting documentation of purchase - namely, purchase order,
receiving order, and vendor invoice.
Answer B is incorrect because analytical procedures are
performed to provide an overall review of the accounts, not to
test the valuation of specific accounts.
Answer C is incorrect because determining whether accounts
payable are classified as current liability addresses the
presentation and disclosure assertion rather than valuation.
Answer D is incorrect because examining subsequent cash
disbursements may reveal unrecorded liabilities but does not
addresses the valuation of those liabilities that are recorded
on the company's books.
75. Which of the following procedures is least likely to be
performed before the balance sheet date?
A. Search for unrecorded liabilities
B. Confirmation of accounts receivable
C. Attendance at the physical inventory count
D. Testing internal control over cash

The auditor searches for unrecorded liabilities by reviewing


cash disbursements made after year-end to determine if they
pertain to unrecorded liabilities as of the client's balance
sheet date.
Answer B is incorrect because, depending on the combined assessed
level of inherent and control risks for receivables,
confirmation may be made before, on, or after the balance sheet
date.
Answer C is incorrect because if the internal control over the
inventory is effective, attendance at the physical inventory
count usually can be made during or at the end of the period
under audit.
Answer D is incorrect because the understanding of internal
control is usually obtained prior to the balance sheet date as
part of the auditor's risk assessment procedures.
76. An auditor performs a test to determine whether all
merchandise for which the client was billed was received. The
population for this test consists of all
A. Receiving reports
B. Vendor's invoices
C. Cancelled checks
D. Merchandise received

A supplier bills the buyer through an invoice that describes the


item(s), amount due, and payment terms. To determine whether all
merchandise for which the client was billed was received, the
auditor's should trace vendor's invoices to the related
receiving reports.
Answer A is incorrect because tracing receiving reports to
vendor's invoices provides audit evidence about whether all
goods received were billed by suppliers.
Answer C is incorrect because trading canceled checks to
receiving reports assures that goods paid for - not goods billed
- were received.
Answer D is incorrect because if the direction of testing is
from goods received, merchandise received but not billed will
not be detected.
77. Which of the following is a substantive procedure than an
auditor would most likely perform to verify the existence and
valuation of recorded accounts payable?

A. Confirming accounts payable balances with known suppliers


who have zero balances.

B. Investigating the open purchase order file to ascertain


that pre-numbered purchase orders are used and accounted
for.

C. Receiving the client's mail, unopened, for a reasonable


period of time after year-end to search for unrecorded
vendor's invoices.

D. Vouching selected entries in the accounts payable


subsidiary ledger to purchase orders and receiving
reports.

Vouching a sample of entries in the accounts payable subsidiary


ledger to purchase orders and receiving reports provides audit
evidence about whether recorded liabilities exist at a given
date and reported at appropriate amounts.
Answer A is incorrect because confirming payables with known
suppliers who have zero balances will probably detect unrecorded
liabilities. Hence, this procedure relates more to management's
completeness assertion.
Answer B is incorrect because investigating the open purchase
order file to ascertain that pre-numbered purchase order files
are used and accounted for is most directly related to
management's completeness assertion.
Answer C is incorrect because the auditor's search for unrecorded
vendor's invoices is most directly related to management's
completeness assertion.
78. When using confirmation to provide evidence about the
completeness assertion for accounts payable, the appropriate
population most likely is

A. Amounts recorded in the accounts payable subsidiary


ledger.

B. Vendors with whom the entity has previously done business.

C. Invoice filed in the entity's open invoice file.

D. Payees' checks drawn in the month subsequent to the balance


sheet date.
The completeness assertion for the account payable addresses
whether all amounts due to vendors that should have been recorded
have been recorded. If confirmation has to be used to verify
this assertion, the population must be all vendors with whom the
entity has previously done business. This means that
confirmation requests will be sent not only to vendors with
large year-end balances, but also to those with small or zero
balances as long as the entity has made purchases of their goods
during the year under audit.
Answers A and C are incorrect because, in testing the
completeness assertion, the auditor is concerned with accounts
payable that have not recognized or invoices that have not been
filed.
Answer D is incorrect because the population for the confirmation
process should consist of vendor's accounts regardless of the
balance -large, small, or zero- at the confirmation date.
79. Unrecorded liabilities are most likely to be found during
the review of which of the following documents?

A. Bills of lading

B. Unpaid bills

C. Unmatched sales invoices

D. Shipping records

The auditor's review of unpaid bills may reveal payables that


should have been recorded or accrued but have not been recognized
as liabilities as of the balance sheet date.
Answers A, C, and D are incorrect because these records and
documents relate to the entities sales, not liabilities.
80. Which of the following audit procedures least likely to
detect an unrecorded liability?

A. Readings the minutes of the meetings of the board of


directors.

B. Analysis and re-computation of interest expense.

C. Analysis and re-computation of depreciation expense.

D. Mailing of bank confirmation forms.

Analysis and re-computation of depreciation expense will provide


audit evidence about the management's valuation and allocation
assertion concerning the entity's property, plant, and
equipment. Thus, this procedure do not relate to the auditor's
objective of detecting unrecorded liabilities.
Answer A is incorrect because the entity's board of directors
usually authorizes transactions that will result to incurrence
of large amount of liabilities.
Answer B is incorrect because the auditor's computation of the
current year's interest expense based on the client's recorded
liabilities may reveal that it is unreasonably lower than the
actual interest expense paid and accrued. The auditor's
investigation of the significant differences may lead to the
detection of unrecorded liabilities.
Answer D is incorrect because the bank confirmation form requests
information about the client's direct liabilities to the bank
which may have not been recognized on the company's books.
81. When title to merchandise in transit has passed to the
audit client, the auditor engaged in the performance of a
purchase cutoff will encounter the greatest difficulty in
gaining assurance with respect to the

A. Quality

B. Quantity

C. Price

D. Terms

The quality of the merchandise can be determined through physical


inspection of the goods already received. Thus, the auditor will
encounter the greatest difficulty in gaining assurance about
quality with respect to goods included in inventory but have not
yet arrived.
Answers B, C, and D are incorrect because supporting purchase
documentation will provide information to the auditor about the
quantity, price, and terms of the purchase.
82. Which of the following audit procedures is best for
identifying unrecorded trade accounts payable?

A. Reconciling vendor's statements to the file of receiving


reports to identify items received just prior to the
balance sheet date.

B. Examining unusual relationships between monthly accounts


payable balances and recorded cash payments.

C. Investigating payables recorded just prior to and just


subsequent to the balance sheet date to determine whether
they are supported by receiving reports.

D. Reviewing cash disbursements recorded subsequent to the


balance sheet date to determine whether the related
payables applied to prior period.

The auditor's search for unrecorded includes the following audit


procedures.
Examining cash disbursements made subsequent to the
balance sheet date and comparing them with recorded account
payables as at year-end.

Sending confirmation to vendors, including those having


zero balances.

Reconciling balances with vendor's documentation.

Answer A is incorrect because reconciling vendor's statements


to the file of receiving reports to identify items received just
prior to the balance sheet date does not provide audit evidence
about whether the items have been recorded.
Answer B is incorrect because examining unusual relationships
between monthly accounts payable balances and recorded cash
payments tests only the recorded payables.
Answer C is incorrect because investigating recorded payables
to determine if they are supported by receiving reports tests
only the recorded payables and does not assure the auditor that
all the goods received have been recorded.
83. In a payables application, checks are authorized and paid
based on matching purchase orders, receiving reports, and
vendor invoices. Partial payments are common. An appropriate
audit procedure for verifying that a purchase order has not
been paid twice is to sort the
A. Check register file by purchase order, compute total
amounts paid by purchase order, compare total amounts paid
with purchase order amounts, and investigate any
discrepancies between total amounts paid and purchase
order amounts.
B. Receiving reports file by vendor invoice amounts and
investigate any discrepancies between the total amounts
received and vendor invoice amounts.
C. Vendor invoice file by purchase order, compute total
amounts paid by purchase order, compare total amounts
invoiced with purchase order amounts, and investigate any
discrepancies between the total amounts invoiced with
purchase order amounts.
D. Receiving report filed by purchase order, compute total
amounts received by purchase order, compare total amounts
received with purchase order amounts, and investigate any
discrepancies between the total amounts received and
purchase order amounts.

Sorting the check register file by purchase order, computing the


total amounts paid by purchase order comparing the total amounts
paid with purchase order amounts, and investigating
discrepancies between the total amounts paid and purchase order
amounts will provide reasonable assurance to the auditor that a
purchase order has not been paid twice.
Answers B, C, and D are incorrect because the audit procedures
described do not include examination of the check register file.
Thus, they do not provide assurance that no overpayments have
been made by the entity.
84. Which of the following procedures relating to the
examination of accounts payable could the auditor delegate
entirely to the clients employee?
A. Mall confirmations for selected account balances.
B. Prepare a schedule of accounts payable.
C. Test footings in the accounts payable ledger.
D. Reconcile unpaid invoices to vendors statements.

The preparation of schedule of accounts payable merely involves


listing of vendors accounts and amounts taken from the accounts
payable subsidiary ledger. This procedure could be delegated
entirely to the clients employees. However, the auditor should
test and review this client-prepared schedule.
Answers A, C, and D, are incorrect because the procedures
described should be performed by the auditor.
85. In an audit of a purchasing department, which of the
following usually is considered a risk factor?
A. Purchase specifications are developed by the department
requesting the material.
B. Purchases are not rotated among suppliers included on an
approved vendor list.
C. Purchases are made from parties related to buyers or other
company officials.
D. Purchases are made against blanket or open purchase orders
for certain types of items.

Purchasing from parties related to buyers or other company


officials is a risk factor because this may heighten the
possibility of fraud.

Answer A is incorrect because the department in need of the


material normally develops specifications.

Answer B is incorrect because rotation is not usually


appropriate. Moreover, maintaining an approved vendor list is a
control to ensure that purchases are made only from approved
vendors.

Answer D is incorrect because blanket or open purchase orders


are normally prepared for materials that are frequently
purchased.

86. The following statements compare confirmation of accounts


payable with vendors and confirmation of accounts receivable
with customers. Which is false?
A. As compared with the confirmation of accounts receivable,
the confirmation of accounts payable will tend to emphasize
accounts with zero balances at the balance sheet date.
B. Statistical sampling techniques are more appropriate in
the confirmation of accounts receivable than in the
confirmation of accounts payable.
C. It is less likely that the confirmation request sent to
the vendor will show the amount owed than the request sent
to the customer will show the amount due.
D. Confirmation of accounts receivable with customer is a
more widely accepted auditing procedure than is
confirmation of accounts payable with vendors.
Statistical sampling techniques are appropriate for large
populations having similar characteristics. This is true for
both confirmation of accounts receivable with customers and
confirmation of accounts payable with vendors.

Answer A is a correct statement. As compared with the


confirmation of accounts receivable, the confirmation of
accounts payable will tend to focus on accounts with zero
balances at the balance sheet date to detect possible under-
statement of payables.

Answer C is a correct statement. So that all purchases may be


confirmed including those current shipments whose bills are
still being processed by the vendor, the confirmation request
for accounts payable generally does not show the amount owed by
the client. For accounts receivable, often only payments of
customer in transit will be reconciled.

Answer D is a correct statement. Confirmation of accounts


receivable is a more widely accepted auditing procedure than is
confirmation of accounts payable. This is because externally
generated vendor documentation is often available for accounts
payable, which may lessen the need to send confirmation requests
to vendors.

87. Which of the following procedures would an auditor least


likely to perform before the balance sheet date?
A. Confirmation of accounts payable
B. Identification of related parties
C. Assessment of control risk
D. Attendance at the physical inventory count

In auditing accounts payable, the most important assertion to


verify is completeness. Thus, accounts payable balances are best
confirmed at year-end.

Answers B, C, and D are incorrect because identification of


related parties, assessment of control risk, and attendance at
the physical inventory count may be performed before the balance
sheet date.

PRODUCTION CYCLE

88. Which of the following statements concerning the auditors


attendance at the physical inventory count is incorrect?
A. A financial statement audit should always include
attendance at the physical inventory count.
B. If the auditor is unable to attend the physical inventory
count on the date planned due to unforeseen circumstances,
he/she should take or observe some physical counts on an
alternative date, and when necessary, perform audit
procedures on intervening transactions.
C. Where attendance is impracticable, due to factors such as
the nature and location of inventory, the auditor should
consider whether alternative procedures provide sufficient
appropriate audit evidence of existence and condition to
conclude that reference to a scope limitation need not be
made.
D. Inventories that are under the custody and control of third
parties (for example, inventories located in public
warehouses)may be verified by obtaining direct
confirmation from the custodians, provided that, depending
on the materiality of the amount involved, additional
procedures should be applied as deemed necessary.
PSA 501 (Audit Evidence Specific Considerations for Selected
Items) states, When inventory is material to the financial
statements, the auditor should obtain sufficient appropriate
audit evidence regarding its existence and condition by
attendance at physical inventory counting unless impracticable.

89. PSA 501 states that in planning attendance at the physical


inventory count, the auditor considers the risks of material
misstatement related to inventory as well as the nature of
the internal control related to inventory. Which of the
following would the auditor also consider?
I. Whether adequate procedures are expected to be established
and proper instructions issued for the physical inventory
count.
II. The timing of the count.
III. The locations at which inventories are held.
IV. Whether an experts assistance is to be sought.

A. I and IV only
B. II and III only
C. II, III, and IV only
D. I, II, III, and IV

90. According to PSA 501, when inventories are under the


custody and control of a third party, the auditor would
ordinarily obtain direct confirmation from the third party as
to the quantities and condition of inventories held on behalf
of the entity. Which of the following would the auditor also
consider?
I. The integrity and independence of the third party.
II. Observing, or arranging for another auditor to observe,
the physical inventory count.
III. Obtaining another auditors report on the adequacy of
the third partys internal control for ensuring that
inventories are correctly counted and adequately
safeguarded.
IV. Inspecting documentation regarding inventories held by
third parties (for example, warehouse receipts) or
obtaining confirmation from other parties when such
inventories have been pledges as collateral.

A. I, II, and IV only


B. I, III and IV only
C. II, III, and IV only
D. I, II, III and IV

91. In an audit of inventories, an auditor is least likely to


verify that
A. The client has used proper inventory pricing.
B. Damaged goods and obsolete items have been properly
accounted for.
C. The financial statement presentation of inventories is
appropriate.
D. All inventory owned by the client is on hand at the time
of the count.

Inventory owned by the client need not be on hand at the time


of the count. For example, some items purchased under FOB
shipping point term may still be in transit on the date of the
count. Also, some goods may have been shipped to customer on
consignment basis.

Answers A and B are incorrect because the auditor should verify


managements valuation and allocation assertion.

Answer C is incorrect because the auditor should verify


managements presentation and disclosure assertion.

92. An auditor selected items for test counts while observing


a clients physical inventory. The auditor then traced the
test counts to the clients inventory listing. This procedure
most likely obtained evidence concerning managements
assertion
A. Existence
B. Rights and obligations
C. Completeness
D. Valuation and allocation

Completeness is an assertion of management that all assets,


liabilities, and equity interests that should have been recorded
have been recorded. Tracing the test counts to the clients
inventory listing assures the auditor that items included in the
observed physical inventory are reflected in the inventory
records.

Answer A is incorrect because the direction of testing must be


from the clients inventory listing to the inventory tags to
obtain evidence that items included in the listing have been
counted, that is, they exist.

Answer B is incorrect because tracing the test counts to the


inventory listing does not provide evidence that the inventory
is owned by the client.

Answer D is incorrect because the valuation and allocation


assertion is verified by determining whether inventory items are
valued at the lower of cost or net realizable value.
CHAPTER 8
Audit Sampling

1. In designing audit procedures, the auditor is required to


determine appropriate means of selecting items for testing to
gather audit evidence. Which of the following means is/are
available to the auditor?
I. Selecting all items (100% examination).
II. Selecting specific items.
III. Audit sampling.

A. I and II only
B. III only
C. I and III only
D. I, II, and III

According to PSA 500 (Audit Evidence), the application of any


one or combination of the above means may be appropriate
depending on the particular circumstances, such as:
The risks of material misstatement related to the
assertion being tested; and
The practicality and efficiency of different means.

2. Which of the following should be considered by the auditor in


deciding which means (or combination of means) to use in
selecting items for testing?
I. The risk of material misstatement related to the
assertion being tested.
II. Audit efficiency.

A. I only
B. II only
C. Both I and II
D. Neither I nor II

3. It will be appropriate to audit all the items that make up a


class of transactions or account balance (100% examination),
except

A. When the class of transactions or account balance consists


of a large number of small value items.
B. When the class of transactions or account balance consists
of a small number of large value items.
C. When there is a significant risk of misstatement and other
selection methods do not provide sufficient appropriate
audit evidence.
D. When the repetitive nature of a calculation or other
process performed automatically by the clients computer
information system (CIS) makes a 100% examination cost
effective.

According to PSA 500, a 100% examination may be appropriate


when:
The population constitutes a small number of large value
items.
There is a significant risk and other means do not
provide sufficient appropriate audit evidence.
The repetitive nature of a calculation or other process
performed automatically by an information system makes
a 100% examination cost effective, for example, through
the use of computer-assisted audit techniques (CAATs).

4. PSA 500 states that the auditor may decide to select specific
items from a population based on such factors as the auditors
understanding of the entity, the assessed risk of material
misstatement, and the characteristics of the population being
tested. Specific items that may be selected for testing
usually include the following, except
A. Items that are of high value.
B. Items that are suspicious, unusual, risk-prone, or have a
history of error.
C. All items whose values do not exceed a certain amount so
as to verify only a small proportion of the total amount
of class of transactions or account balance.
D. Items that provide information about matters such as the
nature of the entity, the nature of transactions, and the
internal control.

According to PSA 500, specific items may include:

High value or key items items that are of high value


or exhibit some other characteristics like those that
are unusual, suspicious, risk-prone, etc.
All items over a certain amount items whose values
exceed a certain amount so as to verify a large
proportion of the total amount of a class of
transactions or an account balance.
Items to obtain information items that provide
specific information like the nature of the entity, the
nature of transactions, and internal control.

5. Audit sampling involves the


A. Selection of all items mover a certain amount.
B. Application of audit procedures to less than 100% of items
within a class of transactions or an account balance such
that all items have a chance of selection.
C. Application of audit procedures to all items over a certain
amount and those that are unusual or have a history of
error.

As defined in PSA 530 (Audit Sampling), audit sampling


involves the application of audit procedures to less than
100% or items within a population of audit relevance such
that all sampling units have a chance of selection in order
to provide the auditor with a reasonable basis on which to
draw conclusions about the entire population.
Answers A and D are incorrect because they involve selection
of specific items for testing to gather audit evidence.
Answer C is incorrect because it involves 100% examination.
6. Population, as defined in PSA 530 (Audit Sampling), means the
entire set of data from which a sample is selected and about
which the auditor wishes to draw conclusions. It is important
for the auditor to ensure that the population is

I. Appropriate to objective of the audit procedure.


II. Complete.

A. I only
B. II only
C. Both I and II
D. Neither I nor II

The auditor should ensure that the population is appropriate


to the objective of the audit procedure, which includes
consideration of the direction of the test to be applied. For
example, the appropriate population to test for overstatement
of accounts payable will be accounts payable listing.
However, of the auditors objective is to test for
understatement of accounts payable, the appropriate
population is not the accounts payable listing but the
subsequent disbursements, unpaid invoices, vendors
statements, or other audit evidence that will satisfy the
objective of the test.
It is also important for the auditor to ensure that the
population is complete. For example, if the sample is to be
drawn from the vouchers file, the auditor should be satisfied
that all vouchers have, in fact, been filed.
7. The two general approaches to audit sampling are:
A. Stratification and value weighted.
B. Random and nonrandom.
C. Statistical and nonstatistical.
D. Precision and reliability.

The two general approaches to audit sampling are statistical


and nonstatistical.
A statistical sampling plan should have the following
characteristics:
a) Random selection of a sample; and
b) Use of probability theory to evaluate sample results,
including measurement of sampling risk.

A nonstatistical sampling plan is any sampling plan which,


according to the standard, does not have the characteristics
of statistical sampling.
Answers A, B, and D are incorrect because stratification and
value weighted; random and nonrandom; and precision and
reliability are not general approaches to audit sampling.
Stratification is the process of dividing a population into
discrete subpopulations (also called strata), each of which
is a group of sampling units which have similar
characteristics (often monetary value).
Value weighted is a selection method in which the sampling
unit is identified as the monetary units that make up a
transaction class or an account balance.
Random selection gives each sampling unit a chance of being
selected. Conversely, nonrandom selection does not give each
sampling unit a chance of being included in the sample.
Precision is the allowance for sampling risk. Reliability
(also called confidence level) is the degree to which the
sample selected is expected to be representative of the
population. It is the mathematical complement of sampling
risk.
8. The principal methods of selecting samples are the use of
I. Random number tables or CAATs.
II. Systematic selection.
III. Haphazard selection.

A. I and II only
B. II and III only
C. I and III only
D. I, II, and III

According to PSA 530, the principal methods of selecting


samples are as follows:
a) Use of computerized random number generator (through
CAATs) or random number tables.
b) Systematic selection, in which every nth item from a
population of sequentially ordered items is selected.
c) Haphazard selection, in which the sample is selected
without following a structured or organized approach,
but also without conscious bias. This selection method
in inappropriate for statistical sampling but may be
useful for nonstatistical sampling plans.

9. An advantage of statistical sampling over nonstatistical


sampling is that statistical sampling helps an auditor to
A. Minimize the failure to detect errors and fraud.
B. Measure the sufficiency of the evidential matter obtained.
C. Eliminate the risk of nonsampling errors.
D. Reduce the level of audit risk and materiality to a
relatively low amount.

Statistical sampling involves application of the laws of


probability that enables the auditor to design an efficient
sample (i.e., a sample that is neither too large nor too
small), to measure the sufficiency of the audit evidence
obtained, and to evaluate the sample results.
Answer A is incorrect because, in some circumstances, a
nonstatistical sampling plan, may be more appropriate to
minimize the failure to detect errors and fraud.
Answer C is incorrect because nonsampling errors do not relate
to audit sampling. These are human errors like the auditors
use of inappropriate procedures or failure to recognize an
error because of misinterpretation of audit evidence
obtained. In addition, nonsampling errors arise because of
the fact that most audit evidence is persuasive rather than
conclusive.
Answer D is incorrect because both statistical and
nonstatistical sampling may be used to reduce audit risk.
Moreover, statistical sampling is irrelevant to materiality.
10. The likelihood of assessing control risk too high is the risk
that the sample selected to test controls
A. Does not support the tolerable error for some or all of
managements assertions.
B. Contains proportionately fewer deviations form prescribed
internal controls than exist in the balance or class as a
whole.
C. Does not support the auditors planned assessed level of
control risk when the true operating effectiveness of
internal control justifies such an assessment.
D. Contains misstatements that could be material to the
financial statements when aggregated with misstatements in
other account balances or transactions classes.

As defined in the standard, sampling risk arises from the


possibility that the auditors conclusion based on a sample
may be different from the conclusion reached if the entire
population were subjected to same audit procedure. It arises
from the fact that a sample may not be representative of the
population from which it was drawn.

In performing tests of controls, the two aspects of sampling


risk are:
1) The risk of assessing control risk too high is the risk
that the assessed level of control risk based on the
sample is greater than the true operating effectiveness
of the control.
2) The risk of assessing control risk too low is the
opposite of assessing control risk too high. It is the
risk that the auditor may be believe that a control is
operating effectively when it is not.

Answers A and D are incorrect because they relate more to


substantive testing. Answer B is incorrect because it relates
to assessing control risk too low.
11. While performing a test of details during an audit, the
auditor determined that the sample results supported that the
conclusion that the recorded account balance was materially
misstated. It was, in fact, not materially misstated. This
situation illustrates the risk of
A. Assessing control risk too low
B. Assessing control risk too high
C. Incorrect acceptance
D. Incorrect rejection

The two aspects of sampling risk in substantive testing are:


1) The risk of incorrect rejection is the risk that a
sample supports the conclusion that the account balance
is materially misstated when, unknown to the auditor,
the account balance is not materially misstated (i.e.,
it is fairly stated).
2) The risk of incorrect acceptance is the risk that a
sample supports the conclusion that the account balance
is not materially misstated (i.e., it is fairly stated)
when, unknown to the auditor, the account balance is
materially misstated.

Answers A and B are incorrect because assessing control too


high or too low related to test of controls.
Answer C is incorrect because the risk of incorrect acceptance
is the risk that the auditor will conclude based on a sample
that the account balance is not materially misstated when, in
fact, it is materially misstated.
12. The risk of incorrect acceptance and the likelihood of
assessing control risk too low relate to the
A. Effectiveness of the audit.
B. Efficiency of the audit.
C. Tolerable misstatement.
D. Preliminary estimates of materiality levels.

If an auditor erroneously accepts an account balance a fairly


stated, it is unlikely that additional audit procedures will
be performed. Thus, the probability that the erroneous
conclusion will be discovered is minimal, thereby decreasing
the effectiveness of the audit.
An auditors rejection of a fairly stated account balance is
most likely to result in performing extended substantive
testing that will ultimately lead to the acceptance of the
balance. Hence, the risk of incorrect rejection affects the
efficiency of the audit.
Assessing control risk too low leads to an unjustified
reduction in substantive testing which, in turn, results in
obtaining insufficient audit evidence, thereby decreasing the
effectiveness of the audit. Conversely, assessing control
risk too high results in an unjustified increase in
substantive testing and thus affecting the efficiency of the
audit.
Answer B is incorrect because the risk of incorrect acceptance
and the likelihood of assessing control risk too low both
relate to audit effectiveness, not audit efficiency. As
indicated above, audit efficiency is affected by the risk of
incorrect rejection and the likelihood of assessing control
risk too high.
Answers C and D are incorrect because the tolerable error and
preliminary estimates of materiality levels are considered
when the auditor plans a sample for substantive testing.
Moreover, the likelihood of assessing control risk too low is
an aspect of sampling risk in performing tests of controls.
13. Which of the following statements is true?
A. Statistical sampling is more convenient to use than
nonstatistical sampling.
B. Statistical sampling aids the auditor in evaluating
results.
C. Statistical sampling requires the auditor to make fewer
judgmental decisions.
D. Statistical sampling will be looked upon by the courts as
providing superior audit evidence.

14. The following are examples of nonsampling risk, except


A. Failure to recognize an error.
B. Obtaining an unrepresentative sample.
C. Use of an audit procedure inappropriate to achieve a given
objective.
D. Failing to evaluate results properly.

15. The following are advantages of using statistical sampling,


except
A. Statistical sampling provides a means for mathematically
measuring the degree of risk that results from examining
only part of a population.
B. Statistical sampling allows the auditor to greatly reduce
substantive testing.
C. Statistical sampling allows the auditor to measure the
sufficiency of the evidential matter obtained.
D. Statistical sampling aids in the design of an efficient
sample.

16. Which of the following methods is most appropriate when


performing tests of controls?
A. Stratified random sampling
B. Unrestricted random sampling with replacement
C. Variable sampling
D. Attribute sampling

17. In the audit of inventory, attribute sampling may be applied


to estimate the
A. Average price of inventory items
B. Physical quantity of inventory items
C. Percentage of slow-moving inventory items
D. Peso value of inventory

18. The risk that the assessed level of control risk based on
the sample is less than the true operating effectiveness of
the control policy or procedure is the risk of
A. Assessing control risk too low.
B. Assessing control risk too high.
C. Incorrect acceptance.
D. Incorrect rejection.

19. An element of sampling risk is


A. Choosing a sample size that is too small to achieve the
sampling objective.
B. Choosing an audit procedure that is inconsistent with the
audit objective.
C. Failing to perform audit procedures that are required by
the sampling plan.
D. Failing to detect a deviation on a document inspected by
the auditor.

20. Statistical sampling provides a technique for


A. Exactly defining materiality.
B. Greatly reducing the extent of substantive testing.
C. Eliminating judgment in testing.
D. Measuring the sufficiency of evidential matter.

21. The distinguishing feature of statistical sampling is that


it
A. Reduces the problems associated with the auditors
judgment concerning materiality.
B. Requires the examination of a smaller number of supporting
documents.
C. Is evaluated in terms of two parameters: statistical means
and random selection.
D. Provides a means for measuring mathematically the degree
of uncertainty that results from examining only part of a
population.

22. Which of the following is true if certain forms are not


consecutively numbered?
A. Systematic sampling may be appropriate.
B. Selection of a random sample probably is not possible.
C. Random number tables cannot be used.
D. Stratified sampling should be used.

23. In examining cash disbursements, an auditor plans to choose


a sample using systematic selection with a random start. The
primary advantage of this selection technique is that
population items
A. May occur more than once in the sample.
B. May occur in a systematic pattern, thus making the sample
more representative.
C. That are indicative of fraud will be included in the
sample.
D. Do not have to be prenumbered in order for the auditor to
use the technique.

24. In attribute sampling, a 5% change in which of the following


factors normally will have the least effect on the size of a
statistical sample?
A. Expected deviation rate.
B. Risk of assessing control risk too low.
C. Population size.
D. Tolerable deviation rate.

25. Given random selection, the same sample size, and the same
tolerable deviation rate for the testing of two unequal
populations, the risk of assessing control risk too low for
the larger population is
A. Higher than the risk of assessing control risk too low for
the smaller population.
B. Lower than the risk of assessing control risk too low for
the smaller population.
C. The same as the risk of assessing control risk too low for
the smaller population.
D. Indeterminable relative to the risk of assessing control
risk too low for the smaller population.

26. If the size of the sample to be used in a particular test of


attributes has not been determined by utilizing statistical
concepts, but the sample has been randomly chosen
A. The auditor will have to evaluate the results by reference
to the principles of discovery sampling.
B. May not use statistical evaluation.
C. The auditor has committed a nonsampling error.
D. No inferences can be drawn from the sample.

27. In determining the number of items to be selected in a sample


for a particular substantive test of details, the auditor
should consider all of the following, except
A. Tolerable misstatement
B. Characteristics of the population
C. Deviation rate
D. Allowable risk of incorrect acceptance

28. Which of the following statements best describes the concept


of sampling risk?
A. A randomly chosen sample may not be representative of the
population as a whole on the characteristic of interest.
B. The documents related to the chosen sample may not be
available for inspection.
C. An auditor may fail to recognize errors in the documents
examined for the chosen sample/
D. An auditor may select audit procedures that are not
appropriate to achieve the specific objective.

According to the standard, sampling risk arises from the


possibility that the auditors conclusion based on a sample
may be different from the conclusion reached if the entire
population were subjected to the same audit procedures. It is
the risk that, unknown to the auditor, the sample selected is
not representative of the population.
Answers B, C, and D are incorrect because they all relate to
nonsampling risk, that is, the risk that the auditor will
reach an incorrect conclusion even if 100% of the population
were examined.
29. In assessing sampling risk, the risk of incorrect rejection
and the risk of assessing control risk too high relate to the
A. Effectiveness of the audit.
B. Efficiency of the audit.
C. Audit quality controls.
D. Selection of the sample.

Both the risk of incorrect rejection and the risk of assessing


control risk too high will typically lead to the performance
of unnecessary audit procedures. Thus, these risks relate to
the efficiency of the audit.
Answer A is incorrect because the risk of incorrect acceptance
and the risk of assessing control risk too low both relate to
the effectiveness of the audit.
Answer C is incorrect because the standards on quality control
do not specifically mention any of these risks.
Answer D is incorrect because these risks do not directly
relate to actual selection of the sample.
30. An underlying feature of random-based selection of items is
that each
A. Item must be systematically selected using replacement.
B. Item in the accounting population should have an
opportunity to be selected.
C. Stratum of the accounting population be given equal
representation in the sample.
D. Item in the accounting population be randomly ordered.

Under random-based selection, each item has known chance of


being selected. The auditor typically uses random number
tables or a computerized random number generator (through
CAATs) in applying this method.
Answer A is incorrect because systematic selection involves
selecting every nth item from the population. Moreover, random
selection may be used without replacement.
Answer C is incorrect because each subpopulation or stratum
need not be given equal representation.
Answer D is incorrect because while items are selected at
random, there is no requirement that the accounting
population be randomly ordered.
31. If certain forms are not consecutively numbered
A. Selection of a random sample probably is not possible.
B. Systematic sampling may be appropriate.
C. Random number tables cannot be used.
D. Stratified sampling should be used.

Systematic selection involves selecting every nth item from


the population. The number of items to skip is determined by
calculating the sampling interval (population size divided by
sample size). Under this method, there is no need to establish
correspondence between population items and random numbers.
Answers A and C are incorrect because the use of random
selection and random number tables is possible even if items
in the population are not consecutively numbered.
Answer D is incorrect because there is no requirement to use
stratified sampling if the forms are not consecutively
numbered.
Stratification involves dividing the population into
different homogenous groups called subpopulation or strata.
Different selection methods are then applied to each
subpopulation or stratum.
32. When performing a test of a control with respect to control
over cash receipts, an auditor may use a systematic sampling
technique, with a start at any randomly selected item. The
biggest disadvantage of this type of sampling is that the
items in the population
A. Must be systematically replaced in the population after
sampling.
B. Must be recorded in a systematic pattern before the sample
can be drawn.
C. May occur in a systematic pattern, thus destroying the
sample randomness.
D. May systematically occur more than once in the sample.

PSA 530 states that when using systematic selection, the


auditor would need to determine that sampling units within
the population are not structured in such a way that the
sampling interval corresponds with a particular pattern in
the population.
Answer A is incorrect because items need not be replaced in
the population.
Answer B is incorrect because systematic sampling is a
selection method and does not relate to the manner in which
items in the population are recorded. Moreover, the
occurrence of a systematic pattern in the population destroys
the sample randomness.
Answer D is incorrect because there is no chance that the
items will systematically occur more than once in the sample
since systematic selection involves selecting every nth item
from the population.
33. For which of the following audit tests would an auditor most
likely use attribute sampling?
A. Selecting accounts receivable for confirmation of account
balances.
B. Examining invoices in support of the valuation of property,
plant, and equipment additions.
C. Making an independent estimate of the amount of FIFO
inventory.
D. Inspecting employee time cards for proper approval by
supervisors.

Attribute sampling is designed to test the rate of deviation


from a prescribed control procedure.
Answers A, B, and C are incorrect because they relate more
directly to variables sampling which is designed to test
whether an account balance is materially misstated.
34. Which of the following sampling methods would be used to
estimate a numerical measurement of a population, such as a
peso value?
A. Variable sampling
B. Attribute sampling
C. Random-number sampling
D. Stop-or-go sampling

A variable sampling plan is designed to test whether an


account balance is materially misstated and therefore
addresses numerical measurements such as a peso value.
Answer B is incorrect because attribute sampling deals with
deviation rates, not numerical measurements.
Answer C is incorrect because random-number sampling is a
selection method and may be used with either an attribute or
a variable sampling plan.
Answer D is incorrect because stop-or-go sampling (also
called sequential sampling) is a form of attribute sampling.
In stop-or-go sampling, the sample is selected in several
steps (i.e., not only a single sample is tested). For each
step, the auditor decided whether to stop the test or to
proceed to the next step.
35. Which of the following combinations results in a decrease in
sample size in an attribute sample?
Allowable Risk
Of Assessing
Control Risk Tolerable Expected Population
Too Low Deviation Rate Deviation Rate
A. Increase Increase Increase
B. Decrease Increase Decrease
C. Increase Increase Decrease
D. Increase Decrease Increase

To determine the sample size for tests of controls, the


auditors considers the following factors:
Factors Conditions Leading to
Smaller Larger
Sample Size Sample Size
1) Planned reliance Lower reliance Higher reliance
on internal control1 on internal on internal
control control
2) Tolerable deviation Higher TDR Lower TDR
rate (TDR)2
3) Allowable risk of Higher allowable Lower allowable
assessing control risk of assessing risk assessing
risk too low control risk control risk
too low too low
4) Expected population
deviation rate
(EDR)3 Lower EDR Higher EDR
5) Number of sampling
units in the Negligible effect on sample size
population unless population is small.
1
The auditor does not perform tests of controls when no
reliance on internal controls is planned.
2 The TDR is the rate of deviation from the prescribed

control activity that the auditor is willing to accept.


3 The EDR is the rate of deviation from the prescribed

control activity that the auditor expects to find in the


population.

36. In determining the number of documents to select for a test


to obtain assurance that all sales returns have been properly
authorized, an auditor should consider the tolerable rate of
deviation from the control activity. The auditor should also
consider the

I. Likely rate of deviations.


II. Allowable risk of assessing control risk too high.
A. I only
B. II only
C. Both I and II
D. Either I or II

The auditor considers the following factors to determine the


sample size for a test of controls:
1) Expected deviation rate
2) Tolerable deviation rate
3) Allowable risk of assessing control risk too low
37. Which of the following factors is usually not considered in
determining the sample size for a test of controls?
A. Expected population deviation rate
B. Risk of assessing control risk too low
C. Tolerable deviation rate
D. Population size, when the population is large

A change in the number of sampling units in the population


has a negligible effect on the required sample size when the
population is large. Therefore, population size is often not
considered in determining the sample size unless it is small.
Answers A, B, and C are incorrect because the risk of
assessing control risk too low, the tolerable deviation rate,
and the expected deviation rate are considered in determining
the sample size for a test of controls.
38. The sample size of a test of controls varies inversely with
Tolerable Expected
Deviation Rate (TDR) Deviation Rate (EDR)
A. No No
B. Yes Yes
C. No Yes
D. Yes No

The TDR is inversely related to the sample size that is, as


the TDR increases, the sample size decreases.
The EDR has a direct effect on the sample size that is, as
the EDR increases, the sample size increases.
39. In planning a statistical sample for a test of controls, an
auditor increased the expected population deviation rate
(EDR) from the prior years rate because of the results of
the prior years tests of controls and the overall control
environment. The auditor most likely would then increase the
planned
A. Risk of assessing control risk too low.
B. Sample size.
C. Allowance for sampling risk.
D. Tolerable deviation rate.

The EDR directly affects the sample size that is, as the
EDR increases, the sample size increases. An increase in the
EDR increases the degree of assurance to be provided by the
sample and therefore increases the sample size.
Answer A is incorrect because the risk of assessing control
risk too low is determined based on the auditors judgment
and does not necessarily increase with the EDR.
Answer C is incorrect because the allowance for sampling risk
is the difference between the maximum deviation rate (also
called upper precision limit) and the sample deviation rate.
Answer D is incorrect because the tolerable deviation rate is
determined based on the auditors judgment. It is a function
of the planned assessed level of control risk and the level
of assurance the evidence is expected to provide. It does not
necessarily increase with the EDR.
40. Which of the following factors does an auditor usually need
to consider in planning a particular audit sample for a test
of controls?
A. Acceptable level of risk of assessing control risk too
low.
B. Tolerable misstatement.
C. Number of sampling units in the population.
D. Total peso amount of the items to be sampled.

To determine the sample size for a test controls, the auditor


considers the risk of assessing control risk too low, the
tolerable deviation rate, and the expected population
deviation rate.
Answer B is incorrect because the auditor considers the
tolerable misstatement in determining the sample size for
substantive tests of details.
Answer C is incorrect because the number of units in the
population has a negligible effect on the sample size unless
it is small.
Answer D is incorrect because the total peso amount of the
items to be sampled is irrelevant to the objective of a test
of controls, that is, to determine the operating
effectiveness of prescribed controls.
41. Which of the following statements is true concerning
statistical sampling in tests of controls?
A. For a given tolerable rate, a larger sample size should be
selected as the expected population deviation rate
decreases.
B. As the population size doubles, the sample size also should
double.
C. The expected population deviation rate has little or no
effect on determining sample size except for very small
populations.
D. The population size has little or no effect on determining
sample size except for very small populations.

A change in the size of the population has a negligible effect


on the required sample size when the population is large.
Answer A is incorrect because the expected population
deviation rate directly affects the sample size. A smaller
sample size should be selected as the expected population
deviation rate decreases.
Answer B is incorrect because, as previously stated, a change
in the population size has a negligible effect on the required
sample size when the population is large.
Answer C is incorrect because, as indicated in answer A, the
expected population deviation rate directly affects the
sample size.
42. Which of the following statements is correct concerning
statistical sampling in tests of controls?
A. In determining the tolerable rate, an auditor considers
detection risk and the sample size.
B. Deviations from specific control activities at a given
rate ordinarily result in misstatements at a lower rate.
C. As the population size increases, the sample size should
increase proportionately.
D. There is an inverse relationship between the expected
population deviation rate and the sample size.

Deviations from a specific control activity increase the risk


of, but do not always result in, misstatement. Hence,
deviations from a specific control activity at a given rate
ordinarily result in misstatement at a lower rate.
Answer A is incorrect because the tolerable rate is a function
of the planned assessed level of control risk and the
assurance sought from the audit evidence.
Answer C is incorrect because as the population size
increases, the sample size increases at a decreasing rate.
Thus, a large population size will have little or no effect
on the sample size.
Answer D is incorrect because the expected deviation rate is
directly related to the required sample size.
43. An auditor plans to examine a sample of 100 purchase orders
for proper approvals as prescribed by the clients internal
control. One of the purchase orders in the chosen sample of
100 cannot be found, and the auditor is unable to use
alternative procedures to test whether the purchase order was
properly approved. The auditor should
A. Choose another purchase order to replace the missing
purchase order in the sample.
B. Select a completely new set of 100 purchase orders.
C. Consider this test of controls invalid and proceed with
substantive tests because internal control is ineffective.
D. Treat the missing purchase order as a deviation for the
purpose of evaluating the sample.

The auditors failure to apply the planned audit procedures


or to perform alternative procedures to selected items
requires consideration of the reasons for the limitation.
Moreover, the auditor considers such items as deviations from
the prescribed control procedures for the purpose of
evaluating the sample.
Answer A is incorrect because choosing another purchase order
is appropriate only if the selected item has been found to be
properly voided or canceled.
Answer B is incorrect because selecting a completely new set
of 100 purchase orders is unnecessary.
Answer C is incorrect because by treating the missing purchase
order as a deviation, the sampling plan could still be
completed.
44. An attribute sampling plan may be used to test the
effectiveness of controls. The auditors evaluation of the
sampling results ordinarily leads to a conclusion concerning
A. The relation of the population deviation rate to the
tolerable rate.
B. Monetary precision exceeding a certain predetermined
amount.
C. The population value not being misstated by more than a
predetermined amount.
D. Population characteristics occurring at least once in the
population.

In an attribute sampling plan, the auditors concern is the


occurrence rate of deviations in the population. This
statistical sampling plan enables the auditor to make an
estimate of the occurrence rate and to arrive at a conclusion
concerning the relation of the population deviation to the
tolerable deviation rate.
Answers B and C are incorrect because variables sampling is
concerned with peso values.
Answer D is incorrect because determining if a population
characteristic occurs at least once in the population will
involve examining items until one is detected.
45. As a result of sampling procedures applied as tests of
controls, an auditor incorrectly assesses control risk lower
than appropriate. Which of the following is the most likely
explanation for this situation?
A. The deviation rate in the auditors sample exceeds the
tolerable rate, but the deviation rate in the population
is less than the tolerable rate.
B. The deviation rate in the auditors sample is less than
the tolerable rate, but the deviation rate in the
population exceeds the tolerable rate.
C. The deviation rates of both the auditors sample and the
population are less than the tolerable rate.
D. The deviation rates of both the auditors sample and the
population exceed the tolerable rate.

If the sample deviation rate is less than the tolerable rate,


the auditor will conclude that the control tested is
functioning effectively. However, if the true population
deviation rate exceeds the tolerable rate, the auditors
assessment of control risk would be lower than appropriate,
that is, too low.
Answer A is incorrect because the auditors assessment of
control risk is too high if the sample deviation rate exceeds
the tolerable rate, but the true population deviation rate is
less than the tolerable rate.
Answers C and D are incorrect because the auditors
conclusions is correct if the sample and population deviation
rates are both greater or less than the tolerable rate.
46. The diagram below depicts the auditors estimated maximum
deviation rate compared with the tolerable rate, and also
depicts the true population deviation rate compared with the
tolerable rate.

Auditors Estimate True State of Population


Based on Deviation Rate Deviation Rate is
Sample Results Exceeds Tolerable Rate Less Than Tolerable
Rate
Maximum Deviation I III
Rate Exceeds
Tolerable Rate

Maximum Deviation II IV
Rate is Less Than
Tolerable Rate

As a result of testing internal controls, the auditor assesses


control risk too high and thereby increases substantive
testing. This is illustrated by situation
A. I
B. II
C. III
D. IV

The auditor would assess control risk too high if, based on
sample results, the maximum deviation rate exceeds the
tolerable rate, but the true population deviation rate is
less than the tolerable deviation rate. Moreover, assessing
control risk too high concerns the efficiency, not the
effectiveness, of the audit because it typically leads to the
performance of additional audit procedures to ultimately
arrive at the correct conclusion.
Answers A and D are incorrect because the auditor will
properly assess control risk at a high level in situation I
and at a low level in situation IV.
Answer B is incorrect because the auditor will assess control
risk too low in situation II.
47. In evaluating an attribute sample, the estimated range that
is expected to contain the population characteristic is the
A. Confidence level
B. Expected deviation rate
C. Precision
D. Upper deviation limit
Precision (also called confidence level) is the range within
which the estimate of the population characteristic is
expected to fail. It is an interval around the sample
statistic that is expected to contain the true population
value.
Answer A is incorrect because confidence level refers to the
auditors measure of how reliable the sample results should
be.
Answer B is incorrect because the expected deviation rate is
the rate of deviation the auditor expects to find in the
population.
Answer D is incorrect because the upper deviation limit, as
it suggests, is the upper limit of the precision or confidence
interval.
48. Which of the following sample planning factors would
influence the sample size for a substantive test of details
for a specific account?
Expected Amount Measure of
of misstatements Tolerable Misstatements
A. Yes Yes
B. No No
C. Yes No
D. No Yes

The auditor considers the following factors in determining


the sample size for substantive tests of details:

Conditions Leading to
Smaller Sample Size Larger Sample Size
a. Reliance on Higher reliance on Lower reliance on
internal control internal control internal control

b. Reliance on other Higher reliance to Lower reliance or


substantive tests be placed on other no reliance to be
directed at the substantive tests placed on other
same assertion substantive tests

c. Measure of Larger measure of Smaller measure of


tolerable error tolerable error tolerable error

d. Expected size Smaller errors or Larger errors or


and frequency lower frequency higher frequency
of errors

e. Population value Smaller monetary Larger monetary


significance to significance to
the financial the financial
information information
f. Acceptable Higher acceptable Lower acceptable
level of risk level of risk level of risk

g. Stratification Stratification of No stratification


the population of the population
when appropriate

h. Number of Negligible effect in sample size unless


sampling units population is small.
in the population

49. An error that arises from an isolated event that has not
recurred other than on specifically identifiable occasions
and is therefore not representative of similar errors in the
population is a/an
A. Anomalous error
B. Isolated error
C. Scandalous error
D. Non-recurring error

50. Which of the following statements concerning projection of


errors found in the sample to the population is incorrect?
A. For tests of details, the auditor should project monetary
errors found in the sample to the population, and should
consider the effect of the projected error on the audit
objective and on other areas of the audit.
B. Anomalous errors found in the sample should be included in
the projection of errors to the population.
C. For tests of controls, no explicit projection of errors is
necessary since the sample deviation rate is also the
projected deviation rate for the population as a whole.
D. If a class of transactions or an account balance has been
divided into strata, the error is projected for each
stratum separately.

PSA 530 states, When an error has been established as an


anomalous error, it may be excluded when projecting sample
errors to the population.
51. In estimation sampling for variables, which of the following
must be known to estimate the appropriate sample size required
to meet the auditors needs in a given situation?
A. The estimated deviation rate in the population.
B. The qualitative aspects of misstatements.
C. The estimated population value.
D. The acceptable level of risk.

When determining the sample size for a substantive test of


details, the auditor should consider the acceptable level of
sampling risk.
Answer A is incorrect because the estimated population
deviation rate is considered in attribute sampling.
Answer B is incorrect because the qualitative aspects of
misstatements are considered in the evaluation of the sample
results.
Answer C is incorrect because estimation of the population
value is the objective of variables sampling.
52. In applying variables sampling, an auditor attempts to
A. Predict a monetary population value within range of
precision.
B. Estimate a qualitative characteristic of interest.
C. Determine various rates of occurrence for specified
attributes.
D. Discover at least one instance of a critical deviation.

Variables sampling is designed to estimate the value of a


population, for example, an account balance.
Answer B is incorrect because variables sampling involves
estimating the value of a population. Hence, the estimate is
quantitative, not qualitative.
Answer C is incorrect because the rate of occurrence of
deviations from a prescribed control activity is determined
when applying attribute sampling.
Answer D is incorrect because discovering at least one
instance of a critical deviation is the objective of discovery
sampling, which is a form of attributes sampling.
53. When planning a sample for s substantive test of details, an
auditor should consider tolerable misstatement for the
sample. This consideration should
A. Not be changed during the audit process.
B. Be related to the auditors business risk.
C. Be related to preliminary judgments about materiality
levels.
D. Not be adjusted for qualitative factors.

The tolerable misstatement is the maximum amount of


misstatement that may exist in an account balance without
causing the financial statements to be materially misstated.
The combined tolerable misstatement for an entire audit
should not exceed the auditors preliminary estimate of
materiality for the financial statements taken as a whole.
Answer A is incorrect because, as the audit progresses, the
auditor may decide to change the tolerable misstatement like
when he/she discovers the incorrect planning assumptions were
used.
Answer B is incorrect because the auditors business risk is
irrelevant to the determination of tolerable misstatement.
Answer D is incorrect because qualitative factors should be
considered when determining the tolerable misstatement for
the sample. For example, the auditor should consider the
nature and cause of misstatements and their impact on other
phases of the audit.
54. An auditor may decide to increase the risk of incorrect
rejection when
A. The cost and effort of selecting additional sample items
are low.
B. Increased reliability from the sample is desired.
C. Many differences (audit value minus recorded value) are
expected.
D. Initial sample results do not support the planned level of
control risk.

The risk of incorrect rejection is the risk that the sample


supports the conclusion that the recorded account balance is
materially misstated, when, in fact, it is fairly stated.
This risk ordinarily result in the performance of additional
audit procedures that will lead the auditor to the proper
conclusion. Thus, the risk of incorrect rejection affects the
efficiency, not the effectiveness of the audit. If the cost
and effort of selecting additional items are low, the auditor
may accept a higher risk of incorrect rejection.

Answer B is incorrect because a decrease in the risk of


incorrect rejection is required if increased reliability
(confidence level) from the sample is desired.

Answer C is incorrect because the account balance is more


likely to be misstated if many differences are expected and
therefore incorrect rejection is less likely.

Answer D is incorrect because control risk relates to tests


of controls whereas incorrect rejection relates to
substantive testing.

55. In statistical sampling methods used in substantive testing,


an auditor most likely would stratify a population into
meaningful groups of
A. Probability-proportional-to-size (PPS) sampling is used.
B. The population has highly variable recorded amounts.
C. The standard deviation of recorded amounts is relatively
small.
D. The auditors estimated tolerable misstatement is
extremely small.

Stratification involves dividing the population into


homogenous groups called strata or subpopulations, thus
reducing the effect of high variability of amounts in the
population. Because the variability of items within each
subpopulation or stratum is reduced, the auditor will be able
to select a smaller sample for each subpopulation.
Answer A is incorrect because the population is automatically
stratified under PPS sampling.
Answer C is incorrect because the auditor would select a
sample from the total population when the standard deviation
of recorded amounts is relatively small.
Answer D is incorrect because a relatively large sample size
will be required when the tolerable misstatement is extremely
small.
56. How would decreases in tolerable misstatement and assessed
level of control risk affect the sample size in a substantive
test of details?
Decrease in Decrease in
Tolerable Misstatement Assessed Level
Of Control Risk
A. Increase sample size Increase sample size
B. Increase sample size Decrease sample size
C. Decrease sample size Increase sample size
D. Decrease sample size Decrease sample size

The tolerable misstatement is inversely related to the sample


size that is, as the tolerable misstatement decreases, the
sample size increases. Also, a decrease in the assessed level
of control risk will allow the auditor to accept a higher
level of detection risk and therefore a small sample size for
substantive testing.
57. Which of the following courses of action would an auditor
most likely follow in planning a sample of cash disbursements
if the auditor is aware of several unusually large cash
disbursements?
A. Continue to draw new samples until all the unusually large
disbursements appear in the sample.
B. Increase the sample size to reduce the effect of the
unusually large disbursements.
C. Stratify the cash disbursements population so that the
unusually large disbursements are selected.
D. Set the tolerable rate of deviation at a lower level than
originally planned.

The auditor should stratify the population to ensure that the


unusually large disbursements will be tested. Sampling
procedures will then be applied to those smaller
disbursements.
Answer A is incorrect because it will be inefficient to
continue to draw new samples to assure inclusion of all
unusually large disbursements.
Answer B is incorrect because the auditor will tend to include
in his/her test those disbursements described as unusually
large.
Answer D is incorrect because the existence of unusually large
disbursements in the population does not affect the tolerable
deviation rate in an attribute sampling application.
58. A number of factors influence the sample size for a
substantive test of details of an account balance. All other
factors being equal, which of the following would lead to a
larger sample size?
A. Smaller measure of tolerable misstatement.
B. Smaller expected frequency of errors.
C. Greater reliance on analytical procedures.
D. Greater reliance on internal control.

The tolerable misstatement is inversely related to the sample


size that is, as the tolerable misstatement decreases, the
sample size increases.
Answer B is incorrect because the expected frequency of errors
directly affects the sample size that is, as the expected
frequency of errors decreases, the sample size also
decreases.
Answer C is incorrect because the auditor will select a
smaller sample size for a substantive test of details if
greater reliance is to be placed on analytical procedures
directed at the same assertion.
Answer D is incorrect because as the degree of reliance on
internal control increases, the acceptable level of detection
also increases. This means that the auditor may restrict
his/her substantive testing by selecting a smaller sample
size.
59. An auditor established a P900,000 tolerable misstatement for
an asset with an account balance of P15,000,000. The auditor
selected a sample of every 20th items from the population that
represented the asset account balance and discovered a net
overstatement of P52,500 (P55,500 overstatements minus P3,000
understatements). Under these circumstances, the auditor most
likely would conclude that
A. The asset account is fairly stated because the tolerable
misstatement exceeds the net of projected actual
overstatements and understatements.
B. The asset account is fairly stated because the total
projected misstatement is less than the tolerable
misstatement.
C. There is an unacceptably high risk that the actual
misstatements in the population exceed the tolerable
misstatement because the total projected misstatement
exceeds the tolerable misstatement.
D. There is an unacceptably high risk that the tolerable
misstatement is more than the sum of actual overstatements
and understatements.

The net overstatement of P52,500 represents 1/20 of the items


in the population. Thus, the projected misstatement will be
P1,050,000 (P52,500 x 20), which exceeds the tolerable
misstatement of P900,000. This circumstance will lead to a
conclusion that there is an unacceptably high risk that the
actual misstatements in the population exceed the tolerable
misstatement.
Answers A and B are incorrect because the calculation above
indicates that the projected misstatement exceeds the
tolerable misstatement.
Answer D is incorrect because even if the sum of actual
overstatements and understatements is to be projected to the
population, the projected misstatement will exceed the
tolerable misstatement. Moreover, if the tolerable
misstatement exceeds the projected misstatement, the auditor
will simply accept the account balance as fairly stated.
60. Which of the following sample selection methods is not
appropriate when using statistical sampling?
A. Random selection
B. Systematic selection
C. Monetary unit sampling
D. Haphazard selection

Haphazard selection method involves selection of the sample


without following a structured approach. This technique is
not appropriate when using statistical sampling.
61. Which of the following sample selection methods cannot
ordinarily be used in audit sampling?
A. Value-weighted selection
B. Random selection
C. Block selection
D. Systematic selection

Block selection involves selection of a block(s) of


contiguous items from within the population. In most
populations, items in a sequence are expected to have similar
characteristics to each other, but different characteristics
from items elsewhere in the population.
Because audit sampling involves drawing valid conclusions
about the whole population based on the sample, block
selection ordinarily cannot be applied in audit sampling.
62. In systematic selection, the number of sampling units in the
population is divided by the sample size to determine the
A. Sampling interval
B. Pattern that may exist in the population
C. Sampling risk
D. Nonsampling risk

In systematic selection, the number of sampling units in the


population is divided by the sample size to determine the
sampling interval. For example, 20 will be the sampling
interval in a population of 5,000 sampling units within which
a sample of 250 items will be selected (5,000 / 250). After
selecting a starting point within the sampling interval of
20, the auditor will then select every 20th item.
63. In audit sampling, ____________ involves dividing the
population into discrete sub-populations which have an
identifying characteristic.
A. Value-weighted selection
B. Stratification
C. Random selection
D. Block selection

The auditor may stratify a population by dividing it into


discrete sub-populations which have an identifying
characteristic.
Stratification of the population may improve audit efficiency
because each stratum will contain homogeneous items that will
allow selection of smaller sample size without increasing
sampling risk.
64. Which of the following statements relating to stratification
is incorrect?
A. When performing tests of details of transactions and
account balances, the population is often stratified by
monetary value.
B. The results of audit procedures applied to a sample of
items within a stratum can be projected to the entire
population.
C. When verifying the valuation assertion for accounts
receivable, account balances mat be stratified by age.
D. Stratification reduces the variability of items within
each stratum.

When using stratified sampling technique, misstatement is


projected for each stratum separately. The projected
misstatements are then combined to determine the possible
effect on the entire population.
65. Audit efficiency may be improved when the sampling unit is
defined as the individual monetary units that comprise the
population. This technique is called
A. Stratification
B. Random selection
C. Systematic selection
D. Value-weighted selection

Value-weighted selection identifies the sampling nit as the


individual monetary units that make up the population. Under
this method, audit effort may be directed to the larger value
items because they will have a greater chance of being
selected. This can result in smaller sample sizes and may
thus improve audit efficiency.
TRUE OR FALSE
1. Nonsampling risk is the risk that audit tests will not uncover
existing exceptions in a sample.
2. For a given tolerable deviation rate, a larger sample size
should be selected as the expected population deviation rate
decreases.

3. The tolerable deviation rate for a test of controls is


generally lower than the expected rate of deviations in the
related accounting records.

4. In statistical on nonstatistical sampling methods used in


substantive testing, an auditor most likely would stratify a
population into meaningful groups if the population contains
both very high and very low recorded amounts.

5. If sample results indicate that the control is operating


effectively, but in fact it is not, control risk will be
assessed too high.

6. To determine if a sample is truly representative of the


population, an auditor would be required to use systematic
sample selection.

7. The risk of incorrect acceptance relates to the effectiveness


of the audit.

8. One of the ways to eliminate nonsampling risk is through the


use of attributes sampling rather than variables sampling.

9. As the amount of misstatements expected in the population


approaches tolerable misstatement, the planned sample size
will increase.

10. The auditors principal objective when using a sample of test


of details of balances is whether the transactions being
audited are free of misstatements

11. The computed upper deviation rate is the sum of the sample
deviation rate and an appropriate allowance for sampling
risk.

12. When selecting a stratified sample, the sample size is


determined for each stratum and selected randomly from the
entire unstratified population.

13. Statistical sampling provides a technique for measuring the


sufficiency of evidential matter.

14. Sampling risk is the risk that audit tests will not uncover
existing exceptions in a sample.

15. Auditors who prefer statistical sampling to nonstatistical


sampling may do so because statistical sampling helps the
auditor eliminate subjectivity in the evaluation of sampling
results.
16. Sampling risk is the risk than an auditor will reach an
incorrect conclusion because a sample is not representation
of the population.

17. The primary objective of using stratified sampling in


auditing is to determine the occurrence rate for a given
characteristic in the population being studied.

18. A sample of all items of a population will eliminate sampling


risk, but increase nonsampling risk.

19. The use of inappropriate audit procedures is a significant


cause of nonsampling risk.

20. The use of an appropriate sample selection technique ensures


a representative sample.

21. The process which requires the calculation of an interval


and then selects the items based on the size of the interval
is random sample selection.

22. Correspondence is established between the random number table


and the population by deciding the number of digits to use in
the random number table and their association with the
population numbering system.

23. It is impossible to draw a six-digit random number from a


table that is separated into columns of five digits.

24. To determine the sample size for a test of controls, an


auditor should consider the tolerable deviation rate, the
desired confidence level, and the expected population
deviation rate.

25. When selecting a three-digit number from a table that is


separated into columns of five digits, it is permissible to
use the first three digits, the middle three digits, or the
last three digits.

KEY ANSWERS

1. D 10. C
6. C
2. C 11. D
7. C
3. A 12. A
8. D
4. C 13. B
9. B
5. B 14. B
32. C
15. B 50. B
33. D
16. D 51. D
34. A
17. C 52. A
35. C
18. A 53. C
36. A
19. A 54. A
37. D
20. D 55. B
38. D
21. D 56. B
39. B
22. A 57. C
40. A
23. D 58. A
24. C 41. D
59. C
25. A 42. B
60. D
26. B 43. D
61. C
27. C 44. A
62. A
28. A 45. B
63. B
29. B 46. C
47. C 64. B
30. B
48. A 65. D
31. B
49. A

TRUE OR FALSE
1. True

2. False

3. False

4. True

5. False

6. False
7. True

8. False

9. True

10. False

11. True

12. False

13. True

14. False

15. False

16. True

17. False

18. False

19. True

20. False

21. False

22. True

23. False

24. True

25. True
CHAPTER 9
COMPLETING THE AUDIT AND POST-AUDIT RESPONSIBILITIES

Analytical Procedures
1. Analytical procedures used in the overall review stage of
the audit generally include:
A. Retesting controls that appeared to be ineffective
during the assessment of control risk.
B. Considering unusual or unexpected account balances that
were not previously identified.
C. Gathering evidence concerning account balances that
have not changed from the prior year.
D. Performing tests of transactions to corroborate
managements financial statement assertions.

PSA 520 (Analytical Procedures) states, The auditor shall


design and perform analytical procedures near the end of
the audit that assist the auditor when forming an overall
conclusion as to whether the financial statements are
consistent with the auditors understanding of the
entity.

Analytical procedures used in the final review stage of


the audit are intended to corroborate the conclusions
formed during the audit of individual components or
elements of financial statements. They assist in arriving
at the overall conclusion as to the reasonableness of the
financial statements. Moreover, analytical procedures may
also identify a previously unrecognized risk of material
misstatement.

Analytical procedures applied as an overall review in the


completion stage of the audit typically include reading the
financial statements and accompanying notes and considering:
1) Unusual or unexpected account balances or
relationships that were not previously identified; and
2) The adequacy of evidence regarding previously
identified unusual or unexpected balances.
Answer A is incorrect because analytical procedures are not
tests of controls.
Answer C is incorrect because there may be nothing unusual or
unexpected concerning account balances that have not changed
from the prior year.
Answer D is incorrect because analytical procedures are not
tests of transactions.
2. Analytical procedures performed in the overall review
stage of an audit suggest that several accounts have
unexpected relationships. The result of these procedures
most likely indicate that:
A. The communication with the audit committee should be
revised.
B. Irregularities exist among the relevant account balances.
C. Additional substantive tests of details are required.
D. Internal control activities are not operating
effectively.
The auditor should perform analytical procedures in the
final review stage of the audit in order to assess the
conclusions reached and evaluate the overall financial
statement presentation.
When analytical procedures disclosed significant
fluctuations or relationships that are consistent with
other relevant information or that deviate from predicted
amounts, the auditor is required to investigate and obtain
adequate explanations and appropriate corroborative
evidence. Thus, additional tests of details are required to
be performed.
The auditors investigation of unusual fluctuations begins
with inquiries of management. In turn, the auditor will
perform the following:
a) Corroboration of managements responses
b) If management is unable to provide an explanation
or if the explanation is not considered adequate, the
auditor should consider the need to apply other audit
procedures based on the result of such inquiries.
Answer A is not correct because the auditor should first
obtain explanations for the unexpected relationships to
determine if the communication with the audit committee will
be revised.
Answer B is incorrect because it is the auditors
investigation, not the analytical procedures performed, that
may uncover the irregularities.

Related Party Transactions


3. The responsibility for the identification and disclosure
of related parties and transactions with such parties
rests with the:
A. Auditor
B. Entitys management
C. Financial Reporting Standards Council (FRSC)
D. Securities and Exchange Commission (SEC)

Management is responsible for the identification and


disclosure of related transactions with such parties.
Management is required to implement adequate internal
control to ensure that related party transactions are
appropriately identified in the information system and
disclosed in the financial statements.

4. The auditor should review information provided by those


charged with governance and management identifying
I. The names of all known parties
II. Related party transactions
A. I only
B. II only
C. Both I and II
D. Neither I nor II
The auditor should review information provided by those
charged with governance and management identifying the
names of all known parties and transactions with such
parties.

5. Which of the following events most likely indicated the


existence of related parties?
A. Making a loan without scheduled terms for payment of the
funds.
B. Discussing merger terms with a company that is a major
competitor.
C. Selling real estate at a price that differs significantly
from its book value.
D. Borrowing a large sum of money at a variable rate of
interest.
The following suggest related party transactions:

Transactions which have abnormal terms of trade, such


as unusual prices, interest rates, guarantees, and
repayment terms.
Transaction which lack an apparent local business
reason for their occurrence.
Transactions in which substance differ from form.
Transactions processed in an unusual manner.
High volume significant transactions with certain
customers or suppliers as compared with others.
Unrecorded transactions such as the receipt or
provision of management services at no charge.
Answer B is incorrect because the parties become related only
after the merger transaction has occurred.
Answer C is incorrect because usually, the real estates fair
value is significantly different from its book value.
Answer D is incorrect because large sums of money are normally
borrowed at variable rates of interest, particularly long-term
borrowings.

6. Which of the following would not necessarily be a related


party transaction?
A. A purchase from another corporation that is controlled by
the corporations chief shareholder.
B. A loan from the corporation to a major shareholder.
C. Sale of land to the corporation by the spouse of a
director.
D. A sale to another corporation with similar name.
According to PSA 24 (Related Party Disclosures), a party is
related to an entity if:
a) Directly, or indirectly through one or more
intermediaries, the party:
i. Controls, is controlled by, or is under common
control with, the entity (this includes parents,
subsidiaries, and fellow subsidiaries);
ii. Has an interest in the entity that gives it
significant influence over the entity; or
iii. Has joint control over the entity;
b) The party is an associate of the entity;
c) The party is a joint venture in which the entity is a
venture;
d) The party is a member of the key management personnel of
the entity or its parent;
e) The party is a close member of the family of any
individual referred to in (a) or (d);
f) The party is an entity that is controlled, jointly
controlled or significantly influenced by, or for which
significant voting power in such entity resides with,
directly or indirectly, any individual referred to in (d)
or (e); or
g) The party is a post-employment benefit plan for the
benefit of employees of the entity, or of any entity that
is a related party of the entity.
Two corporations having a similar name are not necessarily
related.
Answers A, B, and C are incorrect because these
transactions are considered related party transactions.
7. Which of the following procedures should be performed by
the auditor to determine the completeness of information
provided by those charged with governance and management
identifying the names of all known related parties?

I. Review prior years working papers for names of


known related parties.
II. Inquire as to the affiliation of those charged with
governance and officers with other entities.
III. Review minutes of the meetings of shareholders and
those charged with governance.
A. I and II only
B. II and III only
C. I and II only
D. I, II, and III
PSA 550(Related Parties0 requires the auditor to review
information provided by those charged with governance and
management identifying the names of known related parties.
Moreover, the auditor is required to perform the following
procedures to determine the completeness of such information:
a) Review prior years working papers for the names
of known related parties.
b) Review the entitys procedures for
identification of related parties.
c) Inquire as to the affiliation of those charged
with governance and officers with other
entities.
d) Review shareholder records to determine the
names of principal shareholders or, if
appropriate, obtain a listing of principal
shareholders from the share register.
e) Review of minutes of the meetings of
shareholders and those charged with governance
and other relevant statutory records such as
register of directors interests.
f) Inquire of other auditors currently involved in
the audit, or predecessor auditors, as to their
knowledge of additional related parties.
g) Review the entitys income tax returns and other
information supplied to regulatory agencies.

8. Which of the following statements concerning related


party transactions is correct?
A. In the absence of evidence to the contrary, related party
transactions should be assumed to be outside the ordinary
course of business.
B. The audit procedures directed toward identifying related
party transactions should include considering whether
transactions are occurring but are not being given proper
accounting recognition.
C. An auditor should determine whether a particular
transaction would have occurred if the parties had not
been related.
D. An auditor should substantiate that related party
transactions were consummated on terms equivalent to
those that prevail in arms-length transactions.
PSA 550 states that in examining the identified related
party transactions, the auditor should obtain sufficient
appropriate audit evidence as to whether these transactions
have been properly recorded and disclosed.
Answer A is incorrect because the existence of related
parties and transactions with such parties are considered
ordinary features of business.
Answer C is incorrect because, unless the transaction is
routine, it is not ordinarily possible for the auditor to
determine whether a particular transaction would have
occurred if the parties had not been related.
Answer D is incorrect because related party transactions
need not be consummated on terms equivalent to those that
prevail in arms-length transactions.

9. An auditor searching for related party transactions


should obtain an understanding of each subsidiarys
relationship to the total entity because
A. This may permit the audit of intercompany account
balances to be performed as of concurrent dates.
B. This may reveal whether particular transactions would
have taken place if the parties had not been related.
C. The business structure may be deliberately designed to
obscure related party transactions.
D. Intercompany transactions may have been consummated on
terms equivalent to arms-length transactions.
While the existence of related parties and transactions
with such parties and transactions with such parties are
considered ordinary features of business, the auditor
should be aware of them because a related party transaction
may be motivated by other than ordinary business
considerations like profit sharing or even fraud. Thus, the
auditor should consider the possibility that the business
structure may be deliberately designed to obscure related
party transactions.
Answer A is incorrect because the auditor is not required
to conduct a concurrent audit.
Answer B is incorrect because determining whether
particular transactions would have occurred if the parties
had not been related is ordinarily not possible, except for
routine transactions.
Answer D is incorrect because related party transactions
need not be consummated on terms equivalent to arms-length
transactions.

10. After determining that a related party transaction has,


in fact, occurred an auditor should
A. Obtain an understanding of the business purposes of
the transaction.
B. Substantiate that the transaction was consummated on
terms equivalent to an arms-length transaction.
C. Add a separate paragraph to the auditors report to
explain transaction.
D. Perform analytical procedures to verify whether
similar transactions occurred, but were not recorded.
After identifying related party transactions, the auditor
should obtain sufficient appropriate audit evidence to
determine whether such transactions have been properly
recorded and disclosed. The auditor should become satisfied
about their purpose, nature, extent, and effect. Therefore,
the auditor should obtain an understanding of the business
purpose of an identified related party transaction.
Answer B is incorrect because the auditors primary concern is
to determine whether a related party transaction has been
properly recoded and disclosed, not whether such transaction
was consummated on terms equivalent to an arms-length
transaction.
Answer C is incorrect because no modification of the report is
necessary if the related party transaction has been properly
recorded and disclosed. However, the auditor may add an
emphasis of matter paragraph to the audit report to emphasize
that the entity has had significant related party
transactions.
Answer D is incorrect because management is responsible to
ensure that the related party transactions are appropriately
identified in the entitys information system and disclosed in
financial statements. Thus, the auditor is not responsible for
undisclosed, unrecorded related party transactions.

11. Which of the following audit procedure is most likely to


assist an auditor in identifying related party transactions?
A. Inspecting communications with law firms for evidence of
unreported contingent liabilities.
B. Reviewing accounting records for nonrecurring transactions
recognized near the balance sheet date.
C. Retesting ineffective controls previously reported to the
audit committee.
D. Sending second requests for unanswered positive
confirmations of accounts receivable.

The following are examples of audit procedures which may


identify the existence of related party transactions:
Performing detailed tests of transaction balances.
Reviewing minutes of meetings of shareholders and those
charged with governance.
Reviewing accounting records for large or unusual transactions
or balances, paying particular attention to transactions
recognized at or near the end of reporting period.
Reviewing confirmations of loans receivable and payable and
confirmations from banks. Such a review may indicate a
guarantor relationship and other related party transactions.
Reviewing investment transactions, for example, purchase or
sale of an equity interest in a joint venture or other entity.
Answer A is incorrect because the purpose of inspecting
communications with law firms for evidences of unreported
contingent liabilities is to identify potential litigation,
claims, and assessment that may require disclosure in the
financial statements.
Answer C is incorrect because the auditor should not retest
ineffective controls previously reported to the audit
committee.
Answer D is incorrect because confirmation of accounts
receivable do not normally provide audit evidence about the
existence of related party transactions.

12. For a reporting entity that has participated in related


party transactions that are material, disclosure in the
financial statements should include:
A. A reference to deficiencies in the entitys internal
control.
B. A statement to the effect that a transaction was
consummated on terms equivalent to those that prevail
in arms-length transactions.
C. The nature of the relationship and the terms and
manner of settlement.
D. Details of the transactions within major
classifications.
PAS 24 (Related Party Disclosures) provides that if an entity
has participated in material related party transactions, it
should disclose the nature of the relationship, information
about the transaction, and outstanding balances necessary for
an understanding of the potential effect of the relationship
on financial statements.
As minimum, the disclosures should include:
o The amount of the transaction.
o The amount of outstanding balances, their terms
and conditions, whether secured or unsecured,
and the nature of the consideration to be
provided in settlement.
o Provision for doubtful accounts related to the
outstanding balances.
o The expense recognized during the period in
respect of doubtful accounts due from related
parties.
Answer A is incorrect because the company is not required to
make financial statement disclosures about its internal
control.
Answer B is incorrect because a statement to the effect that a
transaction was consummated on terms equivalent to those that
prevail in arms-length transactions is not required. This
representation shall be made only if such terms can be
substantiated.
Answer D is incorrect because there is no requirement to
segregate the transactions into major classifications.

SUBSEQUENT EVENTS REVIEW


13. As used in PSA 560 (Subsequent Events), the term
subsequent events refer to
I. Events occurring between the date of the financial
statements and the date of auditors report.
II. Facts that become known to the auditor after the
date of the auditors report.
A. I only
B. II only
C. Both I and II
D. Neither I nor II
According to PSA 560, the term subsequent events refers to
events occurring between the date of financial statements and
the date of auditors report, and facts that became known to
the auditor after the date of the auditors report.

14. Which of the following statements best describe the date


of the financial statements?
A. The date on which those with the recognized authority
assert that they have prepared the entitys complete
set of financial statements, including the related
notes, and that they have taken responsibility for
them.
B. The date that the auditors report and audited
financial statements are made available to third
parties.
C. The date of the end of the latest period covered by
the financial statements.
D. The date on which the auditor has obtained sufficient
appropriate audit evidence on which to base the
opinion on financial statements.
According to the standard, the date of the financial
statements is the date of the end of the latest period covered
by the financial statements.
Answer A is incorrect because it is the date of the approval
of the financial statements.
Answer B is incorrect because it is the date of issuance of
the financial statements.
Answer D is incorrect because it is usually the date of the
auditors report.

15. The auditor is required to perform procedures designed to


obtain sufficient appropriate audit evidence to identify
all events that may require adjustment of, or disclosure
in, the financial statements up to the
A. Date of auditors report
B. Date of approval of the financial statements
C. Date the financial statements are issued.
D. Date of the financial statements
The standard requires the auditor to perform audit procedures
designed to obtain sufficient appropriate audit evidence that
all events up to the date of auditors report that may require
adjustment of, disclosure in, the financial statements may
have been identified.

16. Which of the following procedures would an auditor most


likely perform to obtain evidence about the occurrence of
subsequent events?
A. Inquiring as to whether any unusual adjustments were
made after the date of financial statements.
B. Confirming a sample of material accounts receivable
after the date of financial statements.
C. Comparing the financial statements being reported on
with those prior period.
D. Investigating personnel charges in the accounting
department occurring after the date of the financial
statements.

The audit procedures to identify subsequent events ordinarily


include the following:

Reviewing the entitys established procedures to


identify subsequent events.
Reviewing minutes of meetings of shareholders, the
board of directors, and committees held subsequent to
the date of the financial statements and inquiring
about matters discussed at meetings for which minutes
are not yet available.
Reading the entitys latest available interim
financial statements, appropriate budgets, and
forecasts, and other related management reports.
Inquiring or extending previous oral or written
inquiries, of the entitys lawyers concerning claims
and litigation.
Inquiring of management as to whether any subsequent
events have occurred which might affect the financial
statements. Examples of such inquiries are:
o The current status of items that were accounted
for on the basis of preliminary or inconclusive
data.
o Whether new commitments, borrowings, or
guarantees have been entered into.
o Whether sales of assets have occurred or are
planned.
o Whether the issue of new shares or debentures or
other agreement to merge or liquidate has been
made or is planned.
o Whether any assets have been appropriated by the
government or destroyed, for example, by fire or
flood.
o Whether there have been any developments
regarding risk areas and contingencies.
o Whether any unusual accounting adjustments have
been made or are contemplated.
o Whether any events have occurred or are likely
to occur in which will bring into question the
appropriateness of the accounting policies
adopted by the entity, for example, events that
may call into question the validity of the going
concern assumption.
Answer B is incorrect because confirmation of receivables is
ordinarily performed before the date of the financial
statements.
Answer C is incorrect because comparison of the financial
statements being reported on with those of the prior period is
an analytical procedure that is performed in the planning
stage of an audit.
Answer D is incorrect because personnel charges occurring
after the date of the financial statements are not considered
significant subsequent events that will require adjustment of,
disclosure in, the financial statements.

17. Which of the following procedures should an auditor


ordinarily perform regarding subsequent events?
A. Review the cut-off bank statements for several months
after the year-end
B. Compare the latest available interim financial
statements with the financial statements being
audited.
C. Send second requests to the clients customers who
failed to respond to initial accounts receivable
confirmation requests.
D. Communicate material weaknesses in internal control to
the clients audit committee.
An analytical procedure that is designed to identify
subsequent events in comparison to the latest interim
financial statements with those statements being reported on.
Answer A is incorrect because the auditor reviews cutoff bank
statements (usually 7 to 10 days from the balance sheet date)
to verify year-end bank reconciliations.
Answer C is incorrect because confirmation or receivables
typically do not relate to the recording of subsequent events.
Answer D is incorrect because communicating material internal
control weaknesses to the audit committee is not a subsequent
events procedure.

18. An auditor is concerned with completing various phases of


the audit after the balance sheet date. This subsequent
period extends to the date of the
A. Delivery of the auditors report to the client
B. Auditors report
C. Final review of the audit working papers
D. Public issuance of financial statements
The auditor is required to perform procedures designed to
identify all subsequent events up to the date of the auditors
report. Hence, the subsequent events period extends from the
date of the financial statements (i.e., the balance sheet
date) to the date of the auditors report.
Answers A and D are incorrect because the delivery of the
auditors report to the client and the public issuance of the
financial statements both occur later than the date of the
auditors report.
Answer C is incorrect because the auditor is required to
perform subsequent events worked up to the date of the
auditors report.

19. Which of the following statements best expresses the


auditors responsibility with respect to facts which
become known to the auditor after the date of the
auditors report but before the date the financial
statements are issued?
A. The auditor should amend the financial statements.
B. If the acts discovered will materially affect the
financial statements, the auditor should issue a new
report which contains either a qualified opinion or an
adverse opinion.
C. The auditor should consider whether the financial
statements need amendment, discuss the matter with
management, and consider taking actions appropriate in
the circumstances.
D. The auditor should withdraw from the engagement.
PSA 560(Subsequent Events) provides that an auditor does not
have any responsibility to perform procedures or make any
inquiry regarding the financial statements after the date of
the auditors report.
It provides further that during the period from the date of
the auditors report to the date the financial statements are
issued, the entitys management has the responsibility to
inform the auditor of facts which may affect the financial
statements.
If during that period the auditor becomes aware of a fact
which may materially affect the financial statements, he/she
should consider whether the financial statements need
amendment, discuss the matter with management, and take the
action appropriate in the circumstances.
Answer A is incorrect because, if necessary, the entitys
management, not the auditor, should amend the financial
statements.
Answer B is incorrect because expressing a qualified opinion
or an adverse opinion is appropriate when management does not
amend the financial statements in circumstances where the
auditor believes they need to be amended and the auditors
report has not been released to the entity. When the auditors
report has been released to the entity, the auditor would
notify those persons ultimately responsible for the overall
direction of the entity not to issue the financial statements
and the auditors report to third parties.
Answer D is incorrect because the discovery of facts after the
date of the auditors report does not necessarily require
withdrawal from the engagement.
20. After issuing a report, an auditor has no obligation to
make continuing inquiries or perform other procedures
concerning the audited financial statements, unless
A. Final determinations or resolutions are made of
contingencies that had been disclosed in the financial
statements.
B. Information about an event that occurred after the
date of the auditors report comes to the auditors
attention.
C. The control environment changes after the issuance of
the report.
D. Information, which existed at the report date and may
affect the report, comes to the auditors attention.
PSA 560 provides that after the financial statements and the
auditors report have been issued, the auditor has no
obligation to make any inquiry concerning such financial
statements.
But, if the auditor becomes aware of a fact which existed at
the date of the auditors report which, if known at that date,
may have caused the auditor to modify the auditors report,
the auditor should consider whether the financial statements
need amendment, discuss the matter with management, and take
the action appropriate in the circumstances.
Answers A and C are incorrect because after the report is
issued, the auditor has no responsibility to consider changes
in the entitys internal control or final determinations or
resolutions of contingencies.
Answer B is incorrect because the auditor does not have
reporting responsibility for events that occurred after the
date of the report.

21. Which of the following events occurring after the


issuance of an auditors report most likely would cause
the auditor to make further inquiries about the
previously issued financial statements?
A. A technological development that could affect the
entitys future ability to continue as a going
concern.
B. The entitys sale of a subsidiary that accounts for
30% of the entitys consolidated sales
C. The discovery of information regarding a contingency
that existed before the financial statements were
issued
D. The final resolution of a lawsuit disclosed in the
notes to the financial statements.
If, after the report has been issued, the auditor becomes
aware of the fact which existed at the report date and which,
if known at that date, may have caused the auditor to modify
the report, the auditor should consider whether the financial
statements need revision, discuss the matter with management,
and take actions appropriate in the circumstances.
Answers A and B are incorrect because the auditor has no
reporting responsibility for events occurring after the
issuance of the auditors report if such events would not
affect the report.
Answer D is incorrect because the auditor has no
responsibility to monitor disclosed contingencies after the
report is issued.

22. After an audit report containing an unmodified opinion on


a clients financial statements was issued, the client
decided to sell the shares of a subsidiary that accounts
for 30% of its revenue and 25% of its net income. The
auditor should
A. Describe the effects of this subsequently discovered
information in a communication with persons known to
be relying on financial statements.
B. Take no action because the auditor has no obligation
to make further inquiries.
C. Determine whether the information is reliable and, if
determined to be reliable, request that revised
financial statements be issued.
D. Notify the entity that the auditors report may no
longer be associated with the financial statements.
After the report has been issued, the auditor has no
responsibility to make any further or continuing inquiry
regarding the financial statements covered by the report,
unless facts that may affect the report come to his/her
attention.
Answers A, C, and D are incorrect because these actions might
be appropriate if the auditor has discovered facts that
existed at the date of the report.
ASSESSMENT OF GOING CONCERN ASSUMPTION
23. PSA 570 (Going Concern) states that a fundamental
principle in the preparation of financial statements is
the going concern assumption. Under this assumption, an
entity is ordinarily viewed as continuing business for
the foreseeable future with neither the intention nor the
necessity of liquidation, ceasing trading or seeking
protection from creditors pursuant to laws and
regulations. The responsibility to make an assessment of
an entitys ability to continue as a going concern rests
with the
A. Auditor
B. Entitys management
C. Securities and Exchange Commission (SEC)
D. Entitys creditors
Under PAS 1(Presentation of Financial Statements), management
is required to make an assessment of an entitys ability to
continue as a going concern.

24. Which of the following statements best describes the


auditors responsibility concerning the appropriateness
of the going concern assumption in the preparation of
financial statements?
A. The auditors responsibility is to make a specific
assessment of the entitys ability to continue as a
going concern.
B. The auditors responsibility is to predict future
events or conditions that may cause the entity to
cease to continue as a going concern.
C. The auditors responsibility is to consider the
appropriateness of managements use of the going
concern as an assumption and consider whether there
are material uncertainties about the entitys ability
to continue as a going concern that need to be
disclosed in the financial statements.
D. The auditors responsibility is to give a guarantee in
the audit report that the entity has the ability to
continue a going concern.
PSA 570 states that the auditors responsibility is to
consider the appropriateness of managements use of the going
concern assumption in the preparation of financial statements,
and consider whether there are material uncertainties about
the entitys ability to continue as a going concern that need
to be disclosed in the financial statements.
Answer A is incorrect because the entitys management is
required to make specific assessment of the entitys ability
to continue as a going concern.
Answers B and D are incorrect because the auditor cannot
predict future events or conditions that may cause the entity
to cease to continue as a going concern. Hence, even if the
auditors report does not make any reference to going concern
uncertainty, it cannot be viewed as a guarantee as to the
entitys ability to continue as a going concern.

25. Which of the following conditions or events most likely


would cause an auditor to have substantial doubt about an
entitys ability to continue as a going concern?
A. Cash flows from operating activities are negative.
B. Stock dividends replace annual cash dividends.
C. Significant related party transactions are pervasive.
D. Research and development projects re postpones.
PSA 570 gives the following examples of conditions or events
which may give rise to business risks, that individually or
collectively, may cast significant doubt about the entitys
ability to continue as a going concern.
Financial

The net liability or current net current liability


position
Fixed-term borrowings approaching maturity without
realistic prospects of renewal or repayment; or
excessive reliance on short-term borrowings to finance
long-term assets
Indications of withdrawal of financial support by
debtors and other creditors
Negative operating cash flows indicated by historical
or prospective financial statements
Adverse key financial ratios
Substantial operating losses or significant
deterioration in the value of assets used to generate
cash flows
Arrears or discontinuance of dividends
Inability to pay creditors on due dates
Inability to comply with the terms of loan agreements
Change from credit to cash-on-delivery (COD)
transactions with suppliers
Inability to obtain financing for essential new
product development or other essential investments
Operational

Loss of key management personnel


Loss of major market, franchise, license, or principal
supplier
Labor difficulties or shortages of important supplies

Other

Non-compliance with capital or other statutory


requirements
Pending legal or regulatory proceedings against the
entity that may, if successful, result in claims that
are likely to be satisfied.
Changes in legislation or government policy expected
to adversely affect the entity
Answer B is incorrect because payments of stock dividend will
not cause going concern problems since they do not require
outflow of cash.
Answer C is incorrect because the auditors concern with
respect to related party transactions is the adequacy of
disclosure in financial statements.
Answer D is incorrect because postponing R&D projects will not
cast significant doubt about the entitys ability to continue
as a going concern.

26. Which of the following audit procedures most likely would


assist an auditor in identifying conditions and events
that may indicate substantial doubt about an entitys
ability to continue as a going concern?
A. Confirming with third parties the details of
arrangements to maintain financial support
B. Comparing the entitys depreciation and asset
capitalization policies to other entities in the
industry
C. Reconciling the cash balance per books with the cutoff
bank statement and the bank confirmation
D. Inspecting the title documents to verify whether any
assets are pledged as collateral
The auditor typically performs the following procedures to
identify conditions and events that may cast significant doubt
about an entitys ability to continue as a going concern:
o Analytical procedures
o Subsequent events review
o Review of compliance with debt and loan
agreements
o Reading minutes of meetings
o Inquiry of legal counsel
o Confirmation with related and third parties of
arrangements for financial support
Answer B is incorrect because comparing the entitys
accounting policies to other entities would not provide
evidence about the entitys continued existence.
Answer C is incorrect because bank reconciliation procedures
would be performed to test the existence of cash.
Answer D is incorrect because inspecting title documents to
verify whether any assets are pledged as a collateral relates
to disclosure, not going concern, issues.

27. Which of the following conditions or events most likely


would cause an auditor to have substantial doubt about an
entitys ability to continue as a going concern?
A. Restrictions on the disposal of principal assets
present
B. Usual trade credit from suppliers denied
C. Significant related party transactions are pervasive.
D. Arrearages in principal stock dividends are paid.
Denial of usual trade credit from suppliers is an indication
that the entity is facing financial difficulties.
Answer A is incorrect because long-term debt arrangements
ordinarily include restrictions on the disposal of principal
assets.
Answer C is incorrect because the existence of related party
transactions concerns disclosure, not going concern, issues.
Answer D is incorrect because the payment of stock dividends
in arrears does not indicate that the entity is financially
distressed.

28. Which of the following audit procedures would most likely


assist an auditor in identifying conditions and events
that may indicate there could be substantial doubt about
an entitys ability to continue as a going concern?
A. Confirmation of bank balances
B. Confirmation of accounts receivable from major
customers
C. Reconciliation of interest expense with debt
outstanding
D. Review of compliance with terms of debt agreements
The auditors review of compliance with terms of debt
agreements may reveal conditions of non-compliance because the
entity is in weak financial condition.
Answers A, B, and C are incorrect because these procedures are
less effective if the auditors objective is to identify
conditions or events that may cast significant doubt about the
entitys ability to continue as a going concern.
29. Harold, CPA, believes there is substantial doubt about
the ability of ABC Co. to continue as a going concern for
a reasonable period of time. In evaluating ABCs plans
for dealing with adverse effects of future conditions and
events, Harold most likely would consider, as a
mitigating factor, ABCs plans to
A. Postpone expenditures for research and development
projects.
B. Purchase production facilities currently being leased
from a related party.
C. Strengthen internal controls over cash disbursements
D. Discuss with lenders the terms of all debt and loan
agreements.
If the auditor believes that there are conditions or events
indicating that substantial doubts exists about an entitys
ability to continue as a going concern, he/she should inquire
to the management concerning its plans to mitigate their
adverse effects. For example, managements plans to dispose of
assets, borrow money, or restructure debt, reduce or delay
expenditures, or increase capital. The auditor should obtain
sufficient appropriate audit evidence that managements plans
are feasible and that the outcome of these plans will improve
the situation.
Answer B is incorrect because the purchase of production
facilities will worsen the companys weak financial condition.
Answer C is incorrect because improvements in internal control
will not solve the companys going concern problems.
Answer D is incorrect because discussion with lenders is not a
sufficient action to improve the situation.

30. Harry, CPA, believes that there is substantial doubt


about the ability of ABC Corp. to continue as a going
concern for a reasonable period of time. In evaluating
ABCs plans for dealing with the adverse effects of
future conditions and events, Harry most likely would
consider, as a mitigating factor, ABCs plans to
A. Purchase equipment and production facilities currently
being leased.
B. Accumulate treasury shares at prices favorable to
ABCs historic price range.
C. Negotiate reductions in required dividends being paid
on preference shares.
D. Accelerate research and development projects related
to future products.

The auditor should inquire of management concerning its plans


to mitigate adverse effects of identified conditions or events
indicating that a substantial doubt exists about an entitys
ability to continue as a going concern.
The auditor should consider managements plans to dispose of
assets, borrow money or restructure debt, reduce or delay
expenditures, and increase capital. Managements plans to
negotiate reductions in required dividends being paid on
preference shares are intended to increase capital.
Answers A, B, and D are incorrect because leasing equipment
and production facilities, increasing capital, and
postponement of R&D projects would be mitigating factors.

31. When an auditor concludes that there is substantial doubt


about a continuing audit clients ability to continue as
a going concern for a reasonable period of time, the
auditors responsibility is to
A. Consider the adequacy of disclosure about the clients
possible inability to continue as a going concern.
B. Issue a qualified or adverse opinion, depending upon
materiality, due to the possible effects on the
financial statements.
C. Report to the clients audit committee that
managements accounting estimates may need to be
adjusted.
D. Reissue prior years auditors report and add an
emphasis of matter paragraph specifically referring to
substantial doubt and going concern.
PSA 570 provides that if the use of the going concern
assumption is appropriate but a material uncertainty exists,
the auditor considers whether the entitys financial
statements:
a) Adequately describe the principal events or
conditions that may give rise to the significant
doubt of an entitys ability to continue in
operation and managements plans to deal with
these events and conditions; and
b) Disclose clearly that there is material
uncertainty related to events or conditions
which may cast significant doubt on the entitys
ability to continue as a going concern, and,
therefore, that it may be unable to realize its
assets and discharge its liabilities in the
normal course of business.

Answer B is incorrect because if adequate disclosure is made


in financial statements, the auditor may still express an
unmodified opinion. However, the auditors report should be
modified by adding an emphasis of matter paragraph that
highlights the going concern issue.
Answer C is incorrect because the going concern issue is not
directly related to managements accounting estimates that may
need adjustment.
Answer D is incorrect because there is no need to change the
prior years audit report since the auditor does not intend to
change his/her opinion on the prior years financial
statements.

OBTAINING WRITTEN REPRESENTATIONS FROM MANAGEMENT


32. Under PSA 580 (Written Representations), the auditor is
required to obtain audit evidence that management
I. Has fulfilled its responsibility for the fair
presentation of the financial statements in
accordance with applicable financial reporting
framework.
II. Has provided the auditor with all relevant
information and access as agreed in the terms of
audit engagement.
A. I only
B. II only
C. Both I and II
D. Neither I nor II
According to PSA 580, the auditor shall request management to
provide a written representation that it has fulfilled its
responsibility for the preparation of financial statements in
accordance with the applicable financial reporting framework,
including where relevant, their fair presentation as set out
in the terms of the audit engagement.
The standard further provides that the auditor shall request
management to provide a written representation that:

It has provided the auditor with all relevant


information and access as agreed in the terms of audit
engagement; and
All transactions have been recorded and are recorded
and are reflected in the financial statements.

33. The date of the written representation shall be:


A. After the date of the auditors report
B. After the date of approval of the entitys financial
statements
C. Before the entitys financial statements are issued.
D. As near as practicable to, but not after the date of
the auditors report on the financial statements.

34. A purpose of a management representation letter is to


reduce
A. The possibility of a misunderstanding concerning
managements responsibility for the financial
statements
B. The scope of an auditors procedures concerning
related party transactions and subsequent events
C. Audit risk to an aggregate level of misstatement that
could be considered material
D. An auditors responsibility to detect material
misstatements only to the extent that the letter is
relied on
The auditor is required to obtain written representations from
management concerning matters material to the financial
statements when other sufficient appropriate audit evidence
cannot reasonably be expected to exist. According to the
standard, the possibility of a misunderstanding between the
auditor and the management is reduced when oral
representations are confirmed by management in writing.
Answers B, C, and D are incorrect because managements written
representations cannot be a substitute for audit evidence that
can be obtained by performing audit procedures.

35. When an audit is made in accordance with PSAs, the


auditor should always
A. Observe the taking of physical inventory on the
balance sheet date
B. Obtain certain written representations from the
management
C. Employ analytical procedures as substantive tests to
obtain evidence about specific assertions related to
account balances
D. Document the understanding of the clients internal
control and the basis for all conclusions about the
assessed level of control risk for financial statement
assertions

PSA 580 requires the auditor to obtain certain written


representations from management. They confirm oral
representations either unsolicited or in response to
specific inquiries received from management.
Answer A is incorrect because attendance at the physical
inventory taking can be performed either during or after the
end of the period covered by the audit like when well-kept
perpetual inventory records are periodically compared with
physical counts.
Answer C is incorrect because the auditor is required to apply
analytical procedures as risk assessment procedures to obtain
an understanding of the entity and its environment and in the
overall review at the end of the audit. Analytical procedures
may also be applied as substantive procedures.
Answer D is incorrect because the auditor is not required to
document the basis if control risk is assessed at the maximum
level for some assertions.

36. A written management representation letter is most likely


to be an auditors best source of corroborative
information of a clients intent to
A. Settle an outstanding lawsuit for an amount less than
the accrued loss contingency
B. Discount a line of business
C. Terminate an employee pension plan
D. Make a public hearing of its ordinary share capital
Written management representations do not substitute for other
audit evidence that the auditor could reasonably expect to be
available. However, in some circumstances, audit evidence
other than that obtained from inquiries may not be reasonably
expected to be available. For example, the auditors
procedures mayb provide little or no evidence to corroborate
managements intention to discontinue a line of business.
Accordingly, written representations will be necessary to
confirm managements intent.
Answers A, C, and D are incorrect because other audit evidence
is expected to be available to corroborate managements intent
to settle an outstanding lawsuit, terminate an employee
pension plan, or make a public offering of an entitys
ordinary share capital.

37. When considering the use of managements written


representations as audit evidence about the completeness
assertion, an auditor should understand that such
representations
A. Constitute sufficient appropriate audit evidence to
support the assertion when considered in combination
with a sufficiently low assessed level of control
risk.
B. Are not part of the audit evidence considered to
support assertion
C. Replace a low assessed level of control risk as audit
evidence to support the assertion
D. Complement, but do not replace, substantive tests
designed to support the assertion.
Management representations cannot be a substitute for other
audit evidence that the auditor could reasonably expect to be
available. Thus, they complement, but do not replace, other
audit evidence to corroborate managements assertions.
Answer A is incorrect because, regardless of the assessed
level of control risk, the auditor should perform substantive
tests to verify managements assertions and detect
misstatements.
Answer B is incorrect because managements written
representations are considered audit evidence.
Answer C is incorrect because managements written
representations complement, but do not replace, substantive
tests of account balances, transaction classes, and disclosure
complements of financial statements. Control risk is assessed
only to determine the acceptable level of detection risk,
which, in turn, is considered in substantive tests.

38. The written representations shall be in the form of


representation letter addressed to the
A. Entitys management
B. Auditor
C. Entitys chief executive officer
D. Entitys chief financial officer

39. A written representation from a clients management that,


among other matters. Acknowledges responsibility for the
fair presentation of financial statements, should
normally be signed by
A. Chief financial officer and the chair of the board of
directors
B. Chief executive officer and chief financial officer
C. Chief executive officer, the chair of the board of
directors, and the clients lawyer
D. Chair of the audit committee
A managements representation letter would ordinarily be
signed by the members of the management who have primary
responsibility for the entity and its financial aspects based
on their best knowledge and belief. These members are
ordinarily the senior executive officer and the senior
financial officer.
However, the auditor is not precluded from obtaining written
representations from other members of the management. For
example, the auditor may obtain written representation about
the completeness of the minutes of meetings of the board of
directors, stockholders, and other committees from the
individual who has custody of such minutes.

40. The following statements are ordinarily included in a


management representation letter, except
A. The completeness and availability of shareholders and
directors meetings
B. Sufficient appropriate audit evidence has been made
available to permit the expression of an unmodified
opinion
C. There have been no irregularities involving management
or employees who have a significant role in internal
control or that could have a material effect on the
financial statements
D. The financial statements are free of material
misstatements, including omissions
The auditor should perform audit procedures to gather
sufficient appropriate audit evidence on which the opinion is
to be based.
Answers A, C, and D are incorrect because they include
materials that are ordinarily included in a managements
representation letter.

41. What type of opinion is the most appropriate when


management does not provide written representations about
its responsibility for the presentation of financial
statements?
A. Qualified opinion
B. Disclaimer of an opinion
C. Adverse opinion
D. Unmodified opinion

42. To which of the following matter would materiality limits


not apply in obtaining written management
representations?
A. Reductions of obsolete inventory to net realizable
value
B. The disclosure of compensating balance arrangements
involving related parties
C. Losses from purchase commitments at prices in excess
of market value
D. The availability of minutes of shareholders and
directors meetings
Written representations from management may be limited to
matters that are considered either individually or
collectively material to the financial statements. With
respect to certain items, it may be necessary for the auditor
to inform management of his/her understanding of materiality.
Materiality limits do not apply to managements
representations concerning the availability of minutes of
shareholders and directors meetings since it is independent
of amounts in the financial statements.

43. Which of the following statements concerning management


representations is incorrect?
A. Representations by management can be a substitute for
other audit evidence that the auditor could reasonably
be expected to be available
B. If the auditor is unable to obtain sufficient
appropriate audit evidence regarding a matter, which
has, or may have, material effect on the financial
statements and such audit evidence is expected to be
available, this will constitute a limitation in the
scope of audit, even if a representation from
management has been received on the matter
C. If a representation by management is contradicted by
other audit evidence, the auditor should investigate
the circumstances and, when necessary, reconsider the
reliability of other representations by management
D. The auditors working papers would ordinarily include
a summary of oral discussions with management or
written representations from management
According to the standard, representations by management
cannot be a substitute for other audit evidence that the
auditor could reasonably expect to be available. For example,
a representation by management as to the cost of an asset is
not a substitute for the audit evidence of such cost that an
auditor would ordinarily expect to obtain.

44. If management does not provide one or more of the


questioned written representations, the auditor shall
I. Discuss the matter with the management.
II. Reevaluate the integrity of the management and
evaluate the effect that this may have on the
reliability of representations (oral or written)
and audit evidence in general
III. Take appropriate actions, including determining the
possible effect on the opinion in the auditors
report
A. I only
B. I and II only
C. I and III only
D. I, II, and III

45. Which of the following statements concerning written


management representations is incorrect?
A. The written representations shall be in the form of a
representation letter addressed to the entitys chief
executive officer and chief financial officer.
B. The auditor shall request written representations from
management with appropriate responsibilities for the
financial statements and knowledge of the matters
concerned.
C. The auditor may consider it necessary to request
management to provide a written representation that it
has communicated to the auditor all deficiencies in
internal control of which management is aware of.
D. Audit evidence obtained during the audit that
management has fulfilled its responsibility for the
presentation of the financial statements in accordance
with the applicable financial reporting framework is
not sufficient without obtaining confirmation from
management that it believes that it has fulfilled its
responsibility.

PROCEDURES REGARDING LITIGATING CLAIMS


46. The primary source of information to be reported about
litigation, claims, and assessments is the
A. Independent auditor
B. Clients management
C. Court records
D. Clients lawyer
Management should adopt policies and procedures to identify,
evaluate, and account for litigation, claims, and assessments
as basis for the preparation of financial statements in
accordance with the applicable framework.
Answer A is incorrect because the auditors responsibility is
to perform audit procedures in order to be aware of any
litigation and claims involving the entity that may result in
a material misstatement of financial statements (PSA 501,
Audit Evidence Additional Considerations for Specific
Items).
Answer C is incorrect because there is no requirement to
examine court orders.
Answer D is incorrect because the clients lawyer corroborates
the information furnished by the clients management.

47. Which of the following procedures is least likely to be


performed by the auditor to identify litigation and
claims involving the entity which may result in a
material misstatement of financial statements?
A. Conform directly to the clients lawyer that all
claims have been recorded in the financial statements.
B. Make appropriate inquiries of management including
obtaining representations.
C. Examine legal expense accounts.
D. Use any information regarding the entitys business
including information obtained from discussions with
any in-house legal department.
A loss contingency is recognized in the financial statements
only if it is probable and the amount can be reasonably
estimated. Therefore, not all contingent claims are recognized
in the financial statements.
Answers B, C, and D are incorrect because they describe
procedures that the auditor would have to carry out to
identify litigation and claims involving the entity.

48. The primary reason an auditor requests that letters of


inquiry be sent to a clients attorney is to provide the
auditor with
A. A description and evaluation of litigation, claims,
and assessments that existed in the balance sheet date
B. The attorneys opinions of the clients historical
experiences in recent similar litigation
C. Corroboration of information furnished by the
management about litigation, claims, and assessments
PSA 501 (Audit Evidence Specific Cosideration for Selected
Items) states that when the auditor assesses a risk of
material misstatement regarding litigation or claims that have
been identified or when the auditor believes they may exist,
the auditor should seek direct communication with the entitys
legal counsel. Such communication will assist in obtaining
sufficient appropriate audit evidence as to whether
potentially material litigation and claims are known and
managements estimate of financial implications, including
costs, are reliable.
Answers A and D are incorrect because, as stated in the
standard, the letter would ordinarily specify the following
when it is considered unlikely that the entitys legal counsel
will respond to a general inquiry:
o A list of litigation and claims
o Managements assessment of the outcome of the
litigation or claim and its estimate of the
financial implications, including costs involved
o A request that the entitys legal counsel
confirm the reasonableness of managements
assessment and provide the auditor with further
information if the list is considered by the
entitys legal counsel to be incomplete or
incorrect.
Answer B is incorrect because the auditor is
primarily concerned with current litigation
involving the entity, not the clients historical
experiences in similar litigation.

49. The letter of audit inquiry should be


A. Prepared sent by the auditor
B. Prepared by management and sent by the auditor
C. Prepared and sent by management
D. Prepared by the auditor and sent by management
The letter of audit inquiry, which should be prepared by
management and sent by the auditor, requests the entitys
legal counsel to communicate directly with the auditor.

50. An auditor should obtain evidence relevant to all of the


following factors concerning third-party litigation
against a client except the
A. Jurisdiction in which the matter will be resolved.
B. Existence of a situation indicating an uncertainty as
to possible loss
C. Profitability of an unfavorable outcome
D. Period in which the underlying cause for legal action
occurred
When performing procedures after year-end concerning
litigation and claims, the auditor is primarily concerned with
the impact of the matter on the fair presentation of the
clients financial statements.
Accordingly, the auditor should obtain sufficient appropriate
audit evidence about the existence, amount, profitability of
an unfavorable outcome, and timing of the cause of the
litigation or claims.
The auditor is least interested in determining the
jurisdiction in which the litigation will be resolved.
Answers B, C, and D are incorrect because the uncertainty as
to the possible loss, the probability of an outcome, and the
period in which the underlying cause for legal action occurred
are matters that should be considered by the auditor to
determine the impact of litigation on the financial
statements.

51. The refusal of a clients lawyer to provide a


representation on the legality of a particular act
committed by the client is ordinarily
A. Proper grounds to withdraw from the engagement
B. Insufficient reason to modify the auditors report
because of the lawyers obligation of confidentiality
C. Considered to be a scope limitation
D. Sufficient reason to issue a subject to opinion
PSA 501 states that where the entitys legal counsel refuses
to respond in an appropriate manner and the auditor is unable
to obtain sufficient appropriate audit evidence by applying
alternative audit procedures, the auditor would consider
whether there is a scope limitation which may lead to a
qualified opinion or a disclaimer of opinion.
Answer A is incorrect because the lawyers failure to
corroborate information furnished by management could lead to
a qualified opinion or a disclaimer of opinion but would not
be considered proper grounds for the withdrawal from the
engagement.
Answer B is incorrect because the auditor could be precluded
from expressing an unmodified opinion when there is a material
scope limitation.
Answer D is incorrect because subject to is not allowed to
be used as a qualifying phrase in an auditors report.

52. Managements refusal to give the auditor permission to


communicate with the entitys legal counsel is most
likely to lead to
A. An adverse opinion
B. A qualified opinion or an adverse opinion
C. An unmodified opinion
D. A qualified opinion or a disclaimer of opinion
If management refuses to give the auditor permission to
communicate with the entitys legal counsel, this would be a
scope limitation and should ordinarily lead to a qualified
opinion or a disclaimer of opinion.

53. In which of the following circumstances would an auditor


most likely meet with the clients legal counsel to
discuss the likely outcome of the litigation and claims?
I. The auditor determines that the matter is a
significant risk.
II. There is a disagreement between management and the
entitys legal counsel.
III. The subject matter of the litigation is complex.
A. I and II only
B. II and III only
C. I and III only
D. I, II, and III
PSA 501 states that, in certain circumstances, for example,
where the auditor determines that the matter is a significant
risk, the matter is complex or there is disagreement between
management and the entitys legal counsel, it may be necessary
for the auditor to meet with the entitys legal counsel to
discuss the likely outcome of litigation and claims. Such
meetings would take place with managements permission and,
preferably, with a representative of management in attendance.

54. Which of the following statements extracted from a


clients lawyers letter concerning litigation, claims,
and assessment most likely would cause the auditor to
request clarification?
A. I believe that the action can be settled for less
than the damages claimed.
B. I believe that the company will be able to defend
this action successfully.
C. I believe that the plaintiffs case against the
company is without merit.
D. I believe that the possible liability to the company
is nominal in amount.
The letter of audit inquiry requests the entitys lawyer to
evaluate the likelihood of an unfavorable outcome, and if
possible, to make an estimate of the amount or range of
potential loss. A statement that the action can be settled for
less than the damages claimed is unclear as to the amount or
range of potential loss.
Answers B and C are incorrect because the responses clearly
state that the entity should not expect any liability to arise
from the lawsuit. Answer D is incorrect because the response
is clear as to the amount or range of possible loss.

55. The auditor should consider the status of legal matters


up to the
A. Balance sheet date
B. Date of the auditors report
C. Date of approval of the financial statements
D. Date of issuance of financial statements
TRUE OR FALSE
1. If potential loss on a contingent liability is remote,
the liability is usually disclosed in the auditors
report but not disclosed in the financial statements.
2. When obtaining evidence regarding litigation against a
client, the CPA would be least interested in
determining the period in which the underlying cause
of litigation occurred.
3. The auditor has no responsibility for events occurring
in the subsequent period unless these events affect
transactions recorded on or before the balance sheet
date.
4. The standard letter of inquiry to the clients legal
counsel should be prepared on auditors stationery and
assigned by an audit partner.
5. The auditors primary means of obtaining corroboration
of managements information concerning litigation is a
letter of audit inquiry to the clients lawyer.
6. When a client will not permit inquiry of outside legal
counsel, the audit report will ordinary contain an
unmodified opinion with an emphasis of matter
paragraph.
7. An auditor has the responsibility to actively search
for subsequent events that occur subsequent to the
date of auditors report.
8. A letter of representations from management provides
written documentation, which is a higher quality of
evidence than managements oral responses to
inquiries.
9. If a lawyer refuses to furnish corroborating
information regarding litigation, claims, and
assessments, the auditor should consider the refusal
to be a scope limitation.
10. After an auditor has issued an audit report, there is
no obligation to make any further audit tests or
inquiries with respect to the audited financial
statements covered by that report unless new
information comes to the auditors attention
concerning an event that occurred prior to the date of
the auditors report that may have affected the
auditors report.
11. The date of the management representation letter
should coincide with the date of auditors report.
12. The management letter is used to allow management to
corroborate oral representations to the auditor.
13. Auditors should perform audit procedures relating to
subsequent events throughout the last day of field
work.
14. The review of audit working papers by the audit
partner is normally related completed immediately as
each working paper is completed.
15. Subsequent events which require adjustment to the
financial statements provide additional information
about significant conditions/events which did not
exist at the balance sheet date.
16. Subsequent events which require adjustment to the
financial statements provide additional information
about significant conditions/events which existed at
the balance sheet date.
17. The date of the management representation letter
received from the client should coincide with the
balance sheet date.
18. Generally, loss contingencies that are judged to be
remote should be disclosed in the notes to financial
statements.
19. To obtain evidence about an entitys subsequent
events, an auditor would reconcile bank activity for
the month after the balance sheet date with cash
activity reflected in accounting records.
20. The auditor is required to confirm directly with the
clients lawyer that all claims have been recorded in
the financial statements.
21. The auditor is required to confirm directly with the
clients lawyer that all claims have been recorded in
the financial statements.
22. A lawyer is responding to an independent auditor as a
result of the audit clients letter of inquiry. The
lawyer may appropriately limit the response to matters
to which the attorney has given substantive attention
in the form of legal consultation or representation.
23. Subsequent events for which the auditor has a
responsibility to actively search are defined as
events which occur subsequent to the balance sheet
date.
24. The letter of representation is signed by high-level
corporate officials, usually the chief executive
officer and chief financial officer.
25. Refusal by a client to prepare and sign the
representation letter would require the auditor to
issue a qualified opinion or a disclaimer.
KEY ANSWERS

1. B 20. D 39. B
2. C 21. C 40. B
3. B 22. B 41. B
4. C 23. B 42. D
5. A 24. C 43. A
6. D 25. A 44. D
7. D 26. A 45. A
8. B 27. B 46. B
9. C 28. D 47. A
10. A 29. A 48. C
11. B 30. C 49. B
12. C 31. A 50. A
13. C 32. C 51. C
14. C 33. D 52. D
15. A 34. A 53. D
16. A 35. B 54. A
17. B 36. B 55. B
18. B 37. D
19. C 38. B

TRUE OR FALSE
1. FALSE 14. FALSE
2. FALSE 15. TRUE
3. FALSE 16. TRUE
4. FALSE 17. FALSE
5. TRUE 18. FALSE
6. FALSE 19. FALSE
7. FALSE 20. FALSE
8. FALSE 21. FALSE
9. TRUE 22. TRUE
10. TRUE 23. FALSE
11. TRUE 24. TRUE
12. FALSE 25. TRUE
13. TRUE
Chapter 10
THE AUDITORS REPORT ON FINANCIAL STATEMENTS

1. A major purpose of the auditors report on financial


statements is to
a. Assure investors of the complete accuracy of the
financial statements
b. Enhance the degree of confidence of intended users
in the financial statements
c. Deter creditors fom extending loans in high-risk
situations
d. Describe the specific auditing procedures
undertaken to gather evidence for the opinion

PSA 200 (Overall Objectives of the Independent Auditor


and the Conduct of an Audit in Accordance with Philippine
Standards on Auditing) states that the purpose of an
audit is to enhance the degree of confidence of intended
users in the financial statements. This is achieved by
the expression of an opinion by the auditor on whether
the financial statements are prepared, in all material
respects, in accordance with an applicable financial
reporting framework.
Answer A is incorrect because an auditors opinion is not
an assurance of complete accuracy of the financial
statements.
Answer C is incorrect because while the auditors report
may contain some information that users of financial
statements may use to make informed decisions, it is not
a substitute for their judgment.
Answer D is incorrect because the Auditors
Responsibility section of the auditors provides only a
general description of how an audit is conducted.

2. If a companys external auditor expresses an unmodified


opinion as a result of the audit of the companys
financial statements, readers of the audit report can
assume that
a. The external auditor found no fraud
b. The company is financially sound and the financial
statements are accurate
c. Internal control is effective
d. The auditor concludes that the financial statements
are prepared, in all material respects, in
accordance with the applicable financial reporting
framework.

An UNMODIFIED OPINION is expressed when the auditor


concludes that the financial statements are prepared, in
all material respects, in accordance with the applicable
financial reporting framework.
If the auditor concludes that, based on the audit
evidence obtained, the financial statements as a whole
are not free from material misstatement or is unable to
obtain sufficient appropriate audit evidence to conclude
that the financial statements as a whole are free from
material misstatement, the auditor should express a
modified opinion (qualified opinion, adverse opinion or
disclaimer of opinion) on the financial statements.
Answer A is incorrect because readers may only assume
that fraud found did not, in the auditors opinion,
materially affect the fair presentation of the
statements.
Answer B is incorrect because the report contains an
opinion on whether the financial statements are prepared,
in all material respects, in accordance with the
applicable financial reporting framework and not an
appraisal of the company. Also, an unmodified opinion
provides reasonable assurance that the statements are
free of material misstatement, not that they are
accurate.
Answer D is incorrect because an unmodified opinion may
be expressed even though the auditees internal control
system is ineffective.
3. When the financial statements contain material but not
pervasive misstatements because the accounting policies
selected are not consistent with the applicable financial
reporting framework, the auditor should
a. Express a qualified opinion and describe the matter
giving rise to the modification in a separate
paragraph
b. Express a qualified opinion and describe the matter
giving rise to the modification within the opinion
paragraph
c. Disclaim an opinion and describe the matter giving
rise to the modification in a separate paragraph
d. Disclaim an opinion and describe the matter giving
rise to the modification within the opinion
paragraph
A QUALIFIED OPINION shall be expressed when, based on
sufficient appropriate audit evidence obtained, the
auditor concludes that misstatements, individually or in
the aggregate, are material, but not pervasive, to the
financial statements.
A qualified opinion is also appropriate when the auditor
is unable to obtain sufficient appropriate audit evidence
on which to base the opinion, but the auditor concludes
that the possible effects of undetected misstatements on
the financial statements, if any, could be material but
not pervasive.
When a qualified opinion is expressed, the auditors
report should include a separate paragraph with the
heading Basis for Qualified Opinion that provides a
description of the matter giving rise to the expression
of a qualified opinion. This paragraph should be placed
immediately before the opinion paragraph.
If the material misstatement relates to specific amounts
in the financial statements (including quantitative
disclosures), the Basis for Qualified Opinion paragraph
should include a description and quantification of the
financial effects of the misstatement, unless
impracticable. If the financial effects cannot be
quantified, the auditor should so state in the paragraph.
Answer B is incorrect because a Basis for Qualified
Opinion paragraph is required.
Answer C and D are incorrect because a DISCLAIMER OF
OPINION is appropriate when the auditor is unable to
obtain sufficient appropriate audit evidence on which to
base the opinion, and the auditor concludes that the
possible effects on the financial statements of
undetected misstatements, if any, could be both material
and pervasive.

4. A CPA engaged to audit financial statements observes that


the accounting for a certain material item is not in
accordance with the applicable financial reporting
framework, although the departure is prominently
disclosed in a note to the financial statements. The CPA
should
a. Express an unmodified opinion but insert an
Emphasis of Matter paragraph emphasizing the matter
by reference to the note
b. Disclaim an opinion
c. Not allow the accounting treatment for this item to
affect the type of opinion because the departure
from the requirement of the applicable financial
reporting framework was disclosed.
d. Modify the opinion because of the departure from
the requirement of the applicable financial
reporting framework
When financial statements contain material misstatements
because the accounting policies applied are not
consistent with the applicable financial reporting
framework, the auditor should express a QUALIFIED OPINION
(if the effect is material but not pervasive) or ADVERSE
OPINION (if the effect is both material and pervasive).
Answer A is incorrect because an unmodified opinion is
inappropriate when financial statements are not free from
material misstatement.
Answer B is incorrect because a disclaimer of opinion is
appropriate when the auditor is unable to obtain
sufficient appropriate audit evidence and the auditor
concludes that the possible effects of misstatements
could be both material and pervasive.
Answer C is incorrect because a note disclosure of the
departure from the requirement of the applicable
financial reporting framework does not eliminate the need
to modify the opinion.

5. Whenever there is a scope limitation, the appropriate


response is to issue a/an
a. Qualified opinion
b. Adverse opinion
c. Disclaimer of opinion
d. Unmodified report, a qualification of scope and
opinion, or a disclaimer, depending on materiality
6. When a qualified opinion is expressed, the implication is
that the auditor
a. Does not believe the financial statements are
presented in accordance with the applicable
financial reporting framework
b. Does not know if the financial statements are
presented in accordance with the applicable
financial reporting framework
c. Believes the financial statements are presented
fairly
d. Believes the financial statements are presented
fairly except for a specific item of them
7. If a misstatement is immaterial to the financial
statements of the company for the current period, but is
expected to have a material effect in future periods, it
is appropriate to express a/an
a. Qualified opinion
b. Unmodified opinion
c. Disclaimer of opinion
d. Adverse opinion
8. On January 2, 2016, the ABC CO. received a notice from
its primary suppliers that effective immediately all
wholesale prices would be increased 10%. On the basis of
the notice, ABC revalued its December 31, 2015 inventory
to reflect the higher costs. As a result, the statement
of financial position reflects inventory stated at an
amount higher than its net realizable value. The
inventory constituted a material proportion of total
assets; however, the effect of the revaluation was
material to current assets but not to total assets or net
income. In reporting on the companys financial
statements for the year ended December 31, 2015, in which
inventory is valued at the adjusted amount, the auditor
would most likely
a. Express an unmodified opinion provided the nature
of the adjustment and the amounts involved are
disclosed in notes to the financial statements
b. Express a qualified opinion
c. Disclaim an opinion
d. Express an adverse opinion
PAS 2 (Inventories) requires that inventories should be
valued at the lower of cost and net realizable value.
Because the effect of the misstatement is material but
not pervasive, the auditor should express a qualified
opinion. The inclusion of the matter in the notes does
not overcome the need for a modified opinion.
Answer A is incorrect because an unmodified opinion is
inappropriate when financial statements are not free from
material misstatement.
Answer C is incorrect because a disclaimer of opinion is
only appropriate when there is no sufficient appropriate
evidence to form an opinion.
Answer D is incorrect because an adverse opinion may not
be appropriate since the effect of revaluation was not
material to total assets or net income, i.e., not
pervasive.
9. ABC CO. completed construction and began to lease a 500-
unit apartment on May 28, 2014. During June, 250 units
were leased, and an additional 150 units were leased in
July 2015.
During the month of May 2014, the company charged to
expense P230,000 for the cost of advertising, a grand
opening party, and the advertising agency fee for
planning the campaign. At December 31, 2015, the
statement of financial position reflected P875,000 of
initial direct costs incurred by the company including
commissions and legal fees paid in negotiating the lease.
These initial direct costs are shown as an addition to
the carrying amount of the leased asset and is being
recognized as an expense over the term of the lease on
the same basis as the lease income.
During your audit of the companys financial statements
for the year ended December 31, 2015 (conducted in
accordance with PSAs), no facts other than those
described above came to your knowledge that would cause
your opinion to be other than that the financial
statements were presented fairly in accordance with
Philippine Financial Reporting Standards.
What type of opinion should your report contain?
a. An adverse opinion
b. An unmodified opinion
c. A disclaimer opinion
d. A qualified opinion

Because the transactions were accounted for properly in


accordance with the requirements of PFRS on leases, an
unmodified opinion is appropriate.
10. When an auditor modifies an opinion because of
inadequate disclosure, the auditor should describe the
nature of the omission in a separate Basis for
Modification paragraph and modify the
Introductory Auditors Opinion
Paragraph Responsibility Paragraph
Paragraph
a. Yes No No
b. Yes Yes No
c. No Yes Yes
d. No No Yes
11. In which of the following situations would an
auditor ordinarily choose between expressing a qualified
opinion or an adverse opinion?
a. The auditor did not observe the entitys physical
inventory and is unable to become satisfied as to
its balance by other auditing procedures.
b. The financial statements fail to disclose
information that is required by the applicable
financial reporting framework.
c. The auditor is asked to report only on the entitys
statement of financial position and not on the
other general purpose financial statements.
d. Events disclosed in the financial statements cause
the auditor to have substantial doubt about the
entitys ability to continue as a going concern.
Material misstatements arising from omission of
disclosures required by the acceptable financial
reporting framework may result in either a qualified or
an adverse opinion.
Answer A is incorrect because a disclaimer of opinion is
appropriate when the auditor is unable to obtain
sufficient appropriate audit evidence on which to base
the opinion, and the auditor concludes that the possible
effects on the financial statements of undetected
misstatements, if any, could be both material and
pervasive.
Answer C is incorrect because audits of single financial
statements may result in the expression of an unmodified
opinion (PSA 805, Special Considerations Audits of
Single Financial Statements and Specific Elements,
Accounts or Items of a Financial Statement).
Answer D is incorrect because substantial doubt about the
entitys continued existence normally results in the
addition of an emphasis of matter paragraph immediately
after the opinion paragraph.
12. Yang, CPA, concludes that there is significant
doubt about ABC CO.s ability to continue as a going
concern. If ABCs financial statements adequately
disclose its financial difficulties, Yangs report should
Include
Emphasis Specifically
Of matter Specifically use the
Paragraph use the words
Following words Going Significant
The Opinion Concern Doubt
Paragraph
a. Yes Yes Yes
b. Yes Yes No
c. Yes No Yes
d. No Yes Yes
PSA 570 (Going Concern) gives the following illustration
of an Emphasis of Matter paragraph when the auditor is
satisfied as to the adequacy of the note disclosure:
Emphasis of Matter
Without qualifying our opinion, we draw attention to Note
X in the financial statements which indicates that the
Company incurred a net loss of Pxxx during the year ended
December 31,20X1 and, as of that date, the Companys
current liabilities exceeded its total assets by Pxxx.
These conditions, along with other matters as set forth
in Note X, indicate the existence of a material
uncertainty that may cast significant doubt about the
Companys ability to continue as a going concern.
13. Which sections of an auditors unmodified report on
financial statements should refer to Philippine Standards
on Auditing (PSA) and Philippine Financial Reporting
Standards (PFRS)?

PSA PFRS
a. Managements opinion
Responsibility auditors responsibility
b. Auditors managements responsibility
Responsibility opinion
c. Opinion managements responsibility
d. Auditors opinion
Responsibility
14. Without affecting the CPAs willingness to express
an unmodified opinion on the clients financial
statements, corporate management may refuse a request to
a. Authorize its attorney to confirm that a list of
pending or threatened litigation prepared by
management includes all items known to the
attorney.
b. Change its basis of accounting for inventories from
first-in, first-out (FIFO) method to weighted
average method.
c. Write down to salvage value certain equipment that
is no longer useful.
d. Allow the CPA to examine tax returns for years
prior to that of the financial statements being
audited.
Even if management refuses to change its inventory
costing method from FIFO to weighted average, the
financial statements would still comply with the
requirements of the PFRS. Therefore,the CPA may still
express an unmodified opinion.
15. In which of the following circumstances would an
auditor most likely add an Emphasis of Matter paragraph
to the auditors report while expressing an unmodified
opinion?
a. The auditor is asked to report on a single
financial statement (e.g., a statement of financial
position).
b. There is significant doubt about the entitys
ability to continue as a going concern.
c. Managements estimates of the effects future events
are unreasonable.
d. Certain transactions cannot be tested because of
managements records retention policy.
When there is significant doubt on the entitys ability
to continue as a going concern, the auditor should
consider the adequacy of disclosure of the material
uncertainty. If adequate disclosure is made in the
financial statements, the auditor shall express an
unmodified opinion and include an Emphasis of Matter
paragraph (after the opinion paragraph) in the report.
Answer A is incorrect because under PSA 805, an auditor
who is engaged to audit a single financial statement may
express an unmodified opinion without ding Emphasis of
Matter paragraph.
Answer C is incorrect because an unmodified opinion would
not be appropriate when financial statements contain
unreasonable estimates.
Answer D is incorrect because a qualified opinion or a
disclaimer of opinion is proper when there is restriction
on the scope of the audit.
16. If an auditor is satisfied that sufficient evidence
supports managements assertions about an uncertainty and
its presentation or disclosure, the auditor should
a. Express a modified opinion with an Emphasis of
Matter paragraph.
b. Express an unmodified opinion with an Emphasis of
Matter paragraph.
c. Express an unmodified opinion with an Other Matter
paragraph.
d. Express a qualified opinion or disclaim an opinion,
depending upon the materiality of the loss.

According to PSA 706 (Emphasis of Matter Paragraphs and


Other Matter Paragraphs in the Independent Auditors
Report), if the auditor considers it necessary to draw
users attention to a matter presented or disclosed in
the financial statements that, in the auditors judgment,
is of such importance that it is fundamental to users
understanding of the financial statements, the auditor
shall include an Emphasis of Matter paragraph in the
auditors report provided that the auditor has obtained
sufficient appropriate audit evidence that the matter is
not materially misstated n the financial statements. The
auditor shall include it immediately after the opinion
paragraph and indicate that the auditors opinion is not
modified in respect of the matter emphasized.
Answer A and D are incorrect because a modified opinion
(qualified, adverse or diclaimer) is appropriate when
financial statements are not free of material
misstatement and/or when the scope of the audit is
limited.
Answer C is incorrect because an Other Matter paragraph
refers to a matter other than those presented or
disclosed in the financial statements that, in the
auditors judgment, is relevant to users understanding
of the audit, the auditors responsibilities or the
auditors report.
17. During the year ended December 31, ABC Co. Reported
its property, plant and equipment at the lower of cost or
market (LCM) because their fair value had declined. The
loss has been included in the income statement and the
adjustment has been fully disclosed in the notes. If a
CPA believes that the values reported in the financial
statements are reasonable, what opinion should be
expressed?
a. An unmodified opinion
b. A subject to qualified opinion
c. An adverse opinion
d. A disclaimer of opinion

Under PAS 16 (Property,Plant and Equipment), a company


may report its property,plant and equipment using the
cost model or the revaluation model. Recording them at
LCM is therefore not consistent with the requirement of
the PAS. Assuming that the effects are material, the
auditor should express either a qualified or adverse
opinion.
Answer A is incorrect because a material misstatement in
the financial statement precludes an unmodified opinion.
Answer B is incorrect because the use of subject to in
a qualified opinion is prohibited.
Answer D is incorrect because a disclaimer is appropriate
when the scope limitation does not permit expression of
an opinion.
18. Reference to financial statements in PSA 700
(Forming an Opinion and Reporting on Financial
Statements), means
a. A complete set of general purpose financial
statements, including the related notes.
b. A complete set of financial statements prepared to
meet the financial information needs of specific
users.
c. A complete set of financial statements prepared in
accordance with a special purpose framework.
d. A complete set of financial statements prepared in
accordance with either a general or special purpose
framework.

Reference to financial statements in PSA 700 means a


complete set of general purpose financial statements,
including the related notes.
General purpose financial statements are financial
statements prepared in accordance with a general purpose
framework, i.e., financial reporting framework designed
to meet the common financial information needs of a wide
range of users.
Answer B, C and D are incorrect because PSA 800 (Special
Considerations Audits of Financial Statements Prepared
in Accordance with Special Purpose Frameworks) applies to
financial statements prepared in accordance with a
special purpose framework, i.e., a financial reporting
framework designed to meet the financial information
needs of specific users.
19. Which of the following statements best describes a
compliance framework?
a. A compliance framework requires compliance with the
requirements of the framework and acknowledges
explicitly or implicitly that, to achieve fair
presentation of the financial statements, it may be
necessary for management to provide disclosures
beyond those specifically required by the
framework.
b. A compliance framework requires compliance with the
requirements of the framework and acknowledges
explicitly that it may be necessary for management
to depart from a requirement of the framework to
achieve fair presentation of the financial
statements.
c. A compliance framework only requires compliance
with the requirements of the framework.
d. A compliance framework refers to a financial
information needs of specific users.

The term fair presentation framework is used to refer


to a financial reporting framework that requires
compliance with the requirements of the framework and:
i. Acknowledges explicitly or implicitly that, to achieve
fair presentation of the financial statements, it may
be necessary for management to provide disclosures
beyond those specifically required by the framework;
or

ii. acknowledges explicitly that that it may be necessary


for management to depart from a requirement of the
framework to achieve fair presentation of the financial
statements.
The term compliance framework is used to refer to a
financial reporting framework that requires compliance
with the requirements of the framework, but does not
contain the acknowledgements in (i) or (ii) above. (PSA
70,par. 7)

20. Which of the following should be considered when


forming an opinion on the audited financial statements?
I.Whether sufficient appropriate audit evidence has been
obtained.
II. Whether uncorrected misstatements are material,
individually or in aggregate.
III. The qualitative aspects of the entitys accounting
practices, including indicators of possible bias in
managements judgments.

a. I only
b. I and III only
c. I and II only
d. I, II and III
21. When reporting on financial statements prepared in
accordance with a compliance framework, the auditor shall
evaluate

A B C D
A. The overall presentation,
Structure and content of
The financial statements No Yes No Yes
B. Whether the financial
Statements, including the
Related notes, represent
The underlying transactions
And events in a manner that
Achieves fair presentation No Yes Yes No

The auditors evaluation should include consideration of


(a) and (b) above when reporting on financial statements
prepared in accordance with a fair presentation framework
(PSA700,par 14).

22. Management and, when appropriate, those charged


with governance have responsibility for
I. The preparation of the financial statements
II. An adequate description of the framework applied in
the preparation of the financial statements
a. I only
b. II only
c. Both I and II
d. Neither I nor II
23. An audit clients description that its financial
statements are prepared in accordance with a particular
applicable financial reporting framework is appropriate
only if
a. The financial statements comply with all the
requirements of that framework that are effective
during the period covered by the financial
statements.
b. The financial statements are in substantial
compliance with that framework.
c. The financial statements adequately disclose the
significant accounting policies selected and
applied.
d. The terminology used in the financial statements,
including the title of each financial statement, is
appropriate.
24. To distinguish it from reports that might be issued
by others,such as by officers of the entity,the board of
directors, or from the reports of other auditors who may
not have to abide by the same ethical requirements the
independent auditor, the auditors report should have an
appropriate
a. Addressee
b. Title
c. Signature
d. Opinion

The auditors report should have a title that clearly


indicates that it is the report of an independent
auditor. The title Independent Auditors Report affirms
that the auditor has met all of the relevant ethical
requirements regarding independence and, therefore,
distinguishes the independent auditors report from
reports issued by others.

25. The audit report date on an unmodified report


indicates
a. The last date on which users may institute a
lawsuit against either the client or the auditor.
b. The last day of the auditors responsibility for
the review of significant events occurring after
the end of the reporting period.
c. The end of the reporting period.
d. The date on which the financial statements were
filed with the Securities and Exchange Commission.
26. The auditors report should be addressed
a. Only to the shareholders of the entity whose
financial statements are being audited.
b. Only to the board of directors of the entity whose
financial statements are being audited.
c. To the CEO or the CFO of the entity whose financial
statements are being audited.
d. Either to the shareholders or the board of
directors of the entity whose financial statements
are being audited.
The auditors report is ordinarily addressed to those for
whom the report is prepared, often either to the
shareholders or to those charged with governance of the
entity whose financial statements are being audited.

27. Which of the following is included in the


introductory or opening paragraph of the auditors
report?
a. Identification of the financial statements audited,
including the date of and period covered by the
financial statements.
b. A statement that the financial statements are the
responsibility of the entitys management.
c. A statement that the audit was conducted in
accordance with Philippine Standards on Auditing.
d. A statement that the responsibility of the auditor
is to express an opinion on the financial
statements based on the audit.

The introductory paragraph shall identify the entity


whose financial statements have been audited and shall
state that the financial statements have been audited.
The introductory paragraph shall also:
a. Identify the title of each of the financial statements
that compromise the complete set of financial
statements;
b. Refer to the summary of significant accounting
policies and other explanatory information; and
c. Specify the date and period covered by the financial
statements.

PSA 700 (Forming an Opinion and Reporting on Financial


Statements) gives the following example of an independent
auditors report:
Independent Auditors Report
(Appropriate Addressee)
Report on the Financial Statements
We have audited the accompanying financial statements of
the ABC Company, which comprise the statement of
financial position as of at December 31,20x1, and the
statement of comprehensive income, statement of changes
in equity and statement of cash flows for the year then
ended, and a summary of significant accounting policies
and other explanatory information.
Managements Responsibility for the Financial Statements
Management is responsible for the preparation and fair
presentation of these financial statements in accordance
with Philippine Financial Reporting Standards, and for
such internal control as management determines is
necessary to enable the preparation of financial
statements that are free from material misstatement,
whether due to fraud or error.
Auditors Responsibility
Our responsibility is to express an opinion on these
financial statements based on our audit. We conducted our
audit in accordance with Philippine Standards on
Auditing. Those standards require that we comply with
ethical requirements and plan and perform the audit to
obtain reasonable assurance about whether the financial
statements are free from material misstatement.
An audit involves performing procedures to obtain
evidence about the amounts and disclosures in the
financial statements. The procedures selected depend on
the auditors judgment, including the assessment of the
risks of material misstatement of the financial
statements, whether due to fraud or error. In making
those risk assessments, the auditor considers internal
control relevant to the entitys preparation and fair
presentation of the financial statements in order to
design audit procedures that are appropriate in the
circumstances, but not for the purpose of expressing an
opinion on the effectiveness of the entitys internal
control. An audit also includes evaluating the
appropriateness of accounting policies used and the
reasonableness of accounting estimates made by
management, as well as evaluating the overall
presentation of the financial statements.
We believe that the audit evidence we have obtained is
sufficient and appropriate to provide a basis for our
audit opinion.
Opinion
In our opinion, the financial statements present fairly,
in all material respects, the financial position of ABC
Company as of December 31,20x1, and of its financial
performance and its cash flows for the year then ended in
accordance with Philippine Financial Reporting Standards.
Report on Other Legal and Regulatory Requirements
(Form and content of this section of the auditors report
will vary depending on the nature of the auditors other
reporting responsibilities.)

(Auditors signature)
(Date of the auditors report)
(Auditors address)

28. An entitys management is responsible for the


preparation and fair presentation of the financial
statements. Its responsibility includes the following,
except
a. Designing, implementing, and maintaining internal
control relevant to the preparation and
presentation of financial statements.
b. Making accounting estimates that are reasonable in
the circumstances.
c. Selecting and applying appropriate accounting
policies.
d. Assessing the risks of material misstatement of the
financial statements.

The independent auditor not the entitys management


should make an assessment of the risks of material
misstatement of the financial statements, whether due to
fraud or error.

29. The opinion paragraph of the auditors report


I. Identifies the applicable financial reporting
framework on which the financial statements
are based.
II. Expresses an opinion on the financial
statements.
a. I only
b. II only
c. Both I and II
d. Neither I nor II

As illustrated in the standard, the opinion paragraph of


the auditors report reads:
In our opinion, the financial statements present
fairly, in all material respects, the financial position
of ABC Company as of December 31,20x1, and its financial
performance and its cash flows for the year then ended in
accordance with Philippine Financial Reporting
Standards.
30. The following statements relate to the date of the
auditors report. Which is false?
a. The auditor should date the report as of the
completion date of the audit.
b. The date of the auditors report should not be
earlier that the date on which the financial
statements are signed or approved by management.
c. The date of the auditors report should not be
later than the date on which the financial
statements are signed or approved by management.
d. The date of the auditors report should always be
later than the date of the financial statements
(i.e., the balance sheet ate).

PSA 700 states that since the auditors opinion is


provided on the financial statements and the financial
statements are responsibility of management, the auditor
is not in a position to conclude that sufficient
appropriate audit evidence has been obtained until the
auditor obtains evidence that a complete set of financial
statements has been prepared and management has accepted
responsibility for them.

31. Which of the following statements best expresses


the objective of the traditional audit of financial
statements?
a. To express an opinion on the fairness with which
the statements present financial position,
financial performance, and cash flows in accordance
with Philippine Financial Reporting Standards.
b. To express an opinion on the accuracy with which
the statements present financial position,
financial performance, and cash flows in accordance
with Philippine Financial Reporting Standards.
c. To make suggestions as to the form or content of
the financial statements or to draft them in whole
or in part.
d. To assure adoption of sound accounting policies and
the establishment and maintenance of internal
control.
The objective of the traditional financial statement
audit is for the auditor to express an opinion (or a
disclaimer of opinion) on the fairness, in all material
respects, of the financial statements prepared and
presented by the clients management.
The auditors report should clearly state whether the
audit was conducted in accordance with PSAs, and whether,
in the opinion of the independent auditor, the financial
statements are presented in accordance with Philippine
Financial Reporting Standards.
Answer B is incorrect because the auditor expresses an
opinion on the fairness (not the accuracy) of financial
statements.
Answer C is incorrect because the clients management is
responsible for adopting sound accounting policies and
establishing and maintaining internal control.

32. Which of the following best describes why an


independent auditor is asked to express an opinion on the
fair presentation of financial statements?
a. It is a customary courtesy that all shareholders
receive an independent report on managements
stewardship in managing the affairs of the
business.
b. The opinion of an independent party is needed
because a company may not be objective with respect
to its own financial statements.
c. It is difficult to prepare financial statements
that fairly present a companys financial position,
financial performance, and cash flows without the
expertise of an independent auditor.
d. It is managements responsibility to seek available
independent aid in the appraisal of the financial
information shown in its financial statements.

The independent auditors opinion lends credibility to


the financial statements prepared and presented by an
entitys management.
Answer A is incorrect because, although it is
customary courtesy that all shareholders receive an
independent report on managements stewardship
function, it is not the primary reason why an
independent report is issued to accompany the
financial statements.
Answer C is incorrect because the preparation of
financial statements is the sole responsibility of an
entitys management.
Answer D is incorrect because the independent
auditors function is to express an opinion on the
fairness of financial statements, not to make an
appraisal of the financial information shown in an
entitys financial statements.
33. How are managements responsibility and the
auditors responsibility represented in the auditors
report?
Managements Auditors
Responsibility Responsibility
a. Implicitly Implicitly
b. Implicitly Explicitly
c. Explicitly Implicitly
d. Explicitly Explicitly

34. In which of the following circumstance would an


auditor most likely add an Emphasis of Matter paragraph
to the auditors report while expressing an unmodified
opinion?
a. There is a substantial doubt about the entitys
ability to continue as a going concern.
b. Managements estimates of the effects of future
vents are unreasonable.
c. No depreciation has been provided in the financial
statements.
d. Certain transactions cannot be tested because of
managements records retention policy.

According to the standard, the auditor should modify


the auditors report by adding an emphasis of matter
paragraph to highlight a material matter regarding a
going concern problem.
PSA 570 (Going Concern) gives the following example of
such a paragraph:
Without qualifying our opinion, we draw attention to
Note X in the financial statements which indicates
that the Company incurred a net loss of Pxxx during
the year ended December 31, 20x1 and as of that date,
the Companys current liabilities exceeded its total
assets by Pxxx. These conditions, along with other
matters as set forth in Note X, indicate the existence
of a material uncertainty which may cast significant
doubt about the Companys ability to continue as a
going concern.
Answer B is incorrect because financial statements
containing unreasonable managements estimates are not
fairly presented and will preclude the auditor from
expressing an unmodified opinion.
Answer C is incorrect because an auditor who is asked
to report on one financial statement may appropriately
express an unmodified opinion without adding an
Emphasis of Matter paragraph in the audit report.
Answer D is incorrect because a scope limitation will
require a qualified opinion or a disclaimer of
opinion.

35. An emphasis of matter paragraph of an auditors


report describes an uncertainty as follows:
Without qualifying our opinion, we draw attention
to Note X to the financial statements. The Company
is the defendant in a lawsuit alleging infringement
of certain patent rights and claiming royalties and
punitive damages. The Company has filed a counter
action and preliminary hearings and discovery
proceedings on both actions are in progress. The
ultimate outcome of the matter cannot presently be
determined and no provision for any liability that
may result has been made in the financial
statements.
What type of opinion should the auditor express
under these circumstances?
a. Unmodified
b. except for qualified
c. subject to qualified
d. Disclaimer of opinion

The addition of an Emphasis of Matter paragraph because


of a significant uncertainty (other than a going concern
problem) does not affect the auditors opinion. As
indicated above, the Emphasis of Matter paragraph refers
to the fact that the auditors opinion is not qualified.
Answer B is incorrect because an uncertainty does not
necessarily lead to a qualified opinion.
Answer C is incorrect because the use of the qualifying
phrase subject to is not allowed.
Answer D is incorrect because a disclaimer of opinion
would be appropriate in extreme cases, such as situations
involving multiple uncertainties that are significant to
the financial statements.

36. An auditors responsibility to express an opinion


on the financial statements is
a. Implicitly represented in the auditors report
b. Explicitly represented in the Auditors
Responsibility section of the auditors report.
c. Explicitly represented in the Managements
Responsibility paragraph of the auditors report.
d. Explicitly represented in the opinion paragraph of
the auditors report.
The Auditors Responsibility section begins with
Our responsibility is to express an opinion on these
financial statements based on our audit.
37. The existence of audit risk is recognized by the
statement in the auditors report that the auditor
a. Is responsible for expressing an opinion on the
financial statements, which are the responsibility
of management.
b. Realizes some matters, either individually or in
the aggregate, are important while other matters
are not important.
c. Obtains reasonable assurance about whether the
financial statements are free from material
misstatement.
d. Assess the accounting principles used and also
evaluates the overall financial statement
presentation.

Audit risk is the risk that the auditor may give an


inappropriate opinion on financial statements that are
materially misstated. The existence of audit risk is
recognized by the statement in the auditors report
that the auditor obtained reasonable (not absolute)
assurance about whether the financial statements are
free of material misstatement.
Answer A, B and D are incorrect because they dont
pertain to audit risk.

38. Which of the following statements is a basic


element of the auditors report?
a. The auditor is responsible for the preparation and
fair presentation of the financial statements.
b. The financial statements are consistent with those
of the prior period.
c. An audit involves performing procedures to obtain
audit evidence about the amounts and disclosures in
the financial statements.
d. The disclosures provide reasonable assurance that
the financial statements are free of material
misstatement.

39. Which paragraphs of an auditors report on


financial statements should refer to Philippine Financial
40.
Reporting Standards?
a. Introductory and Opinion
b. Auditors Responsibility and Managements
Responsibility
c. Introductory and Auditors Responsibility
d. Managements Responsibility and Opinion
40.An independent auditor discovers that a payroll
supervisor of the company being audited has misappropriated
P250,000. The companys total assets and income before tax
are P350 million and P75 million, respectively. Assuming no
other issues affect the report, the auditors report will
most likely contain a/an
e. Unmodified opinion
f. Disclaimer of opinion
g. Adverse opinion
h. Scope qualification

As long as the misappropriation is accounted for


properly, an unmodified opinion will be appropriate
because the financial statements will be fairly
presented in accordance with Philippine Financial
Reporting Standards. Moreover, the amount of
misappropriation is immaterial relative to the
companys assets and income.

41. A client makes test counts on the basis of a


statistical an. The auditor observes such counts as are
deemed necessary and is able to become satisfied as to
the reliability of the clients procedures. In reporting
on the results of the audit, the auditor
a. Must qualify the opinion if the inventories were
material
b. Can express an unmodified opinion
c. Must comment in an Emphasis of Matter paragraph as
to the inability to observe year-end inventories.
d. Is required to disclaim an opinion if the
inventories were material.

The auditor may express an unmodified opinion and


the audit report need not refer to the failure to
observe year-end physical cunt and to the
alternative procedures applied.
Answer A is incorrect because a qualified opinion
is inappropriate if there is no significant scope
limitation and the financial statements are fairly
presented.
Answer C is incorrect because a comment in an
Emphasis of Matter paragraph concerning the
omission of a procedure is not required if the
auditor has become satisfied by performing
alternative procedures.
Answer D is incorrect because, given no significant
scope limitation, a disclaimer of opinion is
unnecessary.

42. A note to the financial statements of the ABC Bank


indicates that all of the records relating to the banks
business operations are stored on magnetic disks, and
that no emergency backup systems or duplicate disks are
stored because the bank and its auditors consider the
occurrence of a catastrophe to be remote. Based upon this
note, the auditors report should express
a. A qualified opinion
b. An unmodified opinion
c. An adverse opinion
d. A subject to opinion

The banks failure to have backup records does not affect


the fairness of the financial statements. Hence, an
unmodified opinion is appropriate.

43. An auditor who uses the work of an expert may refer


to the auditors expert in the auditors report if the
a. Expert is employed by the entity
b. Experts work provides the auditor greater
assurance of reliability
c. Auditor expresses a qualified opinion or an adverse
opinion related to the work of the expert
d. Auditor indicates a division of responsibility
related to the work of the expert

Par.A42 of PSA 620 (Using the Work of an Auditors


Expert) states, It may be appropriate in some
circumstances to refer to the auditors expert in an
auditors report containing a modified opinion to explain
the nature of the modification.in such circumstances, the
auditor may need the permission o the auditors expert
before making such a reference.
Answer A is incorrect because an expert employed by the
entity is a managements expert, not an auditors expert.
Answer B is incorrect because, although the work of an
expert is expected to provide greater assurance of
reliability, reference to the auditors experts work is
justified only when the opinion as a result of the
experts work.
Answer D is incorrect because PSA 620 provides that if
the auditor makes reference to the work of an auditors
expert in the auditors report when such reference is
relevant to an understanding of a modification to the
auditors opinion, the auditor shall indicate in the
auditors report that such reference does not reduce
his/her responsibility for the opinion.

44. When would the auditor refer to the work of an appraiser


in the auditors report?
a. An adverse opinion is expressed based on a difference
of opinion between the client and the outside appraiser
as to the value of certain assets.
b. A disclaimer of opinion is expressed because of a
scope limitation imposed on the auditor by the appraiser
c. a qualified opinion is expressed because of a matter
unrelated to the work of the appraiser
d. An unmodified opinion expressed and an Emphasis of
Matter paragraph is added to disclose the use of the
appraisers work.
According to PSA 620, the auditor may refer to the
auditors expert if a modified report is to be issued as
a result of the experts work.
Answer B is incorrect because the appraiser is not in a
position to limit the scope of the auditors work since
the auditor can engage another appraiser
Answer C is incorrect because the auditors qualification
must be related to the experts work.
Answer D is incorrect because the auditor should not
refer to the work of an expert if an unmodified report is
to be issued.

45.A modified opinion on the financial statements is necessary


when
I. The auditor concludes, based on the audit evidence
obtained, that the financial statements as a whole are
not free from material misstatement.
II. The auditor is unable to obtain sufficient evidence
to conclude that the financial statements as a whole are
free from material misstatement.

a. I only
b. II only
c. Either I or II
d. Neither I nor II

TRUE OR FALSE
1. PSA 700 (Forming an Opinion and Reporting on Financial
Statements) requires that the audit report must be titled
and that the title must include the word independent.
2. An auditor may be unable to express an unmodified opinion
if an immaterial departure from PFRS is present in the
financial statements.
3. If financial statements fail to disclose a material fact,
t auditors may disclose the information in an emphasis of
matter paragraph and issue an unmodified opinion on the
statements.
4. The opening paragraph of the standard unmodified audit
report states that the audit is designed to obtain
reasonable assurance whether the statements are free of
material misstatement.
5. The clients management is responsible for the fairness
of the representations made in financial statements.
6. The audit report date on an unmodified report indicates
the last day of the auditors responsibility for the
review of significant events that occurred subsequent to
the date of the financial statements.
7. When the audited financial statements of the prior year
are presented together with those of the current year,
the continuing auditors report should cover only the
current year.
8. If the financial statements contain a material departure
from PFRS, the auditor usually should no issue an
unmodified opinion.
9. The introductory paragraph of the unmodified audit report
states that the financial statements are the
responsibility of management.
10. If the auditor believes that there is minimal
likelihood that resolution of an uncertainty will have a
material effect on the financial statements, the auditor
would issue an except for qualified opinion.
11. When there is a significant question about a
companys ability to continue as a going concern, the
report issued is usually modified with an emphasis of
matter paragraph.
12. If the statement of financial position of a company
is dated December 31, 2015,the audit report is dated
February 12, 2016, and both are released on February 25,
2016, this indicates that the auditor has searched for
subsequent events occurred up to February 12, 2016.
13. The adverse effects of events causing an auditor to
believe there is a substantial doubt about an entitys
ability to continue as a going concern would likely be
mitigated by evidence relating to the feasibility of
plans to purchase leased equipment at less than market
value.

KEY ANSWERS
1. B
2. D
3. A
4. D
5. D
6. D
7. B
8. B
9. B
10. C
11. B
12. A
13. B
14. B
15. B
16. B
17. C
18. A
19. C
20. D
21. A
22. C
23. A
24. B
25. B
26. D
27. A
28. D
29. C
30. C
31. A
32. B
33. D
34. A
35. A
36. B
37. C
38. C
39. D
40. A
41. B
42. B
43. C
44. A
45. C
46. B
47. C
48. D
49. A
50. D
51. B
52. C
53. C
54. D
55. C
56. B
57. A
58. D
59. A
60. B
61. D
62. A
63. B
64. C
65. A
66. C
67. C
68. A
69. D
70. A
71. D
72. B
73. D
74. A
75. D
76. B
77. C
78. B
79. D
80. A
81. B
82. D
83. C
84. A
85. D
86. C
87. D
88. D
89. A
90. C

TRUE or FALSE
1. True
2. False
3. False
4. False
5. True
6. True
7. False
8. True
9. False
10. False
11. False
12. True
13. False
14. False
15. False
16. False
17. False
18. False
19. False
20. False
21. False
22. True
23. True
24. True
25. True

46. Which of the following terms is used in the standard to


describe the effects on the financial statements of
misstatements or the possible effects on the financial
statements, if any, that are undetected due to an inability to
obtain sufficient appropriate audit evidence?
A. Persuasive
B. Pervasive
C. Material
D. Extensive

According to the standard, pervasive effects on the financial


statements are those that, in the auditors judgment:
1) Are not confined to specific elements, accounts or items
of the financial statements;
2) If so confined, represent or could represent a
substantial proportion of the financial statements; or
3) In relation of disclosures, are fundamental to users
understanding of the financial statements.

47. The auditor shall express an adverse opinion when


A. The auditor, having obtained sufficient appropriate audit
evidence, concludes the misstatements, individually or in
the aggregate, are material, but not persuasive, to the
financial statements.
B. The auditor is unable to obtain sufficient appropriate
audit evidence on which to base the opinion, but the
auditor concludes that the possible effects on the
financial statements of undetected misstatements, if any,
could be material but not persuasive.
C. The auditor, having obtained sufficient appropriate audit
evidence, concludes that misstatements, individually or in
the aggregate, are both material and pervasive to the
financial statements.
D. The auditor is unable to obtain sufficient appropriate
audit evidence on which to base the opinion, and the
auditor concludes that the possible effects on the
financial statements of undetected misstatements, if any,
could be both material and pervasive.

48. A limitation on the scope of the audit may arise from


I. Circumstances beyond the control of the entity.
II. Circumstances relating to the nature and timing of the
auditors work.
III. Limitations imposed by management.

A. I and II only
B. II and III only
C. I and III only
D. I,II, and III

49. The auditors inability to obtain sufficient appropriate


audit evidence (limitation on scope of the audit) arising from
circumstances beyond the control of the entity include when
A. The entitys accounting records have been destroyed.
B. Management prevents the auditor from observing the
counting of the physical inventory.
C. The auditor determines that performing substantive
procedures alone is not sufficient, but the entitys
controls are not effective.
D. The timing of the auditors appointment is such that the
auditor is unable to observe the counting of the physical
inventories.

50. A paragraph included in the auditors report that refers to


a matter appropriately presented or disclosed in the financial
statements that, in the auditor/s judgment, is of such importance
that it is fundamental to users understand of the financial
statements is called
A. Explanatory paragraph
B. Other Matter paragraph
C. Basis for Modified Opinion paragraph
D. Emphasis of Matter paragraph

The following are example of circumstances where the auditor


may consider it necessary to include an Emphasis of Matter
paragraph;
An uncertainty relating to the future outcome of
exceptional litigation or regulatory action.
Early application (where permitted) of a new accounting
standard that has a pervasive effect on the financial
statements in advance of its effective date.
A major catastrophe that has, had, or continues to have,
a significant effect on the entitys financial position.

51. When the auditor includes an Emphasis of Matter paragraph


in the auditors unmodified report, the auditor shall
A. Include it immediately before the Opinion paragraph.
B. Use the heading Emphasis of Matter or other appropriate
heading.
C. Indicate that the authors opinion is modified in respect
of the matter emphasized.
D. Include it immediately after the Managements
Responsibility paragraph.

52. Which of the following should be included in the opinion


paragraph when an author expresses a qualified opinion?
Subject to With Foregoing Explanation
A. Yes No
B. No Yes
C. No No
D. Yes Yes

Par. A22 of PSA 705 states, When the auditor expresses a


qualified opinion, it would not be appropriate to use phrases
such as with the foregoing explanation or subject to in the
opinion paragraph as these are not sufficiently clear or
forceful.

53. In which of the following circumstance would an auditor


usually choose between expressing a qualified opinion or
disclaiming an opinion?
A. Departure from the requirements of the applicable
financial reporting framework.
B. Unreasonable justification for a change in accounting
principle.
C. Inability to obtain sufficient appropriate audit evidence.
D. Inadequate disclosure of accounting policies.
A limitation on the scope of the auditors work whether
imposed by the client or by circumstances such as the inadequacy
of the accounting records, the timing of the auditors work, or
the inability to obtain sufficient appropriate audit evidence
may require expression of a qualified opinion or a disclaimer
of opinion.

54. An auditor decides to express a qualified opinion on an


entitys financial statements because a major inadequacy in its
computerized accounting records prevents the auditor from
applying necessary procedures. The opinion paragraph of the
auditors report should state that the qualification pertains
to
A. A client-imposed scope limitation
B. A departure from PSAs.
C. Inadequate disclosure of necessary information.
D. The possible effects on the financial statements.

When there is a limitation on the scope of the auditors


work that requires expression of a qualified opinion or a
disclaimer of opinion, the auditors report should describe the
limitation and indicate the possible adjustments to the
financial statements that might have been determined to be
necessary had the limitation not existed.
Answer A is incorrect because the opinion paragraph should
state that the qualification pertains to possible effect on the
financial statements and not to the scope limitation itself.
Answer B is incorrect because the audit was conducted in
accordance with PSAs.
Answer C is incorrect because the issue is the auditors
failure to apply necessary procedures and not inadequate
disclosure.

55. Sam, CPA, was engaged to audit the financial statements of


Mantha Corp. after its fiscal year had ended. The timing of
Sams appointment as auditor and the start of field work made
confirmation of accounts receivable by direct communication with
the debtors ineffective. However, Sam applied other procedures
and was satisfied as to the reasonableness of the account
balances. Sams auditors report most likely contained a/an
A. Qualified opinion because of a scope limitation.
B. Qualified opinion because of a departure from PSAs.
C. Unmodified opinion.
D. Unmodified opinion with an Emphasis of Matter paragraph.
By applying alternative procedures, the CPA was satisfied
about the reasonableness of the account balances. Hence, no scope
limitation exists and the CPA may express an unmodified opinion.
The auditors report need not refer to the omission of the
confirmation procedures or the performance of alternative
procedures.
Answers A and B are incorrect because no scope limitation
exists and the CPA has not departed from the PSAs.
Answers D is incorrect because there is no basis given to
add emphasis of matter paragraph in the auditors report.

56. In which of the following situations would an auditor


ordinarily choose between expressing a qualified opinion or an
adverse opinion?
A. The auditor wishes to emphasize an unusually important
subsequent event.
B. The financial statements fail to disclose information that
is required by Philippine Financial Reporting Standards.
C. Events disclosed in the financial statements cause the
auditor to have substantial doubt about the entitys
ability to continue as a going concern.
D. The auditor did not observe the entitys physical inventory
and is unable to become satisfied as to its balance by
other auditing procedures.

A disagreement with management concerning the


acceptability of the accounting policies selected, the method
of their application, or the adequacy of financial statement
disclosures may lead to either a qualified or an adverse opinion.
Answer A is incorrect because the auditor may include an
Emphasis of Matter paragraph in his/her unmodified auditors
report to emphasize an unusually important subsequent event.
Answer C is incorrect because the auditor, without
qualifying the opinion, should add an Emphasis of Matter
paragraph to highlight a material matter regarding a going
concern problem.
Answer D is incorrect because a scope limitation does not
lead to an adverse opinion.

57. Under which of the following circumstances would a disclaimer


of opinion not be appropriate?
A. The financial statements fail to contain adequate
disclosure concerning related party transactions.
B. The auditor is engaged after fiscal year-end and is unable
to observe the physical inventories or apply alternative
procedures to verify their balances.
C. The auditor is unable to determine the amounts associated
with fraud committed by the clients management.
D. The client refuses to permit its attorney to furnish
information requested in a letter of audit inquiry.

A disclaimer of opinion should be expressed when the


possible effect of a limitation on the scope of the auditors
work is so material and pervasive that the auditor has not been
able to obtain sufficient appropriate audit evidence and,
accordingly, is unable to express an opinion on the financial
statements. Inadequate disclosure concerning related party
transactions is not a scope of limitation.

58. When a publicly held company refuses to include in its


audited financial statements any of the segment information that
the auditor believes is required, the auditor should express
a/an
A. Disclaimer of opinion because of the significant scope
limitation.
B. Adverse opinion because of a significant uncertainty.
C. Unmodified opinion with an Emphasis of Matter paragraph
emphasizing the matter.
D. Qualified opinion because of inadequate disclosure.

The auditor should express either a qualified or an adverse


opinion if material information that the auditor believes is
required to be disclosed is not disclosed in the financial
statements.
Answer A is incorrect because inadequate disclosure is a
departure from PFRS, not a scope limitation.
Answer B is incorrect because the auditor should consider
adding an Emphasis of Matter paragraph if there is a significant
uncertainty. However, this does not ordinarily result in an
opinion modification.
Answer C is incorrect because inadequate disclosure leads
to either a qualified opinion or an adverse opinion.

59. When a client will not permit inquiry of outside legal


counsel, the audit report will ordinarily contain a/an
A. Disclaimer of opinion.
B. Adverse opinion.
C. Subject to qualified opinion.
D. Unmodified opinion with an Emphasis of Matter paragraph.

A limitation on the scope of auditors work, whether


imposed by the client of by circumstances, will lead to a
qualified opinion or a disclaimer of opinion.
Answer B is incorrect because a scope limitation will never
lead to an adverse opinion.
Answer C is incorrect because the standard clearly states
that a qualified opinion should be expressed as being except
for the effects of the matter to which the qualification
relates.
Answer D is incorrect because a scope limitation results
in a qualified opinion or a disclaimer of opinion.

60. Which of the following phrases would an auditor most likely


include in the auditors report when expressing a qualified
opinion because of inadequate disclosure.
A. Do not present fairly in all material respects.
B. Except for the omission of the information included in the
Basis for Qualified Opinion paragraph.
C. With the foregoing explanation of these omitted
procedures.
D. Subject to the departure from PFRS, as described above.

Inadequate disclosure, if material to the financial


statements taken as a whole, is a departure from PFRS and will
lead to either a qualified or an adverse opinion.
An auditors report qualified for inadequate disclosure
should include an emphasis of matter paragraph preceding the
opinion paragraph that explains the reason for the
qualification.
The opinion paragraph states, In our opinion, except for
the omission of the information included in the Basis for
Qualified Opinion paragraph. The financial statements present
fairly
Answer A is incorrect because the phrase do not present
fairly is used when the auditor expresses an adverse opinion.
Answer C and D are incorrect because the phrases with the
foregoing explanation and subject to should not be used when
expressing qualified opinion.
61. An auditors report includes the following statement: In
our opinion, because of the effects of the matters discussed
in the Basis for Adverse Opinion paragraph, the financial
statements do not present fairly, in all material respects,
the financial position of ABC Company as of December 31, 20X1,
and of its financial performance and its cash flows for the
year then ended in accordance with Philippine Financial
Reporting Standard. This auditors report contains a/an
A. Adverse opinion with an Emphasis of Matter paragraph.
B. Adverse opinion with an Other Matter paragraph.
C. Adverse opinion with an Explanatory paragraph.
D. Adverse opinion with a Basis for Adverse Opinion
paragraph.
A disagreement with management regarding the acceptability
of the accounting policies selected, the method of their
application or the adequacy of financial statement disclosures
could lead to a qualified or an adverse opinion.
The auditor should express an adverse opinion when the
effect of a disagreement is so material and pervasive to the
financial statements that a qualification is not adequate to
disclose the misleading or incomplete nature of the financial
statements. The statement quoted which includes the phrase do
not present fairly expresses an adverse opinion.

62. In which of the following circumstances would an auditor


be most likely to express an adverse opinion?
A. The financial statements are not in conformity with the
Philippine Accounting Standards (PAS) on capitalization
of leases.
B. Tests of controls show that the entitys internal control
is so poor that it cannot be relied upon.
C. The Chief Executive Officer refuses the auditor access to
minutes of board of directors meetings.
D. Information comes to the auditors attention that raises
substantial doubt about the entitys ability to continue
as a going concern.
The auditor should express an adverse opinion when the
financial statements are not presented fairly in accordance
with Philippine Financial Reporting Standards.
Answer B is incorrect because an ineffective internal
control system is not a basis for an adverse opinion.
Answer C is incorrect because a limitation on the scope of
the auditors work could lead to a qualified opinion or a
disclaimer of opinion.
Answer D is incorrect because significant doubt about the
entitys continued existence, if adequately disclosed in the
financial statements, does not result in the modification of
the opinion but requires an Emphasis of Matter paragraph
following the opinion paragraph.

63. An auditor should disclose the substantive reasons for


expressing an adverse opinion in the Basis for Adverse Opinion
paragraph.
A. Following the opinion paragraph.
B. Preceding the opinion paragraph.
C. Following the introductory paragraph.
D. Within the notes to the financial statements.
When the auditors report contains an adverse opinion, the
opinion paragraph refers to the Basis for Adverse Opinion
paragraph(s) preceding the opinion paragraph. This
paragraph should contain a clear description of the
substantive reasons for the adverse opinion and, unless
impracticable, a quantification of the possible effect(s)
on the financial statements.

64. When an auditor expresses an adverse opinion, the opinion


paragraph should include
A. The principal effects of the departure from the
requirements of the PFRS.
B. The substantive reasons for the financial statements
being misleading.
C. A direct reference to a separate paragraph disclosing the
basis for the opinion.
D. A description of the uncertainty or scope limitation that
prevents an unmodified opinion.
When the auditor expresses an adverse opinion, the opinion
paragraph of the auditors report should include a direct
reference to the Basis for Adverse Opinion paragraph(s)
preceding the opinion paragraph that clearly states all the
substantive reasons for the adverse opinion.
Answers A and B are incorrect because the principal effects
of the departure from the requirements of the PFRS and all
the substantive reasons for the adverse opinion are
disclosed in the Basis for Modified Opinion paragraph(s).
Answer D is incorrect because an uncertainty or a scope
limitation does not lead to an adverse opinion.
65. There are two broad financial reporting frameworks for
comparatives: the corresponding figures and the comparative
financial statements. Which of the following statements is
correct concerning these reporting frameworks?
A. Under the corresponding figures framework, the
corresponding figures for the prior period(s) are
integral part of the current period financial statements.
B. Under the corresponding figures framework, the
corresponding figures for the prior period(s) are
considered separate financial statements.
C. Under the comparative financial statements framework, the
comparative financial statements for the prior period(s)
are intended to be read in conjunction with the amounts
and other disclosures relating to the current period.
D. Under the comparative financial statements framework, the
amounts and other disclosures for the prior period(s)
form part of the current period financial statements.
Under PSA 710 (Comparative Information Corresponding
Figures and Comparative Financial Statements), the two
financial reporting frameworks are:
Corresponding figures where amounts and other disclosures
for the preceding period are included as part of the
current period financial statements, and are intended to be
read in relation to the amounts and other disclosures
relating to the current period.
These corresponding figures are not presented as complete
financial statements capable of standing alone, but are an
integral part of the current period financial statements
intended to be read only in relationship to the current
period figures.
Comparative financial statements where amounts and other
disclosures for the preceding period are included for
comparison with the financial statements of the current
period, but do not form part of the current period
financial statements, the comparative financial statements
are considered separate financial statements.

66. The following statements relate to the auditors reporting


responsibilities regarding comparative information. Which
is/are correct?
I. For corresponding figures, the auditors report only
refers to the financial statements of the current period.
II. For comparative financial statements, the auditors
report refers to each period that financial statements
are presented.

A. I only.
B. II only.
C. Both I and II.
D. Neither I nor II.
Because the corresponding figures are included as part of
the current period financial statements and are intended to
be read in conjunction with the amounts and other
disclosures relating to the current period, the auditors
report only refers to the financial statements of the
current period.
But because the comparative financial statements are
considered separate financial statements, the auditors
report refers to each period that financial statements are
presented.

67. PSA 710 states that the extent of audit procedures


performed on the corresponding figures is significantly less
that for the audit of the current period figures. The
auditors procedures are ordinarily limited to ensuring that
the corresponding figures have been correctly reported and are
appropriately classified, the auditor should assess whether
I. Accounting policies used for the corresponding figures
are consistent with those of the current period or
whether appropriate adjustment and/or disclosures have
been made.
II. Corresponding figures agree with the amounts and other
disclosures presented in the prior period or whether
appropriate adjustments and/or disclosures have been
made.

A. I only.
B. II only.
C. Both I and II.
D. Neither I nor II.

68. In which of the following circumstances would an auditors


report least likely include specific reference to the
corresponding figures?
A. When the auditors report on the prior period, as
previously issued, included a modified opinion and the
matter which gave rise to the modification is resolved
and properly dealt with the financial statements.
B. When the auditors report on the prior period, as
previously issued, included a modified opinion and the
matter which gave rise to the modification is unresolved,
and results in a modification of the auditors report
regarding the current period figures.
C. When the auditors report on the prior period, as
previously issued, included a modified opinion and the
matter which gave rise to the modification is unresolved,
but does not result in a modification of the auditors
report regarding the current period figures.
D. When the auditors report on the prior period financial
statements containing a material misstatement included an
unmodified opinion and the prior period financial
statements have not been revised and reissued, and the
corresponding figures have not been properly restated
and/or appropriate disclosures have not been made.
Par. A3 of PSA 710 states, When the auditors report on
the prior period, as previously issued, included a
qualified opinion, a disclaimer of opinion, or an adverse
opinion is resolved and properly accounted for or disclosed
in the financial statements in accordance with the
applicable financial reporting framework, the auditors
opinion on the current period need not refer to the
previous modification.

69. According to PSA 710, the incoming auditor may refer to


the predecessor auditors report on the corresponding figures
in the incoming auditors report for the current period. The
incoming auditors report should indicate
I. That the financial statements of the prior period were
audited by the predecessor auditor.
II. The type of opinion issued by the predecessor auditor.
III. The date of the predecessor auditors report.

A. I and II only.
B. II and III only.
C. I and III only.
D. I, II and III.

70. When the prior period financial statements are not


audited, the incoming auditor should state in the auditors
report that
I. The corresponding figures are unaudited.
II. The incoming auditor is not required to perform
procedures regarding opening balances of the current
period.

A. I only.
B. II only.
C. Both I and II.
D. Neither I nor II.
PSA 710 states, If the prior period financial statements
were not audited, the auditor shall state in an Other
Matter paragraph in the auditors report that the
corresponding figures are unaudited. Such a statement does
not, however, relieve the auditor of the requirement to
obtain sufficient appropriate audit evidence that the
opening balances do not contain misstatements that
materially affect the current periods financial
statements.

71. J, CPA, audited ABC Companys prior-year financial


statements. These statements are presented with those of the
current year for comparative purposes without Js auditors
report, which expressed a qualified opinion. In drafting the
current years auditors report, S, CPA, the incoming auditor,
should
I. Not name J as the predecessor auditor.
II. Indicate the type of opinion issued by J.
III. Indicate the substantive reasons for Js qualification.
IV. Indicate the date of Js auditors report.

A. I, II and IV only.
B. II, III and IV only.
C. I, II and III only.
D. I, II, III and IV.
PSA 710 states that when the financial statements of the
prior period were audited by another auditor,
a) the predecessor auditor may reissue the audit report
on the prior period with the incoming auditor only
reporting on the current period; or
b) the incoming auditors report shall indicate:
i. that the financial statements of the prior period
were audited by the predecessor auditor;
ii. the type of opinion issued by the predecessor
auditor and, if the opinion was modified, the
reasons therefor;
iii. the date of that report.
It should be emphasized that the requirement in (b) above
is to state the prior years financial statements were
audited by the predecessor auditor, not to name the
predecessor auditor.
72. When comparative financial statements are presented, the
auditors opinion on the financial statements shall refer to
A. Current period only.
B. Current period and those of the other periods presented.
C. Current and immediately preceding period only.
D. Periods presented plus on preceding period.
PSA 710 provides that when comparative financial statements
are presented, the auditors opinion shall refer to each
period for which financial statements are presented an on
which the audit opinion is expressed.

73. Comparative financial statements include in the financial


statements of the prior year that were audited by a
predecessor auditor whose report is not presented, if the
predecessors report was qualified, the incoming auditor
should
A. Express an opinion only on the current years statements
and make no reference to the prior years statements.
B. Issue an updated comparative audit report indicating the
division of responsibility.
C. Request the client to reissue the predecessors report on
the prior years statements.
D. Indicate the substantive reasons for the qualification in
the predecessor auditors opinion.
When the predecessor auditors report is not presented, the
incoming auditors report shall indicate:
i. that the financial statements of the prior period were
audited by the predecessor auditor;
ii. the type of opinion issued by the predecessor auditor
and, if the opinion was modified, the substantive
reasons therefor; and
iii. the date of the report.
Answer A is incorrect because, when comparative financial
statements are presented, the auditors opinion (or
disclaimer of opinion) should be expressed individually on
the financial statements of each period presented.
Answer B is incorrect because the standard does not require
indication of divided responsibility.
Answer C is incorrect because, although the predecessor
auditor may reissue the audit report on the prior period
with the incoming auditor reporting only on the current
period, the situation in the question is that the
predecessor auditors report is not presented.
74. Comparative financial statements include the prior years
statements that were audited by a predecessor auditor whose
report is not presented. If the predecessors report was
unmodified, the incoming auditor should
A. Indicate in the auditors report that the predecessor
auditor expressed an unmodified opinion.
B. Express an opinion on the current years statements alone
and make no reference to the prior years statements.
C. Obtain a letter of representations from the predecessor
auditor concerning any matters that might affect the
incoming auditors opinion.
D. Request the predecessor auditor to reissue the prior
years report.
Answer B is incorrect because, according to PSA 710, when
comparative financial statements are presented, the auditor
should issue a report in which the comparative financial
statements are specifically identified. The auditors
opinion (or disclaimer of opinion) should be expressed
individually on the financial statements of each period
presented.
Answer C is incorrect because the incoming auditor is
required to communicate with the predecessor auditor prior
to accepting the audit engagement.
Answer D is incorrect because, although the predecessor
auditors report may be reissued, the question states that
the predecessor auditors report is not presented.

75. The predecessor auditor, who is satisfied after properly


communicating with the incoming auditor, has reissued his/her
auditors report on prior year financial statements. The
predecessor auditors report should
A. Refer to the work of the incoming auditor in the scope
and opinion paragraphs.
B. Refer to the report of the incoming auditor only in the
scope paragraph.
C. Refer to both the work and the report of the incoming
auditor only in the opinion paragraph.
D. Not refer to the report of the work of the incoming
auditor.
PSA 710 states that when the financial statements of the
prior period were audited by another auditor, the
predecessor auditor may reissue that audit report on the
prior period with the incoming auditor only reporting on
the current period. The reissued report should not refer to
the report or work of the incoming auditor.
Answers A, B, and C are incorrect because the reissued
report should not refer to the report or the work of the
incoming auditor.

76. An auditor expressed a qualified opinion on the prior


years financial statements because of a lack of adequate
disclosure. These financial statements are properly restated
in the current year and presented in comparative form with the
current years financial statements. The auditors updated
report on the prior years financial statements should
A. Make no reference to the type of opinion expressed on the
prior years financial statements.
B. Express an unmodified opinion on the restated financial
statements of the prior year.
C. Be accompanied by the auditors original report on the
prior years financial statements.
D. Continue to express a qualified opinion on the prior
years financial statements.
During the course of the audit of the current period, the
auditor may become aware of circumstances or events that
materially affect the financial statements of a prior
period. For example, if the auditor expressed a qualified
opinion on the prior year financial statements because of
the departure form PFRS and such statements are restated in
the current period to conform with the requirements of the
PFRS, the continuing auditors updated report should refer
to the restatement and express an unmodified opinion.
Moreover, the report should contain an Other Matter
paragraph to disclose the substantive reasons for the
different opinion.
Answer A is incorrect because the auditors report should
contain an Other Matter paragraph to disclose the
substantive reasons for the change and the type of opinion
previously expressed by the auditor.
Answer C is incorrect because the original auditors report
should not be reissued.
Answer D is incorrect because the auditor should express a
different opinion.

77. In performing the audit on the current period financial


statements, the incoming auditor, in certain unusual
circumstances, may become aware of material misstatement that
affects the prior period financial statements on which the
predecessor auditor had previously reported without
modification. In these circumstances, the incoming auditor
should.
I. Discuss the matter with management.
II. Request that the predecessor auditor be informed.

A. I only.
B. II only.
C. Both I and II.
D. Neither I nor II.
According to PSA 710, If the auditor concludes that a
material misstatement exists that affects the prior period
financial statements on which the predecessor auditor had
previously reported without modification, the auditor shall
communicate the misstatement with the appropriate level of
management and those charged with governance and request
that the predecessor auditor be informed. If the prior
period financial statements are amended, and the
predecessor auditor agrees to issue a new auditors report
on the amended financial statements of the prior period,
the auditor shall report only on the current period.

78. The following statements relate to unaudited prior year


financial statements that are presented in comparative form
with audited current year financial statements. Which is
incorrect?
A. The incoming auditor should state in the auditors report
that the comparative financial statements are unaudited.
B. The incoming auditor need not perform audit procedures
regarding opening balances of the current period.
C. Clear disclosure in the financial statements that the
comparative financial statements are unaudited is
encouraged.
D. In situations where the incoming auditor identifies that
the prior year unaudited figures are materially
misstated, the auditor should request management to
revise the prior years figures or if management refuses
to do so, appropriately modify the report.
According to PSA 710, a statement in the auditors report
that the prior period financial statements are unaudited
does not relieve the auditor of the requirement to carry
out appropriate procedures regarding opening balances of
the current period.
79. A client is presenting comparative (two-year) financial
statements. Which of the following is correct concerning
reporting responsibilities of a continuing auditor?
A. The auditor may issue either one audit report on both
presented year, or two audit reports, one on each year.
B. The auditor should issue on audit report, but only on the
most recent year.
C. The auditor should issue two audit reports, one on each
year.
D. The auditor should issue on audit report that is on both
presented years.
When comparative financial statements are presented, the
auditor should issue a report in which the comparatives are
specifically identified because the auditors opinion is
expressed individually on the financial statements of each
period presented.

80. When audited financial statements are presented in a


document (e.g., annual report) containing other information,
the auditor
A. Should read the other information to consider whether it
is inconsistent with the audited financial statements.
B. Has no responsibility for the other information because
it is not part of the basic financial statements.
C. Has an obligation to perform auditing procedures to
corroborate the other information.
D. Is required to express a qualified opinion if the other
information has a material misstatement of fact.

81. An auditor concludes that there is a material


inconsistency in the other information in an annual report to
shareholders containing audited financial statements. If the
auditor concludes that the financial statements do not require
revision, but the client refuses to revise or eliminate the
material inconsistency, the auditor may
A. Disclaim an opinion on the financial statements after
explaining the material inconsistency in an emphasis of
matter paragraph.
B. Revise the auditors report to include an Other Matter
paragraph describing the material inconsistency.
C. Express a qualified opinion after discussing the matter
with the clients directors.
D. Consider the matter closed because the other information
is not in the audited statements.
82. PSA 720 (The Auditors Responsibility in Relation to Other
Information in Documents Containing Audited Financial
Statements) states, If, on reading the other information, the
auditor identifies a material inconsistency, the auditor
should determine whether the audited financial statements or
the other information needs to be revised. What type of
opinion should be expressed if the client refuses to make the
necessary revision in the financial statements?
A. Disclaimer of opinion.
B. Qualified opinion or disclaimer of opinion.
C. Unmodified opinion with an Other Matter paragraph
describing the material inconsistency.
D. Qualified or adverse opinion.

83. Which of the following phrases would an auditor most likely


include in the auditors report when expressing a qualified
opinion because of inadequate disclosure?
A. Subject to the departure from generally accepted
accounting principles, as described above.
B. With the foregoing explanation of these omitted
disclosures.
C. Except for the omission of the information discussed in
the Basis for Qualified opinion paragraph.
D. Do not present fairly in all material respects.

The Qualified Opinion paragraph states. In our opinion, except


for the omission of information discussed in the Basis for
Qualified Opinion paragraph, the financial statement present
fairly.
Answers A and B are incorrect. When the auditor expresses a
qualified opinion. It would not be appropriate to use phrases
such as with the foregoing explanation or subject to in the
opinion paragraph as these are not sufficiently clear or
forceful. (PSA 705 (Revised and Redrafted), par. A22)
Answer D is incorrect because the phrase do not present fairly
is used when expressing an adverse opinion.

84. ABC CO.s financial statements adequately disclose


uncertainties that concern future events, the outcome of which
are not susceptible to reasonable estimation. The auditors
report should include
A. An unmodified opinion
B. A subject to qualified opinion
C. An except for qualified opinion
D. An adverse opinion

AASC ALERT
Series 2 of 2011
reporting on information required by BIR Revenue Regulation
(RR; 15-2010)
85. RR 15-2010 requires disclosure of specific information on
various taxes in the Notes to Financial statements that will
accompany the income tax return to be filed with the BIR. These
disclosure requirements
A. Form part of the disclosure requirements under PFRS
B. Form part of the disclosure requirements under PFRS for
SMEs
C. Form part of the disclosure requirements under PFRS and
PFRS for SMEs
D. Do not form part of the disclosure requirements under PFRS
and other Philippine financial reporting frameworks such
as PFRS for SMEs

86. Under the PSAs, the tax information required by RR 15-2010


that is presented as part of the notes to financial statements
is considered
A. Significant information
B. Other information
C. Supplementary information
D. Material information

Supplementary information, as defined in the glossary of the


PSAs is information that is presented together with the
financial statements that is not required by the applicable
financial reporting framework used to prepare the financial
statements, normally presented in either supplementary schedules
or as additional notes.
The supplementary information may be different from the
financial statements by:

Disclosing the supplementary information in a separate


note at the end of the required notes in the financial
statements which is clearly labeled as Supplementary
information.
Removing any cross references from the financial
statements to the separate note on supplementary
information.

87. The tax information disclosures under RR 15-2010 are required


to be presented
A. Only in the consolidated financial statements.
B. Both in the consolidated financial statements and the
separate financial statements of the parent company and
its subsidiaries.
C. Only in the consolidated financial statements and the
separate financial statements of the parent company.
D. Only in the separate financial statements of the parent
company and the separate financial statements of the
subsidiaries.

The consolidated financial statements are not the financial


statements to be submitted to the BIR accompanying the income
tax returns. Hence, the required disclosures under RR 15-2010
are to be presented in the separate financial statements only.
88. Where the supplementary information required by RR 15-2010
is not clearly differentiated from the financial statements,
such supplementary information shall be
A. Addressed in a separate section in the auditors report
under the sub-title Report on Other Legal and Regulatory
Requirements.
B. Addressed in a separate section in the auditors report
under the sub-title Report on the Supplementary
Information Required under RR 15-2010.
C. Addressed in an emphasis-of-matter paragraph in the
auditors report.
D. Covered by the auditors opinion on the financial
statements.

89. When the supplementary information required under RR 15-2010


is not presented
A. The auditors Report on the Financial Statements. Would
not be affected because such supplementary information is
not part of the basic financial statements.
B. The auditor is precluded from expressing an opinion on the
financial statements.
C. The auditors report should contain a qualified opinion.
D. The auditors report should contain an adverse opinion.

90. The supplementary information required under RR 15-2010 is


clearly differentiated from the audited financial statements.
How would the Report on the Supplementary Information be
affected if the auditors Report on the financial statements
contains an adverse opinion?
A. The auditor should express a qualified opinion on the
supplementary information.
B. To attain consistency in reporting, the auditor should
express an adverse opinion on the supplementary
information.
C. The auditor is precluded from expressing an opinion on the
supplementary information.
D. The auditor should express an unmodified opinion on the
supplementary information when his/her Report on the
Financial Statements contains either an adverse opinion
or a disclaimer of opinion.

The auditor is precluded from expressing an opinion on the


supplementary information when his/her Report on the Financial
Statements contains either an adverse opinion or a disclaimer
of opinion.
The auditors report on the supplementary information should
state that, Because of the significance of the matter described
in the Basis for Adverse (or Disclaimer of) opinion paragraph,
it is inappropriate to and we do not express as opinion on the
information referred to above.
TRUE OR FALSE
1. PSA 700 (Forming an Opinion and Reporting on Financial
Statements) requires that the audit report must be titled and
that the title must include the word independent.
2. An auditor may be unable to express an unmodified opinion if
an immaterial departure from PFRS is present in the financial
statements.
3. If financial statements fail to disclose a material fact, the
auditors may disclose the information in an emphasis of matter
paragraph and issue an unmodified opinion on the statements.
4. The opening paragraph of the standard unmodified audit report
states that the audit is designed to obtain reasonable assurance
whether the statement are free of material misstatement.
5. The clients management is responsible for the fairness of
the representation made in financial statements.
6. The audit report date on an unmodified report indicates the
last day of the auditors responsibility for the review of
significant events that occurred subsequent to the date of the
financial statements.
7. When the audited financial statements of the prior year are
presented together with those of the current year, the continuing
auditors report should cover only the current year.
8. If financial statements contain a material departure from
PFRS, the auditor usually should not issue an unmodified opinion.
9. The introductory paragraph of the unmodified audit report
states that the financial statements are the responsibility of
management.
10. If the auditor believes that there is minimal likelihood
that resolution of an uncertainty will have a material effect
on the financial statements, the auditor would issue an except
for qualified opinion.
11. When there is a significant question about a companys
ability to continue as a going concern, the report issued is
usually modified with an emphasis of matter paragraph.
12. If the statement of financial position of a company is dated
December 31, 2015, the audit report is dated February 12, 2016,
and both are released on February 25, 2016, this indicated that
the auditor has searched for subsequent events that occurred up
to February 12, 2016.
13. The adverse effects of events causing an auditor to believe
there is substantial doubt about an entitys ability to continue
as a going concern would likely be mitigated by evidence relating
to the feasibility of plans to purchase leased equipment at less
than market value.
14. The auditor is required to issue a disclaimer of opinion in
the event of a going concern problem.
15. Audit report issued for financial statements of a company
should refer to PFRS in the auditors responsibility paragraph.
16. The audit report is normally addressed to the companys
chief executive officer or chief financial officer.
17. The auditor is required to issue a disclaimer of opinion in
the event of a material uncertainty.
18. The financial statements most commonly audited by external
auditors are the statement of financial position, the statement
of comprehensive income, and the statement of changes in retained
earnings.
19. A client-imposed scope limitation will generally result in
an adverse opinion.
20. Whenever an auditor issues a qualified report, he or she
must us the term with the foregoing explanation in the opinion
paragraph.
21. When an auditor does not confirm material accounts
receivable, but is satisfied by the application of alternative
auditing procedures, he normally should issue an unmodified
opinion but disclose elsewhere in the report this departure from
customary procedure.
22. When an auditor expresses an adverse opinion, the opinion
paragraph should include a direct reference to a separate
paragraph disclosing the basis for the opinion.
23. Materiality is an essential consideration in determining the
appropriate type of report under a given set of circumstances.
24. An auditor would issue an adverse opinion if the statements
taken as a whole do not fairly present the financial position
and financial performance of the company.
25. The predecessor auditor, after properly communicating with
the successor auditor, has reissued a report because the audit
client desires comparative financial statements. The predecessor
auditors report should make no reference to the report or the
work of the successor auditor.

KEY ANSWERS
1. B 19. C 37. C 55. C 73. D
2. D 21. D 38. C 56. B 74. A
3. A 21. A 39. D 57. A 75. D
4. D 22. C 40. A 58. D 76. B
5. D 23. A 41. B 59. A 77. C
6. D 24. B 42. B 60. B 78. B
7. B 25. B 43. C 61. D 79. D
8. B 26. D 44. A 62. A 80. A
9. B 27. A 45. C 63. B 81. B
10. C 28. D 46. B 64. C 82. D
11. B 29. C 47. C 65. A 83. C
12. A 30. C 48. D 66. C 84. A
13. B 31. A 49. A 67. C 85. D
14. B 32. B 50. D 68. A 86. C
15. B 33. D 51. B 69. D 87. D
16. B 34. A 52. C 70. A 88. D
17. C 35. A 53. C 71. D 89. A
18. A 36. B 54. D 72. B 90. C

TRUE OR FALSE
1. True 6. True 11. False 16. False 21. False
2. False 7. False 12. True 17. False 22. True
3. False 8. True 13. False 18. False 23. True
4. False 9. False 14. False 19. False 24. True
5. True 10. False 15. False 20. False 25. True

Chapter 11
Other reporting responsibilities
PSA 800
Special Considerations Audits of Financial Statements Prepared
in Accordance with Special Purpose Frameworks
1. Financial statement prepared in accordance with financial
reporting framework designed to meet the financial information
needs of specific users are referred to as
A. Special purpose financial statements
B. Special purpose framework
C. General purpose financial statements
D. Specific purpose financial statements
Special purpose financial statements are financial statements
prepared in accordance with a special purpose framework.
Special purpose framework is a financial reporting framework
designed to meet the financial information needs of specific
users.
2. The following are examples of special purpose frameworks,
except
A. A tax basis of accounting for a set of financial statements
that accompany an entitys tax return.
B. The cash receipt and disbursements basis of accounting for
cash flow information that an entity may be requested to
prepare for creditors.
C. Philippines Financial Reporting Standards (PFRS) promulgated
by the Financial Reporting Standards Council (FRSC).
D. The financial reporting provisions of a contract (for
example, a financing agreements).

The following financial reporting frameworks are often


identified as the applicable financial reporting framework in
legislative and regulatory requirements governing the
preparation of general purpose financial statements.

International Financial Reporting Standards (IFRS)


promulgated by the International Accounting Standards
Board (IASB)
International Public Sector Accounting Standards (IPSAs)
promulgated by the International Public Sector Accounting
Standards Board.
Philippine Financial Reporting Standards (PFRS)
promulgated by the Financial Reporting Standards Council
(FRSC).
3. An auditors report on financial statements prepared in
accordance with the financial reporting provisions of a contract
(that is, a special purpose framework) to comply with the
provisions of that contract should include all of the following,
except
A. An opinion as to whether the financial statements are
presented fairly, in all material aspects, in accordance with
the financial reporting provisions of the contract.
B. A statement that indicated the basis of accounting used.
C. An opinion as to whether the basis of accounting used is
appropriate under the circumstances.
D. Reference to the note to the financial statements that
describes the basis of presentation.

The auditors report on financial statements prepared in


accordance with the financial reporting provisions of a contract
should include a statement that indicated the basis of accounting
used or should refer to the note to the financial statements
giving that information.
The opinion should state whether the financial statements are
prepared, in all material respects, in accordance with the
identified basis of accounting. There is no requirement to
express an opinion on the propriety of the basis of accounting
used.
The following is an example of an auditors report on the
complete set of financial statements prepared in accordance with
the financial reporting provisions of a contract:
We have audited the accompanying financial statements of ABC
Company, which compromise the statement of financial potion as
at December 31, 20x1, and the statement of comprehensive income,
statement of changes in equity and statement of cash flows for
the year ended, and a summary of significant accounting policies
and other explanatory information. The financial statements have
been prepared by management based on the financial reporting
provisions of Section Z of the contract dated January 1, 20x1.
4. When an auditor reports on financial statements prepared on
an entitys income tax basis, the auditors report should
A. State the basis of presentation of the financial statements.
B. Disclaim an opinion on whether the statements were examined
in accordance with Philippine Standards on Auditing (PSAs).
C. Not express an opinion on whether the statements are presented
in accordance with the tax basis of accounting used.
D. Include an explanation of how the results of operations differ
from the cash receipt and disbursements basis of accounting.

The auditors report should state the basis of accounting used


or should refer to the note to the financial statements giving
that information.
Answer B is incorrect because the audit should be conducted in
accordance with Philippine Standards of Auditing (PSAs).

Answer C is incorrect because the auditor should express an


opinion on whether the financial statements are prepared, in all
material respects, in accordance with the identified basis of
accounting.
Answer D is incorrect because the auditors report only refers
to the note to the financial statements that explains the basis
of accounting used.

5. An auditor is reporting in a statement of cash receipt and


disbursements. This statement is best referred to in the
opinion paragraph by which of the following descriptions?
A. Results of operations arising from cash transactions.
B. Cash receipts and disbursements
C. Income statement resulting from cash transactions.
D. Statement of cash flows.
The opinion paragraph of a report on a statement of cash receipts
and disbursements states, In our opinion, the financial
statement presents fairly, in all material respects, the cash
receipt and disbursements of ABC Company for the year ended
December 31, 20x1 in accordance with the cash receipts and
disbursements basis of accounting described in Note X.
6. In an audit of special purpose financial statements, the
auditor shall obtain an understanding of
I. The purpose for which the financial statements are
prepared.
II. The intended users.
III. The steps taken by management to determine that the
applicable financial reporting framework is acceptable in the
circumstances.
A. I only
B. II and III only
C. I and II only
D. I, II, and III

7. An auditors report on financial statements prepared on the


cash receipts and disbursements basis of accounting should
include all of the following, except
A. A statement that the audit was conducted in accordance with
Philippine Standards on Auditing.
B. A reference to the note to the financial statements that
describes the cash receipts and disbursements basis of
accounting.
C. A statement that the cash receipts and disbursements basis of
accounting is not a comprehensive basis of accounting.
D. An opinion as to whether the financial statements are
presented fairly, in all material respects, in accordance
with the cash receipts and disbursements basis of accounting.
Answer A, B and D are incorrect because the statement that the
audit was conducted in accordance with PSAs, a reference to the
note to the financial statements that describes the basis of
presentation and an opinion as to whether the statements are
fairly presented should be included in the auditors report.

PSA 805
Special Considerations Audits of Single Financial Statements
and Specific Elements, Accounts or Items of a Financial
Statement
8. A CPA is permitted to accept a separate engagement (not in
conjunction with an audit of financial statements) to audit an
entitys
Schedule of Schedule of
Accounts receivable Profit Participation
A. Yes No
B. No Yes
C. Yes Yes
D. No No
An audit engagement to express an opinion on one or more
components of a financial statement (for example, accounts
receivable, inventory, or a schedule of profit participation)
may be undertaken as a separate engagement or in conjunction
with an audit of the entitys financial statements.
9. Which of the following statements is correct with respect
to an auditors report expressing an opinion on a specific
element on a financial statement?
A. The auditor who was expressed an adverse opinion on the
financial statements as a whole can never express an
unmodified opinion on a specific element in theses financial
statements.
B. The materiality determined for a specific element of a
financial statement may be lower than the maturity determined
for the entitys complete set of financial statements.
C. Such a report can only be issued if the auditor is also
engaged to audit the entire set of financial statements.
D. The attention devoted to the specific element is usually less
than it would be if the financial statements as a whole were
audited.

Answer A is incorrect because when an adverse opinion or a


disclaimer of opinion has been expressed on the entire set of
financial statements, it would be appropriate to express an
unmodified opinion on the specific element only if:
The auditor is not prohibited by law or regulation from
doing so;
That opinion is expressed in an auditors report that is
not published together with the auditors report
containing the adverse opinion or disclaimer of opinion;
and
The specific element does not constitute a major portion
of the entitys complete set of financial statements.

Answer C is incorrect because an engagement to audit one or more


components of financial statements may be undertaken as a
separate engagement or in conjunction with an audit of the
entitys financial statements.

Answer D is incorrect because a special purpose audit engagement


to report on one or more components of financial statements will
ordinarily be more extensive than if the same component(s) were
to be audited in connection with a report on the financial
statements taken as a whole.

10. An auditor may express an opinion on an entitys accounts


receivable balance even if the auditor has disclaimed an
opinion on the financial statements taken as a whole provided
the
A. Report on the accounts receivable is presented separately
from the disclaimer of opinion on the financial statements.
B. Auditor also reports on the current asset portion of the
entitys statement of financial position.
C. Use of the report on the accounts receivable is restricted.
D. Report on the accounts receivable discloses the reason for
the disclaimer of opinion on the financial statements.
The standard states that when an adverse opinion or disclaimer
of opinion on the entire financial statements has been expressed,
the auditor should report on elements of the financial statements
only if those elements are not so extensive as to constitute a
major portion of the financial statements.
Because an engagement to audit a financial statement element
does not result in a report on the financial statements taken
as a whole, a separate report should be presented containing the
auditors opinion on the financial statement element audited.
Answer B is incorrect because a report may be presented on one
or more financial statement elements audited.
Answer C is incorrect because the standards does not require
restriction on the distribution of the report on the financial
statement element audited.
Answer D is incorrect because a separate report on the element
audited should be presented. Moreover, the report need not
describe the reason for the disclaimer of opinion on the
financial statements.
11. When an auditor is requested to express an opinion on the
rental and royalty income of an entity, the auditor may
A. Accept the engagement provided the auditor will comply with
relevant ethical requirements, including those pertaining to
independence, relating to financial statement audit
engagement and all PSAs relevant to the audit.
B. Accept the engagement provided distribution of the auditors
report is limited to the entitys management.
C. Not accept the engagement unless also engaged to audit the
full financial statements of the entity.
D. Not accept the engagement because to do so would be tantamount
to agreeing to express a piecemeal opinion.

Answer B is incorrect because the standard does not require


restrictions on the distribution and use of the auditors report.
Answer C is incorrect because this engagement may be undertaken
as separate engagement or in conjunction with an audit of the
entitys financial statements.
Answer D is incorrect because the auditor is allowed to express
an opinion on one or more financial statement elements if he/she
has not expressed an adverse opinion or disclaimer of opinion
on the financial statements taken as a whole or if the elements
to be reported on are not so extensive as to constitute a major
portion of the financial statements.
12. The following statements are ordinarily included in the
separate auditors report on an entitys compliance with
contractual agreements, except
A. We conduct our audit in accordance with Philippine Standards
on Auditing.
B. In our opinion, the financial statements of the Company are
presented fairly, in all material respects, in accordance
with Philippine Financial Reporting Standards.
C. An audit involves performing procedures to obtain audit
evidence about the amounts and disclosures in the financial
statements.
D. We believe that the audit evidence we have obtained is
sufficient and appropriate to provide a basis for our
opinion.

The auditors report should include a statement that, in the


auditors opinion, the entity has complied with particular
provisions of the contractual agreement.

PSA 810
Engagements to Report on Summary Financial Statements
13. An auditor may report on summary financial statements that
are derived from complete audited financial statements if the
A. Auditor indicated whether the information in the summary
financial statements is consistent with the audited financial
statements from which it was derived.
B. Summary financial statements are distributed only to
management and the board of directors.
C. Auditor describes the additional review procedures performed
on the summary financial statements.
D. Summary financial statements are presented in comparative
form with the prior years summarized financial statements.
According to the standard, the auditors report on the summary
financial statements shall include the following basic elements:
1. A title clearly indicating it is the report of an independent
auditor;
2. An addressee;
3. An introductory paragraph that;
a. Identifies the summary financial statements on which
the auditor is reporting, including the title of each
statement included in the summary financial
statements;
b. Identifies the audited financial statements;
c. Refers to the auditors report on the audited
financial statements, the date of that report, and
the fact that an unmodified opinion is expressed on
the audited financial statements;
d. If the date of the auditors report on the summary
financial statements is later than the date of the
auditors report on the audited financial
statements, states that the summary financial
statements and the audited financial statements do
not reflect the effects of events that occurred
subsequent to the date of the auditors report on
the audited financial statements; and
e. A statement indicating that the summary financial
statements do not contain all the disclosure required
by the financial reporting framework applied in the
preparation of the audited financial statements, and
that reading the summary financial statements is not
a substitute for reading the audited financial
statements.
4. A description is managements responsibility for the summary
financial statements, explaining the management is
responsible for the preparation of the summary financial
statements in accordance with the applied criteria.
5. A statement that the auditor is responsible for expressing an
opinion on the summary financial statements based on the
procedures required by the PSA.
6. A paragraph clearly expressing an opinion.
7. The auditors signature.
8. The date of auditors report.
9. The auditors address.
14. In the auditors report on summary financial statements that
are derived from an entitys audited financial statements, a CPA
should indicate that the
A. CPA has audited and expressed an opinion on the complete
financial statements.
B. CPA expressed limited assurance that the financial statements
are presented in accordance with PRFS.
C. Summary financial statements are not fairly presented in all
material respects.
D. Summary financial statements are prepared in accordance with
special purpose financial reporting framework.
PSRE 2400
Engagements to Review Financial Statements
15. In a review engagement, the practitioner and the client
should agree on the terms of the engagement. The agreed terms
would be recorded in an engagement letter or other suitable form
such as a contract. The engagement letter should include all of
the following, except
A. A provision that the engagement cannot be relied upon to
disclose errors, fraud, or illegal acts.
B. A provision that any errors, fraud, or noncompliance with
laws and regulations that come to the practitioners
attention need not be reported.
C. A sample of the report expected to be rendered.
D. The objective of the service to be performed.
Although a review engagement cannot be relied upon to disclose
whether fraud, errors, or noncompliance with the laws and
regulations exist, the engagement letter should indicate to the
client that the practitioner will inform management or the board
of directors of any material matters that will come to the
practitioners attention.
Matters that would be included in the engagement letter include:
The objective of the service to be performed.

Managements responsibility for the financial statements.

The scope of the review, including reference to PSRE 2400.

Unrestricted access to whatever records, documentation,


and other information requested in connection with the
review.

A sample of the report expected to be rendered.

The fact that the engagement cannot be relied upon to


disclose errors. Illegal acts, or other irregularities,
for example, fraud or defalcations that may exist.

A statement that the audit is not to be performed and that


an audit opinion will not be rendered. To emphasize that
point and to avoid confusion, the practitioner may also
consider pointing out that a review engagement will not
satisfy any statutory or third party requirements for an
audit.
16. Which of the following procedures should a practitioner
perform during an engagement to review an entitys financial
statements?
A. Examining cash disbursements in the subsequent period for
unrecorded liabilities.
B. Sending bank confirmation letters to the entitys financial
institutions.
C. Obtaining a client representation letter from members of
management.
D. Communicating material internal control weaknesses during the
assessment of control risk.
A review of financial statements consists principally of
inquiries and analytical procedures. When considered
appropriate, the practitioner should obtain written
representations from the members of management who have
reasonability for financial and accounting matters.
Answers A, B and D are incorrect because examining subsequent
cash disbursements, sending confirmation requests to financial
institutions, and communicating material weaknesses in internal
control are performed in an audit.
17. Which of the following procedures is a practitioner lease
likely to perform during a review engagement?
A. Comparing the financial statements with anticipated results
in budgets and forecasts.
B. Studying the relationships of financial statement elements
expected to conform to predictable patterns.
C. Inquiring of management about actions taken at the board of
directors meetings.
D. Observing the safeguards over access to and use of assets and
records.
18. Which of the following inquiries or analytical procedures
ordinarily is performed in an engagement to review an entitys
financial statements?
A. Inquiries concerning the entitys procedures for recording
and summarizing transactions.
B. Analytical procedures designed to test the accounting records
by obtaining corroborating evidential matter.
C. Analytical procedures designed to test managements
assertions regarding continued existence.
D. Inquiries of the entitys attorney concerning contingent
liabilities.
19. Which of the following procedures is usually performed by
the practitioner in a review engagement of an entity?
A. Sending a letter of inquiry to the entitys lawyer.
B. Confirming a significant percentage of receivables by direct
communication with debtors.
C. Comparing the financial statements with statements for
comparable prior periods.
D. Communicating material weaknesses in the design or
implementation of internal control.
20. Which of the following procedures most likely would not be
included in a review engagement of an entity?
A. Assessing control risk.
B. Considering whether the financial statements are in
accordance with PFRS.
C. Obtaining a management representation letter.
D. Inquiring about subsequent events.
21. When providing limited assurance that the financial
statements of an entity require no material modifications to be
in accordance with PFRS, the practitioner should
A. Confirm with the entitys lawyer that material loss
contingencies are disclosed.
B. Understand the accounting principles of the industry in which
the entity operates.
C. Develop audit programs to determine whether the entitys
financial statements are fairly presented.
D. Assess the risk that a material misstatement could occur in
a financial statement assertion.
22. Which of the following would not be included in a
practitioners report based upon a review of an entitys
financial statements?
A. A statement that the financial statements are the
responsibility of the companys management.
B. A statement describing the principal procedures performed.
C. A statement that the review was conducted in accordance with
PSA
D. A statement describing the practitioners conclusions based
upon the results of the review.
The review report includes a statement that the review was
conducted in accordance with the Philippine Standard on Review
Engagement 2400. Moreover, the report indicates that a review
consists principally of inquiries and analytical procedures and
provides less assurance than an audit.
23. The date of the review report should
A. Not be earlier than the date on which the financial statements
were approved by management.
B. Be earlier than the date on which the financial statements
were approved by management.
C. Coincide with the date of the financial statements.
D. Not be later than the date of the financial statements.
According to the standard, a practitioner should date the review
report as of the date the review is completed, which includes
performing procedures relating to events occurring up to the
date of the report. However, since the practitioners
responsibility is to report on the financial statements as
prepared and presented by the management, the practitioner
should not date the report earlier than the date on which the
financial statements were approved by management.
24. During an engagement to review the financial statements of
an entity, a practitioner becomes aware of a material departure
from PFRS. If the practitioner decides to modify the review
report because management will not revise the financial
statements, the practitioner should
A. Express negative assurance on accounting principle not
conforming with PFRS.
B. Express positive assurance on accounting principles
conforming with PFRS.
C. Express a qualified opinion.
D. Express a qualification of the negative assurance provided or
give an adverse statement that the financial statements are
not presented fairly, in all material respects, in accordance
with PFRS.
If the practitioner becomes aware of a material departure from
PFRS, he/she should describe the nature of the departure in a
separate paragraph, including, unless impracticable, a
quantification of the possible effect(s) on the financial
statements. In addition, the practitioner should either:
a. Express a qualification of the negative assurance
provided in the report; or
b. When the effect of the departure is so material and
pervasive that a qualification is believed to be
inadequate to disclose the misleading or incomplete
nature of the financial statements, give an adverse
statement that the financial statements are not
presented fairly, in all material respects, in
accordance with PFRS.
Answer A is incorrect because the practitioner should provide
negative assurance when his/her review procedures did not
disclose material departure from PFRS. The review report should
state that nothing has come to the practitioners attention based
on the review that caused the practitioner to believe the
financial statements are not presented fairly, in all material
respects, in accordance with PFRS.
25. If there has been a significant limitation on the
practitioners review of an entitys financial statements, the
practitioner should describe the limitation and
I. Express q qualification of the negative assurance.
II. Provide no assurance.
A. I only
B. II only
C. Either I or II
D.Neither I or II
The standard provides that if there has been a material scope
limitation, the review report should describe the limitation and
either:
a. Express a qualification of the negative assurance provided
regarding the possible adjustments to the financial
statements that might have been determined to be necessary
had the limitation not existed; or
b. When the possible effect of the limitation is so
significant and pervasive that the practitioner concludes
that no level of assurance can be provided, not provide
any assurance.
26. For the purpose of expressing negative assurance in the
review report, the practitioner should obtain sufficient
appropriate evidence primarily through
A. Inquiry and confirmation
B. Analytical procedures and substantive tests of details of
transactions and account balances
C. Confirmation and tests of controls
D. Inquiry and analytical procedures
27. PSRE 2400 (Engagements to Review Financial Statements), as
amended by the AASC in February 2008, applies to
A. Reviews of any historical financial information of an audit
client.
B. Reviews of any historical financial information by a
practitioner other than the entitys auditor.
C. Review of historical financial or other information by a
practitioner other than the entitys auditor.
D. Review of historical financial or other information of an
audit client.
PSRE 2400 (Engagement to Review Financial Statements) and PSRE
2410 (Review of Interim Financial Information Performed by the
Independent Auditor of the Entity) were amended by the AASC in
February 2008. The objective of the amendments made is to clarify
to which engagements each of the standards is to be applied. The
effect of the amendments is summarized as follows:

PSRE 2400 applies to reviews of historical financial


information by a practitioner other than entitys auditor.

PSRE 2410 applies to reviews of historical financial


information by the entitys auditor.

Reviews of other historical information fall under PSAE


3000 (Revised). Assurance Engagements other than Audits or
Reviews of Historical Financial Information.
28. A practitioners review of an entitys financial statements
does not provide assurance that he/she will become aware of all
significant matters that would be disclosed in an audit. However,
if the practitioner has become aware that information coming to
his/her attention may be materially misstated, the practitioner
should
A. Carry out additional or more extensive procedures as are
necessary to achieve limited assurance.
B. Withdraw immediately from the engagement.
C. Perform a complete audit and issue modified auditors
report.
D. Downgrade the engagement to a compilation and issue the
appropriate report.
According to PSRE 2400, if the practitioner has reason to believe
that the information subject to review may be materially
misstated, he/she should carry out additional or more extensive
procedures as are necessary to be able to express negative
assurance or to confirm that a modified report is required.
PSRE 2410
Review of Interim Financial Information Performed by the
independent Auditor of the Entity

29. Which of the following statements concerning objective of


an engagement to review interim financial information is
correct?
A. To obtain reasonable assurance that the interim financial
information is free from material misstatement.
B. To enable the auditor to express a conclusion whether, on
the basis of the review, anything has come to the auditors
attention that causes the auditor to believe that the
interim financial information is not prepared, in all
material respects, in accordance with an applicable
financial reporting framework.
C. To provide a basis of expressing an opinion whether the
interim financial information is presented fairly, in all
material respects, in accordance with an applicable
financial reporting framework.
D. The objective of a review of interim financial information
is similar to that of an audit conducted in accordance
with PSAs.
According to PSRE 2410 The objective of an engagement to review
interim financial information is to enable the auditor to express
a conclusion whether, on the basis of the review, anything has
come to the auditors attention that causes the auditor to
believe that the interim financial information is not prepared,
in all material respects, in accordance with an applicable
financial reporting framework.
Answer A is incorrect because a review of interim financial
information, in contrast to an audit, is not designed to obtain
reasonable assurance that the interim financial information is
free from material misstatement. A review consists of making
inquiries, primarily of persons responsible for financial and
accounting matters, and applying analytical and other review
procedures. It does not provide all of the evidence that would
be required in an audit.
Answer C is incorrect because a review of interim financial
information does not provide a basis for expressing an opinion
whether the financial information is presented fairly, in all
material respects, in accordance with an applicable financial
reporting framework.
Answer D is incorrect because the objective of a review of
interim financial information differs significantly from that
of an audit conducted in accordance with PSAs.
30. Which of the following procedures ordinarily should be
applied when an independent auditor conducts a review of interim
financial information of an entity?
A. Verify changes in key account balances.
B. Perform cut-off tests for cash receipts and disbursements.
C. Read the minutes of the board of directs meetings.
D. Inspect the open purchase order file.
A review of interim financial information includes reading the
minutes of meetings of shareholders, those charged with
governance, and other appropriate committees to identify matters
that may affect the interim financial information. It also
involves inquiring about matters dealt with at meetings for which
minutes are not available that may affect the interim financial
information.
Answers A, B, and D are incorrect because verifying changes in
key account balances, performing cut-offs test, and inspecting
open purchase order file are verification procedures ordinarily
performed in an audit.
31. An independent auditor who conducts a review of an entitys
interim financial information should have an understanding of
the entity and its environment, including its internal control,
as it relates to the preparation of both annual and interim
financial information. This enables the auditor to
I. Identify the types of potential misstatements and
consider the likelihood of their occurrence.
II. Select the inquiries, analytical and other review
procedures.
A. I only
B. II only
C. Both I and II
D. Neither I nor II
31. The following procedures are ordinarily performed in an
engagement to review interim financial information, except
A. Tests of the accounting records through inspection,
observation, or confirmation
B. Obtaining an understanding of the entity and its environment,
including its internal control, as it relates to the
preparation of both annual and interim financial information.
C. Inquiring of members of management responsible for financial
and accounting matters.
D. Communication with other auditors who are performing a review
of the interim financial information if the reporting
entitys significant components.
Procedures for performing a review of interim financial
information are ordinarily limited to making inquiries,
primarily of persons responsible for financial and accounting
matters, and applying analytical and other reviews procedures.
A review ordinarily does not require test of the accounting
records through inspection, observation, or confirmation.
Answer B, C and D are incorrect because they describe procedures
that are ordinarily performed in an engagement to review interim
financial information.
The following procedures are ordinarily performed in conducting
a review of interim financial information:

Reading the minutes of the meetings of shareholders, those


charged with governance, and other appropriate committees
to identify matters that may affect the interim financial
information, and inquiring about matters dealt with at
meetings for which minutes are not available that may
affect the interim financial information.

Considering the effect, if any, of matters giving rise to


modification of the audit or review report, accounting
adjustments or unadjusted misstatements at the time of the
previous audit or reviews.

Communicating, where appropriate, with other auditors who


are performing a review of the interim financial
information of the reporting entitys significant
components.

Inquiring of members of management responsible for


financial and accounting matters.

Applying analytical procedures to the interim financial


information designed to identify relationships and
individual items that appear to be unusual and that may
reflect a material misstatement in the interim financial
information.

Reading the interim financial information, and considering


whether anything has come to the auditors attention that
cause the auditor to believe that the interim financial
information is not prepared, in all material respects, in
accordance with applicable financial reporting framework.
PSRS 4400
Engagements on Agreed-upon Procedures
33. A report may be based upon applying agreed-upon procedures
to specified elements, accounts, or items of a financial
statement. The users of the report should participate in
establishing the procedures to be performed. If the auditor
cannot discuss the procedures with all the parties who will
receive the report, he/she may
I. Discuss the procedures to be applied with appropriate
representatives of the parties involved.
II. Review relevant correspondence from the parties
involved.
III. Distribute a draft of the type of report that will be
issued to the parties involved.

A. I and II only
B. I and III only
C. II and III only
D. I, II and III
PSRS 4400 (Engagements on Agreed-upon Procedures) states, In
certain circumstances, for example, when the procedures have
been agreed to between the regulator, industry representatives
and representatives of the accounting profession, the auditor
may not be able to discuss the procedures with all the parties
who will receive the report. In such cases, the auditor may
consider, for example, discussing the procedures to be applies
with appropriate representatives of the parties involved,
reviewing relevant correspondence from such parties or sending
them a draft of the type of report that will be issued.
34. The auditor may accept an engagement to perform specified
procedures on the specific subject matter of specified elements,
accounts, or items of a financial statement if
A. The report does not list the procedures performed.
B. The financial statements are prepared in accordance with
s special purpose framework.
C. Use of the report is restricted.
D. The auditor is also the entitys continuing auditor.
PSRS 4400 states that the report is restricted to those parties
that have agreed to the procedures to be performed since others,
unaware of the reasons for the procedures, may misinterpret the
results.
Answer A is incorrect because the report should include a listing
of the specific procedures performed.
Answer B is incorrect because the financial statements need not
be prepared in accordance with a special purpose framework.
Answer D is incorrect because the auditor need not be the
entitys continuing auditor.
35. The distribution of which of the following types of reports
is unrestricted?
A. Identification of the purpose for which the agreed-upon
procedures were performed.
B. An expression of positive assurance based on the specific
procedures performed.
C. A statement that the auditor is independent of the entity.
D. A general description of the procedures performed.
According to PSRS 4400, the report on an agreed-upon procedures
engagement needs to describe the purpose and the agreed-upon
procedures of the engagement in sufficient detail to enable the
users of the report to understand the nature and extent of the
work performed.
Answer B is incorrect because the report should include a
statement that the procedures performed do not constitute either
an audit or a review and, as such, no assurance is expressed.
Answer C is incorrect because the report should contain a
statement that the auditor is not independent of the entity if
such is the case.
Answer D is incorrect because the report should include a listing
of the specific procedures performed.
37. An agreed-upon procedures engagement may involve the auditor
in performing certain procedures concerning
I. Individual items of financial data.
II. A financial statement.
III. A complete set of financial statements.

A. I and II only
B. II and III only
C. I and III only
D. I, II, and III
PSRS 4400 (Engagements to Perform Agreed-upon Procedures
Regarding Financial Information) states, An engagement to
perform agreed-upon procedures may involve the auditor in
performing certain procedures concerning individual items of
financial data (for example, accounts payable, account
receivable, purchases from related parties and sales and profits
of a segment of an entity), a financial statement (for example,
a statement of financial position) or even a complete set of
financial statements.
38. Negative assurance may be expressed when an accountant is
engaged to report agreed-upon procedures to specified

I. Elements of a financial statement.


II. Accounts of a financial statement.
A. I only
B. II only
C. Both I and II
D. Neither I nor II
According to PSRS 4400, the objective of an agreed-upon
procedures engagement is for the auditor to carry out procedures
of an audit nature to which the auditor and the entity and any
appropriate third parties have agreed and to report on factual
findings. The accountant does not provide negative or other forms
of assurance. Users of the report assess for themselves the
procedures and findings of the accountant and draw their own
conclusions.
39. An accountant may accept an engagement to apply agreed-upon
procedures that are not sufficient to express an opinion on one
or more financial statements provided that
A. The accountant is also the entitys continuing auditor.
B. Distribution of the accountants report is restricted.
C. The financial statements are prepared in accordance with
a special purpose financial reporting framework.
D. The accountants report does not enumerate the procedures
performed.

An accountant may accept an agreed-upon procedures engagement


provided that the parties involved have a clear understanding
of the procedures to be performed. This is to prevent
misinterpretation of the results by those who are unaware of the
reasons for the procedures performed.
Answer A is incorrect because the accountant need not be the
entitys continuing auditor.
Answer B is incorrect because the financial statements need not
be prepared using the special purpose financial reporting
framework.
Answer D is incorrect because the accountants report should
include a listing of the specific procedures performed.
40. Which of the following is least likely to be included in an
agreed-upon procedures engagement report?
A. Identification of the purpose for which the agreed-upon
procedures were performed.
B. A summary of procedures performed.
C. Limited assurance on the information presented.
D. Use of the report is restricted.

The accountants report should include a statement that the


procedures performed do not constitute either an audit or a
review and, as such, no assurance is expressed.
According to PSRS 4400, the report of factual findings should
contain:
a. Title;
b. Addressee (ordinarily the client who engaged the auditor
to perform the agreed-upon procedures);
c. Identification of specific financial or non-financial
information to which the agreed-upon procedures have been
applied;
d. A statement that the procedures performed were those
agreed-upon with the recipient;
e. A statement that the engagement was performed in accordance
with the PSRS applicable to agreed-upon procedures
engagements;
f. A statement that the auditor is not independent if such is
the case;
g. Identification of the purpose for which the agreed-upon
procedures were performed;
h. A listing of the agreed-upon procedures performed;
i. A description of the auditors factual findings including
sufficient details of errors and exceptions found;
j. A statement that the procedures performed do not constitute
either an audit or a review and, as such, no assurance is
expressed.
k. A statement that had the auditor performed additional
procedures, an audit or a review, other matters might have
come to light that would have been reported;
l. A statement that the report is restricted to those parties
that have agreed to the procedures to be performed;
m. A statement (when applicable) that the report relates only
to the elements, accounts, items or financial and non-
financial information specified and that it does not extend
to the entitys financial statements taken as a whole;
n. Date of the report;
o. Auditors address; and
p. Auditors signature

PSRS 4410
Engagements to Compile Financial Information

41. When performing a compilation engagement, the accountant


is required to
A. Assess internal controls.
B. Make inquiries of management to assess the reliability
and completeness of the information provided.
C. Verify matters and explanations.
D. Obtain a general knowledge of the business and
operations of the entity

According to PSRS 4110 (Engagements to Compile Financial


Information), The accountant should obtain a general knowledge
of the business and operations of the entity and should be
familiar with the accounting principles and practices of the
industry in which the entity operates and with the form and
content of the financial information that is appropriate in the
circumstances.
The standard further provides that, The accountant ordinarily
obtains knowledge of these matters through experience with the
entity or inquiry of the entitys personnel.

PSRS 4110, par. 13 provides that the accountant is not ordinarily


required to:
a) make any inquiries of management to assess the reliability
and completeness of the information provided;
b) assess internal controls;
c) verify any matters; or
d) verify any explanations.

42. Independence is a requirement for which of the following


engagements?

Compilation Review Agreed-upon Procedures


A. No Yes No
B. No No No
C. Yes No Yes
D. Yes Yes Yes

Independence is not a requirement for compilation and agreed-


upon procedures engagements. However, where the accountant or
auditor is not independent, a statement to that effect would be
made in the report.

43. Which of the following require compliance with the


requirements of the Code of Ethics for Professional
accountants in the Philippines?

Compilation Review Agreed-upon Procedures


A. Yes Yes No
B. No No Yes
C. No No No
D. Yes Yes Yes

44. An accountant who performs a compilation engagement


A. Should read the compiled information and consider
whether it appears to be appropriate in form and free
from obvious material misstatements.
B. Should use his/her auditing expertise in testing the
assertions underlying the compiled financial
information.
C. Include in his/her report a listing of the specific
procedures performed.
D. Need not to obtain an acknowledgement from management
of its responsibility for the appropriate presentation
of financial information.

45. Each page of the financial information compiled by the


accountant should include the following reference, except
A. Unaudited
B. Compiled without Audit or Review
C. Refer to Compilation Report
D. Compiled, Negative Assurance Expressed

According to PSRS 4110 (Engagements to Compile Financial


Information), the financial information compiled by the
accountant should contain a reference such as:
Unaudited;
Compiled without Audit or Review; or
Refer to Compilation Report
on each page of the financial information or on the front of the
complete set of financial statements.

46. The objective of a compilation engagement is


A. For the accountant to use accounting expertise, as
opposed to auditing expertise, to collect, classify,
and summarize financial information.
B. For the auditor to carry out procedures of an audit
nature to which the auditor and the entity and any
appropriate third parties have agreed and to report on
factual findings.
C. To enable an auditor to state, on the basis of the
procedures which do not provide all the evidence that
would be required in an audit, anything has come to the
auditors attention that causes the auditor to believe
that the financial statements are not prepared, in all
material respects, in accordance with an identified
financial reporting framework.
D. For the auditor to provide a high, but not absolute,
level of assurance that the financial information is
free of material misstatement.

PSRS 4110 (Engagements to Compile Financial Information) states,


The objective of a compilation engagement is for the accountant
to use accounting expertise, as opposed to auditing expertise,
to collect, classify, and summarize financial information.
Answer B is incorrect because it describes the objective of an
agreed-upon procedures engagement.

Answer C is incorrect because it describes negative (or limited)


assurance provided in a review engagement.

Answer D is incorrect because an audit provides a high, but not


absolute level of assurance that the financial information is
free of material misstatement.

47. When compiling an entitys financial statements, an


accountant would be least likely to:
A. Obtain an acknowledgment from management of its
responsibility for the financial statements.
B. Perform analytical procedures designed to identify
relationships that appear to be unusual.
C. Plan the work.
D. Read compiled financial statements and consider whether
they appear to include adequate disclosures.
Analytical procedures are necessary in review and audit
engagements; not in compilation engagements.

Answer A is incorrect because, according to PSRS 4410, the


accountant should obtain an acknowledgement from management of
its responsibility for the appropriate presentation and of its
approval of the financial information.

Answer C is incorrect because the work should be planned to


ensure that an effective engagement will be performed.

Answer D is incorrect because the accountant should read the


compiled financial statements and consider they are free from
obvious material misstatements, including:
Mistakes in the application of PFRS.
Nondisclosure of PFRS and any known departures thereof.
Nondisclosure of any other significant matters of which
the accountant has become aware.

48. When compiling the financial statements of an entity, an


accountant should
A. Understand the accounting principles and practices of
the entitys industry.
B. Inquire of key personnel concerning related parties and
subsequent events.
C. Perform ratio analyses of the financial data of
comparable periods.
D. Review agreements with financial institutions for
restriction of cash balances.

PSRS 4410 states, The accountant should obtain general


knowledge of the business and operations of the entity and should
be familiar with the accounting principles and practices of the
industry in which the entity operates and with the form and
content of the financial information that is appropriate in the
circumstances.

Answer B and D are incorrect because inquiries concerning related


parties and subsequent events, and procedures to obtain
corroborating evidence about restriction on cash balances are
appropriate in an audit.

Answer C is incorrect because analytical procedures such as ratio


analyses are appropriate in review and audit engagements.

49. Which of the following should not be included in an


accountants report based upon the compilation of an
entitys financial statements?
A. A statement that a compilation of the companys
financial statements was made in accordance with the
PSA applicable to compilation engagements.
B. A statement that management is responsible for the
financial statements.
C. A statement that the accountant has not audited or
review the statements.
D. A statement that the accountant does not express an
opinion but provides only a negative assurance on the
statements.

The accountants report should indicate that since no audit or


review was performed, no assurance is expressed.

PSRS 4410 gives the following example of a compilation report:

We have compiled the accompanying financial statements of


ABC Company based on information you have provided. These
financial statements comprises the statement of financial
position of ABC Company as at December 31, 20X1, the
statement of comprehensive income, statement of changes in
equity and statement of cash flows for the year ended, and
a summary of significant accounting policies and other
explanatory information.

We performed this compilation engagement in accordance


with Philippine Standard on Related Services 4410,
Compilation Engagements.

We have applied our expertise in accounting and financial


reporting to assist you in the preparation and presentation
of these financial statements in accordance with
Philippine Financial Reporting Standards. We have compiled
relevant ethical requirements, including principles of
integrity, objectivity, professional competence and due
care.

These financial statements and the accuracy and


completeness of the information used to compile them are
your responsibility.
Since a compilation engagement is not an assurance
engagement, we are not required to verify the accuracy or
completeness of the information you provided to us to
compile these financial statements. Accordingly, we do not
express and audit opinion or a review conclusion on whether
these financial statements are prepared in accordance with
PFRS.

50. In performing a compilation of financial statements of an


entity, the accountant decides that modification of the
report is not adequate to indicate deficiencies in the
financial statements taken as a whole, and the client is
not willing to correct deficiencies. The accountant should
therefore
A. Express an adverse audit opinion.
B. Express a qualification of negative assurance.
C. Withdraw from the engagement.
D. Perform a review of the financial statements.
PSRS 4410 States that if the accountant becomes aware that
information supplied by management is incorrect, incomplete, or
otherwise unsatisfactory, he/she should request management to
provide additional information or correct that deficiencies. The
accountant should withdraw from the engagement if the management
refuses to do so.

Answers A and B are incorrect because the accountant should not


express any form of assurance on complied financial statements.

Answer D is incorrect because the accountant has no


responsibility to upgrade the engagement to a review.

PSAE 3400
The Examination of Prospective Financial Information

51. Prospective financial information means financial


information based on assumptions about events that may
occur in the future and possible actions by the entity. It
can be in the form of a projection, a forecast, or a
combination of both. A forecast
A. Presents estimates given one or more hypothetical
assumptions.
B. It is based on assumptions reflecting conditions
expected to exist and courses of action expected to be
taken.
C. Unlike a projection, may contain range.
D. Is based on the most conservative estimates.

According to PSAE (Philippine Standards on Assurance


Engagements) 3400 [The Examination of Prospective Financial
Information], a forecast means prospective financial
information prepared on the basis of assumptions as to future
events which management expects to take place and actions
management expects to take as of the date the information is
prepared (best-estimate assumptions).

A projection, as defined in the standard, means prospective


financial information prepared on the basis of:
a) Hypothetical assumption about future events and
management actions which are not necessarily expected
to take place, such as when some entities are in a
start-up phase or are considering a major change in the
nature of operations; or
b) A mixture of best-estimate and hypothetical
assumptions.

Answer A is incorrect because, as indicated above, a projection


(not a forecast) is based on hypothetical assumptions about
future events and management actions which are not necessarily
expected to take place.

Answer C is incorrect because both forecasts and projections can


be expressed in terms of a range.
Answer D is incorrect because a forecast is based on the entitys
best-estimate assumptions.

52. The party responsible for assumptions identified in the


preparation of prospective financial statements is usually
A. The clients management.
B. The clients independent auditor.
C. The reporting accountant.
D. A third-party lending institution.

According to PSAE 3400, management is responsible for the


preparation and presentation of prospective financial
information, including the identification and disclosure of the
assumptions on which it is based. The auditor examines and
reports on the prospective financial information to enhance its
credibility whether it is intended for use by third parties or
internal purposes.

Answers B, C, and D are incorrect because the party responsible


for assumptions identified in preparation of prospective
financial information is usually the entitys management.

53. Given one or more hypothetical assumptions, a responsible


party may prepare, to the best of its knowledge and belief,
an entitys expected financial position, result of
operations, and cash flows. Such prospective financial
statements are known as
A. Partial presentations
B. Financial projections
C. Financial forecast
D. Pro forma financial statements

Financial projections are prepared on the basis on hypothetical


assumptions which are not necessarily expected to take place.

54. An examination of a financial forecast is a professional


service that involves
A. Assuming responsibility to update management on key
events for one year after the reports date.
B. Compiling or assembling a financial forecast that is
based on managements assumptions.
C. Limiting the distribution of the accountants report to
management and board of directors.
D. Evaluating the preparation of a financial forecast and
the support underlying managements assumption.

Prospective financial information includes financial forecasts


and projections. The examination of prospective financial
information requires the accountant to obtain sufficient
appropriate evidence as to whether:
a) Managements best estimate assumptions on which the
prospective financial information is based are not
unreasonable and, in the case of hypothetical
assumption, such assumptions are consistent with the
purpose of the information;
b) The prospective financial information is properly
prepared on the basis of the assumptions.
c) The prospective financial information is properly
presented and all material assumptions are adequately
disclosed, including a clear indication as to whether
they are best-estimate assumptions or hypothetical
assumptions; and
d) The prospective financial information is prepared on a
consistent basis with historical financial statements,
using appropriate accounting principles.

Answer A is incorrect because the accountant does not have


responsibility as to update management on key events after the
reports date.

Answer B is incorrect because an examination of a financial


forecast entails evaluation of its preparation and the support
underlying managements assumptions.

Answer C is incorrect because there is no requirement to restrict


distribution of forecasts.

55. The accountant should not accept, or should withdraw from,


an engagement to examine prospective financial information
when
I. The assumptions are clearly unrealistic.
II. The accountant believes that the prospective financial
information will be inappropriate for its intended use.

A. I only
B. II only
C. Either I or II
D. Neither I nor II

PSAE 3400 states, The auditor should not accept, or should


withdraw from, an engagement when the assumptions are clearly
unrealistic or when the auditor believes that the prospective
financial information will be inappropriate for the intended
use.

56. When the examination of prospective financial information


is affected by conditions that preclude application of one
or more procedures considered necessary in the
circumstances, the auditor should withdraw from engagement
or
A. Disclaim the opinion
B. Express an adverse opinion
C. Express a qualified opinion
D. Issue an unmodified report
A limitation on the scope of the accountants examination of
prospective financial information may lead to either withdrawal
of the accountant from the engagement or disclaimer of opinion.

57. When the accountant believes that the presentation and


disclosure of the prospective financial information is no
adequate, the auditor should
I. Express a qualified or adverse opinion.
II. Withdraw from the engagement.

A. I only
B. II only
C. Either I or II
D. Neither I nor II

PSAE 3400 states, When the auditor believes that the


presentation and disclosure of the prospective financial
information is not adequate, the auditor should express a
qualified or adverse opinion in the report on the prospective
financial information, or withdraw from the engagement as
appropriate.

58. The report on examination of prospective financial


information should include
A. A statement that the accountant is responsible for the
prospective financial information, including the
assumptions on which it is based.
B. A statement of positive assurance as to whether the
assumptions provide a reasonable basis for the
prospective financial information.
C. Appropriate caveats concerning the achievability of the
results indicated by the prospective financial
information.
D. A statement that the examination was conducted in
accordance with PSAs.
According to PSAE 3400, the report on an examination of
prospective financial information should include the following:
a) Title;
b) Addressee;
c) Identification of the prospective financial information;
d) A reference to the Philippine Standard on Assurance
Engagements applicable to the examination of prospective
financial information;
e) A statement that management is responsible for the
prospective financial information including the
assumptions on which it is based;
f) When applicable, a reference to the purpose and/or
restricted distribution of the prospective financial
information;
g) A statement of negative assurance as to whether the
assumptions provide a reasonable basis for the prospective
financial information;
h) An opinion as to whether the prospective financial
information is properly prepared on the basis of the
assumptions and is presented in accordance with the
relevant financial reporting framework;
i) Appropriate caveats concerning the achievability of the
results indicated by the prospective financial
information;
j) Date of the report which should be the date the procedures
have been completed;
k) Auditors address; and
l) Signature.

59. Before accepting an engagement to examine the prospective


financial information, the auditor would consider
I. The intended use of the information.
II. The nature of assumptions.
III. The period covered by the information.

A. I only
B. II only
C. I and III only
D. I, II, and III

PSAE 3400 states that before accepting an engagement to examine


prospective financial information, the auditor would consider,
among other things:
The intended use of the information.
Whether the information will be general or limited
distribution.
The nature of the assumptions, that is, whether they are
best-estimate or hypothetical assumptions.
The elements to be included in the information.
The period covered by the information.

60. Which of the following statements concerning an


examination of prospective financial information is
incorrect?
A. The auditor should consider the period of time covered
by the financial information.
B. The auditor should obtain a sufficient level of
knowledge of the business to be able to evaluate whether
all significant assumptions required for the
preparation of the prospective financial information
have been identified.
C. The auditor need not obtain written representations from
management.
D. The auditor should consider the extent to which reliance
on entitys historical financial information is
justified.

According to PSAE 3400, The auditor should obtain a written


representations from the management regarding the intended use
of the prospective financial information, the completeness of
significant management assumptions and management assumptions
and managements acceptance of its responsibility for the
prospective financial information.

TRUE OR FALSE

1. A CPA firm can issue a compilation report only if the


engagement partner has no direct or material indirect
financial interest in the client.
2. A compilation of financial statements provides negative
assurance regarding the financial statements.
3. A review engagement is not currently an acceptable form of
association with prospective financial statements.
4. The statement that Nothing came to our attention which would
indicate that these statements are not fairly presented
expresses a disclaimer of opinion.
5. When a practitioner examines projected financial statements,
the practitioners report should include a separate
paragraph that describes the limitation on the usefulness
of the presentation.
6. The concept of limited assurance is provided for in agreed-
upon procedures engagement.
7. Use of agreed-upon procedures report is restricted to the
specified users.
8. An audit review requires less evidence related to internal
control than a review.
9. A practitioner who reviews the financial statements of an
entity should issue a report stating that a review provides
only limited assurance that a financial statements are
fairly presented.
10. An accountants report issued after compiling the financial
statements of an entity should state that a compilation is
limited to presenting in the form of financial statements
information that is the representation of management.
11. Before performing a review of an entitys financial
statements, a practitioner should obtain a sufficient level
of knowledge of the accounting principles and practices of
the industry in which the entity operates.
12. An accountants report issued after compiling the financial
statements of an entity should state that a compilation is
substantially less scope than an audit in accordance with
PSA, the objective of which is the expression of an opinion.
13. Negative assurance is not permissible in reports based upon
a review engagement.
14. Compilation reports are normally dated as of the clients
balance sheet date.
15. An accountants report issued after compiling the financial
statements of an entity should state that a compilation
consists principally of inquiries of company personnel and
analytical procedures.
16. Accepting an engagement to examine an entitys financial
projection most likely would be appropriate if the
projection were to be distributed to a bank with which the
entity is negotiating for a loan.
17. A non-audit engagement in which the accountant undertakes
to present, in the form of financial statements, information
that is the representation of management, without
undertaking to express any assurance on the statements is
called an agreed-upon procedures engagement.
18. A CPA must be independent to issue a review report.
19. A practitioners report on agreed-upon procedures that is
in the form of procedures and findings should contain an
acknowledgement of the practitioners responsibility for the
sufficiency of the procedures.
20. When performing compilation services, the accountant is
required to obtain an understanding of the clients internal
control.
21. An agreed-upon procedures engagement is one in which the
auditor and management or a third party agree that the
auditor will apply his/her judgement to determine procedures
to be performed.
22. The CPA may issue a report on whether the summary financial
statements derived from the audited financial statements are
consistent, in all material respects, with those financial
statements, in accordance with PFRS.
23. When an accountant compiles a financial forecast, the
accountants report should include a caveat that the
prospective results of the financial forecast may not be
achieved.
24. General use statements are prepared for use by known
contractual parties.
25. Financial projections can be provided for general use.

KEY ANSWERS

1. A 19. C 37. D
2. C 20. A 38. D
3. C 21. B 39. B
4. A 22. C 40. C
5. B 23. A 41. D
6. D 24. D 42. A
7. C 25. C 43. D
8. C 26. D 44. A
9. B 27. B 45. D
10. A 28. A 46. A
11. A 29. B 47. B
12. B 30. C 48. A
13. A 31. C 49. D
14. A 32. A 50. C
15. B 33. D 51. B
16. C 34. C 52. A
17. D 35. D 53. B
18. A 36. A 54. D
55. C 5. True 18. True
56. A 6. False 19. False
57. C 7. True 20. False
58. C 8. False 21. False
59. D 9. False 22. True
60. C 10. True 23. True
11. True 24. False
TRUE OR FALSE 12. False 25. False
13. False
1. False 14. False
2. False 15. False
3. True 16. True
4. False 17. False

You might also like